test one ms, test 2 med surge 3, RESPIRTORY TEST 3, test, Test 5 practice questions, med-surg chapter 31 test bank, Iggy Chapter 45, 40 MedSurg, A&C Med Surg 10th Ch 38, Iggy Chapter 35, Iggy10th Ch 33, A&C Med Surg Iggy Ch 10, Chapter 11: Concepts o...

Ace your homework & exams now with Quizwiz!

2. A client in the emergency department is having a stroke. The client weighs 225 pounds. After the initial bolus of t-Pa, at what rate should the nurse set the IV pump? (Record your answer using a decimal rounded to the nearest tenth.) ____ mL/hr

ANS: 1.4 mL/hr The client weighs 102 kg. The dose of t-PA is 0.9 mg/kg with a maximum of 90 mg, so the client's dose is 90 mg. 10% of the dose is given as a bolus IV over the first minute (9 mg). That leaves 81 mg to run in over 59 minutes. , which rounds to 1.4 mL/hr. DIF: Applying/Application REF: 939 KEY: Neurologic disorders| drug calculation| thrombolytic agents MSC: Integrated Process: Nursing Process: Implementation NOT: Client Needs Category: Physiological Integrity: Pharmacological and Parenteral Therapies

The nurse is caring for a client with suspected septic shock. What does the nurse prepare to do within 1 hour of the client being identified as possibly having sepsis? (Select all that apply.) a. Administer antibiotics. b. Draw serum lactate levels. c. Infuse vasopressors. d. Measure central venous pressure. e. Obtain blood cultures. f. Administer rapid bolus of IV crystalloids.

a. Administer antibiotics. b. Draw serum lactate levels. c. Infuse vasopressors. e. Obtain blood cultures. f. Administer rapid bolus of IV crystalloids.

A nurse cares for a client with an intravenous temporary pacemaker for bradycardia. The nurse observes the presence of a pacing spike but no QRS complex on the client's electrocardiogram. What action would the nurse take next? a. Administer intravenous diltiazem. b. Assess vital signs and level of consciousness. c. Administer sublingual nitroglycerin. d. Assess capillary refill and temperature.

B In temporary pacing, the wires are threaded onto the epicardial surface of the heart and exit through the chest wall. The pacemaker spike would be followed immediately by a QRS complex. Pacing spikes seen without subsequent QRS complexes imply loss of capture. If there is no capture, then there is no ventricular depolarization and contraction. The nurse would assess for cardiac output via vital signs and level of consciousness. The other interventions would not determine if the client is tolerating the loss of capture.

A nurse plans care for a client with a halo fixator. Which interventions would the nurse include in this client's plan of care? (Select all that apply.) a. Remove the vest for client bathing. b. Assess the pin sites for signs of infection. c. Loosen the pins when sleeping. d. Decrease the patient's oral fluid intake. e. Assess the chest and back for skin breakdown.

B, E

The nurse learning about cancer development remembers characteristics of normal cells. Which characteristics does this include? (Select all that apply.) a. Differentiated function b. Large nucleus-to-cytoplasm ratio c. Loose adherence d. Nonmigratory e. Specific morphology f. Orderly and specific growth

Differentiated function Nonmigratory Specific morphology Orderly and specific growth

A nurse teaches a client with diabetes mellitus about foot care. Which statements would the nurse include in this client's teaching? (Select all that apply.) a. "Do not walk around barefoot." b. "Soak your feet in a tub each evening." c. "Trim toenails straight across with a nail clipper." d. "Treat any blisters or sores with Epsom salts." e. "Wash your feet every other day."

a. "Do not walk around barefoot." c. "Trim toenails straight across with a nail clipper."

A client has a serum potassium level of 6.5 mEq/L (6.5 mmol/L), a serum creatinine level of 2 mg/dL (176 mcmol/L), and a urine output of 350 mL/day. What is the best action by the nurse? a. Place the client on a cardiac monitor immediately. b. Teach the client to limit high-potassium foods. c. Continue to monitor the client's intake and output. d. Ask to have the laboratory redraw the blood

a. Place the client on a cardiac monitor immediately.

2. The emergency department team is performing cardiopulmonary resuscitation on a client when the client's spouse arrives. Which action would the nurse take first? a. Request that the client's spouse sit in the waiting room. b. Ask the spouse if he or she wishes to be present during the resuscitation. c. Suggest that the spouse begin to pray for the patient. d. Refer the client's spouse to the hospital's crisis team.

b. Ask the spouse if he or she wishes to be present during the resuscitation. If resuscitation efforts are still under way when the family arrives, one or two family members may be given the opportunity to be present during lifesaving procedures. The other options do not give the spouse the opportunity to be present for the client or to begin to have closure.

A client arrives in the emergency department after being in a car crash with fatalities. The client has a nearly amputated leg that is bleeding profusely. What action by the nurse takes priority? a. Apply direct pressure to the bleeding. b. Ensure the client has a patent airway. c. Obtain a pulse oximetry reading d. Start two large-bore IV catheters.

b. Ensure the client has a patent airway. Airway is the priority, followed by breathing (pulse oximetry) and circulation (IVs and direct pressure).

5. A client is going to be admitted for a scheduled surgical procedure. Which action does the nurse explain is the most important thing the client can do to protect against errors? a. Bring a list of all medications and what they are for. b. Keep the doctor's phone number by the telephone. c. Make sure all providers wash hands before entering the room. d. Write down the name of each caregiver who comes in the room.

a. Bring a list of all medications and what they are for.

A home health care nurse is planning an exercise program with an older adult who lives at home independently but whose mobility issues prevent much activity outside the home. Which exercise regimen would be most beneficial to this adult? a. Building strength and flexibility b. Improving exercise endurance c. Increasing aerobic capacity d. Providing personal training

a. Building strength and flexibility This older adult is mostly homebound. Exercise regimens for homebound clients include things to increase functional fitness and ability for activities of daily living. Strength and flexibility will help the client to be able to maintain independence longer. The other plans are good but will not specifically maintain the client's functional abilities.

A new nurse reports to the nurse preceptor that a client requested pain medication, and when the nurse brought it, the client was sound asleep. The nurse states the client cannot possibly sleep with the severe pain the client described. Which response by the experienced nurse is best? a. "Being able to sleep doesn't mean pain doesn't exist." b. "Have you ever experienced any type of pain?" c. "The client should be assessed for drug addiction." d. "You're right; I would put the medication back."

a."Being able to sleep doesn't mean pain doesn't exist."

A client presents to the emergency department after prolonged exposure to the cold. The client is difficult to arouse and speech is incoherent. What action would the nurse take first? a. Reposition the client into a prone position. b. Administer warmed intravenous fluids to the client. c. Wrap the client's extremities in warm blankets. d. Initiate extracorporeal rewarming via hemodialysis.

b. Administer warmed intravenous fluids to the client. Moderate hypothermia manifests with muscle weakness, increased loss of coordination, acute confusion, apathy, incoherence, stupor, and impaired clotting. Moderate hypothermia is treated by core rewarming methods, which include administration of warm IV fluids; heated oxygen; and heated peritoneal, pleural, gastric, or bladder lavage. The client's trunk would be warmed prior to the extremities to prevent peripheral vasodilation. Extracorporeal warming with cardiopulmonary bypass or hemodialysis is a treatment for severe hypothermia.

A nurse plans care for a client admitted with a snakebite to the right leg. With whom would the nurse collaborate? a. The facility's neurologist b. The poison control center c. The physical therapy department d. A herpetologist (snake specialist)

b. The poison control center For the client with a snakebite, the nurse would contact the regional poison control center immediately for specific advice on antivenom administration and client management.

When working with women who are taking hormonal birth control, what health promotion measures does the nurse teach to prevent possible pulmonary embolism (PE)? (Select all that apply.) a. Avoid drinking alcohol. b. Eat more omega-3 fatty acids. c. Exercise on a regular basis. d. Maintain a healthy weight. e. Stop smoking cigarettes.

c. Exercise on a regular basis. d. Maintain a healthy weight. e. Stop smoking cigarettes.

After teaching a client who is recovering from pancreas transplantation, the nurse assesses the client's understanding. Which statement made by the client indicates a need for further teaching? a. "If I develop an infection, I should stop taking my corticosteroid." b. "If I have pain over the transplant site, I will call the surgeon immediately." c. "I should avoid people who are ill or who have an infection." d. "I should take my cyclosporine exactly the way I was taught."

a. "If I develop an infection, I should stop taking my corticosteroid."

The nurse is assessing a client with acute pyelonephritis. What assessment findings would the nurse expect? (Select all that apply.) a. Fever b. Chills c. Tachycardia d. Tachypnea e. Flank or back pain f. Fatigue

a. Fever b. Chills c. Tachycardia d. Tachypnea e. Flank or back pain f. Fatigue

A nurse assesses a client who experienced a spinal cord injury at the T5 level 12 hours ago. Which assessment findings would the nurse correlate with neurogenic shock? (Select all that apply.) a. Heart rate of 34 beats/min b. Blood pressure of 185/65 mm Hg c. Urine output less than 30 mL/hr d. Decreased level of consciousness e. Increased oxygen saturation

A, C, D,

A nurse auscultates a harsh hollow sound over a client's trachea and larynx. What action would the nurse take first? a. Document the findings. b. Administer oxygen therapy. c. Position the client in high-Fowler position. d. Administer prescribed albuterol.

a. Document the findings.

The nurse plans care for a client who has acute pancreatitis and is prescribed nothing by mouth (NPO). With which health care team members would the nurse collaborate to provide appropriate nutrition to this client? (Select all that apply.) a. Registered dietitian nutritionist b. Nursing assistant c. Clinical pharmacist d. Certified herbalist e. Primary health care provider

a. Registered dietitian nutritionist c. Clinical pharmacist e. Primary health care provider Clients who are prescribed NPO while experiencing an acute pancreatitis episode may need enteral or parenteral nutrition. The nurse would collaborate with the registered dietitian nutritionist, clinical pharmacist, and primary health care provider to plan and implement the more appropriate nutritional interventions. The nursing assistant and certified herbalist would not assist with this clinical decision

A nurse cares for a postmenopausal client who has had two episodes of bacterial urethritis in the last 6 months. The client asks, "I never have urinary tract infections. Why is this happening now?" How would the nurse respond? a. "Your immune system becomes less effective as you age." b. "Low estrogen levels can make the tissue more susceptible to infection." c. "You should be more careful with your personal hygiene in this area." d. "It is likely that you have an untreated sexually transmitted disease."

b. "Low estrogen levels can make the tissue more susceptible to infection."

A nurse assesses a client with diabetes mellitus. Which assessment finding would alert the nurse to decreased kidney function in this client? a. Urine specific gravity of 1.033 b. Presence of protein in the urine c. Elevated capillary blood glucose level d. Presence of ketone bodies in the urine

b. Presence of protein in the urine

A nurse develops a dietary plan for a client with diabetes mellitus and new-onset microalbuminuria. Which component of the client's diet would the nurse decrease? a. Carbohydrates b. Proteins c. Fats d. Total calories

b. Proteins

A client is admitted with acute pancreatitis. What priority problem would the nurse expect the client to report? a. Nausea and vomiting b. Severe boring abdominal pain c. Jaundice and itching d. Elevated temperature

b. Severe boring abdominal pain The client who has acute pancreatitis reports severe boring abdominal pain that is often rated by clients as a 10+ on a 0-10 pain scale. Nausea, vomiting, and fever may also occur, but that is not the client's priority for care

After teaching a client who has diverticulitis, a nurse assesses the client's understanding. Which statement made by the client indicates a need for further teaching? a. "I'll ride my bike or take a long walk at least three times a week." b. "I must try to include at least 25 g of fiber in my diet every day." c. "I will take a laxative nightly at bedtime to avoid becoming constipated." d. "I should use my legs rather than my back muscles when I lift heavy objects."

c. "I will take a laxative nightly at bedtime to avoid becoming constipated." Laxatives are not recommended for patients with diverticulitis because they can increase pressure in the bowel, causing additional outpouching of the lumen. Exercise and a high-fiber diet are recommended for clients with diverticulitis because they promote regular bowel function. Using the leg muscles rather than the back for lifting prevents abdominal straining

12. A nurse has educated a client on isoniazid. What statement by the client indicates that teaching has been effective? a. "I need to take extra vitamin C while on isoniazid." b. "I should take this medicine with milk or juice." c. "I will take this medication on an empty stomach." d. "My contact lenses will be permanently stained."

c. "I will take this medication on an empty stomach." Isoniazid needs to be taken on an empty stomach, either 1 hour before or 2 hours after meals. Extra vitamin B needs to be taken while on the drug. Staining of contact lenses commonly occurs while taking rifampin.

A new nurse reads a client has a wound "healing by second intention" and asks what that means. Which description by the charge nurse is most accurate? a. "The wound edges have been approximated and stitched together." b. "The wound was stapled together after an infection was cleared up." c. "The wound is an open cavity that will fill in with granulation tissue." d. "The wound was contaminated by debris and can't be closed at all."

c. "The wound is an open cavity that will fill in with granulation tissue." Wounds healing by second intention are deeper wounds that leave open cavities. These wounds heal as connective tissue fills in the dead space. A wound that has its edges brought together (approximated) and sutured or stapled together is said to be healing by first intention. A wound that was left open while an infection healed and then is closed is an example of healing by third intention. A wound that cannot be closed at all would be left to heal by second intention.

A nurse is caring for a client whose spouse died in a recent mass casualty accident. The client says, "I can't believe that my spouse is gone and I am left to raise my children all by myself." How would the nurse respond? a. "Please accept my sympathies for your loss." b. "I can call the hospital chaplain if you wish." c. "You sound anxious about being a single parent." d. "At least your children still have you in their lives."

c. "You sound anxious about being a single parent." Therapeutic communication includes active listening and honesty. This statement demonstrates that the nurse recognizes the client's distress and has provided an opening for discussion. Extending sympathy and offering to call the chaplain do not give the client the opportunity to discuss feelings. Stating that the children still have one parent discounts the client's feelings and situation.

A client is started on continuous venovenous hemofiltration (CVVH). Which finding would require immediate action by the nurse? a. Potassium level of 5.5 mEq/L (5.5 mmol/L) b. Sodium level of 138 mEq/L (138 mmol/L) c. Blood pressure of 76/58 mm Hg d. Pulse rate of 88 beats/min

c. Blood pressure of 76/58 mm Hg

A client is starting hormonal therapy with tamoxifen to lower the risk for breast cancer. What information needs to be explained by the nurse regarding the action of this drug? a. It blocks the release of luteinizing hormone. b. It interferes with cancer cell division. c. It selectively blocks estrogen in the breast. d. It inhibits DNA synthesis in rapidly dividing cells.

c. It selectively blocks estrogen in the breast.

A nurse learns that the fastest growing subset of the older population is which group? a. Elite old b. Middle old c. Old old d. Young old

c. Old old The old old is the fastest growing subset of the older population. This is the group comprising those 85 to 99 years of age. The young old are between 65 and 74 years of age; the middle old are between 75 and 84 years of age; and the elite old are over 100 years of age.

A client with diabetes mellitus type 2 has been well controlled with metformin. The client is scheduled for magnetic resonance imaging (MRI) scan with contrast. What priority would the nurse take at this time? a. Teach the client about the purpose of the MRI. b. Assess the client's blood urea nitrogen and creatinine. c. Tell the client to withhold metformin for 24 hours before the MRI. d. Ask the client if he or she is taking antibiotics.

c. Tell the client to withhold metformin for 24 hours before the MRI.

4. While triaging clients in a crowded emergency department, a nurse assesses a client who presents with symptoms of tuberculosis. Which action would the nurse take first? a. Apply oxygen via nasal cannula. b. Administer intravenous 0.9% saline solution. c. Transfer the client to a negative-pressure room. d. Obtain a sputum culture and sensitivity.

c. Transfer the client to a negative-pressure room. A client with signs and symptoms of tuberculosis or other airborne pathogens would be placed in a negative-pressure room to prevent contamination of staff, clients, and family members in the crowded emergency department. The client may or may not need oxygen or an IV. A sputum culture would be obtained but is not the priority.

After teaching a patient with type 2 diabetes mellitus who is prescribed nateglinide, the nurse assesses the client's understanding. Which statement made by the patient indicates a correct understanding of the prescribed therapy? a. "I'll take this medicine during each of my meals." b. "I must take this medicine in the morning when I wake." c. "I will take this medicine before I go to bed." d. "I will take this medicine immediately before I eat."

d. "I will take this medicine immediately before I eat."

A nurse teaches a client with diabetes mellitus who is experiencing numbness and reduced sensation. Which statement would the nurse include in this client's teaching to prevent injury? a. "Examine your feet using a mirror every day." b. "Rotate your insulin injection sites every week." c. "Check your blood glucose level before each meal." d. "Use a bath thermometer to test the water temperature."

d. "Use a bath thermometer to test the water temperature."

A nurse assesses a client admitted with a brown recluse spider bite. Which assessment does the nurse perform to identify complications of this bite? a. Ask the client about pruritus at the bite site. b. Inspect the bite site for a bluish purple vesicle. c. Assess the extremity for redness and swelling. d. Monitor the client's temperature every 4 hours.

d. Monitor the client's temperature every 4 hours. Fever and chills indicate systemic toxicity, which can lead to hemolytic anemia, thrombocytopenia, DIC, and death. Assessing for a fever would indicate this complication. All other symptoms are normal for a brown recluse bite and would be assessed, but they do not provide information about complications from the bite.

A nurse plans care for a client who is experiencing dyspnea and must stop multiple times when climbing a flight of stairs. Which intervention would the nurse include in this client's plan of care? a. Assistance with activities of daily living b. Physical therapy activities every day c. Oxygen therapy at 2 L per nasal cannula d. Complete bedrest with frequent repositioning

a. Assistance with activities of daily living

A nurse prepares to discharge a client who is newly diagnosed with a chronic inflammatory bowel disease. Which questions would the nurse ask in preparation for discharge? (Select all that apply.) a. Does your gym provide yoga classes? b. When should you contact your provider? c. What do you plan to eat for dinner? d. Do you have a scale for daily weights? e. How many bathrooms are in your home?

a. Does your gym provide yoga classes? b. When should you contact your provider? c. What do you plan to eat for dinner? e. How many bathrooms are in your home? A home assessment for a client who has a chronic inflammatory bowel disease would include identifying adequacy and availability of bathroom facilities, opportunities for rest and relaxation, and the client's knowledge of dietary therapy, and when to contact the primary health care provider. The client does not need to perform daily weights

A nurse assesses a client who is recovering from a Whipple procedure. Which assessment finding alerts the nurse to immediately contact the primary health care provider? a. Drainage from a fistula b. Diminished bowel sounds c. Pain at the incision site d. Nasogastric (NG) tube drainage

a. Drainage from a fistula Complications of a Whipple procedure include secretions that drain from a fistula and peritonitis. Absent bowel sounds, pain at the incision site, and NG tube drainage are normal postoperative findings.

A client appears dyspneic, but the oxygen saturation is 97%. What action by the nurse is best? a. Assess for other signs of hypoxia. b. Change the sensor on the pulse oximeter. c. Obtain a new oximeter from central supply. d. Tell the client to take slow, deep breaths.

A. Assess for other manifestations of hypoxia.

A nurse on the medical-surgical unit has received a hand-off report. Which client would the nurse see first? a. Client being discharged later on a complicated analgesia regimen. b. Client with new-onset abdominal pain, rated as an 8 on a 0-10 scale. c. Postoperative client who received oral opioid analgesia 45 minutes ago. d. Client who has returned from physical therapy and is resting in the recliner.

B. Client with new-onset abdominal pain, rated as an 8 on a 0-10 scale.

A client is admitted with a pulmonary embolism (PE). The client is young, healthy, and active and has no known risk factors for PE. What action by the nurse is most appropriate? a. Encourage the client to walk 5 minutes each hour. b. Refer the client to smoking cessation classes. c. Teach the client about factor V Leiden testing. d. Tell the client that sometimes no cause for disease is found.

C.Teach the client about factor V Leiden testing.

A nurse prepares to defibrillate a client who is in ventricular fibrillation. Which intervention is appropriate for the nurse to perform prior to defibrillating this client? a. Make sure that the defibrillator is set to synchronous mode. b. Administer 1 mg of intravenous epinephrine. c. Test the equipment by delivering a smaller shock at 100 J. d. ensure that everyone is clear of contact with the client and the bed

D To avoid injury, the rescuer commands that all personnel clear contact with the client or the bed and ensures their compliance before delivery of the shock. Defibrillation is done in asynchronous mode. Equipment would not be tested before a client is defibrillated because this is an emergency procedure; equipment would be checked on a routine basis. Defibrillation takes priority over any medications.

10. A nurse cares for a client who has packing inserted for posterior nasal bleeding. What action would the nurse take first? a. Assess the client's pain level. b. Keep the client's head elevated. c. Teach the client about the causes of nasal bleeding. d. Assess the client's airway.

D. Assess the client's airway. If the packing slips out of place, it may obstruct the client's airway. The other options are good interventions, but ensuring that the airway is patent in the priority objective.

A nurse assesses a client who is prescribed levothyroxine for hypothyroidism. Which assessment finding alerts the nurse that drug therapy is effective? a. Thirst is recognized and fluid intake is appropriate. b. Weight has been the same for 3 weeks. c. Total white blood cell count is 6000 cells/mm3 (6 × 109 /L). d. Heart rate is 76 beats/min and regular.

D.Heart rate is 76 beats/min and regular.

The nurse is preparing a client who has chronic pancreatitis about how to prevent exacerbations of the disease. Which health teaching will the nurse include? (Select all that apply.) a. "Avoid alcohol ingestion." b. "Be sure and balance rest with activity." c. "Avoid caffeinated beverages." d. "Avoid green, leafy vegetables." e. "Eat small meals and high-calorie snacks."

a. "Avoid alcohol ingestion." b. "Be sure and balance rest with activity." c. "Avoid caffeinated beverages." e. "Eat small meals and high-calorie snacks." Clients who have chronic pancreatitis need to avoid GI stimulants, including alcohol, caffeine, and nicotine. Food and snacks need to be high-calorie to prevent additional weight loss. Green vegetables can be consumed if tolerated by the client.

The nurse teaches a client who has viral gastroenteritis. Which dietary instruction would the nurse include in the health teaching? a. "Drink plenty of fluids to prevent dehydration." b. "You should only drink 1 L of fluids daily." c. "Increase your protein intake by drinking more milk." d. "Sips of cola or tea may help to relieve your nausea."

a. "Drink plenty of fluids to prevent dehydration. The client should drink plenty of fluids to prevent dehydration. Milk products may not be tolerated. Caffeinated beverages increase intestinal motility and should be avoided

A nurse teaches a client about self-management after experiencing a urinary calculus treated by lithotripsy. Which statements would the nurse include in this client's discharge teaching? (Select all that apply.) a. "Finish the prescribed antibiotic even if you are feeling better." b. "Drink at least 3 L of fluid each day." c. "The bruising on your back may take several weeks to resolve." d. "Report any blood present in your urine." e. "It is normal to experience pain and difficulty urinating."

a. "Finish the prescribed antibiotic even if you are feeling better." b. "Drink at least 3 L of fluid each day." c. "The bruising on your back may take several weeks to resolve."

A nurse cares for a client who had a colostomy placed in the ascending colon 2 weeks ago. The client states, "The stool in my pouch is still liquid." How would the nurse respond? a. "The stool will always be liquid with this type of colostomy." b. "Eating additional fiber will bulk up your stool and decrease diarrhea." c. "Your stool will become firmer over the next couple of weeks." d. "This is abnormal. I will contact your primary health care provider."

a. "The stool will always be liquid with this type of colostomy."

A client is unsure of the decision to undergo peritoneal dialysis (PD) and wishes to discuss the advantages of this treatment with the nurse. Which statements by the nurse are correct regarding PD? (Select all that apply.) a. "You will not need vascular access to perform PD." b. "There is less restriction of protein and fluids." c. "You will have no risk for infection with PD." d. "You have flexible scheduling for the exchanges." e. "It takes less time than hemodialysis treatments."

a. "You will not need vascular access to perform PD." b. "There is less restriction of protein and fluids." d. "You have flexible scheduling for the exchanges."

A nurse teaches a client with hyperthyroidism. Which dietary modifications should the nurse include in this client's health teaching? (Select all that apply.) a. Increased carbohydrates b. Decreased fats c. Increased calorie intake d. Supplemental vitamins e. Increased proteins

a. Increased carbohydrates c. Increased calorie intake e. Increased proteins

A nurse cares for a client recovering from prosthetic valve replacement surgery. The client asks, "Why will I need to take anticoagulants for the rest of my life?" How should the nurse respond? a. "The prosthetic valve places you at greater risk for a heart attack." b. "Blood clots form more easily in artificial replacement valves." c. "The vein taken from your leg reduces circulation in the leg." d. "The surgery left a lot of small clots in your heart and lungs."

b. "Blood clots form more easily in artificial replacement valves." Synthetic valve prostheses and scar tissue provide surfaces on which platelets can aggregate easily and initiate the formation of blood clots.

A client is preparing to have a fecal occult blood test (FOBT). What health teaching would the nurse include prior to the test? a. "This test will determine whether you have colorectal cancer." b. "You need to avoid red meat and NSAIDs for 48 hours before the test." c. "You don't need to have this test because you can have a virtual colonoscopy." d. "This test can determine your genetic risk for developing colorectal cancer."

b. "You need to avoid red meat and NSAIDs for 48 hours before the test."

A nurse plans care for a client who is nearing end of life. Which question should the nurse ask when developing this clients plan of care? a. Is your advance directive up to date and notarized? b. Do you want to be at home at the end of your life? c. Would you like a physical therapist to assist you with range-of-motion activities? d. Have your children discussed resuscitation with your health care provider?

b. Do you want to be at home at the end of your life? When developing a plan of care for a dying client, consideration should be given for where the client wants to die. Advance directives do not need to be notarized. A physical therapist would not be involved in end-of-life care. The client should discuss resuscitation with the health care provider and children; do-not-resuscitate status should be the clients decision, not the familys decision.

A nurse assesses a client who is recovering from an open traditional Whipple surgical procedure. Which assessment finding(s) alert(s) the nurse to a complication from this surgery? (Select all that apply.) a. Clay-colored stools b. Substernal chest pain c. Shortness of breath d. Lack of bowel sounds or flatus e. Urine output of 20 mL/6 hr

b. Substernal chest pain c. Shortness of breath d. Lack of bowel sounds or flatus e. Urine output of 20 mL/6 hr Myocardial infarction (chest pain), pulmonary embolism (shortness of breath), adynamic ileus (lack of bowel sounds or flatus), and acute kidney injury (urine output of 20 mL/6 hr) are common complications for which the nurse must assess the client after the Whipple procedure. Clay-colored stools are associated with cholecystitis and are not a complication of a Whipple procedure

The nurse is teaching a client how to increase the flow of dialysate into the peritoneal cavity during dialysis. Which statement by the client demonstrates a correct understanding of the teaching? a. "I should leave the drainage bag above the level of my abdomen." b. "I could flush the tubing with normal saline if the flow stops." c. "I should take a stool softener every morning to avoid constipation." d. "My diet should have low fiber in it to prevent any irritation."

c. "I should take a stool softener every morning to avoid constipation."

A nurse is caring for a client who has a diagnosis of multiple organ dysfunction syndrome (MODS) who will be receiving sodium nitroprusside via IV infusion. What action by the nurse causes the charge nurse to intervene? a. Assessing the IV site before giving the drug b. Obtaining a programmable ("smart") IV pump c. Removing the IV bag from the brown plastic cover d. Taking and recording a baseline set of vital signs

c. Removing the IV bag from the brown plastic cover Nitroprusside degrades in the presence of light, so it must be protected by leaving it in the original brown plastic bag when infusing. The other actions are correct

After teaching a client who has diabetes mellitus with retinopathy, nephropathy, and peripheral neuropathy, the nurse assesses the client's understanding. Which statement made by the client indicates a correct understanding of the teaching? a. "I have so many complications; exercising is not recommended." b. "I will exercise more frequently because I have so many complications." c. "I used to run for exercise; I will start training for a marathon." d. "I should look into swimming or water aerobics to get my exercise."

d. "I should look into swimming or water aerobics to get my exercise."

The nurse teaches a client about how to prevent transmission of gastroenteritis. Which statement by the nurse indicates a need for further teaching? a. "I won't let anyone use my dishes or glasses." b. "I'll wash my hands with antibacterial soap." c. "I'll keep my bathroom extra clean." d. "I'll cook all the meals for my family."

d. "I'll cook all the meals for my family." All of these statements are correct except for that the client should not prepare meals for others to help prevent transmission of gastroenteritis

A new nurse asks for an explanation of "refractory hypoxemia." What answer by the staff development nurse is best? a. "It is chronic hypoxemia that accompanies restrictive airway disease." b. "It is hypoxemia from lung damage due to mechanical ventilation." c. "It is hypoxemia that continues even after the client is weaned from oxygen." d. "It is hypoxemia that persists even with 100% oxygen administration.

d. "It is hypoxemia that persists even with 100% oxygen administration.

A nurse teaches a client with functional urinary incontinence. Which statement would the nurse include in this client's teaching? a. "You must clean around your catheter daily with soap and water." b. "You will need to be on your drug therapy for life." c. "Operations to repair your bladder are available, and you can consider these." d. "You might want to get pants with elastic waistbands."

d. "You might want to get pants with elastic waistbands."

The nurse is admitting a client who has acute glomerulonephritis caused by beta streptococcus. What drug therapy would the nurse expect to be prescribed for this client? a. Antihypertensives b. Antilipidemics c. Antidepressants d. Antibiotics

d. Antibiotics

During dressing changes, the nurse assesses a client who had breast reconstruction. Which finding would cause the nurse to take immediate action? a. Slightly reddened incisional area b. Blood pressure of 128/75 mm Hg c. Temperature of 99° F (37.2° C) d. Dusky color of the breast flap

d. Dusky color of the breast flap

A nurse learns that which of the following is the single biggest risk factor for developing cancer? a. Exposure to tobacco b. Advancing age c. Occupational chemicals d. Oncovirus infection

Advancing age

A nurse is caring for a dying client whose adult child confides frequent crying episodes to the nurse. How does the nurse respond?

" your feelings are completely normal and may continue for a long time."

A nurse plans care for a client who is nearing end-of-life. Which question will the nurse ask when developing this clients plan of care?

"Do you want to be at home at the end of your life?"

A client calls the clinic to report exposure to poison ivy and an itchy rash that is not helped with over-the-counter antihistamines. What response by the nurse is most appropriate? A. Antihistamines do not help poison ivy B. There are different antihistamines to try C. You should be seen in the clinic right away D. You will need to take some IV steroids

A Antihistamines do not help, corticosteroids do since histamines are NOT a mediator in this type of reaction

A nurse assesses a client who is recovering from an open traditional anterior cervical fusion. Which assessment findings would alert the nursing to a complication from this procedure? (Select all that apply.) a. Difficulty swallowing b. Hoarse voice c. Constipation d. Bradycardia e. Hypertension

A, B

The nurse is assessing a client for signs and symptoms of systemic lupus erythematous. Which of the following would be consistent with this disorder? Select all that apply A. Discoid rash on skin exposed to sunlight B. Urinalysis positive for casts and protein C. Painful deformed small joints D. Pain on inspiration E. Thrombocytosis F. Serum positive for antinuclear antibodies (ANA)

A, B, D, F S&S of SLE include but are not limited to - a discoid rash on skin exposed to the sun - UA with casts and protein - pleurisy as manifested by pain on inspiration - positive ANA titers in the blood

A nurse promotes the prevention of lower back pain by teaching clients at a community center. Which statement(s) would the nurse include in this education? (Select all that apply.) a. "Participate in an exercise program to strengthen back muscles." b. "Purchase a mattress that allows you to adjust the firmness." c. "Wear flat instead of high-heeled shoes to work each day." d. "Keep your weight within 20% of your ideal body weight." e. "Avoid prolonged standing or sitting, including driving."

A, C, E

The nurse has educated a community group of risk factors for ovarian cancer. Which statement by a participant shows the need for reviewing the information? A. "This is a disease of young women." B. "Never being pregnant increases my risk." C. "Difficulty conceiving is a risk factor." D. "Having endometriosis is one of the risks."

A. "This is a disease of young women." Ovarian cancer usually strikes women who are middle age or older. Nulliparity, difficulty conceiving, and endometriosis all increase risk and are correct statements.

7. A nurse is demonstrating suctioning a tracheostomy during the annual skills review. What action by the student demonstrates that more teaching is needed? a. Applying suction while inserting the catheter b. Preoxygenating the client prior to suctioning c. Suctioning for a total of three times if needed d. Suctioning for only 10 to 15 seconds each time

A. Applying suction while inserting the catheter Suction would only be applied while withdrawing the catheter. The other actions are appropriate.

Which of the following are risk factors for breast cancer? (Select all that apply). A. Early menses B. Nulliparity C. Increased age D. Hormone replacement therapy (HRT) E. Use of an IUD F. Recent oral contraceptive use

A. Early menses C. Increased age D. Hormone replacement therapy (HRT) F. Recent oral contraceptive use Early menses, increased age, hormone replacement therapy and recent use of oral contraceptives are all risk factors for breast cancer. Nulliparity is a risk factor for endometrial and ovarian cancer. use of an IUD is a risk factor for endometrial and cervical cancer.

A client is interested in learning about the risk factors for prostate cancer. Which factors does the nurse include in the teaching? (Select all that apply.) A. First-degree relative with prostate cancer B. Smoking C. Obesity D. Advanced age E. Eating too much red meat F. Race

A. First-degree relative with prostate cancer D. Advanced age E. Eating too much red meat F. Race Risk factors for prostate cancer include having a first-degree relative with the disease, advanced age, and African-American race. Smoking, obesity, and eating too much red meat are not considered risk factors. Research is exploring the relationship with diet.

3. A nurse is working with a client who takes clopidogrel. The client's recent laboratory results include a blood urea nitrogen (BUN) of 33 mg/dL and creatinine of 2.8 mg/dL. What action by the nurse is best? a. Ask if the client eats grapefruit. b. Assess the client for dehydration. c. Facilitate admission to the hospital. d. Obtain a random urinalysis.

ANS: A There is a drug-food interaction between clopidogrel and grapefruit that can lead to acute kidney failure. This client has elevated renal laboratory results, indicating some degree of kidney involvement. The nurse would assess if the client eats grapefruit or drinks grapefruit juice. Dehydration can cause the BUN to be elevated, but the elevation in creatinine is more specific for a kidney injury. The client does not necessarily need to be admitted. A urinalysis may or may not be ordered.

27. The nurse is assessing a client on admission to the hospital. The client's leg appears as shown below: What action by the nurse is best? a. Assess the client's ankle-brachial index. b. Elevate the client's leg above the heart. c. Obtain an ice pack to provide comfort. d. Prepare to teach about heparin sodium.

ANS: A This client has dependent rubor, a classic finding in peripheral arterial disease. The nurse would measure the client's ankle-brachial index. Elevating the leg above the heart will further impede arterial blood flow. Ice will cause vasoconstriction, also impeding circulation and perhaps causing tissue injury. Heparin sodium is not the drug of choice for this condition.

The nurse is assessing a client with long-term rheumatoid arthritis (RA) who has been taking prednisone for 10 years. For which complications of chronic drug therapy would the nurse assess? (Select all that apply.) a. Osteoporosis b. Diabetes mellitus c. Glaucoma d. Hypertension e. Hypokalemia f. Decreased immunity

ANS: A, B, C, D, E, F Prednisone is a corticosteroid that is sometimes used for autoimmune disorders like RA when other drugs are not effective or cannot be tolerated. However, it can cause many complications when used long-term, including all of the health problems listed in the choices.

A hospice nurse plans care for a client who is experiencing pain. Which complementary therapies should the nurse incorporate in this clients pain management plan? (Select all that apply.) a. Play music that the client enjoys. b. Massage tissue that is tender from radiation therapy. c. Rub lavender lotion on the clients feet. d. Ambulate the client in the hall twice a day. e. Administer intravenous morphine.

ANS: A, C Complementary therapies for pain management include massage therapy, music therapy, Therapeutic Touch, and aromatherapy. Nurses should not massage over sites of tissue damage from radiation therapy. Ambulation and intravenous morphine are not complementary therapies for pain management.

A client with rheumatoid arthritis (RA) has an acutely swollen, red, and painful joint. What nonpharmacologic intervention does the nurse recommend? a. Heating pad b. Ice packs c. Splint d. Paraffin dip

ANS: B Ice is best for acute inflammation. Heat often helps with joint stiffness. Splinting helps preserve joint function. A paraffin dip is used to provide warmth to the joint which is more appropriate for chronic pain and stiffness.

4. A client with a stroke is being evaluated for fibrinolytic therapy. What information from the client or family is most important for the nurse to obtain? a. Loss of bladder control b. Other medical conditions c. Progression of symptoms d. Time of symptom onset

ANS: D The time limit for initiating fibrinolytic therapy for a stroke is 3 to 4.5 hours, so the exact time of symptom onset is the most important information for this client. The other information is not as critical. DIF: Applying/Application REF: 938 KEY: Stroke| neurologic disorders| nursing assessment| fibrinolytic therapy MSC: IntegratedProcess:NursingProcess:Assessment NOT: Client Needs Category: Physiological Integrity: Pharmacological and Parenteral Therapies

A nurse assesses a client with multiple sclerosis after administering prescribed fingolimod. For which common side effect would the nurse monitor? a. Peripheral edema b. Facial flushing c. Tachycardia d. Fever

B

A client is on intravenous heparin to treat a pulmonary embolism. The clients most recent partial thromboplastin time (PTT) was 25 seconds. What order should the nurse anticipate? a. Decrease the heparin rate. b. Increase the heparin rate. c. No change to the heparin rate. d. Stop heparin; start warfarin.

B. Increase the heparin rate

What type of drugs are useful in a Type IV reaction?

Corticosterioids are the most helpful in reducing discomfort Histamine antagonists are not useful because these reactions are not caused by histamines

The nurse initiates care for a client with a cervical spinal cord injury who arrives via emergency medical services. What action would the nurse take first? a. Assess level of consciousness. b. Obtain vital signs. c. Administer oxygen therapy. d. Evaluate respiratory status

D

A nurse asks why several clients are getting more than one type of pain medication instead of very high doses of one medication. Which response by the charge nurse is best? a. "A multimodal approach is the preferred method of control." b. "Clients are consumers and they demand lots of pain medicine." c. "We are all much more liberal with pain medications now." d. "Pain is so complex it takes different approaches to control it."

D. "Pain is so complex it takes different approaches to control it."

1. A nurse is assessing a client who has suffered a nasal fracture. Which assessment would the nurse perform first? a. Facial pain b. Vital signs c. Bone displacement d. Airway patency

D. Airway patency A patent airway is the priority. The nurse first would make sure that the airway is patent and then would determine whether the client is in pain and whether bone displacement or blood loss has occurred.

An older adult has diabetic neuropathy and often reports unbearable foot pain. About which medication would the nurse plan to educate the client? a. Desipramine b. Duloxetine c. Morphine sulfate d. Nortriptyline

Duloxetine

A nurse is assessing a client with glioblastoma. What assessment is most important? a. Abdominal palpation b. Abdominal percussion c. Lung auscultation d. Neurologic examination

Neurologic examination

In Type 1 hypersensitivity there is a reaction of what immunoglobulin?

Reaction of IgE antibody on basophils with antigen, this results in release of mediators, especially histamine

A client is in the oncology clinic for a first visit since being diagnosed with cancer. The nurse reads in the client's chart that the cancer classification is TISN0M0. What does the nurse conclude about this client's cancer? a. The primary site of the cancer cannot be determined. b. Regional lymph nodes could not be assessed. c. There are multiple lymph nodes involved already. d. There are no distant metastases noted in the report.

There are no distant metastases noted in the report.

Poison Ivy, Graft rejection, positive TB skin test (PPD) and sarcoidosis are all examples of what type of hypersensitivity? What is reacting?

Type IV Delayed Reaction of sensitized T cells with antigen and release of lymphokine she which activates macrophages and induces inflammation

A client with chronic kidney disease (CKD) has an elevated serum phosphorus level. What drug would the nurse anticipate to be prescribed for this client? a. Calcium acetate b. Doxycyline c. Magnesium sulfate d. Lisinopril

a. Calcium acetate

What factor best predicts a nurse's willingness to employ critical thinking? a. Caring b. Knowledge c. Presence d. Skills

a. Caring All attributes are important in nursing, however; the nurse's willingness to think critically is predicted by caring behaviors, self-reflection, and insight.

A nurse assesses a client with mitral valve stenosis. What clinical manifestation should alert the nurse to the possibility that the client's stenosis has progressed? a. Oxygen saturation of 92% b. Dyspnea on exertion c. Muted systolic murmur d. Upper extremity weakness

b. Dyspnea on exertion Dyspnea on exertion develops as the mitral valvular orifice narrows and pressure in the lungs increases.

The nurse is caring for a client with probable colorectal cancer (CRC). What assessment findings would the nurse expect? (Select all that apply.) a. Weight gain b. Rectal bleeding c. Anemia d. Change in stool shape e. Electrolyte imbalances f. Abdominal discomfort

b. Rectal bleeding c. Anemia d. Change in stool shape f. Abdominal discomfort

3. Which teaching point is most important for the client with a peritonsillar abscess? a. Gargle with warm salt water. b. Take all antibiotics as directed. c. Let us know if you want liquid medications. d. Wash hands frequently.

b. Take all antibiotics as directed. Any client on antibiotics must be instructed to complete the entire course of antibiotics. Not completing them can lead to complications or drug-resistant strains of bacteria. The other instructions are appropriate, just not the most important.

A nurse teaches a client who is at risk for colorectal cancer. Which dietary recommendation would the nurse teach the client? a. "Eat low-fiber and low-residual foods." b. "White rice and bread are easier to digest." c. "Add vegetables such as broccoli and cauliflower to your diet." d. "Foods high in animal fat help to protect the intestinal mucosa."

c. "Add vegetables such as broccoli and cauliflower to your diet."

A nurse has taught a female client about the modifiable risk factors for breast cancer. Which statement made by the client indicates that more teaching is needed? a. "I am fortunate that I breast-fed each of my three children for 12 months." b. "It looks as though I need to start working out at the gym more often." c. "I am glad that we can still have wine with every evening meal." d. "When I have menopausal symptoms, I must avoid hormone replacement therapy."

c. "I am glad that we can still have wine with every evening meal."

The nurse is caring for a client with urinary incontinence. The client states, "I am so embarrassed. My bladder leaks like a young child's bladder." How would the nurse respond? a. "I understand how you feel. I would be mortified." b. "Incontinence pads will minimize leaks in public." c. "I can teach you strategies to help control your incontinence." d. "More people experience incontinence than you might think."

c. "I can teach you strategies to help control your incontinence."

A nurse cares for a client who has pyelonephritis. The client states, "I am embarrassed to talk about my symptoms." How would the nurse respond? a. "I am a professional. Your symptoms will be kept in confidence." b. "I understand. Elimination is a private topic and shouldn't be discussed." c. "Take your time. It is okay to use words that are familiar to you." d. "You seem anxious. Would you like a nurse of the same gender to care for you?

c. "Take your time. It is okay to use words that are familiar to you."

A new nurse asks the precepting nurse "What is the best way to assess a client's pain?" Which response by the nurse is best? a. Numeric pain scale b. Behavioral assessment c. Client's self-report d. Objective observation

c. Client's self-report

A nurse cares for a client experiencing diabetic ketoacidosis who presents with Kussmaul respirations. What action would the nurse take? a. Administration of oxygen via facemask b. Intravenous administration of 10% glucose c. Implementation of seizure precautions d. Administration of intravenous insulin

d. Administration of intravenous insulin

A nurse cares for a client who has kidney stones from gout ricemia. Which medication does the nurse anticipate administering? a. Phenazopyridine b. Doxycyline c. Tolterodine d. Allopurinol

d. Allopurinol

After teaching a patient with diverticular disease, a nurse assesses the client's understanding. Which menu selection indicates the client correctly understood the teaching? a. Roasted chicken with rice pilaf and a cup of coffee with cream b. Spaghetti with meat sauce, a fresh fruit cup, and hot tea c. Garden salad with a cup of bean soup and a glass of low-fat milk d. Baked fish with steamed carrots and a glass of apple juice

d. Baked fish with steamed carrots and a glass of apple juice Clients who have diverticular disease are prescribed a low-residue diet. Whole grains (rice pilaf), uncooked fruits and vegetables (salad, fresh fruit cup), and high-fiber foods (cup [240 mL] of bean soup) would be avoided with a low-residue diet. Canned or cooked vegetables are appropriate. Apple juice does not contain fiber and is acceptable for a low-residue diet

A client is admitted with possible sepsis. Which action will the nurse perform first? a. Administer antibiotics. b. Give an antipyretic. c. Place the client in isolation. d. Obtain specified cultures.

d. Obtain specified cultures.

The nurse is caring for a client who is scheduled for a paracentesis. Which action is appropriate for the nurse to take? a. Have the client sign the informed consent form. b. Get the patient into a chair before the procedure. c. Help the client lie flat in bed on the right side. d. Assist the client to void before the procedure

d. Assist the client to void before the procedure For safety, the patient would void just before a paracentesis to prevent bladder damage to the procedure. The primary health care provider would have the client sign the consent form. The proper position for a paracentesis is sitting upright in bed or, alternatively, sitting on the side of the bed and leaning over the bedside table

A client with acute kidney injury (AKI) has a blood pressure of 76/55 mm Hg. The primary health care provider prescribed 1000 mL of normal saline to be infused over 1 hour to maintain perfusion. The client starts to develop shortness of breath. What is the nurse's priority action? a. Calculate the mean arterial pressure (MAP). b. Ask for insertion of a pulmonary artery catheter. c. Take the client's pulse. d. Decrease the rate of the IV infusion.

d. Decrease the rate of the IV infusion.

A nurse is caring for a client on mechanical ventilation and finds the client agitated and thrashing about. What action by the nurse is most appropriate? a. Assess the cause of the agitation. b. Reassure the client that he or she is safe. c. Restrain the client's hands. d. Sedate the client immediately

A. Assess the cause of the agitation.

The nurse recalls that the risk factors for acute gastritis include which of the following? (Select all that apply.) A: Alcohol B: Caffeine C: Corticosteroids D: Fruit juice E: Nonsteroidal anti-inflammatory drugs (NSAIDs)

A: Alcohol B: Caffeine C: Corticosteroids E: Nonsteroidal anti-inflammatory drugs (NSAIDs) Risk factors for acute gastritis include alcohol, caffeine, corticosteroids, and chronic NSAID use. Fruit juice is not a risk factor, although in some people it does cause distress.

The nurse is caring for a client who is starting on propylthiouracil for hyperthyroidism. What statement by the client indicates a need for further teaching? a. "I will let my provider know if I have weight gain and cold intolerance." b. "I will let my provider know if I have a metallic taste or stomach upset." c. "I will avoid crowds and other people who have infection." d. "I am aware that if the drug changes the color of my urine, I should stop it."

B. "I will let my provider know if I have a metallic taste or stomach upset."

Nurses at a conference learn the process by which pain is perceived by the client. Which processes are included in the discussion? (Select all that apply.) a. Induction b. Modulation c. Sensory perception d. Transduction e. Transmission f. Transition

B.Modulation C.Sensory perception D.Transduction F.Transmission

The nurse plans care for a client with Crohn disease who has a heavily draining fistula. Which intervention would be the nurse's priority action? a. Low-fiber diet b. Skin protection c. Antibiotic administration d. Intravenous glucocorticoids

b. Skin protection Protecting the client's skin is the priority action for a patient who has a heavily draining fistula. Intestinal fluid enzymes are caustic and can cause skin breakdown or fungal infections if the skin is not protected. The plan of care for a client who has Crohn disease also includes adequate nutrition focused on high-calorie, high-protein, high-vitamin, and low-fiber meals, antibiotic administration, and glucocorticoids.

A client has rheumatoid arthritis (RA) and the nurse is conducting a home assessment. What options can the nurse suggest for the client to maintain independence in activities of daily living (ADLs)? (Select all that apply.) a. Grab bars to reach high items b. Long-handled bath scrub brush c. Soft rocker-recliner chair d. Toothbrush with built-up handle e. Wheelchair cushion for comfort

ANS: A, B, D Grab bars, long-handled bath brushes, and toothbrushes with built-up handles all provide modifications for daily activities, making it easier for the client with RA to complete ADLs independently. The rocker-recliner and wheelchair cushion are comfort measures but do not help increase independence. Most clients who have RA are not wheelchair-bound.

5. A client has mucositis. What actions by the nurse will improve the clients nutrition? (Select all that apply.) a. Assist with rinsing the mouth with saline frequently. b. Encourage the client to eat room-temperature foods. c. Give the client hot liquids to hold in the mouth. d. Provide local anesthetic medications to swish and spit. e. Remind the client to brush teeth gently after each meal.

ANS: A, B, D, E Mucositis can interfere with nutrition. The nurse can help with rinsing the mouth frequently with water or saline; encouraging the client to eat cool, slightly warm, or room-temperature foods; providing swish-and-spit anesthetics; and reminding the client to keep the mouth clean by brushing gently after each meal. Hot liquids would be painful for the client.

7. After administering newly prescribed captopril (Capoten) to a client with heart failure, the nurse implements interventions to decrease complications. Which priority intervention should the nurse implement for this client? a. Provide food to decrease nausea and aid in absorption. b. Instruct the client to ask for assistance when rising from bed. c. Collaborate with unlicensed assistive personnel to bathe the client. d. Monitor potassium levels and check for symptoms of hypokalemia.

ANS: B Administration of the first dose of angiotensin-converting enzyme (ACE) inhibitors is often associated with hypotension, usually termed first-dose effect. The nurse should instruct the client to seek assistance before arising from bed to prevent injury from postural hypotension. ACE inhibitors do not need to be taken with food. Collaboration with unlicensed assistive personnel to provide hygiene is not a priority. The client should be encouraged to complete activities of daily living as independently as possible. The nurse should monitor for hyperkalemia, not hypokalemia, especially if the client has renal insufficiency secondary to heart failure. DIF: Applying/Application REF: 685 KEY: Heart failure| angiotensin-converting enzyme (ACE) inhibitor| medication| patient education MSC: Integrated Process: Nursing Process: Implementation NOT: Client Needs Category: Physiological Integrity: Pharmacological and Parenteral Therapies

18. The nurse has taught a client with cancer ways to prevent infection. What statement by the client indicates that more teaching is needed? a. I should take my temperature daily and when I dont feel well. b. I will wash my toothbrush in the dishwasher once a week. c. I wont let anyone share any of my personal items or dishes. d. Its alright for me to keep my pets and change the litter box.

ANS: D Clients should wash their hands after touching their pets and should not empty or scoop the cat litter box. The other statements are appropriate for self-management.

1. A nurse caring for a client removes the client's oxygen as prescribed. The client is now breathing what percentage of oxygen in the room air? a. 14% b. 21% c. 28% d. 31%

B. 21% Oxygen content of atmospheric or "room air" is about 21%.

A client has been diagnosed with a very large pulmonary embolism (PE) and has a dropping blood pressure. What medication would the nurse being most beneficial? a. Alteplase b. Enoxaparin c. Unfractionated heparin d. Warfarin sodium

a. Alteplase

After teaching a client with a high thoracic spinal cord injury, the nurse assesses the client's understanding. Which statement by the client indicates a correct understanding of how to prevent respiratory problems at home? a. "I'll use my incentive spirometer every 2 hours while I'm awake." b. "I'll drink thinned fluids to prevent choking." c. "I'll take cough medicine to prevent excessive coughing." d. "I'll position myself on my right side so I don't aspirate

A

The nurse is caring for a 60-year-old female client who sustained a thoracic spinal cord injury 10 years ago. For which potential complication will the nurse assess during this client's care? a. Fracture b. Malabsorption c. Delirium d. Anemia

A

The nurse is caring for a client who is diagnosed with urinary tract infection (UTI). What common urinary signs and symptoms does the nurse expect? (Select all that apply.) a. Dysuria b. Frequency c. Burning d. Fever e. Chills f. Hematuria

a. Dysuria b. Frequency c. Burning f. Hematuria

A nurse is teaching a client with heart failure who has been prescribed enalapril (Vasotec). Which statement should the nurse include in this client's teaching? a. "Avoid using salt substitutes." b. "Take your medication with food." c. "Avoid using aspirin-containing products." d. "Check your pulse daily."

a. "Avoid using salt substitutes." Angiotensin-converting enzyme (ACE) inhibitors such as enalapril inhibit the excretion of potassium.

4. A nurse is caring for a client who is experiencing moderate metabolic alkalosis. Which action should the nurse take? a. Monitor daily hemoglobin and hematocrit values. b. Administer furosemide (Lasix) intravenously. c. Encourage the client to take deep breaths. d. Teach the client fall prevention measures.

d. Teach the client fall prevention measures. The priority nursing care for a client who is experiencing moderate metabolic alkalosis is providing client safety. Clients with metabolic alkalosis have muscle weakness and are at risk for falling. The other nursing interventions are not appropriate for metabolic alkalosis.

An assistive personnel asks why brushing client s' teeth with a toothbrush in the intensive care unit is important to infection control. What response by the registered nurse is best? a. "It mechanically removes biofilm on teeth." b. "It's easier to clean all surfaces with a brush." c. "Oral care is important to all our clients." d. "Toothbrushes last longer than oral swabs."

ANS: A Biofilms are a complex group of bacteria that function within a slimy gel on surfaces such as teeth. Mechanical disruption (i.e., toothbrushing with friction) is the best way to control them. The other answers are not accurate.

4. A nurse asks a client to take deep breaths during an electroencephalography. The client asks, "Why are you asking me to do this?" How would the nurse respond? a. "Hyperventilation causes vascular dilation of cerebral arteries, which decreases electoral activity in the brain." b. "Deep breathing helps you to relax and allows the electroencephalograph to obtain a better waveform." c. "Hyperventilation causes cerebral vasoconstriction and increases the likelihood of seizure activity." d. "Deep breathing will help you to blow off carbon dioxide and decreases intracranial pressures."

ANS: C Hyperventilation produces cerebral vasoconstriction and alkalosis, which increase the likelihood of seizure activity. The client is asked to breathe deeply 20 times for 3 minutes. The other responses are not accurate.

A nurse is caring for a dying client. The client's spouse states," I think he's choking to death." How would the nurse respond?

"I will have another nurse assist me to turn your spouse onto the side."

Allergens can be contacted in many ways, what are those ways?

-inhaled -ingested -injected -skin or mucous membrane contracted

A nurse prepares to discharge a client with a cardiac dysrhythmia who is prescribed home health care services. Which priority information would be communicated to the home health nurse upon discharge? a. Medication orders for home b. Immunization history c. Religious beliefs d. Nutrition preferences

A The home health nurse needs to know current medications the client is taking to ensure assessment, evaluation, and further education related to these medications. The other information might be used to plan care, but not as the priority.

5. A nurse cares for a client who has hypertension that has not responded well to several medications. The client states compliance is not an issue. What action would the nurse take next? a. Assess the client for obstructive sleep apnea. b. Arrange a home sleep apnea test. c. Encourage the client to begin exercising. d. Schedule a polysomnography

A. Assess the client for obstructive sleep apnea. Hypertension not responding to medications can be a sign of obstructive sleep apnea (OSA). The nurse would assess the client using an evidence-based tool, such as the STOP-Bang Sleep Apnea Questionnaire, the Epworth Sleepiness Scale, the Pittsburgh Sleep Quality Index, and the Multiple Sleep Latency Test. If the results of the assessment indicate OSA may be a problem, the nurse would consult the primary health care provider for further testing. An at-home sleep-study is often done prior to a polysomnography. Excessive weight can contribute to OSA so exercising is always encouraged, but this is not specific to assessing for OSA.

A client is taking furosemide 40 mg/day for management of early chronic kidney disease (CKD). To assess the therapeutic effect of the medication, what action of the nurse is best? a. Obtain daily weights of the client. b. Auscultate heart and breath sounds. c. Palpate the client's abdomen. d. Assess the client's diet history.

a. Obtain daily weights of the client.

A nurse provides diabetic education at a public health fair. Which disorders would the nurse include as complications of diabetes mellitus? (Select all that apply.) a. Stroke b. Kidney failure c. Blindness d. Respiratory failure e. Cirrhosis

a. Stroke b. Kidney failure c. Blindness

A nurse teaches a client who is starting urinary bladder training. Which statement would the nurse include in this client's teaching? a. "Use the toilet when you first feel the urge, rather than at specific intervals." b. "Initially try to use the toilet at least every half hour for the first 24 hours." c. "Try to consciously hold your urine until the scheduled toileting time." d. "The toileting interval can be increased once you have been continent for a week."

c. "Try to consciously hold your urine until the scheduled toileting time."

9. A nurse is triaging clients in the emergency department. Which client would be considered "urgent"? a. A 20-year-old female with a chest stab wound and tachycardia b. A 45-year-old homeless man with a skin rash and sore throat c. A 75-year-old female with a cough and a temperature of 102° F (38.9° C) d. A 50-year-old male with new-onset confusion and slurred speech

c. A 75-year-old female with a cough and a temperature of 102° F (38.9° C) A client with a cough and a temperature of 102° F (38.9° C) is urgent. This client is at risk for deterioration and needs to be seen quickly, but is not in an immediately life-threatening situation. The client with a chest stab wound and tachycardia and the client with new-onset confusion and slurred speech would be triaged as emergent. The client with a skin rash and a sore throat is not at risk for deterioration and would be triaged as nonurgent.

The nurse is assessing a client for risk of developing metabolic syndrome. Which risk factor is associated with this health condition? a. Hypotension b. Hyperthyroidism c. Abdominal obesity d. Hypoglycemia

c. Abdominal obesity

The charge nurse is orienting a new nurse about care for an assigned client with an arteriovenous (AV) fistula for hemodialysis in her left arm. Which action by the float nurse would be considered unsafe? a. Palpating the access site for a bruit or thrill b. Using the right arm for a blood pressure reading c. Administering intravenous fluids through the AV fistula d. Checking distal pulses in the left arm

c. Administering intravenous fluids through the AV fistula

A client is admitted with a diagnosis of possible strangulated inguinal hernia. For which complication would the nurse monitor? a. Paralytic ileus b. Bowel volvulus c. Sepsis d. Colitis

c. Sepsis

look at test bank

test bank

look at test bank for picture

test bank

test bank

test bank

teat bank

test bank pics

test bank pics

test bank pics

A client asks the nurse if eating only preservative- and dye-free foods will decrease cancer risk. What response by the nurse is best? a. "Maybe; preservatives, dyes, and preparation methods may be risk factors." b. "No; research studies have never shown those things to cause cancer." c. "There are other things you can do that will more effectively lower your risk." d. "Yes; preservatives and dyes are well known to be carcinogens."

"Maybe; preservatives, dyes, and preparation methods may be risk factors."

A client who has multiple sclerosis reports increased severe muscle spasticity and tremors. What nursing action is most appropriate to manage this client's concern? a. Request a prescription for an antispasmodic drug such as baclofen. b. Prepare the client for deep brain stimulation surgery. c. Refer the client to a massage therapist to relax the muscles. d. Consult with the occupational therapist for self-care assistance.

A

A nurse assesses a client who is recovering from an open anterior cervical discectomy and fusion. Which complication would alert the nurse to urgently communicate with the primary health care provider? a. Auscultated stridor b. Weak pedal pulses c. Difficulty swallowing d. Inability to shrug shoulders

A

A nurse has educated a client on an epinephrine auto-injector. What statement by the client indicated additional instruction is needed? A. I don't need to go to the hospital after using it B. I must carry two auto-injectors at all times C. I will write the expiration date on my calendar D. This can be injected right through my clothes

A Clients would be instructed to call 911 and go to the hospital for monitoring after using the auto-injector. The medication may wear off before the offending agent has cleared the clients system. All other statements show a good understanding.

A nurse is providing community education on the seven warning signs of cancer. Which signs are included? (Select all that apply.) a. A sore that does not heal b. Changes in menstrual patterns c. Indigestion or trouble swallowing d. Near-daily abdominal pain e. Obvious change in a mole f. Frequent indigestion

A sore that does not heal Changes in menstrual patterns Indigestion or trouble swallowing Obvious change in a mole Frequent indigestion

The nurse receives a hand-off report. One client is described as a drug seeker who is obsessed with even tiny changes in physical condition and is "on the light constantly" asking for more pain medication. When assessing this client's pain, which statement or question by the nurse is most appropriate? a. "Help me understand how pain is affecting you right now." b. "I wish I could do more; is there anything I can get for you?" c. "You cannot have more pain medication for 3 hours." d. "Why do you think the medication is not helping your pain?"

A. "Help me understand how pain is affecting you right now."

While assessing a client with Graves disease, the nurse notes that the client's temperature has risen 1° F (1° C). What does the nurse do first? a. Turn the lights down and shut the patient's door. b. Call for an immediate electrocardiogram (ECG). c. Calculate the client's apical-radial pulse deficit. d. Administer a dose of acetaminophen.

A. Turn the lights down and shut the patient's door.

9. A client with HIV-III is hospitalized and has weeping Kaposi sarcoma lesions. The nurse dresses them with sterile gauze. When changing these dressings, which action is most important for the nurse's safety? a. Adhering to Standard Precautions b. Assessing tolerance to dressing changes c. Performing hand hygiene before and after care d. Disposing of soiled dressings properly

ANS: A All of the actions are important, but due to the infectious nature of this illness, the nurse would ensure he or she is following Standard Precautions (and Transmission-Based Precautions when necessary) to avoid a potential exposure.

8. The nurse working with oncology clients understands that which age-related change increases the older clients susceptibility to infection during chemotherapy? a. Decreased immune function b. Diminished nutritional stores c. Existing cognitive deficits d. Poor physical reserves

ANS: A As people age, there is an age-related decrease in immune function, causing the older adult to be more susceptible to infection than other clients. Not all older adults have diminished nutritional stores, cognitive dysfunction, or poor physical reserves.

A client is to receive a fecal microbiota transplantation tomorrow (FMT). What action by the nurse is best? a. Administer bowel cleansing as prescribed. b. Educate the client on immunosuppressive drugs. c. Inform the client he/she will drink a thick liquid. d. Place a nasogastric tube to intermittent suction.

ANS: A The usual route of delivering an FMT is via colonoscopy, so the client would have a bowel cleansing as prescribed for that procedure. The client will not need immunosuppressant drugs, to drink the material, or have an NG tube inserted.

The nurse assesses a client after a total hip arthroplasty. The client's surgical leg is visibly shorter than the other one and the client reports extreme pain. While a co-worker calls the surgeon, what action by the nurse is appropriate? a. Assess neurovascular status in both legs. b. Elevate the surgical leg and apply ice. c. Prepare to administer pain medication. d. Try to place the surgical leg in abduction.

ANS: A This client has signs and symptoms of hip dislocation, a potential complication of this surgery. Hip dislocation can cause neurovascular compromise. The nurse would assess neurovascular status while comparing both legs. The nurse would not try to move the extremity to elevate or abduct it. Pain medication may be administered if possible, but first the nurse would thoroughly assess the client.

11. A client has just been informed of a positive HIV test. The client is distraught and does not know what to do. What intervention by the nurse is best? a. Assess the client for support systems. b. Determine if a clergy member would help. c. Explain legal requirements to tell sex partners. d. Offer to tell the family for the client.

ANS: A This client needs the assistance of support systems. The nurse would help the client identify them and what role they can play in supporting him or her. A clergy member may or may not be welcome. Positive HIV test results are reportable in all 50 states, Washington, D.C., and Canada but the nurse works with the client to support his or her choices in disclosure. The nurse would not tell the family for the client.

A client asks the nurse about what medications may be included for nonopioid multimodal analgesia following a total knee arthroplasty. What medications may be given to the client? (Select all that apply.) a. Gabapentin b. Ketorolac c. Hydrocodone d. Ketamine e. Morphine f. Bupivacaine

ANS: A, B, D, F All of the choices are appropriate to use for nonopioid multimodal analgesia except for the two opioid drugs—hydrocodone and morphine. The nonopioid medications are used to decrease inflammation and pain.

3. A client on interferon therapy is reporting severe skin itching and irritation. What actions does the nurse delegate to the unlicensed assistive personnel (UAP)? (Select all that apply.) a. Apply moisturizers to dry skin. b. Apply steroid creams to the skin. c. Bathe the client using mild soap. d. Help the client with a hot water bath. e. Teach the client to avoid sunlight.

ANS: A, C The nurse can delegate applying unscented moisturizer and using mild soap for bathing. Steroid creams are not used for this condition. Hot water will worsen the irritation. Client teaching is a nursing function.

15. A client with HIV-III has been hospitalized with suspected cryptosporidiosis. What physical assessment would be most important with this condition? a. Auscultating the lungs b. Assessing mucous membranes c. Listening to bowel sounds d. Performing a neurologic examination

ANS: B Cryptosporidiosis can cause diarrhea and wasting with extreme loss of fluids and electrolytes. The nurse would assess signs of hydration/dehydration as the priority, including checking the client's mucous membranes for dryness. The nurse will perform the other assessments as part of a comprehensive assessment.

21. A client is admitted with superior vena cava syndrome. What action by the nurse is most appropriate? a. Administer a dose of allopurinol (Aloprim). b. Assess the clients serum potassium level. c. Gently inquire about advance directives. d. Prepare the client for emergency surgery.

ANS: C Superior vena cava syndrome is often a late-stage manifestation. After the client is stabilized and comfortable, the nurse should initiate a conversation about advance directives. Allopurinol is used for tumor lysis syndrome. Potassium levels are important in tumor lysis syndrome, in which cell destruction leads to large quantities of potassium being released into the bloodstream. Surgery is rarely done for superior vena cava syndrome.

27. The nurse assesses a client's Glasgow Coma Scale (GCS) score and determines it to be 12 (a 4 in each category). What care should the nurse anticipate for this client? a. Can ambulate independently b. May have trouble swallowing c. Needs frequent re-orientation d. Will need near-total care

ANS: C This client will most likely be confused and need frequent re-orientation. The client may not be able to ambulate at all but should do so independently, not because of mental status. Swallowing is not assessed with the GCS. The client will not need near-total care. DIF: Analyzing/Analysis REF: 934 KEY: Neurologic disorders| neurologic assessment MSC: IntegratedProcess:NursingProcess:Assessment NOT: Client Needs Category: Physiological Integrity: Reduction of Risk Potential

12. A nurse works with clients who have alopecia from chemotherapy. What action by the nurse takes priority? a. Helping clients adjust to their appearance b. Reassuring clients that this change is temporary c. Referring clients to a reputable wig shop d. Teaching measures to prevent scalp injury

ANS: D All of the actions are appropriate for clients with alopecia. However, the priority is client safety, so the nurse should first teach ways to prevent scalp injury.

The nurse assesses a client with rheumatoid arthritis (RA) and Sjögren syndrome. What assessment would be most important for this client? a. Abdominal assessment b. Oxygen saturation c. Breath sounds d. Visual acuity

ANS: D Sjögren syndrome may be seen in clients with RA and manifests with dryness of the eyes, mouth, and vagina in females. Visual disturbances can occur. The other assessments are not related to Sjögren syndrome.

3. A new nurse has been assigned a client who is in the hospital to receive iodine-131 treatment. Which action by the nurse is best? a. Ensure the client is placed in protective isolation. b. Hand off a pregnant client to another nurse. c. No special action is necessary to care for this client. d. Read the policy on handling radioactive excreta.

ANS: D This type of radioisotope is excreted in body fluids and excreta (urine and feces) and should not be handled directly. The nurse should read the facilitys policy for handling and disposing of this type of waste. The other actions are not warranted.

A reaction that involves all the blood vessels and bronchiolar smooth muscle, and causes wides spread blood vessel dilation, decreased cardiac output and bronchoconstriction

Anaphylaxis

The nurse is caring for clients on the medical surgical unit. What action by the nurse will help prevent a client from having a type II hypersensitivity reaction? A. Administering steroids for a positive TB test B. Correctly identifying the client prior to a blood transfusion C. Keeping the client free of the offending agent D. Providing a latex-free environment for the client

B A classic example of a type II hpersensitivity reaction is a blood transfusion reaction. These can be prevented by correctly identifying the client and cross checking the unit of blood to be administered. A positive type of IV response is a positive TB test. Avoidance therapy is the cornerstone of treatment for a type IV reaction

A client has been newly diagnosed with systemic Lupus erythematosus and is reviewing self-care measures with the nurse. Which statement by the client indicates a need to review the material? A. "I will avoid direct sunlight as much as possible" B. "Baby powder is good for the contestant sweating" C. " grouping errands will help prevent fatigue" D. "Rest time will have to become a priority"

B Constant sweating is NOT a sign of SLE and powders are drying so they should not be used at least not in excess

A nurse is assessing clients on a medical-surgical unit. Which client would the nurse identify as being at greatest risk for atrial fibrillation? a. A 45-year-old who takes an aspirin daily. b. A 50-year-old who is post coronary artery bypass graft surgery. c. A 78-year-old who had a carotid endarterectomy. d. An 80-year-old with chronic obstructive pulmonary disease.

B Atrial fibrillation occurs commonly in clients with cardiac disease. Other risk factors include hypertension (HTN), previous ischemic stroke, transient ischemic attack (TIA) or other thromboembolic event, diabetes mellitus, heart failure, obesity, hyperthyroidism, chronic kidney disease, excessive alcohol use, and mitral valve disease. The other conditions do not place these clients at higher risk for atrial fibrilation

A nurse caring for clients with systemic lupus erythematous plans care understanding the most common cause of death for these clients is which of the following? Select all that apply A. Infection B. Cardiovascular impairment C. Vasculitis D. Chronic kidney disease E. Liver failure F. Blood dyscrasias

B, D Any and all organs and tissue may be affected in systemic lupus erythematous but the MOST COMMON causes of death are Cardiovascular impairment And CKD (chronic kidney disease)

A nurse is preparing to admit a client on mechanical ventilation from the emergency department. What action by the nurse takes priority? a. Assessing that the ventilator settings are correct b. Ensuring that there is a bag-valve-mask in the room c. Obtaining personal protective equipment d. Planning to suction the client upon arrival to the room

B, Ensuring there is a bag-valve-mask in the room

The nurse has provided postvasectomy discharge instructions to the client. What statement by the client demonstrates good understanding? A. "We can have unprotected intercourse as soon as I have healed." B. "An ice pack to my scrotum will help with the swelling." C. "I need to report signs of infection, swelling, or bruising right away." D. "The stitches can be removed here in the office in 7 to 10 days."

B. "An ice pack to my scrotum will help with the swelling." After vasectomy, clients are instructed to use birth control until the 3-month semen analysis shows that the procedure has worked, to use an ice pack intermittently for 24 to 48 hours, that swelling and bruising are normal, and the bandage can be removed in 48 hours. There are no sutures to be removed.

A nurse is caring for a client who received intraspinal analgesia. Which action by the nurse is most important to ensure client safety? a. Assess and record vital signs every 4 hours. b. Instruct the client to report any unrelieved pain. c. Monitor for numbness and tingling in the legs. d. Perform frequent neurologic assessments.

B. Instruct the client to report any unrelieved pain.

A client is hospitalized with a second episode of pulmonary embolism (PE). Recent genetic testing reveals the client has an alteration in the gene CYP2C19. What action by the nurse is best? a. Instruct the client to eliminate all vitamin K from the diet. b. Prepare preoperative teaching for an inferior vena cava (IVC) filter. c. Refer the client to a chronic illness support group. d. Teach the client to use a soft-bristled toothbrush.

B. Prepare preoperative teaching for an inferior vena cava (IVC) filter

A client with a broken arm had ice placed on it for 20 minutes. A short time after the ice was removed, the client reports that the effect has worn off and requests pain medication, which cannot be given yet. Which actions by the nurse are most appropriate? (Select all that apply.) a. Ask for a physical therapy consult. b. Educate the client on cold therapy. c. Offer to provide a heating pad. d. Repeat the ice application. e. Teach the client relaxation techniques. f. Offer the client headphones with music.

B.Educate the client on cold therapy. D.Repeat the ice application. E.Teach the client relaxation techniques.

A client has a recurrence of gastric cancer and is crying. What response by the nurse is most appropriate? A: "Do you have family or friends for support?" B: "Would you tell me what you are feeling now." C: "Well, we knew this would probably happen." D: "Would you like me to refer you to hospice?"

B: "Would you tell me what you are feeling now." The nurse assesses the client's emotional state with open-ended questions and statements and shows a willingness to listen to the client's concerns. Asking about support people is very limited in nature, and "yes-or-no" questions are not therapeutic. Stating that this was expected dismisses the client's concerns. The client may or may not be ready to hear about hospice, and this is another limited, yes-or-no question.

The nurse is caring for a client with a long history of peptic ulcer disease. What assessment findings would the nurse anticipate if the client experiences upper gastrointestinal (GI) bleeding? (Select all that apply.) A: Decreased heart rate B: Decreased blood pressure C: Bounding radial pulse D: Dizziness E: Hematemesis F: Decreased urinary output

B: Decreased blood pressure D: Dizziness E: Hematemesis F: Decreased urinary output The client who has upper GI bleeding would likely have vomiting that contains blood (hematemesis), and would have signs and symptoms of dehydration such as a decreased blood pressure, dizziness, and/or decreased urinary output. The heart rate increases rather than decreases and the pulse is weak rather than bounding in clients who are dehydrated.

The nurse caring for oncology clients knows that which form of metastasis is the most common? a. Bloodborne b. Direct invasion c. Lymphatic spread d. Via bone marrow

Bloodborne

A client with multiple sclerosis is being discharged from rehabilitation. Which statement would the nurse include in the client's discharge teaching? a. "Be sure that you use a wheelchair when you go out in public." b. "Wear an undergarment brief at all times in case of incontinence." c. "Avoid overexertion, stress, and extreme temperature if possible." d. "Avoid having sexual intercourse to conserve energy."

C

The nurse is preparing to teach a client recently diagnosed with multiple sclerosis about taking glatiramer acetate. Which statement by the client indicates a need for further teaching? a. "I will rotate injection sites to prevent skin irritation." b. "I need to avoid large crowds and people with infection." c. "I should report any flulike symptoms to my primary health care provider." d. "I will report any signs of infection to my primary health care provider."

C

A nurse administers prescribed adenosine to a client. Which response would the nurse assess for as the expected therapeutic response? a. Decreased intraocular pressure b. Increased heart rate c. Short period of asystole d. Hypertensive crisis

C Clients usually respond to adenosine with a short period of asystole, bradycardia with long pauses, nausea, or vomiting. Adenosine has no impact on intraocular pressure nor does it cause increased heart rate or hypertensive crisis.

The nurse asks a client who has experienced ventricular dysrhythmias about substance abuse. The client asks, "Why do you want to know if I use cocaine?" What is the nurse's best response? a. "Substance abuse puts clients at risk for many health issues." b. "The hospital requires that I ask you about cocaine use." c. "Clients who use cocaine are at risk for fatal dysrhythmias." d. "We can provide services for cessation of substance abuse."

C Clients who use cocaine or illicit inhalants are particularly at risk for potentially fatal dysrhythmias. The other responses do not adequately address the client's question.

A client has recently been diagnosed with type II endometrial cancer and will be treated with brachytherapy. What statement by the client indicates a need for further education on this treatment? A. "Each treatment will take only 20 to 30 minutes." B. "I have to be alone in the room during treatment so I don't expose others." C. "I can get up and walk around or read in a chair during the treatments." D. "I need to report any heavy vaginal bleeding or severe diarrhea."

C. "I can get up and walk around or read in a chair during the treatments." Type II endometrial cancer is likely to invade the uterine wall and metastasize. Treatment with brachytherapy is intended to prevent recurrence. During the treatment, which lasts 20 to 30 minutes each, the woman must remain on bedrest to avoid dislodging the radioactive source. The source emits radiation while it is in place, so the woman is in the treatment room by herself. Once it is removed, she has no restrictions on being around others. She would need to report any heavy vaginal bleeding or severe diarrhea.

A nurse is caring for four clients receiving pain medication. After the hand-off report, which client would the nurse see first? a. Client who is crying and agitated b. Client with a heart rate of 104 beats/min c. Client with a Pasero Scale score of 4 d. Client with a verbal pain report of 9

C. Client with a Pasero Scale score of 4

A client with peptic ulcer disease is in the emergency department and reports gastric pain that has gotten much worse over the last 24 hours. The client's blood pressure when lying down is 112/68 mm Hg and when standing is 98/52 mm Hg. What action by the nurse is most appropriate? A: Administer a proton pump inhibitor (PPI). B: Call the Rapid Response Team. C: Start a large-bore IV with normal saline. D: Tell the patient to remain lying down.

C: Start a large-bore IV with normal saline. This client has orthostatic changes to the blood pressure, indicating fluid volume loss. The nurse would start a large-bore IV with isotonic solution. PPIs are not a treatment for an ulcer. The Rapid Response Team is not needed at this point. The client should be put on safety precautions, which includes staying in bed, but this is not the most appropriate action at this time.

What type of symptoms would you typically see with a Type IV reaction?

Edema Induration Ischemia Tissue damage at the site of exposure

A nurse is caring for a client who has lung cancer and is dying. Which prescription does the nurse question?

Morphine 10 mg of sublingual every six hours PRN for pain level greater than five

A nurse cares for a dying client. Which manifestation of dying does the nurse treat first?

Pain

Which statement about carcinogenesis is accurate? a. An initiated cell will always become clinical cancer. b. Cancer becomes a health problem once it is 1 cm in size. c. Normal hormones and proteins do not promote cancer growth. d. Tumor cells need to develop their own blood supply.

Tumor cells need to develop their own blood supply.

Hay fever, allergic asthma, anaphylaxis and angioedema are all examples of what type of hypersensitivity?

Type 1 rapid or immediate

A nurse prepares to discharge a client who has heart failure. Which questions should the nurse ask to ensure this client's safety prior to discharging home? (Select all that apply.) a. "Are your bedroom and bathroom on the first floor?" b. "What social support do you have at home?" c. "Will you be able to afford your oxygen therapy?" d. "What spiritual beliefs may impact your recovery?" e. "Are you able to accurately weigh yourself at home?"

a. "Are your bedroom and bathroom on the first floor?" b. "What social support do you have at home?" d. "What spiritual beliefs may impact your recovery?"

A client is in shock and the nurse prepares to administer insulin for a blood glucose reading of 208 mg/dL (11.6 mmol/L). The spouse asks why the client needs insulin as the client is not a diabetic. What response by the nurse is best? a. "High glucose is common in shock and needs to be treated." b. "Some of the medications we are giving are to raise blood sugar." c. "The IV solution has lots of glucose, which raises blood sugar." d. "The stress of this illness has made your spouse a diabetic."

a. "High glucose is common in shock and needs to be treated." High glucose readings are common in shock, and best outcomes are the result of treating them and maintaining glucose readings in the range of 140 to 180 mg/dL (7.7 to 10 mmol/L. Medications and IV solutions may raise blood glucose levels, but this is not the most accurate answer. The stress of the illness has not "made" the client diabetic.

A nurse assesses a client who has had two episodes of bacterial cystitis in the last 6 months. Which question(s) would the nurse ask? (Select all that apply.) a. "How much water do you drink every day?" b. "Do you take estrogen replacement therapy?" c. "Does anyone in your family have a history of cystitis?" d. "Are you on steroids or other immune-suppressing drugs?" e. "Do you drink grapefruit juice or orange juice daily?"

a. "How much water do you drink every day?" b. "Do you take estrogen replacement therapy?" d. "Are you on steroids or other immune-suppressing drugs?"

A nurse teaches a client who is diagnosed with diabetes mellitus. Which statement would the nurse include in this client's plan of care to delay the onset of microvascular and macrovascular complications? a. "Maintain tight glycemic control and prevent hyperglycemia." b. "Restrict your fluid intake to no more than 2 L a day." c. "Prevent hypoglycemia by eating a bedtime snack." d. "Limit your intake of protein to prevent ketoacidosis."

a. "Maintain tight glycemic control and prevent hyperglycemia."

A nurse assesses clients on a cardiac unit. Which client should the nurse identify as being at greatest risk for the development of left-sided heart failure? a. A 36-year-old woman with aortic stenosis b. A 42-year-old man with pulmonary hypertension c. A 59-year-old woman who smokes cigarettes daily d. A 70-year-old man who had a cerebral vascular accident

a. A 36-year-old woman with aortic stenosis

The nurse understands which information regarding patient-centered care? a. A competency recognizing the client as the source of control of his or her care b. A project addressing challenges in implementing patient-centered care c. Purposeful, informed, and outcome-focused care of clients or families d. The ability to use best evidence and practice when making care-related decisions

a. A competency recognizing the client as the source of control of his or her care Patient-centered care is a QSEN competency that recognizes the patient or caregiver as the source of control and full partner in providing compassionate and coordinated care based on respect for the patient's preferences, values, and needs. QSEN is a project addressing the challenge of preparing future nurses with the knowledge, skills, and attitudes (KSAs) necessary to continuously improve the quality and safety of the health care systems in which they work. Critical thinking is the application of purposeful, informed, and outcome-focused care. The ability to use best evidence and practice when making care-related decisions is evidence-based practice.

The nurse studying shock understands that the common signs and symptoms of this condition are directly related to which problems? (Select all that apply.) a. Anaerobic metabolism b. Hyperglycemia c. Hypotension d. Impaired renal perfusion e. Increased systemic perfusion

a. Anaerobic metabolism c. Hypotension

The nurse is formulating a teaching plan according to evidence-based breast cancer screening guidelines for a 50-year-old woman with low risk factors. Which diagnostic methods would be included in the plan? (Select all that apply.) a. Annual mammogram b. Magnetic resonance imaging (MRI) c. Breast ultrasound d. Breast self-awareness e. Clinical breast examination f. Self-breast examination

a. Annual mammogram d. Breast self-awareness e. Clinical breast examination

6. Which action by the nurse working with a client best demonstrates respect for autonomy? a. Asks if the client has questions before signing a consent b. Gives the client accurate information when questioned c. Keeps the promises made to the client and family d. Treats the client fairly compared to other clients

a. Asks if the client has questions before signing a consent

6. A nurse assesses a client who is admitted with an acid-base imbalance. The clients arterial blood gas values are pH 7.32, PaO2 85 mm Hg, PaCO2 34 mm Hg, and HCO3 16 mEq/L. What action should the nurse take next? a. Assess clients rate, rhythm, and depth of respiration. b. Measure the clients pulse and blood pressure. c. Document the findings and continue to monitor. d. Notify the physician as soon as possible.

a. Assess clients rate, rhythm, and depth of respiration. Progressive skeletal muscle weakness is associated with increasing severity of acidosis. Muscle weakness can lead to severe respiratory insufficiency. Acidosis does lead to dysrhythmias (due to hyperkalemia), but these would best be assessed with cardiac monitoring. Findings should be documented, but simply continuing to monitor is not sufficient. Before notifying the physician, the nurse must have more data to report.

A client is in the early stages of shock and is restless. What comfort measures does the nurse delegate to the assistive personnel (AP)? (Select all that apply.) a. Bringing the client warm blankets b. Giving the client hot tea to drink c. Massaging the client's painful legs d. Reorienting the client as needed e. Sitting with the client for reassurance

a. Bringing the client warm blankets b. Giving the client hot tea to drink d. Reorienting the client as needed e. Sitting with the client for reassurance

The nurse is caring for five clients on the medical-surgical unit. Which clients would the nurse consider to be at risk for postrenal acute kidney injury (AKI)? (Select all that apply.) a. Client with prostate cancer b. Client with blood clots in the urinary tract c. Client with ureterolithiasis d. Client with severe burns e. Client with lupus

a. Client with prostate cancer b. Client with blood clots in the urinary tract c. Client with ureterolithiasis .

A nurse manager institutes the Fulmer SPICES Framework as part of the routine assessment of older adults in the hospital. The nursing staff assesses for which factors? (Select all that apply.) a. Confusion b. Evidence of abuse c. Incontinence d. Problems with behavior e. Sleep disorders

a. Confusion c. Incontinence e. Sleep disorders SPICES stands for sleep disorders, problems with eating or feeding, incontinence, confusion, and evidence of falls.

The nurse assists the wound care/ostomy nurse assess a client prior to ostomy surgery. Which assessments would the nurse complete before marking the placement for the ostomy? (Select all that apply.) a. Contour of the abdomen when standing b. Location of the client's belt line c. Contour of the abdomen when lying d. Location of abdominal muscles e. Contour of the abdomen when sitting

a. Contour of the abdomen when standing b. Location of the client's belt line c. Contour of the abdomen when lying e. Contour of the abdomen when sitting

8. A client has been diagnosed with tuberculosis (TB). What action by the nurse takes highest priority? a. Educating the client on adherence to the treatment regimen b. Encouraging the client to eat a well-balanced diet c. Informing the client about follow-up sputum cultures d. Teaching the client ways to balance rest with activity

a. Educating the client on adherence to the treatment regimen The treatment regimen for TB often ranges from 26 weeks, but can be up to 2 years, making adherence problematic for many people. The nurse would stress the absolute importance of following the treatment plan for the entire duration of prescribed therapy. The other options are appropriate topics to educate this client on but do not take priority.

An emergency department nurse assesses a client admitted after a lightning strike. The client is awake but somewhat confused. Which assessment would the nurse complete first? a. Electrocardiogram (ECG) b. Wound inspection c. Creatinine kinase d. Computed tomography of head

a. Electrocardiogram (ECG) Clients who survive a lightning strike can have serious myocardial injury, which can be manifested by ECG and myocardial perfusion abnormalities. The nurse would prioritize the ECG over the other assessments which would be completed later.

A nurse working with older adults in the community plans programming to improve morale and emotional health in this population. What activity would best meet this goal? a. Exercise program to improve physical function b. Financial planning seminar series for older adults c. Social events such as dances and group dinners d. Workshop on prevention from becoming an abuse victim

a. Exercise program to improve physical function All activities would be beneficial for the older population in the community. However, failure in performing one's own activities of daily living and participating in society has direct effects on morale and life satisfaction. Those who lose the ability to function independently often feel worthless and empty. An exercise program designed to maintain and/or improve physical functioning would best address this need.

The nurse is caring for a client with early encephalopathy due to cirrhosis of the liver. Which factors may contribute to increased encephalopathy for which the nurse would assess? (Select all that apply.) a. Infection b. GI bleeding c. Irritable bowel syndrome d. Constipation e. Anemia f. Hypovolemia

a. Infection b. GI bleeding d. Constipation f. Hypovolemia Anemia and irritable bowel syndrome are unrelated to developing or worsening encephalopathy, which is caused by increased protein which breaks down into ammonia. Infection can cause hypovolemia which would increase serum protein concentration. Constipation and GI bleeding causes a large protein load in the intestines

A nurse is working with a client who has a painful rash consisting of grouped weeping and crusting lesions in distinct lines. What actions by the nurse are most appropriate? (Select all that apply.) a. Instruct the client to report lesions near the eyes. b. Have the client take long, hot baths to soak the lesions. c. Show the client how to make a baking soda compress. d. Advise the client to avoid exposure to UV light rays. e. Demonstrate proper use of antifungal medications. f. Review appropriate hygiene measures.

a. Instruct the client to report lesions near the eyes. c. Show the client how to make a baking soda compress. This client has herpes zoster (shingles). Eye infection is possible, so the client should be taught to report any lesions erupting near the eyes. Comfort measures can include compresses, calamine lotions, and baking soda. Long hot baths are not recommended. Avoiding UV lighting is important for herpes simplex. Herpes zoster is a viral disorder, so antifungal medications are not used. Hygiene is not an issue causing an outbreak.

7. A nurse cares for a client who is prescribed an intravenous prostacyclin agent for pulmonary artery hypertension. What actions would the nurse take to ensure the client's safety while on this medication? (Select all that apply.) a. Keep an intravenous line dedicated strictly to the infusion. b. Teach the client that this medication increases pulmonary pressures. c. Ensure that there is always a backup drug cassette available. d. Start a large-bore peripheral intravenous line. e. Use strict aseptic technique when using the drug delivery system.

a. Keep an intravenous line dedicated strictly to the infusion. c. Ensure that there is always a backup drug cassette available. e. Use strict aseptic technique when using the drug delivery system. Intravenous prostacyclin agents would be administered to a client with pulmonary artery hypertension through a central venous catheter with a dedicated intravenous line for this medication. Death has been reported when the drug delivery system is interrupted even briefly; therefore, a backup drug cassette would also be available. The nurse would use strict aseptic technique when using the drug delivery system. The nurse would teach the client that this medication decreases pulmonary pressures and increases lung blood flow.

The nurse assesses a client with ulcerative colitis. Which complications are paired correctly with their physiologic processes? (Select all that apply.) a. Lower gastrointestinal bleeding—erosion of the bowel wall b. Abscess formation—localized pockets of infection develop in the ulcerated bowel lining c. Toxic megacolon—transmural inflammation resulting in pyuria and fecaluria d. Nonmechanical bowel obstruction—paralysis of colon resulting from colorectal cancer e. Fistula—dilation and colonic ileus caused by paralysis of the colon

a. Lower gastrointestinal bleeding—erosion of the bowel wall b. Abscess formation—localized pockets of infection develop in the ulcerated bowel lining d. Nonmechanical bowel obstruction—paralysis of colon resulting from colorectal cancer Lower GI bleeding can lead to erosion of the bowel wall. Abscesses are localized pockets of infection that develop in the ulcerated bowel lining. Nonmechanical bowel obstruction is paralysis of the colon that results from colorectal cancer. When the inflammation is transmural, fistulas can occur between the bowel and bladder resulting in pyuria and fecaluria. Paralysis of the colon causing dilation and subsequent colonic ileus is known as a toxic megacolon.

The nurse assesses a patient who is recovering from an ileostomy placement. Which assessment finding would alert the nurse to immediately contact the primary health care provider? a. Pale and bluish stoma b. Liquid stool c. Ostomy pouch intact d. Blood-tinged output

a. Pale and bluish stoma The nurse would assess the stoma for color and contact the primary health care provider if the stoma is pale, bluish, or dark because these changes indicate possible lack of perfusion. The nurse would expect the client to have an intact ostomy pouch with dark green liquid stool that may contain some blood

A nurse assesses an older client who is scratching and rubbing white ridges on the skin between the fingers and on the wrists. Which action would the nurse take? a. Request a prescription for permethrin. b. Administer an antihistamine. c. Assess the client's airway. d. Apply gloves to minimize friction.

a. Request a prescription for permethrin. The client's presentation is most likely to be scabies, a contagious mite infestation. The drugs used to treat this infestation are ivermectin and permethrin. The nurse would contact the primary care provider to request a prescription for one of the medications. Secondary interventions may include medication to decrease the itching. The client's airway is not at risk with this skin disorder. Applying gloves will help prevent transmission.

A nurse reviews the electronic health record of a client who has Crohn disease and a draining fistula. Which documentation would alert the nurse to urgently contact the primary health care provider for additional prescriptions? a. Serum potassium of 2.6 mEq/L (2.6 mmol/L) b. Client ate 20% of breakfast meal c. White blood cell count of 8200/mm3 (8.2 109/L) d. Client's weight decreased by 3 lb (1.4 kg)

a. Serum potassium of 2.6 mEq/L (2.6 mmol/L) Fistulas place the patient with Crohn disease at risk for hypokalemia which can lead to serious dysrhythmias. This potassium level is low and would cause the nurse to intervene. The white blood cell count is normal. The other two findings are abnormal and also warrant intervention, but the potassium level takes priority

2. A hospital nurse is participating in a drill during which many "clients" with inhalation anthrax are being admitted. What drugs would the nurse anticipate administering? (Select all that apply.) a. Vancomycin b. Ciprofloxacin c. Doxycycline d. Ethambutol e. Sulfamethoxazole-trimethoprim (SMX-TMP)

a. Vancomycin b. Ciprofloxacin c. Doxycycline Vancomycin, ciprofloxacin, and doxycycline are all possible treatments for inhalation anthrax. Ethambutol is used for tuberculosis. SMX-TMP is commonly used for urinary tract infections and other common infections.

A nurse assesses a client who is prescribed varenicline for smoking cessation. Which signs or symptoms would the nurse identify as adverse effects of this medication? (Select all that apply.) a. Visual hallucinations b. Tachycardia c. Decreased cravings d. Manic behavior e. Increased thirst f. Orangish urine

a. Visual hallucinations d. Manic behavior

After treating several young women for urinary tract infections (UTIs), the college nurse plans an educational offering on reducing the risk of getting a UTI. What information does the nurse include? (Select all that apply.) a. Void before and after each act of intercourse. b. Consider changing to spermicide from birth control pills. c. Do not douche or use scented feminine products. d. Wear loose-fitting nylon panties. e. Wipe or clean the perineum from front to back.

a. Void before and after each act of intercourse. e. Wipe or clean the perineum from front to back.

A nurse teaches a patient about self-monitoring of blood glucose levels. Which statement would the nurse include in this client's teaching to prevent bloodborne infections? a. "Wash your hands after completing each test." b. "Do not share your monitoring equipment." c. "Blot excess blood from the strip with a cotton ball." d. "Use gloves when monitoring your blood glucose."

b. "Do not share your monitoring equipment."

A nurse is assessing a client who has a recent diagnosis of melanoma for understanding of treatment choices. What statement by the client indicates good understanding of the information? a. "Dermabrasion or chemical peels can be done in the office." b. "I may need lymph node resection during Mohs surgery." c. "This needs only a small excision with local anesthetic." d. "After surgery I will need 8 weeks of radiation therapy."

b. "I may need lymph node resection during Mohs surgery." Melanoma is usually treated with Mohs micrographic surgery, in which tissue is sectioned horizontally in layers and examined histologically, layer by layer, to assess for cancer cells. Dermabrasion and chemical peels can be used on actinic keratoses. Local anesthetic for small excisions is generally used on basal or squamous cell carcinomas. Radiation is usually not used with melanoma.

A nurse assesses a client who is recovering from extracorporeal shock-wave lithotripsy for renal calculi. The nurse notes an ecchymotic area on the client's right lower back. What action would the nurse take? a. Administer fresh-frozen plasma. b. Apply an ice pack to the site. c. Place the client in the prone position. d. Obtain serum coagulation test results.

b. Apply an ice pack to the site.

A nurse assesses a client who has mitral valve regurgitation. For which cardiac dysrhythmia should the nurse assess? a. Preventricular contractions b. Atrial fibrillation c. Symptomatic bradycardia d. Sinus tachycardia

b. Atrial fibrillation Atrial fibrillation is a clinical manifestation of mitral valve regurgitation and stenosis.

7. A nurse is caring for a client who has the following arterial blood values: pH 7.12, PaO2 56 mm Hg, PaCO2 65 mm Hg, and HCO3 22 mEq/L. Which clinical situation should the nurse correlate with these values? a. Diabetic ketoacidosis in a person with emphysema b. Bronchial obstruction related to aspiration of a hot dog c. Anxiety-induced hyperventilation in an adolescent d. Diarrhea for 36 hours in an older, frail woman

b. Bronchial obstruction related to aspiration of a hot dog Arterial blood gas values indicate that the client has acidosis with normal levels of bicarbonate, suggesting that the problem is not metabolic. Arterial concentrations of oxygen and carbon dioxide are abnormal, with low oxygen and high carbon dioxide levels. Thus, this client has respiratory acidosis from inadequate gas exchange. The fact that the bicarbonate level is normal indicates that this is an acute respiratory problem rather than a chronic problem, because no renal compensation has occurred.

The nurse is assessing a client with a diagnosis of prerenal acute kidney injury (AKI). Which condition would the nurse expect to find in the patient's recent history? a. Pyelonephritis b. Dehydration c. Bladder cancer d. Kidney stones

b. Dehydration

1. A new nurse is working with a preceptor on an inpatient medical-surgical unit. The preceptor advises the student that which is the priority when working as a professional nurse? a. Attending to holistic client needs b. Ensuring client safety c. Not making medication errors d. Providing client-focused care

b. Ensuring client safety

5. A nurse is planning care for a client who is anxious and irritable. The clients arterial blood gas values are pH 7.30, PaO2 96 mm Hg, PaCO2 43 mm Hg, and HCO3 19 mEq/L. Which questions should the nurse ask the client and spouse when developing the plan of care? (Select all that apply.) a. Are you taking any antacid medications? b. Is your spouses current behavior typical? c. Do you drink any alcoholic beverages? d. Have you been experiencing any vomiting? e. Are you experiencing any shortness of breath?

b. Is your spouses current behavior typical? c. Do you drink any alcoholic beverages? e. Are you experiencing any shortness of breath? This clients symptoms of anxiety and irritability are related to a state of metabolic acidosis. The nurse should ask the clients spouse or family members if the clients behavior is typical for him or her, and establish a baseline for comparison with later assessment findings. The nurse should also assess for alcohol intake because alcohol can change a clients personality and cause metabolic acidosis. The other options are not causes of metabolic acidosis.

The nurse is planning health teaching for a client starting on levothyroxine. What health teaching about this drug would the nurse include? a. The need to take the drug when the client feels fatigued and weak. b. The need to report chest pain and dyspnea when starting the drug. c. The need to check blood pressure and pulse every day. d. The need to rotate injection sites when giving self the drug.

b. The need to report chest pain and dyspnea when starting the drug.

A nurse is teaching a client and family about self-care at home for the client's wound infected with methicillin-resistant Staphylococcus aureus. What statement by the client indicates a need to review the information? a. "I will keep dry bandages on the wound and change them when drainage appears." b. "I will shower instead of taking a bath in the bathtub each day." c. "If the dressing is dry, I can sit or sleep anywhere in the house." d. "I will clean exposed household surfaces with a bleach and water mixture."

c. "If the dressing is dry, I can sit or sleep anywhere in the house." The client should not sit on upholstered furniture or sleep in the same bed as another person until the infection has cleared. The other statements show good understanding.

A nurse is assessing a client who is suspected of having ARDS. The nurse is confused that although the client appears dyspneic and the oxygen saturation is 88% on 6 L/min of oxygen, the client's lungs are clear. What explanation does the more senior nurse provide? a. "The client is too dehydrated for moist-sounding lungs." b. "The client hasn't started having any bronchospasm yet." c. "Lung edema is in the interstitial tissues, not the airways." d. "Clients with ARDS usually have clear lung sounds.

c. "Lung edema is in the interstitial tissues, not the airways."

The nurse is teaching assistive personnel (AP) about fluid restriction for a client who has acute kidney injury (AKI). The client's 24-hour urinary output is 120 mL. How much fluid would the client be allowed to have over the next 24 hours? a. 380 mL b. 500 mL c. 620 mL d. 750 mL

c. 620 mL

The nurse assesses a client who is admitted with a pelvic fracture. Which assessments would the nurse monitor to prevent or detect a complication of this injury? (Select all that apply.) a. Temperature b. Urinary output c. Blood pressure d. Pupil reaction e. Skin color

c. Blood pressure d. Pupil reaction e. Skin color

A nurse assesses a client who has a 15-year history of diabetes and notes decreased tactile sensation in both feet. What action would the nurse take first? a. Document the finding in the client's chart. b. Assess tactile sensation in the client's hands. c. Examine the client's feet for signs of injury. d. Notify the primary health care provider.

c. Examine the client's feet for signs of injury.

The nurse is caring for a client who has chronic pyelonephritis. What assessment finding would the nurse expect? a. Fever b. Flank pain c. Hypertension d. Nausea and vomiting

c. Hypertension

The nurse documents the vital signs of a client diagnosed with acute pancreatitis: Apical pulse = 116 beats/min Respirations = 28 breaths/min Blood pressure = 92/50 What complication of acute pancreatitis would the nurse suspect that the client might have? a. Electrolyte imbalance b. Pleural effusion c. Internal bleeding d. Pancreatic pseudocyst

c. Internal bleeding The client is exhibiting signs of hypovolemia most likely due to internal bleeding or hemorrhage. Due to decreased blood volume, the blood pressure is low and the heart rate increases to compensate for hypovolemia to ensure organ perfusion. Respirations often increase to increase oxygen in the blood

A hospitalized older adult has been assessed at high risk for skin breakdown. Which actions does the registered nurse (RN) delegate to the assistive personnel (AP)? (Select all that apply.) a. Assess skin redness when turning. b. Document Braden Scale results. c. Keep the client's skin dry. d. Obtain a pressure-relieving mattress. e. Turn the client every 2 hours.

c. Keep the client's skin dry. d. Obtain a pressure-relieving mattress. e. Turn the client every 2 hours. The nurses' aide or AP can assist in keeping the client's skin dry, order a special mattress on direction of the RN, and turn the client on a schedule. Assessing the skin is a nursing responsibility, although the aide would be directed to report any redness noticed. Documenting the Braden Scale results is the RN's responsibility as the RN is the one who performs that assessment.

The nurse is caring for a client who has a risk gene for developing cirrhosis. Which racial/ethnic group has this gene most often? a. Blacks b. Asian/Pacific Islanders c. Latinos d. French

c. Latinos The Patatin-like phospholipase domain containing 3 gene (PNPLA3) has been identified as a risk gene for cirrhosis, which occurs most often in Latinos when compared to other populations

A client is placed on fluid restriction because of chronic kidney disease (CKD). Which assessment finding would alert the nurse that the client's fluid balance is stable at this time? a. Decreased calcium levels b. Increased phosphorus levels c. No adventitious sounds in the lungs d. Increased edema in the legs

c. No adventitious sounds in the lungs

Which finding in a female client by the nurse would receive the highest priority for further diagnostics? a. Tender moveable masses throughout the breast tissue b. Nipple discharge without a palpable mass c. Nontender fixed mass in the upper outer quadrant of the breast d. Small, painful mass under warm reddened skin and nipple discharge

c. Nontender fixed mass in the upper outer quadrant of the breast

A nurse caring for an older client on a medical-surgical unit notices the client reports frequent constipation and only wants to eat softer foods such as rice, bread, and puddings. What assessment would the nurse perform first? a. Auscultate bowel sounds. b. Check skin turgor. c. Perform an oral assessment. d. Weigh the client.

c. Perform an oral assessment. Poorly fitting dentures and other dental problems are often manifested by a preference for soft foods and constipation from the lack of fiber. The nurse would perform an oral assessment to determine if these problems exist. The other assessments are important, but will not yield information specific to the client's food preferences as they relate to constipation.

A nurse admits an older adult to the hospital who lives at home with family. The nurse assesses that the client is malnourished. What actions by the nurse are best? (Select all that apply.) a. Contact Adult Protective Services or hospital social work. b. Request the primary health care provider prescribes tube feedings. c. Perform and document results of a Braden Scale assessment. d. Request a dietary consultation from the health care provider. e. Suggest a high-protein oral supplement between meals. f. Assess the client's own teeth or the dentures for proper fit.

c. Perform and document results of a Braden Scale assessment. d. Request a dietary consultation from the health care provider. e. Suggest a high-protein oral supplement between meals. f. Assess the client's own teeth or the dentures for proper fit. Malnutrition in the older population is multifactorial and has several potential adverse outcomes. Appropriate actions by the nurse include assessing the client's risk for skin breakdown with the Braden Scale, requesting a consultation with a dietitian, suggesting a high-protein meal supplement, and assessing the client's dentures or own teeth. There is no evidence that the client is being abused or needs a feeding tube at this time.

9. A nurse working on a cardiac unit delegated taking vital signs to an experienced unlicensed assistive personnel (UAP). Four hours later, the nurse notes the client's blood pressure is much higher than previous readings, and the client's mental status has changed. What action by the nurse would most likely have prevented this negative outcome? a. Determining if the UAP knew how to take blood pressure b. Double-checking the UAP by taking another blood pressure c. Providing more appropriate supervision of the UAP d. Taking the blood pressure instead of delegating the task

c. Providing more appropriate supervision of the UAP

A nurse assesses a client after administering isosorbide mononitrate (Imdur). The client reports a headache. Which action should the nurse take? a. Initiate oxygen therapy. b. Hold the next dose of Imdur. c. Instruct the client to drink water. d. Administer PRN acetaminophen.

d. Administer PRN acetaminophen. The vasodilating effects of isosorbide mononitrate frequently cause clients to have headaches during the initial period of therapy.

The nurse assesses a client with gastroenteritis. What risk factor would the nurse consider as the most likely cause of this disorder? a. Consuming too much fruit b. Consuming fried or pickled foods c. Consuming dairy products d. Consuming raw seafood

d. Consuming raw seafood Raw seafood is often contaminated and unless cooked can would most likely cause gastroenteritis. Any of the other food can also become contaminated if not stored properly or contaminated by workers/cooks who contaminate these foods

A nurse assesses a client with a pelvic fracture. Which assessment finding would the nurse identify as a complication of this injury? a. Hypertension b. Diarrhea c. Infection d. Hematuria

d. Hematuria

18. A nurse cares for a client who has a pleural chest tube. What action would the nurse take to ensure safe use of this equipment? a. Strip the tubing to minimize clot formation and ensure patency. b. Secure tubing junctions with clamps to prevent accidental disconnections. c. Connect the chest tube to wall suction as prescribed by the primary health care provider. d. Keep padded clamps at the bedside for use if the drainage system is interrupted.

d. Keep padded clamps at the bedside for use if the drainage system is interrupted. Padded clamps would be kept at the bedside for use if the drainage system becomes dislodged or is interrupted. The nurse would never strip the tubing. Tubing junctions would be taped, not clamped. Wall suction would be set at the level indicated by the device's manufacturer, not the primary health care provider.

A client is brought to the emergency department after sustaining injuries in a severe car crash. The client's chest wall does not appear to be moving normally with respirations, oxygen saturation is 82%, and the client is cyanotic. What action does the nurse take first? a. Administer oxygen and reassess. b. Auscultate the client's lung sounds. c. Facilitate a portable chest x-ray. d. Prepare to assist with intubation.

d. Prepare to assist with intubation.

A nurse has taught a client about dietary changes that can reduce the chances of developing cancer. What statement by the client indicates the nurse needs to provide additional teaching? a. "Foods high in vitamin A and vitamin C are important." b. "I'll have to cut down on the amount of bacon I eat." c. "I'm so glad I don't have to give up my juicy steaks." d. "Vegetables, fruit, and high-fiber grains are important."

"I'm so glad I don't have to give up my juicy steaks."

A nurse reviews the chart and new prescriptions for a client with diabetic ketoacidosis: Vital Signs and Assessment Blood pressure: 90/62 mm Hg Pulse: 120 beats/min Respiratory rate: 28 breaths/min Urine output: 20 mL/hr via catheter Laboratory Results Serum potassium: 2.6 mEq/L (2.6 mmol/L) Medications Potassium chloride 40 mEq/L (40 mmol/L) IV bolus STAT Increase IV fluid to 100 mL/hr What action would the nurse take? a. Administer the potassium and then consult with the primary health care provider about the fluid prescription. b. Increase the intravenous rate and then consult with the primary health care provider about the potassium prescription. c. Administer the potassium first before increasing the infusion flow rate for the client. d. Increase the intravenous flow rate before administering the potassium to the client.

A b. Increase the intravenous rate and then consult with the primary health care provider about the potassium prescription.

A nurse assesses a client with tachycardia. Which clinical manifestation requires immediate intervention by the nurse? a. Midsternal chest pain b. Increased urine output c. Mild orthostatic hypotension d. P wave touching the T wave

A Chest pain, possibly angina, indicates that tachycardia may be increasing the client's myocardial workload and oxygen demand to such an extent that normal oxygen delivery cannot keep pace. This results in myocardial hypoxia and pain. Increased urinary output and mild orthostatic hypotension are not life-threatening conditions and therefore do not require immediate intervention. The P wave touching the T wave indicates significant tachycardia and would be assessed to determine the underlying rhythm and cause; this is an important assessment but is not as critical as chest pain, which indicates cardiac cell death.

A nurse teaches a client who experiences occasional premature atrial contractions (PACs) accompanied by palpitations that resolve spontaneously without treatment. Which statement would the nurse include in this client's teaching? a. "Minimize or abstain from caffeine." b. "Lie on your side until the attack subsides." c. "Use your oxygen when you experience PACs." d. "Take amiodarone daily to prevent PACs."

A PACs usually have no hemodynamic consequences. For a client experiencing infrequent PACs, the nurse would explore possible lifestyle causes, such as excessive caffeine intake and stress. Lying on the side will not prevent or resolve PACs. Oxygen is not necessary. Although medications may be needed to control symptomatic dysrhythmias, for infrequent PACs, the client first would try lifestyle changes to control them.

A nurse supervises an assistive personnel (AP) applying electrocardiographic monitoring. Which statement would the nurse provide to the AP related to this procedure? a. "Clean the skin and clip hairs if needed." b. "Add gel to the electrodes prior to applying them." c. "Place the electrodes on the posterior chest." d. "Turn off oxygen prior to monitoring the client."

A To ensure the best signal transmission, the skin would be clean and hairs clipped. Electrodes would be placed on the anterior chest, and no additional gel is needed. Oxygen has no impact on electrocardiographic monitoring.

A nurse is teaching a client who has premature ectopic beats. Which education would the nurse include in this client's teaching? (Select all that apply.) a. Smoking cessation b. Stress reduction and management c. Avoiding vagal stimulation d. Adverse effects of medications e. Foods high in potassium f. Types of aerobic exercise

A, B, D A client who has premature beats or ectopic rhythms would be taught to stop smoking, manage stress, take medications as prescribed, and report adverse effects of medications. Clients with premature beats are not at risk for vasovagal attacks or potassium imbalances. While exercise is beneficial, aerobic exercise is not specifically linked to this client's educational needs.

A nurse teaches a client with a new permanent pacemaker. Which instructions would the nurse include in this client's teaching? (Select all that apply.) a. "Until your incision is healed, do not submerge your pacemaker. Only take showers." b. "Report any pulse rates lower than your pacemaker settings." c. "If you feel weak, apply pressure over your generator." d. "Have your pacemaker turned off before having magnetic resonance imaging (MRI)." e. "Do not lift your left arm above the level of your shoulder for 8 weeks."

A, B, E The client would not submerge in water until the site has healed; after the incision is healed, the client may take showers or baths without concern for the pacemaker. The client would be instructed to report changes in heart rate or rhythm, such as rates lower than the pacemaker setting or greater than 100 beats/min. The client would be advised of restrictions on physical activity for 8 weeks to allow the pacemaker to settle in place. The client would never apply pressure over the generator and would avoid tight clothing. The client would never have MRI because, whether turned on or off, the pacemaker contains metal. The client would be advised to inform all health care providers that he or she has a pacemaker.

The nurse is studying hypersensitivity reactions. Which reactions are correctly matched with their hypersensitivity types? Select all that apply A. Type I- examples include hay fever and anaphylaxis B. Type II- mediated by action of IgM C. Type III- immune complex deposits in blood vessel walls D. Type IV- examples are poison ivy and transplant rejection E. Type IV- involve both antibodies and complement

A, C, D Type I reactions are mediated by immunoglobulin E (IgE) and include hay fever, anaphylaxis, and allergic asthma. Type III reactions consist of immune complexes that form and deposit in the walls of blood vessels. Type IV reactions include responses to poison ivy exposure, positive tuberculosis tests, and graft rejection. Type II reactions are mediated by immunoglobulin G, not IgM. Type IV hypersensitivity reactions do not involve either antibodies or complement

A nurse assesses a client who recently experienced a traumatic spinal cord injury. Which assessment data would the nurse obtain to assess the client's coping strategies? (Select all that apply.) a. Spiritual beliefs b. Level of pain c. Family support d. Level of independence e. Annual income f. Previous coping strategies

A, C, D, F

A nurse cares for a client with congestive heart failure who has a regular cardiac rhythm of 128 beats/min. For which physiologic alterations would the nurse assess? (Select all that apply.) a. Decrease in cardiac output b. Increase in cardiac output c. Decrease in blood pressure d. Increase in blood pressure e. Decrease in urine output f. Increase in urine output

A, C, E Elevated heart rates in a healthy client initially cause blood pressure and cardiac output to increase. However, in a client who has congestive heart failure or a client with long-term tachycardia, ventricular filling time, cardiac output, and blood pressure eventually decrease. As cardiac output and blood pressure decrease, urine output will fall.

A client is being administered the first dose of belimumab for a systemic lupus erythematous flare. What actions by the nurse are most appropriate? Select all that apply A. Observe the client for at least 2 hours afterward B. Instruct the client about the monthly infusion schedule C. Inform the client not to drive or sign legal papers for 24 hours D. Ensure emergency equipment is working and nearby E. Make a follow up appointment for a lipid panel in 2 months F. Instruct the client to hold other medications for 72 hours

A, D This drug is a monoclonal antibody to tumor necrosis factor. The first dose would be administered in a place where sever allergic reactions and/or anaphylaxis can be managed. This includes having emergency equipment nearby. The client would be observed for at least 2 hours after this first dose.

A nurse assesses cerebrospinal fluid leaking onto a client's surgical dressing. What actions would the nurse take? (Select all that apply.) a. Place the client in a flat position. b. Monitor vital signs for hypotension. c. Utilize a bedside commode. d. Assess for abdominal distension. e. Report the leak to the surgeon.

A, E

The nurse is teaching an uncircumcised 65-year-old client about self-management of a urinary catheter in preparation for discharge to his home. What statement indicates the client needs more information? A. "I have to wash the outside of the catheter once a day with soap and water. B. "I should take extra time to clean the catheter site by pushing the foreskin back." C. "The drainage bag needs to be changed at least once a week and as needed." D. "I should pour a solution of vinegar and water through the tubing and bag."

A. "I have to wash the outside of the catheter once a day with soap and water. The first few inches (centimeters) of the catheter must be washed daily starting at the penis and washing outward with soap and water. The other options are correct for self-management of a urinary catheter in the home setting.

The nurse is educating a client on the prevention of toxic shock syndrome (TSS). Which statement by the client indicates a lack of understanding? A. "I need to change my tampon every 8 hours during the day." B. "At night, I should use a feminine pad rather than a tampon." C. "If I don't use tampons, I should not get TSS." D. "It is best if I wash my hands before inserting the tampon."

A. "I need to change my tampon every 8 hours during the day." Tampons need to be changed every 3 to 6 hours to avoid infection by such organisms as Staphylococcus aureus. All of the other responses are correct: use of feminine pads at night, not using tampons at all, and washing hands before tampon insertion are all strategies to prevent TSS.

The nurse is giving discharge instructions to a client who had a total abdominal hysterectomy with a vaginal repair. Which statements by the client indicate a need for further teaching? (Select all that apply.) A. "I should not have any problems driving to see my mother, who lives 3 hours away." B. "Now that I have time off from work, I can return to my exercise routine next week." C. "My granddaughter weighs 23 lb (10.5 kg) so I need to refrain from picking her up." D. "I will have to limit the number of times that I climb our stairs at home to fewer than five times a day." E ."I need to refrain from sexual intercourse for 4 to 6 weeks." F ."When I do resume intercourse, I will use a water-based lubricant and go slowly."

A. "I should not have any problems driving to see my mother, who lives 3 hours away." B. "Now that I have time off from work, I can return to my exercise routine next week." Driving and sitting for extended periods of time should be avoided until the surgeon gives permission. For 2 to 6 weeks, exercise participation should also be avoided. All of the other responses demonstrate adequate knowledge for discharge. The client should not lift anything heavier than 10 lb (4.5 kg), should limit stair climbing, and should refrain from sexual intercourse. When intercourse is resumed, the client should use water-based lubricant and proceed slowly as the vaginal walls are tighter. This may temporarily cause some pain.

A nurse has taken an informed consent to a woman who is having a transvaginal repair of a prolapsed uterus. What client statement indicates a need for more information? A. "The mesh they use may become infected." B. "I may still need to do my Kegel exercises." C. "I will watch for any signs of infection." D. "I know how to use the incentive spirometer."

A. "The mesh they use may become infected." Mesh is not used in the transvaginal approach as it has been discontinued in this country. The other statements show good understanding.

The nurse is teaching a client with benign prostatic hyperplasia (BPH). What statement indicates the client needs further information? A. "There should be no problem with drinking wine with dinner each night." B. "I am so glad that I weaned myself off of coffee about a year ago." C ."I need to inform my allergist that I cannot take my normal antihistamine." D. "My routine of drinking a quart (liter) of water first thing in the morning needs to change."

A. "There should be no problem with drinking wine with dinner each night." Caffeine and alcohol have diuretic effects and so the nurse would teach about avoiding or limiting their intake. The statement about drinking wine indicates a need for further instruction. Antihistamines can cause urinary retention. Clients are taught to avoid drinking large quantities of fluid at one time.

A client comes to the clinic with concerns about an enlarged left testicle and heaviness in his lower abdomen. Which diagnostic test would the nurse expect to be ordered? A. Alpha-fetoprotein (AFP) B. Prostate-specific antigen (PSA) C. Serum acid phosphatase (PAP) D. C-reactive protein (CRP)

A. Alpha-fetoprotein (AFP) These are symptoms of possible testicular cancer. AFP is a tumor marker that is elevated in testicular cancer. PSA and PAP testing is used in testing for prostate cancer and its metastasis. CRP is diagnostic for inflammatory conditions.

11. A client is receiving oxygen at 4 L per nasal cannula. What comfort measure may the nurse delegate to assistive personnel (AP)? a. Apply water-soluble ointment to nares and lips. b. Periodically turn the oxygen down or off. c. Replaces the oxygen tubing with a different type. d. Turn the client every 2 hours or as needed.

A. Apply water-soluble ointment to nares and lips. Oxygen can be drying, so the UAP can apply water-soluble lubricant to the client's lips and nares. The AP would not adjust the oxygen flow rate or replace the tubing. Turning the client is not related to comfort measures for oxygen.

A nurse uses the Checklist of Nonverbal Pain Indicators to assess pain in a nonverbal client with advanced dementia but no other medical history except well-controlled hypertension and high cholesterol. The client scores a zero. Which action by the nurse is best? a. Assess physiologic indicators and vital signs. b. Do not give pain medication as no pain is indicated. c. Document the findings and continue to monitor. d. Try a small dose of analgesic medication for pain.

A. Assess physiologic indicators and vital signs.

5. An assistive personnel (AP) was feeding a client with a tracheostomy. Later that evening, the UAP reports that the client had a coughing spell during the meal. What action by the nurse is best? a. Assess the client's lung sounds. b. Assign a different AP to the client. c. Report the AP to the manager. d. Request thicker liquids for meals.

A. Assess the client's lung sounds. The best action is to check the client's oxygenation because he or she may have aspirated. Once the client has been assessed, the nurse would notify the primary health care provider of possible aspiration and would consult with the registered dietitian about appropriately thickened liquids. The UAP should have reported the incident immediately, but addressing that issue is not the immediate priority.

3. A client has a tracheostomy that is 3 days old. Upon assessment, the nurse notes that the client's face is puffy and the eyelids are swollen. What action by the nurse takes best? a. Assess the client's oxygen saturation. b. Notify the Rapid Response Team. c. Oxygenate the client with a bag-valve-mask. d. Palpate the skin of the upper chest.

A. Assess the client's oxygen saturation. This client may have subcutaneous emphysema, which is air that leaks into the tissues surrounding the tracheostomy. The nurse would first assess the client's oxygen saturation and other indicators of oxygenation. If the client is stable, the nurse can palpate the skin of the upper chest to feel for the air. If the client is unstable, the nurse calls the Rapid Response Team. Using a bag-valve-mask device may or may not be appropriate for the unstable client.

2. A nurse assesses a client who has a nasal fracture. The client reports constant nasal drainage, a headache, and difficulty with vision. What action would the nurse take next? a. Collect the nasal drainage on a piece of filter paper. b. Encourage the client to blow his or her nose. c. Perform a test focused on a neurologic examination. d. Palpate the nose, face, and neck.

A. Collect the nasal drainage on a piece of filter paper. The client with nasal drainage after facial trauma could have a skull fracture resulting in leakage of cerebrospinal fluid (CSF). CSF can be differentiated from regular drainage by the fact that it forms a halo when dripped on filter paper and tests positive for glucose. The other actions would be appropriate but are not as high a priority as assessing for CSF. A CSF leak would increase the patient's risk for infection.

9. While assessing a client who has facial trauma, the nurse auscultates stridor. The client is anxious and restless. What action would the nurse take first? a. Contact the primary health care provider and prepare for intubation. b. Administer prescribed albuterol nebulizer therapy. c. Place the client in high-Fowler position. d. Ask the client to perform deep-breathing exercises.

A. Contact the primary health care provider and prepare for intubation. Facial and neck tissue edema can occur in clients with facial trauma. Airway patency is the highest priority. Clients who experience stridor and hypoxia, manifested by anxiety and restlessness, would be immediately intubated to ensure airway patency. Albuterol decreases bronchi and bronchiole inflammation, not facial and neck edema. Although putting the client in high-Fowler position and asking the client to perform breathing exercises may temporarily improve the patient's comfort, these actions will not decrease the underlying problem or improve airway patency.

Which of the following are risk factors for cervical cancer? (Select all that apply). A. HPV infection B. BRCA2 or BRCA2 gene mutation C. Multiparity D. First full-term pregnancy <17 years E. Obesity F. Smoking

A. HPV infection C. Multiparity D. First full-term pregnancy <17 years E. Obesity F. Smoking HPV infection, multiparty, first full-term pregnancy before the age of 17, obesity and smoking are all risk factors for cervical cancer. Additional risk factors include: chlamydia infection, being immunocompromised, women whose mothers took DES between 1940 and 1971, long-term use of contraceptives, IUD use, and family history. BRCA 1 or BRCA 2 gene mutations are a risk factor for breast and ovarian cancers

The nurse is conducting a history on a male client to determine the severity of symptoms associated with prostate enlargement. Which finding is cause for prompt action by the nurse? A. Hematuria B. Urinary hesitancy C. Postvoid dribbling D. Weak urinary stream

A. Hematuria Hematuria, especially at the start or end of the urine stream, could indicate infection due to possible urine retention and would cause the nurse to act promptly. Common symptoms of benign prostatic hyperplasia are urinary hesitancy, postvoid dribbling, and a weak urinary stream due to the enlarged prostate causing bladder outlet obstruction.

Which of the following are risk factors for endometrial cancer? (Select all that apply). A. Late menopause B. Tamofixen C. BRCA1 or BRCA2 gene mutation D. PCOS E. HPV infection F. Obesity

A. Late menopause B. Tamofixen D. PCOS F. Obesity Late menopause, use to tamoxifen, PCOS and obesity are all risk factor for endometrial cancer. Other risk factors include: early menarche, use or oral contraceptives, IUDs, nulliparity, diabetes mellitus, endometrial hyperplasia, and radiation to the pelvic area. BRCA1 and BRCA2 gene mutations are risk factors for ovarian cancer and HPV is a risk fact for cervical cancer.

A client came to the clinic with erectile dysfunction. What are some possible causes of this condition that the nurse could discuss with the client during history taking? (Select all that apply.) A. Recent prostatectomy B. Long-term hypertension C. Diabetes mellitus D. Hour-long exercise sessions E. Consumption of beer each night F. Taking long hot baths

A. Recent prostatectomy B. Long-term hypertension C. Diabetes mellitus E. Consumption of beer each night Organic erectile dysfunction can be caused by surgical procedures, vascular diseases such as hypertension and its treatment, diabetes mellitus, and alcohol consumption. There is no evidence that exercise or hot baths are related to this problem.

The nurse is taking the history of a 24-year-old client diagnosed with cervical cancer. What possible risk factors would the nurse assess? (Select all that apply.) A. Smoking B. Multiple births C. Poor diet D. Nulliparity E. Younger than 18 at first intercourse F. Infections with HPV

A. Smoking B. Multiple births F. Infections with HPV Smoking, multiple births, and infection with HPV are all risk factors for cervical cancer. Nulliparity is a risk factor for endometrial cancer. Poor diet could lead to decreased immunity, which is a risk, but is not directly related. Giving birth before the age of 17 is a risk factor.

A 28-year-old client is diagnosed with uterine leiomyoma and is experiencing severe symptoms. Which actions by the nurse are the most appropriate at this time? (Select all that apply.) a. Teach nonpharmacologic comfort measures. b. Discuss the high risk of infertility with this diagnosis. c. Relieve anxiety by relaxation techniques and education. d. Discuss in detail the side effects of laparoscopic surgery. e. Review complete blood count for possible iron deficiency anemia.

A. Teach nonpharmacologic comfort measures. c. Relieve anxiety by relaxation techniques and education e. Review complete blood count for possible iron deficiency anemia. With uterine leiomyomas or fibroids, heavy bleeding is the predominant symptom, with anxiety occurring because of fears of cancer or infertility. Interventions would be directed to the heavy bleeding and anxiety relief, such as relaxation techniques and education about the pathophysiology and possible treatment of the fibroids. While many women do not experience pain with this condition, some do, so the nurse would teach nonpharmacologic comfort measures. The nurse could suggest resources to give more information about the diagnosis. Discussion of the possibility of infertility and side effects of laparoscopic surgery is premature and may increase the anxiety.

A nurse plans care for a client with hyperparathyroidism. Which intervention does the nurse include in this client's plan of care? a. Use a lift sheet to assist the client with position changes in bed. b. Ask the client to ambulate in the hallway twice a day. c. Provide the client with a soft-bristled toothbrush for oral care. d. Instruct the assistive personnel to strain the patient's urine for stones.

A. Use a lift sheet to assist the client with position changes in bed.

A nurse receives hand-off report on four postoperative clients who each had total hysterectomies. Which client would the nurse assess first upon initial rounding? A. Vaginal hysterectomy: two saturated perineal pads in 2 hours B. Abdominal: temperature of 99° F (37.2° C), blood pressure of 116/74 mm Hg C. Vaginal: opened incisional edges and moderate bleeding D. Abdominal: urinary catheter output of 150 mL in the last 3 hours

A. Vaginal hysterectomy: two saturated perineal pads in 2 hours Normal vaginal bleeding after a vaginal hysterectomy should be less than one saturated perineal pad in 4 hours. Two saturated pads in such a short time could indicate hemorrhage, which is a priority. The client with the slight temperature elevation needs to be assessed for possible infection, but not as the priority. A vaginal hysterectomy would not result in an incision the nurse could observe separating. The urinary output is normal.

A nurse learns the concepts of addiction, tolerance, and dependence. Which information is accurate? (Select all that apply.) a. Addiction is a chronic physiologic disease process. b. Physical dependence and addiction are the same thing. c. Pseudoaddiction can result in withdrawal symptoms. d. Tolerance is a normal response to regular opioid use. e. Tolerance is said to occur when opioid effects decrease. f. Physical dependence occurs after repeated doses of an opioid.

A.Addiction is a chronic physiologic disease process. B.Tolerance is a normal response to regular opioid use. TE.olerance is said to occur when opioid effects decrease. F.Physical dependence occurs after repeated doses of an opioid.

A nurse on the postoperative unit administers many opioid analgesics. Which actions by the nurse are best to prevent unwanted sedation as a complication of these medications? (Select all that apply.) a. Avoid using other medications that cause sedation. b. Delay giving medication if the client is sleeping. c. Give the lowest dose that produces good control. d. Identify clients at high risk for unwanted sedation. e. Use an oximeter to monitor clients receiving analgesia.

A.Avoid using other medications that cause sedation. C.Give the lowest dose that produces good control. D.Identify clients at high risk for unwanted sedation. E.Use an oximeter to monitor clients receiving analgesia.

A client reports a great deal of pain following a fairly minor operation. The surgeon leaves a prescription for the nurse to administer a placebo instead of pain medication. Which actions by the nurse are most appropriate? (Select all that apply.) a. Consult with the surgeon and voice objections. b. Delegate administration of the placebo to another nurse. c. Give the placebo and reassess the client's pain. d. Notify the nurse manager of the placebo prescription. e. Tell the client what medications were prescribed.

A.Consult with the surgeon and voice objections. D.Notify the nurse manager of the placebo prescription.

A nurse learns that there are physical consequences to unrelieved pain. Which factors are included in this problem? (Select all that apply.) a. Decreased immune response b. Development of chronic pain c. Increased gastrointestinal (GI) motility d. Possible immobility e. Slower healing f. Negative quality of life

A.Decreased immune response B.Development of chronic pain D.Possible immobility E.Slower healing F.Negative quality of life

A nurse is studying pain sources. Which statements accurately describe different types of pain? (Select all that apply.) a. Neuropathic pain sometimes accompanies amputation. b. Nociceptive pain originates from abnormal pain processing. c. Deep somatic pain is pain arising from bone and connective tissues. d. Somatic pain originates from skin and subcutaneous tissues. e. Visceral pain is often diffuse and poorly localized.

A.Neuropathic pain sometimes accompanies amputation. C.Deep somatic pain is pain arising from bone and connective tissues. D.Somatic pain originates from skin and subcutaneous tissues. E.Visceral pain is often diffuse and poorly localized.

Which of the following is (are) (a) risk factor(s) for gastric cancer? (Select all that apply.) A: Achlorhydria B: Chronic atrophic gastritis C: H. pylori infection D: Iron deficiency anemia E: Pernicious anemia

A: Achlorhydria B: Chronic atrophic gastritis C: H. pylori infection E: Pernicious anemia Achlorhydria, chronic atrophic gastritis, H. pylori infection, and pernicious anemia are all risk factors for developing gastric cancer. Iron deficiency anemia is not a risk factor.

A nurse is preparing to administer pantoprazole intravenously to prevent stress ulcers during surgery. What action(s) by the nurse is (are) most appropriate? (Select all that apply.) A: Administer the drug through a separate IV line. B: Infuse pantoprazole using an IV pump. C: Keep the drug in its original brown container. D: Take vital signs frequently during infusion. E: Use an in-line IV filter when infusing.

A: Administer the drug through a separate IV line. B: Infuse pantoprazole using an IV pump. E: Use an in-line IV filter when infusing. When infusing pantoprazole, use a separate IV line, a pump, and an in-line filter. A brown wrapper and frequent vital signs are not needed.

What action(s) by the nurse is (are) appropriate to promote nutrition in a client who had a partial gastrectomy? (Select all that apply.) A: Administer vitamin B12 injections. B: Ask the primary health care provider about folic acid replacement. C: Educate the client on enteral feedings. D: Obtain consent for total parenteral nutrition. E: Provide iron supplements for the client.

A: Administer vitamin B12 injections. B: Ask the primary health care provider about folic acid replacement. E: Provide iron supplements for the client. After a partial or total gastrectomy, clients are at high risk for anemia due to vitamin B12 deficiency, folic acid deficiency, or iron deficiency. The nurse would provide supplements for all these nutrients. The client does not need enteral feeding or total parenteral nutrition.

A client has dumping syndrome. What menu selections indicate the client understands the correct diet to manage this condition? (Select all that apply.) A: Apricots B: Coffee cake C: Milk shake D: Potato soup E: Steamed broccoli

A: Apricots D: Potato soup Canned apricots and potato soup are appropriate selections as they are part of a high-protein, high-fat, and low- to moderate-carbohydrate diet. Coffee cake and other sweets must be avoided. Milk products and sweet drinks such as shakes must be avoided. Gas-forming foods such as broccoli must also be avoided.

A client has dumping syndrome after a partial gastrectomy. Which action by the nurse would be appropriate? A: Arrange a dietary consult. B: Increase fluid intake. C: Limit the client's foods. D: Make the client NPO.

A: Arrange a dietary consult. The client with dumping syndrome after a gastrectomy has multiple dietary needs. A referral to the registered dietitian nutritionist will be extremely helpful. Food and fluid intake is complicated and needs planning. The client should not be NPO.

The nurse is caring for a client who has frequent gastric pain and dyspepsia. Which procedure would the nurse expect for the client to make an accurate diagnosis? A: Esophagogastroduodenoscopy (EGD) B: Abdominal arteriogram C: Nuclear medicine scan D: Magnetic resonance imaging (MRI)

A: Esophagogastroduodenoscopy (EGD) The gold standard for diagnosing disorders of the stomach is an EGD which allows direct visualization by the endoscopist into the esophagus, stomach, and duodenum.

The nurse is caring for a client experiencing upper gastrointestinal (GI) bleeding. What is the priority action for the client's care? A: Maintain airway, breathing, and circulation. B: Monitor vital signs, including orthostatic blood pressures. C: Draw blood for hemoglobin and hematocrit immediately. D: Insert a nasogastric (NG) tube and connect to intermittent suction.

A: Maintain airway, breathing, and circulation. The priority action for any client experiencing deterioration or an emergent situation is monitor and maintain airway, breathing, and circulation (ABCs). Taking orthostatic blood pressures would not be appropriate, but the nurse would monitor vital signs carefully and draw blood for hemoglobin and hematocrit. An NG tube would also need to be inserted and connected to gastric suction to rest the GI tract. However, none of these actions take priority over maintaining ABCs.

The nurse is taking a history on an older adult. Which factors would the nurse assess as potential risks for low back pain? (Select all that apply.) a. Scoliosis b. Spinal stenosis c. Hypocalcemia d. Osteoporosis e. Osteoarthritis

ALL

SHORT ANSWER 1. A client in the emergency department is having a stroke and the provider has prescribed the tissue plasminogen activator (t-PA) alteplase (Activase). The client weighs 146 pounds. How much medication will this client receive? (Record your answer using a whole number.) _____ mg

ANS: 60 mg The dose of t-PA is 0.9 mg/kg with a maximum dose of 90 mg. The client weighs 66.4 kg. 0.9 mg × 66.4 = 59.76 mg, which rounds to 60 mg. DIF: Applying/Application REF: 939 KEY: Neurologic disorders| thrombolytic agents| drug calculation MSC: Integrated Process: Nursing Process: Analysis NOT: Client Needs Category: Physiological Integrity: Pharmacological and Parenteral Therapies

12. A nurse is caring for four clients in the neurologic intensive care unit. After receiving the hand-off report, which client should the nurse see first? a. Client with a Glasgow Coma Scale score that was 10 and is now is 8 b. Client with a Glasgow Coma Scale score that was 9 and is now is 12 c. Client with a moderate brain injury who is amnesic for the event d. Client who is requesting pain medication for a headache

ANS: A A 2-point decrease in the Glasgow Coma Scale score is clinically significant and the nurse needs to see this client first. An improvement in the score is a good sign. Amnesia is an expected finding with brain injuries, so this client is lower priority. The client requesting pain medication should be seen after the one with the declining Glasgow Coma Scale score. DIF: Applying/Application REF: 952 KEY: Neurologic disorders| neurologic assessment| critical rescue MSC: Integrated Process: Nursing Process: Analysis NOT: Client Needs Category: Safe and Effective Care Environment: Management of Care

25. A nurse is caring for a client with a nonhealing arterial lower leg ulcer. What action by the nurse is best? a. Consult with the wound care nurse. b. Give pain medication prior to dressing changes. c. Maintain sterile technique for dressing changes. d. Prepare the client for eventual amputation.

ANS: A A nonhealing wound needs the expertise of the wound care nurse. Premedicating prior to painful procedures and maintaining sterile technique are helpful, but if the wound is not healing, more needs to be done. The client may need an amputation, but other options need to be tried first.

3. The nurse providing direct client care uses specific practices to reduce the chance of acquiring infection with human immune deficiency virus (HIV) from clients. Which practice is most effective? a. Consistent use of Standard Precautions b. Double-gloving before body fluid exposure c. Labeling charts and armbands "HIV+" d. Wearing a mask within 3 feet (1 m) of the client

ANS: A According to The Joint Commission, the most effective preventative measure to avoid HIV exposure is consistent use of Standard Precautions. Standard Precautions are required by the CDC. Double-gloving is not necessary. Labeling charts and armbands in this fashion is a violation of the Health Information Portability and Accountability Act (HIPAA). Wearing a mask within 3 feet (1 m) of the client is not necessary with every client contact.

6. A nurse is interested in providing community education and screening on hypertension. In order to reach a priority population, to what target audience would the nurse provide this service? a. African-American churches b. Asian-American groceries c. High school sports camps d. Women's health clinics

ANS: A African Americans in the United States have one of the highest rates of hypertension in the world. The nurse has the potential to reach this priority population by providing services at African-American churches. Although hypertension education and screening are important for all groups, African Americans are the priority population for this intervention.

12. A nurse is teaching a female client about alcohol intake and how it affects hypertension. The client asks if drinking two beers a night is an acceptable intake. What answer by the nurse is best? a. "No, women should only have one beer a day as a general rule." b. "No, you should not drink any alcohol with hypertension." c. "Yes, since you are larger, you can have more alcohol." d. "Yes, two beers per day is an acceptable amount of alcohol."

ANS: A Alcohol intake should be limited to two drinks a day for men and one drink a day for women. A "drink" is classified as one beer, 1.5 ounces of hard liquor, or 5 ounces of wine. Limited alcohol intake is acceptable with hypertension. The woman's size does not matter.

A nursing manager is concerned about the number of infections on the hospital unit. What action by the manager would best help prevent these infections? a. Auditing staff members' hand hygiene practices b. Ensuring clients are placed in appropriate isolation c. Establishing a policy to remove urinary catheters quickly d. Teaching staff members about infection control methods

ANS: A All methods will help prevent infection; however, health care workers' lack of hand hygiene is the biggest cause of health care-associated infections. The manager can start with a hand hygiene audit to see if this is a contributing cause.

The nurse is teaching a client who is prescribed acetaminophen for control of osteoarthritic joint pain. What statement by the client indicates a need for further teaching? a. "I won't take more than 5000 mg of this drug each day." b. "I'll follow up to get my lab tests done to check my liver." c. "I'll check drugs that I take for acetaminophen in them." d. "I can use topical patches and creams to help relieve pain."

ANS: A All of the choices are correct about acetaminophen except that the maximum daily dosage is 4000 mg. For older adults, 3000 mg are recommended due to slower drug metabolism by the liver.

A nurse is teaching a client newly diagnosed with osteoarthritis (OA) about drugs used to treat the disease. For which drug does the nurse plan health teaching? a. Acetaminophen b. Cyclobenzaprine hydrochloride c. Hyaluronate d. Ibuprofen

ANS: A All of these drugs may be appropriate to treat OA. However, the first-line drug is acetaminophen. Cyclobenzaprine is a muscle relaxant given to treat muscle spasms. Hyaluronate is a synthetic joint fluid implant. Ibuprofen is a nonsteroidal anti-inflammatory drug.

20. A nurse working with clients who experience alopecia knows that which is the best method of helping clients manage the psychosocial impact of this problem? a. Assisting the client to pre-plan for this event b. Reassuring the client that alopecia is temporary c. Teaching the client ways to protect the scalp d. Telling the client that there are worse side effects

ANS: A Alopecia does not occur for all clients who have cancer, but when it does, it can be devastating. The best action by the nurse is to teach the client about the possibility and to give the client multiple choices for preparing for this event. Not all clients will have the same reaction, but some possible actions the client can take are buying a wig ahead of time, buying attractive hats and scarves, and having a hairdresser modify a wig to look like the clients own hair. Teaching about scalp protection is important but does not address the psychosocial impact. Reassuring the client that hair loss is temporary and telling him or her that there are worse side effects are both patronizing and do not give the client tools to manage this condition.

1. A nurse assesses clients on a cardiac unit. Which client should the nurse identify as being at greatest risk for the development of left-sided heart failure? a. A 36-year-old woman with aortic stenosis b. A 42-year-old man with pulmonary hypertension c. A 59-year-old woman who smokes cigarettes daily d. A 70-year-old man who had a cerebral vascular accident

ANS: A Although most people with heart failure will have failure that progresses from left to right, it is possible to have left-sided failure alone for a short period. It is also possible to have heart failure that progresses from right to left. Causes of left ventricular failure include mitral or aortic valve disease, coronary artery disease, and hypertension. Pulmonary hypertension and chronic cigarette smoking are risk factors for right ventricular failure. A cerebral vascular accident does not increase the risk of heart failure. DIF: Applying/Application REF: 679 KEY: Heart failure| health screening MSC: IntegratedProcess:NursingProcess:Assessment NOT: Client Needs Category: Safe and Effective Care Environment: Management of Care

15. A nurse prepares a client for lumbar puncture (LP). Which assessment finding would alert the nurse to contact the primary health care provider? a. Shingles infection on the client's back b. Client is claustrophobic c. Absence of intravenous access d. Paroxysmal nocturnal dyspnea

ANS: A An LP would not be performed if the client has a skin infection at or near the puncture site because of the risk of cerebrospinal fluid infection. A nurse would want to notify the primary health care provider if shingles were identified on the client's back. If a client has shortness of breath when lying flat, the LP can be adapted to meet the client's needs. Claustrophobia, absence of IV access, and paroxysmal nocturnal dyspnea have no impact on whether an LP can be performed.

2. A client had an embolic stroke and is having an echocardiogram. When the client asks why the provider ordered "a test on my heart," how should the nurse respond? a. "Most of these types of blood clots come from the heart." b. "Some of the blood clots may have gone to your heart too." c. "We need to see if your heart is strong enough for therapy." d. "Your heart may have been damaged in the stroke too."

ANS: A An embolic stroke is caused when blood clots travel from one area of the body to the brain. The most common source of the clots is the heart. The other statements are inaccurate. DIF: Understanding/Comprehension REF: 931 KEY: Neurologic disorders| stroke| patient education MSC: IntegratedProcess:Teaching/Learning NOT: Client Needs Category: Physiological Integrity: Physiological Adaptation

2. The nurse is presenting information to a community group on safer sex practices. The nurse would teach that which sexual practice is the riskiest? a. Anal intercourse b. Masturbation c. Oral sex d. Vaginal intercourse

ANS: A Anal intercourse is the riskiest sexual practice because the fragile anal tissue can tear, creating a portal of entry for human immune deficiency virus in addition to providing mucus membrane contact with the virus.

6. A nurse is teaching a client with heart failure who has been prescribed enalapril (Vasotec). Which statement should the nurse include in this client's teaching? a. "Avoid using salt substitutes." b. "Take your medication with food." c. "Avoid using aspirin-containing products." d. "Check your pulse daily."

ANS: A Angiotensin-converting enzyme (ACE) inhibitors such as enalapril inhibit the excretion of potassium. Hyperkalemia can be a life-threatening side effect, and clients should be taught to limit potassium intake. Salt substitutes are composed of potassium chloride. ACE inhibitors do not need to be taken with food and have no impact on the client's pulse rate. Aspirin is often prescribed in conjunction with ACE inhibitors and is not contraindicated. DIF: Applying/Application REF: 685 KEY: Heart failure| angiotensin-converting enzyme (ACE) inhibitor| medication| patient education MSC: IntegratedProcess:Teaching/Learning NOT: Client Needs Category: Physiological Integrity: Pharmacological and Parenteral Therapies

9. A nurse is assessing a client with peripheral artery disease (PAD). The client states that walking five blocks is possible without pain. What question asked next by the nurse will give the best information? a. "Could you walk further than that a few months ago?" b. "Do you walk mostly uphill, downhill, or on flat surfaces?" c. "Have you ever considered swimming instead of walking?" d. "How much pain medication do you take each day?"

ANS: A As PAD progresses, it takes less oxygen demand to cause pain. Needing to cut down on activity to be pain free indicates that the client's disease is worsening. The other questions are useful, but not as important.

11. A nurse admits a client who is experiencing an exacerbation of heart failure. Which action should the nurse take first? a. Assess the client's respiratory status. b. Draw blood to assess the client's serum electrolytes. c. Administer intravenous furosemide (Lasix). d. Ask the client about current medications.

ANS: A Assessment of respiratory and oxygenation status is the priority nursing intervention for the prevention of complications. Monitoring electrolytes, administering diuretics, and asking about current medications are important but do not take priority over assessing respiratory status. DIF: Applying/Application REF: 687 KEY: Heart failure| respiratory distress/failure| assessment/diagnostic examination MSC: Integrated Process: Nursing Process: Implementation NOT: Client Needs Category: Safe and Effective Care Environment: Management of Care

28. After a stroke, a client has ataxia. What intervention is most appropriate to include on the client's plan of care? a. Ambulate only with a gait belt. b. Encourage double swallowing. c. Monitor lung sounds after eating. d. Perform post-void residuals.

ANS: A Ataxia is a gait disturbance. For the client's safety, he or she should have assistance and use a gait belt when ambulating. Ataxia is not related to swallowing, aspiration, or voiding. DIF: Applying/Application REF: 934 KEY: Neurologic disorders| patient safety MSC: Integrated Process: Nursing Process: Implementation NOT: Client Needs Category: Safe and Effective Care Environment: Safety and Infection Control

21. A client has a brain abscess and is receiving phenytoin (Dilantin). The spouse questions the use of the drug, saying the client does not have a seizure disorder. What response by the nurse is best? a. "Increased pressure from the abscess can cause seizures." b. "Preventing febrile seizures with an abscess is important." c. "Seizures always occur in clients with brain abscesses." d. "This drug is used to sedate the client with an abscess."

ANS: A Brain abscesses can lead to seizures as a complication. The nurse should explain this to the spouse. Phenytoin is not used to prevent febrile seizures. Seizures are possible but do not always occur in clients with brain abscesses. This drug is not used for sedation. DIF: Understanding/Comprehension REF: 955 KEY: Neurologic disorders| antiseizure medications MSC: IntegratedProcess:Teaching/Learning NOT: Client Needs Category: Physiological Integrity: Pharmacological and Parenteral Therapies

8. The nurse is performing an assessment of cranial nerve III. Which testing is appropriate? a. Pupil constriction b. Deep tendon reflexes c. Upper muscle strength d. Speech and language

ANS: A CN III is the oculomotor nerve which controls eye movement, pupil constriction, and eyelid movement.

5. A nurse assesses a client recovering from a cerebral angiography via the right femoral artery. Which assessment would the nurse complete? a. Palpate bilateral lower extremity pulses. b. Obtain orthostatic blood pressure readings. c. Perform a funduscopic examination. d. Assess the gag reflex prior to eating.

ANS: A Cerebral angiography is performed by threading a catheter through the femoral or brachial artery. The extremity is kept immobilized after the procedure. The nurse checks the extremity for adequate circulation by noting skin color and temperature, presence and quality of pulses distal to the injection site, and capillary refill. Clients usually are on bedrest; therefore, orthostatic blood pressure readings cannot be performed. The funduscopic (eye) examination would not be affected by cerebral angiography. The client is not given general anesthesia; therefore, the client's gag reflex would not be compromised.

1. A nurse in the oncology clinic is providing preoperative education to a client just diagnosed with cancer. The client has been scheduled for surgery in 3 days. What action by the nurse is best? a. Call the client at home the next day to review teaching. b. Give the client information about a cancer support group. c. Provide all the preoperative instructions in writing. d. Reassure the client that surgery will be over soon.

ANS: A Clients are often overwhelmed at a sudden diagnosis of cancer and may be more overwhelmed at the idea of a major operation so soon. This stress significantly impacts the clients ability to understand, retain, and recall information. The nurse should call the client at home the next day to review the teaching and to answer questions. The client may or may not be ready to investigate a support group, but this does not help with teaching. Giving information in writing is important (if the client can read it), but in itself will not be enough. Telling the client that surgery will be over soon is giving false reassurance and does nothing for teaching.

A hospitalized client is placed on Contact Precautions. The client needs to have a computed tomography (CT) scan. What action by the nurse is most appropriate? a. Ensure that the radiology department is aware of the Isolation Precautions. b. Plan to travel with the client to ensure appropriate precautions are used. c. No special precautions are needed when this client leaves the unit. d. Notify the primary health care provider that the client cannot leave the room.

ANS: A Clients in isolation will leave their rooms only when necessary, such as for a CT scan that cannot be done portably in the room. The nurse will ensure that the receiving department is aware of the Isolation Precautions needed to care for the client. The other options are not needed.

26. A client has peripheral arterial disease (PAD). What statement by the client indicates misunderstanding about self-management activities? a. "I can use a heating pad on my legs if it's set on low." b. "I should not cross my legs when sitting or lying down." c. "I will go out and buy some warm, heavy socks to wear." d. "It's going to be really hard but I will stop smoking."

ANS: A Clients with PAD should never use heating pads as skin sensitivity is diminished and burns can result. The other statements show good understanding of self-management.

14. Which statement would the nurse include when teaching the assistive personnel (AP) about how to care for a client with cranial nerve II impairment? a. "Tell the client where food items are on the breakfast tray." b. "Place the client in a high-Fowler position for all meals." c. "Make sure the client's food is visually appetizing." d. "Assist the client by placing the fork in the left hand."

ANS: A Cranial nerve II, the optic nerve, provides central and peripheral vision. A patient who has cranial nerve II impairment will have decreased visual acuity, so the AP would tell the client where different food items are on the meal tray. The other options are not appropriate for client with cranial nerve II impairment.

5. A nurse cares for a client with right-sided heart failure. The client asks, "Why do I need to weigh myself every day?" How should the nurse respond? a. "Weight is the best indication that you are gaining or losing fluid." b. "Daily weights will help us make sure that you're eating properly." c. "The hospital requires that all inpatients be weighed daily." d. "You need to lose weight to decrease the incidence of heart failure."

ANS: A Daily weights are needed to document fluid retention or fluid loss. One liter of fluid equals 2.2 pounds. The other responses do not address the importance of monitoring fluid retention or loss. DIF: Remembering/Knowledge REF: 683 KEY: Heart failure| patient education MSC: Integrated Process: Teaching/Learning NOT: Client Needs Category: Physiological Integrity: Physiological Adaptation

9. A client with a stroke has damage to Broca's area. What intervention to promote communication is best for this client? a. Assess whether or not the client can write. b. Communicate using "yes-or-no" questions. c. Reinforce speech therapy exercises. d. Remind the client not to use neologisms.

ANS: A Damage to Broca's area often leads to expressive aphasia, wherein the client can understand what is said but cannot express thoughts verbally. In some instances the client can write. The nurse should assess to see if that ability is intact. "Yes-or-no" questions are not good for this type of client because he or she will often answer automatically but incorrectly. Reinforcing speech therapy exercises is good for all clients with communication difficulties. Neologisms are made-up "words" often used by clients with sensory aphasia. DIF: Applying/Application REF: 943 KEY: Neurologic disorders| stroke| communication MSC: Integrated Process: Nursing Process: Implementation NOT: Client Needs Category: Psychosocial Integrity

21. A nurse cares for an older adult client with heart failure. The client states, "I don't know what to do. I don't want to be a burden to my daughter, but I can't do it alone. Maybe I should die." How should the nurse respond? a. "Would you like to talk more about this?" b. "You are lucky to have such a devoted daughter." c. "It is normal to feel as though you are a burden." d. "Would you like to meet with the chaplain?"

ANS: A Depression can occur in clients with heart failure, especially older adults. Having the client talk about his or her feelings will help the nurse focus on the actual problem. Open-ended statements allow the client to respond safely and honestly. The other options minimize the client's concerns and do not allow the nurse to obtain more information to provide client-centered care. DIF: Applying/Application REF: 683 KEY: Heart failure| support| psychosocial response MSC: IntegratedProcess:Caring NOT: Client Needs Category: Psychosocial Integrity

A client who has rheumatoid arthritis is prescribed etanercept. What health teaching by the nurse about this drug is appropriate? a. Giving subcutaneous injections b. Having a chest x-ray once a year c. Taking the medication with food d. Using heat on the injection site

ANS: A Etanercept is given as a subcutaneous injection twice a week. The nurse would teach the client how to self-administer the medication. The other options are not appropriate for etanercept.

23. A nurse works on an oncology unit and delegates personal hygiene to an unlicensed assistive personnel (UAP). What action by the UAP requires intervention from the nurse? a. Allowing a very tired client to skip oral hygiene and sleep b. Assisting clients with washing the perianal area every 12 hours c. Helping the client use a soft-bristled toothbrush for oral care d. Reminding the client to rinse the mouth with water or saline

ANS: A Even though clients may be tired, they still need to participate in hygiene to help prevent infection. The other options are all appropriate.

8. The nurse is caring for four hypertensive clients. Which drug-laboratory value combination would the nurse report immediately to the health care provider? a. Furosemide/potassium: 2.1 mEq/L b. Hydrochlorothiazide/potassium: 4.2 mEq/L c. Spironolactone/potassium: 5.1 mEq/L d. Torsemide/sodium: 142 mEq/L

ANS: A Furosemide is a loop diuretic and can cause hypokalemia. A potassium level of 2.1 mEq/L is quite low and would be reported immediately. Spironolactone is a potassium-sparing diuretic that can cause hyperkalemia. A potassium level of 5.1 mEq/L is on the high side, but it is not as critical as the low potassium with furosemide. The other two laboratory values are normal.

16. A client had a femoral-popliteal bypass graft with a synthetic graft. What action by the nurse is most important to prevent wound infection? a. Appropriate hand hygiene before giving care b. Assessing the client's temperature every 4 hours c. Clean technique when changing dressings d. Monitoring the client's daily white blood cell count

ANS: A Hand hygiene is the best way toGRprAevDeEntSiLnfAecBti.oCnsOiMn hospitalized clients. Dressing changes would be done with sterile technique. Assessing vital signs and white blood cell count will not prevent infection.

24. A nurse is providing community screening for risk factors associated with stroke. Which client would the nurse identify as being at highest risk for a stroke? a. A 27-year-old heavy cocaine user b. A 30-year-old who drinks a beer a day c. A 40-year-old who uses seasonal antihistamines d. A 65-year-old who is active and on no medications

ANS: A Heavy drug use, particularly cocaine, is a risk factor for stroke. Heavy alcohol use is also a risk factor, but one beer a day is not considered heavy drinking. Antihistamines may contain phenylpropanolamine, which also increases the risk for stroke, but this client uses them seasonally and there is no information that they are abused or used heavily. The 65-year-old has only age as a risk factor. DIF: Remembering/Knowledge REF: 933 KEY: Neurologic disorders| stroke| health screening MSC: IntegratedProcess:NursingProcess:Assessment NOT: Client Needs Category: Health Promotion and Maintenance

A client is hospitalized and on multiple antibiotics. The client develops frequent diarrhea. What action by the nurse is most important? a. Consult with the primary health care provider about obtaining stool cultures. b. Delegate frequent perianal care to assistive personnel. c. Place the client on NPO status until the diarrhea resolves. d. Request a prescription for an antidiarrheal medication.

ANS: A Hospitalized clients who have three or more stools a day for 2 or more days are suspected of having infection with Clostridium difficile. The nurse will inform the primary health care provider and request stool cultures. Frequent perianal care is important and can be delegated but is not the most important action. The client does not necessarily need to be NPO; if the client is NPO, the nurse ensures he or she is getting appropriate IV fluids to prevent dehydration. Antidiarrheal medication may or may not be appropriate as the diarrhea serves as the portal of exit for the infection.

14. A client had a percutaneous angioplasty for renovascular hypertension 3 months ago. What assessment finding by the nurse indicates that an important outcome for this client has been met? a. Client is able to decrease blood pressure medications. b. Insertion site has healed without redness or tenderness. c. Most recent lab data show BUN: 19 mg/dL and creatinine 1.1 mg/dL. d. Verbalizes understanding of postprocedure lifestyle changes.

ANS: A Hypertension can be caused by renovascular disease. Opening up a constricted renal artery can lead to decreased blood pressure, manifested by the need for less blood pressure medication. The other findings are normal and desired, but not specifically related to hypertension caused by renal disease.

12. A nurse performs an assessment of pain discrimination on an older adult. The client correctly identifies, with eyes closed, a sharp sensation on the right hand when touched with a pin. Which action would the nurse take next? a. Touch the pin on the same area of the left hand. b. Contact the primary health care provider with the assessment results. c. Ask the client about current and past medications. d. Continue the assessment on the client's feet and legs.

ANS: A If testing is begun on the right hand and the client correctly identifies the pain stimulus, the nurse would continue the assessment on the left hand. This is a normal finding and does not need to be reported to the provider, but instead documented in the client's medical record. Medications do not need to be assessed in response to this finding. The nurse would assess the left hand prior to assessing the feet.

The nurse caring for clients admitted for infectious diseases understands what information about emerging global diseases and bioterrorism? a. Many infections are or could be spread by international travel. b. Safer food preparation practices have decreased foodborne illnesses. c. The majority of Americans have adequate innate immunity to smallpox. d. Plague produces a mild illness and generally has a low mortality rate.

ANS: A Increased global travel has resulted in the spread of many emerging diseases and has the potential to spread diseases caused by bioterrorism. Foodborne illnesses are on the increase. Many people in the United States have never been vaccinated against smallpox, and those who have are not guaranteed life-long protection. Plague can be fatal.

13. A client is receiving interleukins along with chemotherapy. What assessment by the nurse takes priority? a. Blood pressure b. Lung assessment c. Oral mucous membranes d. Skin integrity

ANS: A Interleukins can cause capillary leak syndrome and fluid shifting, leading to intravascular volume depletion. Although all assessments are important in caring for clients with cancer, blood pressure and other assessments of fluid status take priority.

6. A client is receiving chemotherapy through a peripheral IV line. What action by the nurse is most important? a. Assessing the IV site every hour b. Educating the client on side effects c. Monitoring the client for nausea d. Providing warm packs for comfort

ANS: A Intravenous chemotherapy can cause local tissue destruction if it extravasates into the surrounding tissues. Peripheral IV lines are more prone to this than centrally placed lines. The most important intervention is prevention, so the nurse should check hourly to ensure the IV site is patent, or frequently depending on facility policy. Education and monitoring for side effects such as nausea are important for all clients receiving chemotherapy. Warm packs may be helpful for comfort, but if the client reports that an IV site is painful, the nurse needs to assess further.

A nurse discusses inpatient hospice with a client and the clients family. A family member expresses concern that her loved one will receive only custodial care. How should the nurse respond? a. The goal of palliative care is to provide the greatest degree of comfort possible and help the dying person enjoy whatever time is left. b. Palliative care will release you from the burden of having to care for someone in the home. It does not mean that curative treatment will stop. c. A palliative care facility is like a nursing home and costs less than a hospital because only pain medications are given. d. Your relative is unaware of her surroundings and will not notice the difference between her home and a palliative care facility.

ANS: A Palliative care provides an increased level of personal care designed to manage symptom distress. The focus is on pain control and helping the relative die with dignity.

13. A client is in the clinic for a follow-up visit after a moderate traumatic brain injury. The client's spouse is very frustrated, stating that the client's personality has changed and the situation is intolerable. What action by the nurse is best? a. Explain that personality changes are common following brain injuries. b. Ask the client why he or she is acting out and behaving differently. c. Refer the client and spouse to a head injury support group. d. Tell the spouse this is expected and he or she will have to learn to cope.

ANS: A Personality and behavior often change permanently after head injury. The nurse should explain this to the spouse. Asking the client about his or her behavior isn't useful because the client probably cannot help it. A referral might be a good idea, but the nurse needs to do something in addition to just referring the couple. Telling the spouse to learn to cope belittles the spouse's concerns and feelings. DIF: Applying/Application REF: 953 KEY: Neurologic disorders| therapeutic communication| psychosocial response| coping MSC: Integrated Process: Communication and Documentation NOT: Client Needs Category: Psychosocial Integrity

13. A client is receiving rituximab. What assessment by the nurse takes priority? a. Blood pressure b. Temperature c. Oral mucous membranes d. Pain

ANS: A Rituximab can cause infusion-related reactions, including hypotension, so monitoring blood pressure is the priority. Other complications of this drug include fever with chills/rigors, headache and abdominal pain, shortness of breath, bronchospasm, nausea and vomiting, and rash. Assessing the client's temperature and for pain are both pertinent assessments, but do not take priority over the blood pressure. Oral mucus membrane assessment is important for clients with cancer, but are not specific for this treatment.

25. A client has a shoulder injury and is scheduled for a magnetic resonance imaging (MRI). The nurse notes the presence of an aneurysm clip in the client's record. What action by the nurse is best? a. Ask the client how long ago the clip was placed. b. Have the client sign an informed consent form. c. Inform the provider about the aneurysm clip. d. Reschedule the client for computed tomography.

ANS: A Some older clips are metal, which would preclude the use of MRI. The nurse should determine how old the clip is and relay that information to the MRI staff. They can determine if the client is a suitable candidate for this examination. The client does not need to sign informed consent. The provider will most likely not know if the client can have an MRI with this clip. The nurse does not independently change the type of diagnostic testing the client receives. DIF: Applying/Application REF: 940 KEY: Neurologic disorders| patient safety| communication| nursing assessment MSC: IntegratedProcess:NursingProcess:Assessment NOT: Client Needs Category: Safe and Effective Care Environment: Safety and Infection Control

17. An HIV-negative client who has an HIV-positive partner asks the nurse about receiving tenofovir/emtricitabine. What information is most important to teach the client about this drug? a. Does not reduce the need for safe sex practices. b. Has been taken off the market due to increases in cancer. c. Reduces the number of HIV tests you will need. d. Is only used for postexposure prophylaxis.

ANS: A Tenofovir/emtricitabine is a newer drug used for preexposure prophylaxis and appears to reduce transmission of human immune deficiency virus (HIV) from known HIV-positive people to HIV-negative people. The drug does not reduce the need for practicing safe sex. Since the drug can lead to drug resistance if used, clients will still need HIV testing every 3 months. This drug has not been taken off the market and is not used for postexposure prophylaxis.

5. A nurse is talking with a client about a negative enzyme-linked immunosorbent assay (ELISA) test for human immune deficiency virus (HIV). The test is negative and the client states "Whew! I was really worried about that result." What action by the nurse is most important? a. Assess the client's sexual activity and patterns. b. Express happiness over the test result. c. Remind the client about safer sex practices. d. Tell the client to be retested in 3 months.

ANS: A The ELISA test can be falsely negative if testing occurs after the client has become infected but prior to making antibodies to HIV. This period of time is known as the window period and can last up to 21 days. The confirmatory Western Blot test takes an additional 7 days, so using that testing algorithm, the client's status may not truly be known for up to 28 days. The client may have had exposure that has not yet been confirmed. The nurse needs to assess the client's sexual behavior further to determine the proper response. The other actions are not the most important, but discussing safer sex practices is always appropriate. Testing would be recommended every 3 months for someone engaging in high risk behaviors.

17. A nurse assesses a client and notes the client's position as indicated in the illustration below: How would the nurse document this finding? a. Decorticate posturing b. Decerebrate posturing c. Atypical hyperreflexia d. Spinal cord degeneration

ANS: A The client is demonstrating decorticate posturing, which is seen with interruption in the corticospinal pathway. This finding is abnormal and is a sign that the client's condition has deteriorated. The primary health care provider, the charge nurse/team leader, and other health care team members would be notified immediately of this change in status. Decerebrate posturing consists of external rotation and extension of the extremities. Hyperreflexes present as increased reflex responses. Spinal cord degeneration presents frequently with pain and discomfort.

2. A nurse reads on a hospitalized clients chart that the client is receiving teletherapy. What action by the nurse is best? a. Coordinate continuation of the therapy. b. Place the client on radiation precautions. c. No action by the nurse is needed at this time. d. Restrict visitors to only adults over age 18.

ANS: A The client needs to continue with radiation therapy, and the nurse can coordinate this with the appropriate department. The client is not radioactive, so radiation precautions and limiting visitors are not necessary.

13. A client has been hospitalized with an opportunistic infection secondary to HIV-III. The client's partner is listed as the emergency contact, but the client's mother insists that she should be listed instead. What action by the nurse is best? a. Contact the social worker to assist the client with advance directives. b. Ignore the mother; the client does not want her to be involved. c. Let the client know, gently, that nurses cannot be involved in these disputes. d. Tell the client that, legally, the mother is the emergency contact.

ANS: A The client should make his or her wishes known and formalize them through advance directives. The nurse would help the client by contacting someone to help with this process. Ignoring the mother or telling the client that nurses cannot be involved does not help the situation. Legal statutes vary by state, but the nurse would be the client's advocate and help ensure his or her wishes are met.

A client is getting out of bed into the chair for the first time after an uncemented total hip arthroplasty. What action by the nurse is appropriate? a. Have adequate help to transfer the patient. b. Provide socks so the patient can slide easier. c. Tell the patient full weight bearing is allowed. d. Use a footstool to elevate the patient's leg.

ANS: A The client with an uncemented hip will be on toe-touch only after surgery. The nurse would ensure there is adequate help to transfer the patient while preventing falls. Slippery socks may cause a fall. Elevating the leg is not going to assist with the client's transfer.

15. A nurse cares for a client with infective endocarditis. Which infection control precautions should the nurse use? a. Standard Precautions b. Bleeding precautions c. Reverse isolation d. Contact isolation

ANS: A The client with infective endocarditis does not pose any specific threat of transmitting the causative organism. Standard Precautions should be used. Bleeding precautions or reverse or contact isolation is not necessary. DIF: Applying/Application REF: 697 KEY: Infection| Standard Precautions MSC: Integrated Process: Nursing Process: Implementation NOT: Client Needs Category: Safe and Effective Care Environment: Safety and Infection Control

11. A client hospitalized for chemotherapy has a hemoglobin of 6.1 mg/dL. What medication should the nurse prepare to administer? a. Epoetin alfa (Epogen) b. Filgrastim (Neupogen) c. Mesna (Mesnex) d. Oprelvekin (Neumega)

ANS: A The clients hemoglobin is low, so the nurse should prepare to administer epoetin alfa, a colony-stimulating factor that increases production of red blood cells. Filgrastim is for neutropenia. Mesna is used to decrease bladder toxicity from some chemotherapeutic agents. Oprelvekin is used to increase platelet count.

14. A client with HIV-II is hospitalized for an unrelated condition, and several medications are prescribed in addition to the regimen already being used. What action by the nurse is most important? a. Consult with the pharmacy about drug interactions. b. Ensure that the client understands the new medications. c. Give the new drugs without considering the old ones. d. Schedule all medications at standard times.

ANS: A The drug regimen for someone with HIV/AIDS is complex and consists of many medications that must be given at specific times of the day, and that have many interactions with other drugs and food. The nurse would consult with a pharmacist about possible interactions. Client teaching is important but does not take precedence over ensuring the medications do not interfere with each other, which could lead to drug resistance or a resurgence of symptoms.

23. A client has been diagnosed with a deep vein thrombosis and is to be discharged on warfarin. The client is adamant about refusing the drug because "it's dangerous." What action by the nurse is best? a. Assess the reason behind the client's fear. b. Remind the client about laboratory monitoring. c. Tell the client that drugs are safer today than before. d. Warn the client about consequences of noncompliance.

ANS: A The first step is to assess the reason behind the client's fear, which may be related to the experience of someone the client knows who took warfarin or misinformation. If the nurse cannot address the specific rationale, teaching will likely be unsuccessful. Laboratory monitoring once every few weeks may not make the client perceive the drug to be safe. General statements like "drugs are safer today" do not address the root cause of the problem. Warning the client about possible consequences of not taking the drug is not therapeutic and is likely to lead to an adversarial relationship.

29. A client in the emergency department is having a stroke and needs a carotid artery angioplasty with stenting. The client's mental status is deteriorating. What action by the nurse is most appropriate? a. Attempt to find the family to sign a consent. b. Inform the provider that the procedure cannot occur. c. Nothing; no consent is needed in an emergency. d. Sign the consent form for the client.

ANS: A The nurse should attempt to find the family to give consent. If no family is present or can be found, under the principle of emergency consent, a life-saving procedure can be performed without formal consent. The nurse should not just sign the consent form. DIF: Applying/Application REF: 938 KEY: Neurologic disorders| informed consent| ethics MSC: Integrated Process: Communication and Documentation NOT: Client Needs Category: Safe and Effective Care Environment: Management of Care

4. While assessing a client on a cardiac unit, a nurse identifies the presence of an S3 gallop. Which action should the nurse take next? a. Assess for symptoms of left-sided heart failure. b. Document this as a normal finding. c. Call the health care provider immediately. d. Transfer the client to the intensive care unit.

ANS: A The presence of an S3 gallop is an early diastolic filling sound indicative of increasing left ventricular pressure and left ventricular failure. The other actions are not warranted. DIF: Remembering/Knowledge REF: 683 KEY: Heart failure| assessment/diagnostic examination MSC: IntegratedProcess:NursingProcess:Assessment NOT: Client Needs Category: Physiological Integrity: Reduction of Risk Potential

15. A nurse is caring for a client who has chronic emphysema and is receiving oxygen therapy at 6 L/min via nasal cannula. The following clinical data are available: Arterial Blood Gases Vital Signs pH = 7.28 Pulse rate = 96 beats/min PaO2 = 85 mm Hg Blood pressure = 135/45 PaCO2 = 55 mm Hg Respiratory rate = 6 breaths/min HCO3 = 26 mEq/L O2 saturation = 88% Which action should the nurse take first? a. Notify the Rapid Response Team and provide ventilation support. b. Change the nasal cannula to a mask and reassess in 10 minutes. c. Place the client in Fowlers position if he or she is able to tolerate it. d. Decrease the flow rate of oxygen to 2 to 4 L/min, and reassess.

ANS: A The primary trigger for respiration in a client with chronic respiratory acidosis is a decreased arterial oxygen level (hypoxic drive). Oxygen therapy can inhibit respiratory efforts in this case, eventually causing respiratory arrest and death. The nurse could decrease the oxygen flow rate; eventually, this might improve the clients respiratory rate, but the priority action would be to call the Rapid Response Team whenever a client with chronic carbon dioxide retention has a respiratory rate less than 10 breaths/min. Changing the cannula to a mask does nothing to improve the clients hypoxic drive, nor would it address the clients most pressing need. Positioning will not help the client breathe at a normal rate or maintain client safety.

9. The nurse has educated a client on precautions to take with thrombocytopenia. What statement by the client indicates a need to review the information? a. "I will be careful if I need enemas for constipation." b. "I will use an electric shaver instead of a razor." c. "I should only eat soft food that is either cool or warm." d. "I won't be able to play sports with my grandkids."

ANS: A The thrombocytopenic client is at high risk for bleeding even from minor trauma. Due to the risk of injuring rectal and anal tissue, the client should not use enemas or rectal thermometers. This statement would indicate the client needs more information. The other statements are appropriate for the thrombocytopenic client.

11. A client has a traumatic brain injury. The nurse assesses the following: pulse change from 82 to 60 beats/min, pulse pressure increase from 26 to 40 mm Hg, and respiratory irregularities. What action by the nurse takes priority? a. Call the provider or Rapid Response Team. b. Increase the rate of the IV fluid administration. c. Notify respiratory therapy for a breathing treatment. d. Prepare to give IV pain medication.

ANS: A These manifestations indicate Cushing's syndrome, a potentially life-threatening increase in intracranial pressure (ICP), which is an emergency. Immediate medical attention is necessary, so the nurse notifies the provider or the Rapid Response Team. Increasing fluids would increase the ICP. The client does not need a breathing treatment or pain medication. DIF: Applying/Application REF: 952 KEY: Neurologic disorders| Rapid Response Team| critical rescue MSC: Integrated Process: Communication and Documentation NOT: Client Needs Category: Safe and Effective Care Environment: Management of Care

17. A client who had a severe traumatic brain injury is being discharged home, where the spouse will be a full-time caregiver. What statement by the spouse would lead the nurse to provide further education on home care? a. "I know I can take care of all these needs by myself." b. "I need to seek counseling because I am very angry." c. "Hopefully things will improve gradually over time." d. "With respite care and support, I think I can do this."

ANS: A This caregiver has unrealistic expectations about being able to do everything without help. Acknowledging anger and seeking counseling show a realistic outlook and plans for accomplishing goals. Hoping for improvement over time is also realistic, especially with the inclusion of the word "hopefully." Realizing the importance of respite care and support also is a realistic outlook. DIF: Evaluating/Synthesis REF: 957 KEY: Neurologic disorders| discharge teaching| psychosocial response| coping MSC: IntegratedProcess:Caring NOT: Client Needs Category: Psychosocial Integrity

24. A client with a history of heart failure and hypertension is in the clinic for a follow-up visit. The client is on lisinopril and warfarin. The client reports new-onset cough. What action by the nurse is most appropriate? a. Assess the client's lung sounds and oxygenation. b. Instruct the client on another antihypertensive. c. Obtain a set of vital signs and document them. d. Remind the client that cough is a side effect of lisinopril.

ANS: A This client could be having an exacerbation of heart failure or experiencing a side effect of lisinopril (and other angiotensin-converting enzyme inhibitors). The nurse would assess the client's lung sounds and other signs of oxygenation first. The client may or may not need to switch antihypertensive medications. Vital signs and documentation are important, but the nurse would assess the respiratory system first. If the cough turns out to be a side effect, reminding the client is appropriate, but then more action needs to be taken.

17. A client with a history of prostate cancer is in the clinic and reports new onset of severe low back pain. What action by the nurse is most important? a. Assess the clients gait and balance. b. Ask the client about the ease of urine flow. c. Document the report completely. d. Inquire about the clients job risks.

ANS: A This client has manifestations of spinal cord compression, which can be seen with prostate cancer. This may affect both gait and balance and urinary function. For client safety, assessing gait and balance is the priority. Documentation should be complete. The client may or may not have occupational risks for low back pain, but with his history of prostate cancer, this should not be where the nurse starts investigating.

20. A nurse assesses a client with the National Institutes of Health (NIH) Stroke Scale and determines the client's score to be 36. How should the nurse plan care for this client? a. The client will need near-total care. b. The client will need cuing only. c. The client will need safety precautions. d. The client will be discharged home.

ANS: A This client has severe neurologic deficits and will need near-total care. Safety precautions are important but do not give a full picture of the client's dependence. The client will need more than cuing to complete tasks. A home discharge may be possible, but this does not help the nurse plan care for a very dependent client. DIF: Analyzing/Analysis REF: 935 KEY: Neurologic disorders| neurologic assessment MSC: Integrated Process: Nursing Process: Analysis NOT: Client Needs Category: Physiological Integrity: Reduction of Risk Potential

25. A client in the emergency department reports difficulty breathing. The nurse assesses the clients appearance as depicted below: What action by the nurse is the priority? a. Assess blood pressure and pulse. b. Attach the client to a pulse oximeter. c. Have the client rate his or her pain. d. Start high-dose steroid therapy.

ANS: A This client has superior vena cava syndrome, in which venous return from the head, neck, and trunk is blocked. Decreased cardiac output can occur. The nurse should assess indicators of cardiac output, including blood pressure and pulse, as the priority. The other actions are also appropriate but are not the priority.

1. The nurse is caring for a client diagnosed with HIV-II. The client's CD4+ cell count is 399/mm3 (0.399 109/L). What action by the nurse is best? a. Counsel the client on safer sex practices/abstinence. b. Encourage the client to abstain from alcohol. c. Facilitate genetic testing for CD4+ CCR5/CXCR4 co-receptors. d. Help the client plan high-protein/iron meals.

ANS: A This client is in the Centers for Disease Control and Prevention HIV-II case definition group. He or she remains highly infectious and would be counseled on either safer sex practices or abstinence. Abstaining from alcohol is healthy but not required, although some medications may need to be taken while abstaining. Genetic testing is not commonly done, but an alteration on the CCR5/CXCR4 co-receptors is seen in long-term nonprogressors. High-protein/iron meals are important for people who are immunosuppressed, but helping to plan them does not take precedence over stopping the spread of the disease.

23. A client has a subarachnoid bolt. What action by the nurse is most important? a. Balancing and recalibrating the device b. Documenting intracranial pressure readings c. Handling the fiberoptic cable with care to avoid breakage d. Monitoring the client's phlebostatic axis

ANS: A This device needs frequent balancing and recalibration in order to read correctly. Documenting readings is important, but it is more important to ensure the device's accuracy. The fiberoptic transducer-tipped catheter has a cable that must be handled carefully to avoid breaking it, but ensuring the device's accuracy is most important. The phlebostatic axis is not related to neurologic monitoring. DIF: Applying/Application REF: 956 KEY: Neurologic disorders| neurologic assessment| equipment safety MSC: Integrated Process: Nursing Process: Implementation NOT: Client Needs Category: Safe and Effective Care Environment: Safety and Infection Control

A nurse is caring for a client who has methicillin-resistant Staphylococcus aureus (MRSA) infection cultured from the urine. What action by the nurse is most appropriate? a. Prepare to administer vancomycin. b. Strictly limit visitors to immediate family only. c. Wash hands only after taking off gloves after care. d. Wear a respirator when handling urine output

ANS: A Vancomycin is one of a few drugs approved to treat MRSA. The others include linezolid and ceftaroline fosamil. Delafloxacin is a new antibiotic approved to treat MRSA. Visitation does not need to be limited to immediate family only. Hand hygiene is performed before and after wearing gloves. A respirator is not needed, but if splashing is anticipated, a face shield can be used.

A nurse is caring for a client who is terminally ill. The clients spouse states, I am concerned because he does not want to eat. How should the nurse respond? a. Let him know that food is available if he wants it, but do not insist that he eat. b. A feeding tube can be placed in the nose to provide important nutrients. c. Force him to eat even if he does not feel hungry, or he will die sooner. d. He is getting all the nutrients he needs through his intravenous catheter.

ANS: A When family members understand that the client is not suffering from hunger and is not starving to death, they may allow the client to determine when, what, or if to eat. Often, as death approaches, metabolic needs decrease and clients do not feel the sensation of hunger. Forcing them to eat frustrates the client and the family.

10. A nurse is seeing many clients in the neurosurgical clinic. With which clients should the nurse plan to do more teaching? (Select all that apply.) a. Client with an aneurysm coil placed 2 months ago who is taking ibuprofen (Motrin) for sinus headaches b. Client with an aneurysm clip who states that his family is happy there is no chance of recurrence c. Client who had a coil procedure who says that there will be no problem following up for 1 year d. Client who underwent a flow diversion procedure 3 months ago who is taking docusate sodium (Colace) for constipation e. Client who underwent surgical aneurysm ligation 3 months ago who is planning to take a Caribbean cruise

ANS: A, B After a coil procedure, up to 20% of clients experience re-bleeding in the first year. The client with this coil should not be taking drugs that interfere with clotting. An aneurysm clip can move up to 5 years after placement, so this client and family need to be watchful for changing neurologic status. The other statements show good understanding. DIF: Evaluating/Synthesis REF: 940 KEY: Neurologic disorders MSC: Integrated Process: Nursing Process: Evaluation NOT: Client Needs Category: Physiological Integrity: Physiological Adaptation

8. A nurse assesses a client who is recovering from a heart transplant. Which assessment findings should alert the nurse to the possibility of heart transplant rejection? (Select all that apply.) a. Shortness of breath b. Abdominal bloating c. New-onset bradycardia d. Increased ejection fraction e. Hypertension

ANS: A, B, C Clinical manifestations of heart transplant rejection include shortness of breath, fatigue, fluid gain, abdominal bloating, new-onset bradycardia, hypotension, atrial fibrillation or flutter, decreased activity tolerance, and decreased ejection fraction. DIF: Remembering/Knowledge REF: 703 KEY: Transplant| heart failure MSC: IntegratedProcess:NursingProcess:Assessment NOT: Client Needs Category: Physiological Integrity: Reduction of Risk Potential

2. A nurse is preparing to administer IV chemotherapy. What supplies does this nurse need? (Select all that apply.) a. Chemo gloves b. Facemask c. Isolation gown d. N95 respirator e. Shoe covers

ANS: A, B, C The Occupational Safety and Health Administration (OSHA) and the Oncology Nurses Society have developed safety guidelines for those preparing or administering IV chemotherapy. These include double gloves (or chemo gloves), a facemask, and a gown. An N95 respirator and shoe covers are not required.

6. A clients family members are concerned that telling the client about a new finding of cancer will cause extreme emotional distress. They approach the nurse and ask if this can be kept from the client. What actions by the nurse are most appropriate? (Select all that apply.) a. Ask the family to describe their concerns more fully. b. Consult with a social worker, chaplain, or ethics committee. c. Explain the clients right to know and ask for their assistance. d. Have the unit manager take over the care of this client and family. e. Tell the family that this secret will not be kept from the client.

ANS: A, B, C The clients right of autonomy means that the client must be fully informed as to his or her diagnosis and treatment options. The nurse cannot ethically keep this information from the client. The nurse can ask the family to explain their concerns more fully so everyone understands the concerns. A social worker, chaplain, or ethics committee can become involved to assist the nurse, client, and family. The nurse should explain the clients right to know and ask the family how best to proceed. The nurse should not abdicate responsibility for this difficult situation by transferring care to another nurse. Simply telling the family that he or she will not keep this secret sets up an adversarial relationship. Explaining this fact along with the concept of autonomy would be acceptable, but this by itself is not.

7. A client has meningitis following brain surgery. What comfort measures may the nurse delegate to the unlicensed assistive personnel (UAP)? (Select all that apply.) a. Applying a cool washcloth to the head b. Assisting the client to a position of comfort c. Keeping voices soft and soothing d. Maintaining low lighting in the room e. Providing antipyretics for fever

ANS: A, B, C, D The client with meningitis often has high fever, pain, and some degree of confusion. Cool washcloths to the forehead are comforting and help with pain. Allowing the client to assume a position of comfort also helps manage pain. Keeping voices low and lights dimmed also helps convey caring in a nonthreatening manner. The nurse provides antipyretics for fever. DIF: Applying/Application REF: 962 KEY: neurologic disorders| delegation| comfort measures| communication| unlicensed assistive personnel (UAP) MSC: Integrated Process: Communication and Documentation NOT: Client Needs Category: Physiological Integrity: Basic Care and Comfort

9. A client has a small-bore feeding tube (Dobhoff tube) inserted for continuous enteral feedings while recovering from a traumatic brain injury. What actions should the nurse include in the client's care? (Select all that apply.) a. Assess tube placement per agency policy. b. Keep the head of the bed elevated at least 30 degrees. c. Listen to lung sounds at least every 4 hours. d. Run continuous feedings on a feeding pump. e. Use blue dye to determine proper placement.

ANS: A, B, C, D All of these options are important for client safety when continuous enteral feedings are in use. Blue dye is not used because it can cause lung injury if aspirated. DIF: Applying/Application REF: 955 KEY: Neurologic disorders| enteral feedings MSC: Integrated Process: Nursing Process: Implementation NOT: Client Needs Category: Safe and Effective Care Environment: Safety and Infection Control

A client who had a recent total knee arthroplasty will be using a continuous passive motion (CPM) machine after discharge at home. What health teaching about the CPM machine will the nurse include? (Select all that apply.) a. "Keep the machine padded well to prevent skin breakdown." b. "Ensure that your leg is placed properly on the machine." c. "Use the machine as prescribed but not at mealtime." d. "When the machine is not being used, do not store it on the floor." e. "Check that the cycle and range of motion is kept at the level prescribed."

ANS: A, B, C, D, E Although not used as often today, some clients are prescribed to use the CPM machine to increase range of motion in the surgical knee. All of these teaching points are important for any client who uses a CPM machine.

The nurse is teaching assistive personnel about postoperative care for an older adult who had a posterolateral total hip arthroplasty. What teaching will the nurse include? (Select all that apply.) a. "Move the client slowly to prevent dizziness and a possible fall." b. "Encourage the client to deep breathe and cough at least every 2 hours." c. "Help the client use the incentive spirometer at least every 2 hours." d. "Keep the abduction pillow in place at all times while the client is in bed." e. "Let me know if the client has an elevated temperature or pulse." f. "Keep in mind that the client may be a little confused after surgery." g. "Please let me know if you see any reddened or open skin areas during bathing."

ANS: A, B, C, D, E, F Older adults are at risk for complications of decreased mobility after surgery, including atelectasis, pneumonia, pressure injuries, and orthostatic hypotension. Therefore these precautions are to help keep the client safe and avoid complications that could be life threatening.

A new graduate nurse has started working on a medical-surgical unit. What actions would the nurse take to be prepared for a disaster? (Select all that apply.) a. Know the institution's Emergency Response Plan. b. Participate in the institution's disaster drill. c. Develop a personal preparedness plan. d. Understand that nurses play a role in every phase of a disaster. e. Be prepared to report immediately to the emergency department. f. Be willing to be flexible working during a crisis situation.

ANS: A, B, C, D, F a. Know the institution's Emergency Response Plan. b. Participate in the institution's disaster drill. c. Develop a personal preparedness plan. d. Understand that nurses play a role in every phase of a disaster. Nurses play a major role in disaster and need to be prepared for any type of disaster. Knowing the institution's emergency management plan and participating in disaster drills will help the nurse be prepared for a disaster. Concerns for their home and family can impact the willingness to report in an emergency and can be diminished by being prepared with a personal preparedness plan. Nurses play key roles before, during, and after a disaster in the development of emergency management plan in defining specific nursing roles. During a crisis, nurses may be assigned to different areas of the facility or to different job functions and must remain flexible while working to their best ability.

A nurse asks the supervisor why older adults are more prone to infection than other adults. What reasons does the supervisor give? (Select all that apply.) a. Age-related decrease in immune function b. Decreased cough and gag reflexes c. Diminished acidity of gastric secretions d. Increased lymphocytes and antibodies e. Thinning skin that is less protective f. Higher rates of chronic illness

ANS: A, B, C, E, F Older adults have several age-related changes making them more susceptible to infection, including decreased immune function, decreased cough and gag reflex, decreased acidity of gastric secretions, thinning skin, fewer lymphocytes and antibodies, and higher rates of chronic illness.

3. A nurse assesses clients on a cardiac unit. Which clients should the nurse identify as at greatest risk for the development of acute pericarditis? (Select all that apply.) a. A 36-year-old woman with systemic lupus erythematosus (SLE) b. A 42-year-old man recovering from coronary artery bypass graft surgery c. A 59-year-old woman recovering from a hysterectomy d. An 80-year-old man with a bacterial infection of the respiratory tract e. An 88-year-old woman with a stage III sacral ulcer

ANS: A, B, D Acute pericarditis is most commonly associated acute exacerbations of systemic connective tissue disease, including SLE; with Dressler's syndrome, or inflammation of the cardiac sac after cardiac surgery or a myocardial infarction; and with infective organisms, including bacterial, viral, and fungal infections. Abdominal and reproductive surgeries and pressure ulcers do not increase clients' risk for acute pericarditis. DIF: Applying/Application REF: 699 KEY: Inflammatory response| health screening MSC: IntegratedProcess:NursingProcess:Assessment NOT: Client Needs Category: Safe and Effective Care Environment: Management of Care

4. A client has thrombocytopenia. What actions does the nurse delegate to the unlicensed assistive personnel (UAP)? (Select all that apply.) a. Apply the clients shoes before getting the client out of bed. b. Assist the client with ambulation. c. Shave the client with a safety razor only. d. Use a lift sheet to move the client up in bed. e. Use the Waterpik on a low setting for oral care.

ANS: A, B, D Clients with thrombocytopenia are at risk of significant bleeding even with minor injuries. The nurse instructs the UAP to put the clients shoes on before getting the client out of bed, assist with ambulation, shave the client with an electric razor, use a lift sheet when needed to reposition the client, and use a soft-bristled toothbrush for oral care.

6. A nurse prepares to discharge a client who has heart failure. Based on the Heart Failure Core Measure Set, which actions should the nurse complete prior to discharging this client? (Select all that apply.) a. Teach the client about dietary restrictions. b. Ensure the client is prescribed an angiotensin-converting enzyme (ACE) inhibitor. c. Encourage the client to take a baby aspirin each day. d. Confirm that an echocardiogram has been completed. e. Consult a social worker for additional resources.

ANS: A, B, D The Heart Failure Core Measure Set includes discharge instructions on diet, activity, medications, weight monitoring and plan for worsening symptoms, evaluation of left ventricular systolic function (usually with an echocardiogram), and prescribing an ACE inhibitor or angiotensin receptor blocker. Aspirin is not part of the Heart Failure Core Measure Set and is usually prescribed for clients who experience a myocardial infarction. Although the nurse may consult the social worker or case manager for additional resources, this is not part of the Core Measures. DIF: Understanding/Comprehension REF: 689 KEY: Heart failure| discharge| Core Measures| The Joint Commission MSC: Integrated Process: Nursing Process: Analysis NOT: Client Needs Category: Safe and Effective Care Environment: Management of Care

A hospice nurse is caring for a dying client and her family members. Which interventions should the nurse implement? (Select all that apply.) a. Teach family members about physical signs of impending death. b. Encourage the management of adverse symptoms. c. Assist family members by offering an explanation for their loss. d. Encourage reminiscence by both client and family members. e. Avoid spirituality because the clients and the nurses beliefs may not be congruent.

ANS: A, B, D The nurse should teach family members about the physical signs of death, because family members often become upset when they see physiologic changes in their loved one. Palliative care includes management of symptoms so that the peaceful death of the client is facilitated. Reminiscence will help both the client and family members cope with the dying process. The nurse is not expected to explain why this is happening to the familys loved one. The nurse can encourage spirituality if the client is agreeable, regardless of whether the clients religion is the same.

7. A nurse prepares to discharge a client who has heart failure. Which questions should the nurse ask to ensure this client's safety prior to discharging home? (Select all that apply.) a. "Are your bedroom and bathroom on the first floor?" b. "What social support do you have at home?" c. "Will you be able to afford your oxygen therapy?" d. "What spiritual beliefs may impact your recovery?" e. "Are you able to accurately weigh yourself at home?"

ANS: A, B, D To ensure safety upon discharge, the nurse should assess for structural barriers to functional ability, such as stairs. The nurse should also assess the client's available social support, which may include family, friends, and home health services. The client's ability to adhere to medication and treatments, including daily weights, should also be reviewed. The other questions do not address the client's safety upon discharge. DIF: Applying/Application REF: 689 KEY: Heart failure| discharge| safety MSC: Integrated Process: Nursing Process: Analysis NOT: Client Needs Category: Safe and Effective Care Environment: Safety and Infection Control

A nurse is visiting a client discharged home after a total hip arthroplasty. What safety precautions would the nurse recommend to the client and family? (Select all that apply.) a. Buy and install an elevated toilet seat. b. Install grab bars in the shower and by the toilet. c. Step into the bathtub with the affected leg first. d. Remove all throw rugs throughout the house. e. Use a shower chair while taking a shower.

ANS: A, B, D, E Buying and installing an elevated toilet seat, installing grab bars, removing throw rugs, and using a shower chair will all promote safety for this client. The client is still on partial weight bearing, so he or she cannot step into the bathtub leading with the operative side.

8. A nurse is working with many stroke clients. Which clients would the nurse consider referring to a mental health provider on discharge? (Select all that apply.) a. Client who exhibits extreme emotional lability b. Client with an initial National Institutes of Health (NIH) Stroke Scale score of 38 c. Client with mild forgetfulness and a slight limp d. Client who has a past hospitalization for a suicide attempt e. Client who is unable to walk or eat 3 weeks post-stroke

ANS: A, B, D, E Clients most at risk for post-stroke depression are those with a previous history of depression, severe stroke (NIH Stroke Scale score of 38 is severe), and post-stroke physical or cognitive impairment. The client with mild forgetfulness and a slight limp would be a low priority for this referral. DIF: Applying/Application REF: 935 KEY: Neurologic disorders| stroke| psychosocial response| depression| nursing assessment MSC: IntegratedProcess:NursingProcess:Assessment NOT: Client Needs Category: Psychosocial Integrity

The nurse learns that effective antimicrobial therapy requires which factors to be present? (Select all that apply.) a. Appropriate drug b. Proper route of administration c. Standardized peak levels d. Sufficient dose e. Sufficient length of treatment f. Appropriate trough levels

ANS: A, B, D, E In order to be effective, antimicrobial therapy must use the appropriate drug in a sufficient dose, for a sufficient length of time, and given via the appropriate route. Some antimicrobials do require monitoring for peak and trough levels, but not all.

A nurse cares for several clients on an inpatient unit. Which infection control measures will the nurse implement? (Select all that apply.) a. Wear a gown when contact of clothing with body fluids is anticipated. b. Teach clients and visitors respiratory hygiene techniques. c. Obtain powered air purifying respirators for all staff members. d. Do not use alcohol-based hand rub between client contacts. e. Disinfect frequently touched surfaces in client-care areas.

ANS: A, B, E Infection control measures appropriate to all clients include hand hygiene with alcohol-based hand rub or soap between client contact, procedures for routine care, cleaning and disinfection of frequently contaminated surfaces, and wearing personal protective equipment when contamination is anticipated. Client and visitors would be instructed on appropriate respiratory hygiene and cough etiquette. No information in the stem indicates the clients need anything more than Standard Precautions.

MULTIPLE RESPONSE 1. A nurse is assessing a client with left-sided heart failure. For which clinical manifestations should the nurse assess? (Select all that apply.) a. Pulmonary crackles b. Confusion, restlessness c. Pulmonary hypertension d. Dependent edema e. Cough that worsens at night

ANS: A, B, E Left-sided heart failure occurs with a decrease in contractility of the heart or an increase in afterload. Most of the signs will be noted in the respiratory system. Right-sided heart failure occurs with problems from the pulmonary vasculature onward including pulmonary hypertension. Signs will be noted before the right atrium or ventricle including dependent edema. DIF: Remembering/Knowledge REF: 682 KEY: Heart failure| assessment/diagnostic examination MSC: IntegratedProcess:NursingProcess:Assessment NOT: Client Needs Category: Physiological Integrity: Physiological Adaptation

2. A nurse evaluates laboratory results for a client with heart failure. Which results should the nurse expect? (Select all that apply.) a. Hematocrit: 32.8% b. Serum sodium: 130 mEq/L c. Serum potassium: 4.0 mEq/L d. Serum creatinine: 1.0 mg/dL e. Proteinuria f. Microalbuminuria

ANS: A, B, E, F A hematocrit of 32.8% is low (should be 42.6%), indicating a dilutional ratio of red blood cells to fluid. A serum sodium of 130 mEq/L is low because of hemodilution. Microalbuminuria and proteinuria are present, indicating a decrease in renal filtration. These are early warning signs of decreased compliance of the heart. The potassium level is on the high side of normal and the serum creatinine level is normal. DIF: Applying/Application REF: 683 KEY: Heart failure| assessment/diagnostic examination MSC: Integrated Process: Nursing Process: Evaluation NOT: Client Needs Category: Physiological Integrity: Reduction of Risk Potential

A client is being admitted with suspected tuberculosis (TB). What actions by the nurse are best? (Select all that apply.) a. Admit the client to a negative-airflow room. b. Maintain a distance of 3 feet (1 m) from the client at all times. c. Obtain specialized respirators for caregiving. d. Other than wearing gloves, no special actions are needed. e. Wash hands with chlorhexidine after providing care. f. Assure client has a respirator for moving between departments.

ANS: A, C A client with suspected TB is admitted to Airborne Precautions, which includes a negative-airflow room and special N95 or PAPR masks to be worn when providing care. A 3-foot (1 m) distance without a mask is required for Droplet Precautions (a nurse providing direct care cannot ensure that he or she will never need to be within 3 feet of the client). Chlorhexidine is used for clients with a high risk of infection. When moving between departments, the client wears a surgical mask.

The nurse is caring for an older client who had a total knee arthroplasty. Prior to surgery, the client lived alone independently. With which interprofessional health care team members will the nurse collaborate to ensure positive client outcomes? (Select all that apply.) a. Case manager b. Mental health counselor c. Physical therapist d. Occupational therapist e. Speech-language pathologist f. Clergy/Spiritual leader

ANS: A, C The client was independent and living alone prior to surgery but will likely need help for a short time at home. However, if the client was ADL independent, he or she will not need referral to an occupational therapist. Therefore, a case manager can assess the living situation and identify any special needs to be addressed. The physical therapist will help the client learn to ambulate independently with a walker. There is no indication that the client needs referral for mental, spiritual, or speech-language services.

MULTIPLE RESPONSE 1. A nursing student studying the neurologic system learns which information? (Select all that apply.) a. An aneurysm is a ballooning in a weakened part of an arterial wall. b. An arteriovenous malformation is the usual cause of strokes. c. Intracerebral hemorrhage is bleeding directly into the brain. d. Reduced perfusion from vasospasm often makes stroke worse. e. Subarachnoid hemorrhage is caused by high blood pressure.

ANS: A, C, D An aneurysm is a ballooning of the weakened part of an arterial wall. Intracerebral hemorrhage is bleeding directly into the brain. Vasospasm often makes the damage from the initial stroke worse because it causes decreased perfusion. An arteriovenous malformation (AVM) is unusual. Subarachnoid hemorrhage is usually caused by a ruptured aneurysm or A VM. DIF: Remembering/Knowledge REF: 932 KEY: Neurologic disorders MSC: Integrated Process: Teaching/Learning NOT: Client Needs Category: Physiological Integrity: Physiological Adaptation

6. A nurse cares for older clients who have traumatic brain injury. What should the nurse understand about this population? (Select all that apply.) a. Admission can overwhelm the coping mechanisms for older clients. b. Alcohol is typically involved in most traumatic brain injuries for this age group. c. These clients are more susceptible to systemic and wound infections. d. Other medical conditions can complicate treatment for these clients. e. Very few traumatic brain injuries occur in this age group.

ANS: A, C, D Older clients often tolerate stress poorly, which includes being admitted to a hospital that is unfamiliar and noisy. Because of decreased protective mechanisms, they are more susceptible to both local and systemic infections. Other medical conditions can complicate their treatment and recovery. Alcohol is typically not related to traumatic brain injury in this population; such injury is most often from falls and motor vehicle crashes. The 65- to 76-year-old age group has the second highest rate of brain injuries compared to other age groups. DIF: Remembering/Knowledge REF: 951 KEY: Neurologic disorders| older adults| trauma MSC: IntegratedProcess:Teaching/Learning NOT: Client Needs Category: Health Promotion and Maintenance

5. A nurse collaborates with an unlicensed assistive personnel (UAP) to provide care for a client with congestive heart failure. Which instructions should the nurse provide to the UAP when delegating care for this client? (Select all that apply.) a. "Reposition the client every 2 hours." b. "Teach the client to perform deep-breathing exercises." c. "Accurately record intake and output." d. "Use the same scale to weigh the client each morning." e. "Place the client on oxygen if the client becomes short of breath."

ANS: A, C, D The UAP should reposition the client every 2 hours to improve oxygenation and prevent atelectasis. The UAP can also accurately record intake and output, and use the same scale to weigh the client each morning before breakfast. UAPs are not qualified to teach clients or assess the need for and provide oxygen therapy. DIF: Applying/Application REF: 684 KEY: Heart failure| delegation| interdisciplinary team| unlicensed assistive personnel (UAP) MSC: Integrated Process: Communication and Documentation NOT: Client Needs Category: Safe and Effective Care Environment: Management of Care

A nurse admits an older adult client to the hospital. Which criterion should the nurse use to determine if the client can make his own medical decisions? (Select all that apply.) a. Can communicate his treatment preferences b. Is able to read and write at an eighth-grade level c. Is oriented enough to understand information provided d. Can evaluate and deliberate information e. Has completed an advance directive

ANS: A, C, D To have decision-making ability, a person must be able to perform three tasks: receive information (but not necessarily oriented 4); evaluate, deliberate, and mentally manipulate information; and communicate a treatment preference. The client does not have to read or write at a specific level. Education can be provided at the clients level so that he can make the necessary decisions. The client does not need to complete an advance directive to make his own medical decisions. An advance directive will be necessary if he wants to designate someone to make medical decisions when he is unable to.

2. The nurse working in the emergency department assesses a client who has symptoms of stroke. For what modifiable risk factors should the nurse assess? (Select all that apply.) a. Alcohol intake b. Diabetes c. High-fat diet d. Obesity e. Smoking

ANS: A, C, D, E Alcohol intake, a high-fat diet, obesity, and smoking are all modifiable risk factors for stroke. Diabetes is not modifiable but is a risk factor that can be controlled with medical intervention. DIF: Remembering/Knowledge REF: 933 KEY: Neurologic disorders| stroke| nursing assessment MSC: IntegratedProcess:NursingProcess:Assessment NOT: Client Needs Category: Physiological Integrity: Physiological Adaptation

7. A client receiving chemotherapy has a white blood cell count of 1000/mm3. What actions by the nurse are most appropriate? (Select all that apply.) a. Assess all mucous membranes every 4 to 8 hours. b. Do not allow the client to eat meat or poultry. c. Listen to lung sounds and monitor for cough. d. Monitor the venous access device appearance with vital signs. e. Take and record vital signs every 4 to 8 hours.

ANS: A, C, D, E Depending on facility protocol, the nurse should assess this client for infection every 4 to 8 hours by assessing all mucous membranes, listening to lung sounds, monitoring for cough, monitoring the appearance of the venous access device, and recording vital signs. Eating meat and poultry is allowed. 1. A nurse in the oncology clinic is providing preoperative education to a client just diagnosed with cancer. The client has been scheduled for surgery in 3 days. What action by the nurse is best? a. Call the client at home the next day to review teaching. b. Give the client information about a cancer support group. c. Provide all the preoperative instructions in writing. d. Reassure the client that surgery will be over soon.* ANS: A Clients are often overwhelmed at a sudden diagnosis of cancer and may be more overwhelmed at the idea of a major operation so soon. This stress significantly impacts the client's ability to understand, retain, and recall information. The nurse would call the client at home the next day to review the teaching and to answer questions. The client may or may not be ready to investigate a support group, but this does not help with teaching. Giving information in writing is important (if the client can read it), but in itself will not be enough. Telling the client that surgery will be over soon is giving false reassurance and does nothing for teaching.

A nurse is planning postoperative care for a client following a total hip arthroplasty. What nursing interventions would help prevent venous thromboembolism for this client? (Select all that apply.) a. Early ambulation b. Fluid restriction c. Quadriceps-setting exercises d. Compression stockings/devices e. Anticoagulant drug therapy

ANS: A, C, D, E Early ambulation, leg exercises, and compression stockings/devices promote venous return and peripheral circulation which helps prevent deep vein thrombi. Anticoagulants such as subcutaneous low-molecular-weight heparin (LMWH) or factor Xa inhibitors are used for all clients who have a total lower extremity joint arthroplasty. The nurse would encourage fluids to expand blood volume and promote circulation; fluids would not be restricted.

A nurse is providing health education at a community center. Which instructions does the nurse include in teaching about prevention of lightning injuries during a storm? (Select all that apply.) a. Seek shelter inside a building or vehicle. b. Hide under a tall tree. c. Do not take a bath or shower. d. Turn off the television. e. Remove all body piercings. f. Put down golf clubs or gardening tools.

ANS: A, C, D, F a. Seek shelter inside a building or vehicle. c. Do not take a bath or shower. d. Turn off the television. f. Put down golf clubs or gardening tools. When thunder is heard, individuals should seek shelter in a safe area such as a building or an enclosed vehicle. Electrical equipment such as TVs and stereos should be turned off. Stay away from plumbing, water, and metal objects such as golf clubs or gardening tools. Do not stand under an isolated tall tree or a structure such as a flagpole. Body piercings will not increase a person's chances of being struck by lightning.

4. After teaching a client with congestive heart failure (CHF), the nurse assesses the client's understanding. Which client statements indicate a correct understanding of the teaching related to nutritional intake? (Select all that apply.) a. "I'll read the nutritional labels on food items for salt content." b. "I will drink at least 3 liters of water each day." c. "Using salt in moderation will reduce the workload of my heart." d. "I will eat oatmeal for breakfast instead of ham and eggs." e. "Substituting fresh vegetables for canned ones will lower my salt intake."

ANS: A, D, E Nutritional therapy for a client with CHF is focused on decreasing sodium and water retention to decrease the workload of the heart. The client should be taught to read nutritional labels on all food items, omit table salt and foods high in sodium (e.g., ham and canned foods), and limit water intake to a normal 2 L/day. DIF: Applying/Application REF: 682 KEY: Heart failure| patient education MSC: Integrated Process: Teaching/Learning NOT: Client Needs Category: Physiological Integrity: Basic Care and Comfort

5. A nursing student studying traumatic brain injuries (TBIs) should recognize which facts about these disorders? (Select all that apply.) a. A client with a moderate trauma may need hospitalization. b. A Glasgow Coma Scale score of 10 indicates a mild brain injury. c. Only open head injuries can cause a severe TBI. d. A client with a Glasgow Coma Scale score of 3 has severe TBI. e. The terms "mild TBI" and "concussion" have similar meanings.

ANS: A, D, E "Mild TBI" is a term used synonymously with the term "concussion." A moderate TBI has a Glasgow Coma Scale (GCS) score of 9 to 12, and these clients may need to be hospitalized. Both open and closed head injuries can cause a severe TBI, which is characterized by a GCS score of 3 to 8. DIF: Remembering/Knowledge REF: 947 KEY: Neurologic disorders| trauma MSC: Integrated Process: Teaching/Learning NOT: Client Needs Category: Physiological Integrity: Physiological Adaptation

4. A nurse has applied to work at a hospital that has National Stroke Center designation. The nurse realizes the hospital adheres to eight Core Measures for ischemic stroke care. What do these Core Measures include? (Select all that apply.) a. Discharging the client on a statin medication b. Providing the client with comprehensive therapies c. Meeting goals for nutrition within 1 week d. Providing and charting stroke education e. Preventing venous thromboembolism

ANS: A, D, E Core Measures established by The Joint Commission include discharging stroke clients on statins, providing and recording stroke education, and taking measures to prevent venous thromboembolism. The client must be assessed for therapies but may go elsewhere for them. Nutrition goals are not part of the Core Measures. DIF: Remembering/Knowledge REF: 945 KEY: Neurologic disorders| stroke| Core Measures MSC: IntegratedProcess:Teaching/Learning NOT: Client Needs Category: Safe and Effective Care Environment: Management of Care

7. A nurse is caring for a client with a nonhealing arterial ulcer. The primary health care provider has informed the client about possibly needing to amputate the client's leg. The client is crying and upset. What actions by the nurse are best? (Select all that apply.) a. Ask the client to describe his or her current emotions. b. Assess the client for support systems and family. c. Offer to stay with the client if he or she desires. d. Relate how smoking contributed to this situation. e. Tell the client that many people have amputations. f. Arrange for an amputee to come visit the client.

ANS: A,B,C When a client is upset, the nurse would offer self by remaining with the client if desired. Other helpful measures include determining what and whom the client has for support systems and asking the client to describe what he or she is feeling. Telling the client how smoking has led to this situation will only upset the client further and will damage the therapeutic relationship. Telling the client that many people have amputations belittles the client's feelings. It is too early to send an amputee to visit the client as the decision to amputate has not yet been made.

1. A nurse is learning about human immune deficiency virus (HIV) infection. Which statements about HIV infection are correct? (Select all that apply.) a. CD4+ cells begin to create new HIV virus particles. b. Antibodies produced are incomplete and do not function well. c. Macrophages stop functioning properly. d. Opportunistic infections and cancer are leading causes of death. e. People with HIV-I disease are not infectious to others. f. The CD4+ T-cell is only affected when the disease has progressed to HIV-III

ANS: A,B,C,D In HIV, CD4+ cells begin to create new HIV particles. Antibodies the client produce are incomplete and do not function well. Macrophages also stop functioning properly. Opportunistic infections and cancer are the two leading causes of death in client's with HIV infection. People infected withGHRIVADarEeSinLfeAcBtio.uCsOinMall stages of the disease. The CD4+ T-cell is the immune system cell most affected by infection with the HIV virus.

9. A nurse begins a job at a Veterans Administration Hospital and asks why so much emphasis is on HIV testing for the veterans. What reasons is this nurse given? (Select all that apply.) a. Veterans have a high prevalence of substance abuse. b. Many veterans may engage in high risk behaviors. c. Many older veterans may not know their risks. d. Everyone should know their HIV status. e. Belief that the VA has tested them and would notify them if positive.

ANS: A,B,C,D,E All options are correct for the veteran population. The nurse interacting with veteran would ensure they know about the HIV testing offered by the VA.

2. A nurse is preparing to administer IV chemotherapy. What supplies does this nurse need? (Select all that apply.) a. "Chemo" gloves b. Face mask c. Impervious gown d. N95 respirator e. Shoe covers f. Eye protection

ANS: A,B,C,F The Occupational Safety and Health Administration (OSHA) and the Oncology Nurses Society have developed safety guidelines for those preparing or administering IV chemotherapy. These include double gloves (or "chemo" gloves), eye protection, a face mask, and a gown. An N95 respirator and shoe covers are not required.

4. A nurse is caring for a client on IV infusion of heparin. What actions does this nurse include in the client's plan of care? (Select all that apply.) a. Assess the client for bleeding. b. Monitor the daily activated partial thromboplastin time (aPTT) results. c. Stop the IV for aPTT above baseline. d. Use an IV pump for the infusion. e. Weigh the client daily on the same scale.

ANS: A,B,D Assessing for bleeding, monitoring aPTT, and using an IV pump for the infusion are all important safety measures for heparin to prevent injury from bleeding. The aPTT needs to be 1.5 to 2.5 times normal in order to demonstrate that the heparin is therapeutic. Weighing the client is not related.

11. The nurse is reviewing risk factors in a client who has atherosclerosis. Which findings are most concerning? (Select all that apply.) a. Elevated low-density lipoprotein b. Decreased levels of high-density lipoprotein cholesterol (HDL-C) c. Asian ethnicity d. History of smoking e. Blood pressure: 142/92 mm Hg on one occasion

ANS: A,B,D Elevated levels of lipids (fats) such as low-density lipoprotein cholesterol (LDL-C) and decreased levels of high-density lipoprotein cholesterol can cause chemical damage to blood vessel walls. Smoking can cause endothelial damage in addition to increasing a client's carbon monoxide levels. African American and Hispanic ethnicities carry an increased risk for atherosclerosis. Hypertension does increase atherosclerosis risk, but an elevated reading on one occasion is not classified as hypertension.

3. A client has been bedridden for several days after major abdominal surgery. What action does the nurse delegate to the assistive personnel (AP) for deep vein thrombosis (DVT) prevention? (Select all that apply.) a. Apply compression stockings. b. Assist with ambulation. c. Encourage coughing and deep breathing. d. Offer fluids frequently. e. Teach leg exercises.

ANS: A,B,D The AP can apply compression stockings, assist with ambulation, and offer fluids frequently to help prevent DVT. The AP can also encourage the client to do pulmonary exercises, but these do not decrease the risk of DVT. Teaching is a nursing function.

1. A nurse assesses a client with an injury to the medulla. Which clinical manifestations would the nurse expect to find? (Select all that apply.) a. Decreased respiratory rate b. Impaired swallowing c. Visual changes d. Inability to shrug shoulders e. Loss of gag reflex

ANS: A,B,D,E Cranial nerves IX (glossopharyngeal), X (vagus), XI (accessory), and XII (hypoglossal) emerge from the medulla, as do portions of cranial nerves VII (facial) and VIII (acoustic). Damage to these nerves causes decreased respirations, impaired swallowing, inability to shrug shoulders, and loss of the gag reflex. The other manifestations are not associated with damage to the medulla.

7. The nurse is educating a client with HIV-II and the partner on self-care measures to prevent infection when blood counts are low. What information does the nurse provide? (Select all that apply.) a. Do not work in the garden or with houseplants. b. Do not empty the kitty litter boxes. c. Clean your toothbrush in the dishwasher daily. d. Bathe daily using antimicrobial soap. e. Avoid people who are sick and large crowds. f. Make sure meat, fish, and eggs are cooked well.

ANS: A,B,D,E,F Ways to avoid infection when immunocompromised include not working in the garden or with houseplants; not emptying litter boxes; running the toothbrush through the dishwasher at least weekly; bathing daily using antimicrobial soap; avoiding sick people and large crowds; and making sure meat, fish, and eggs are cooked well prior to eating them.

2. Which findings are AIDS-defining characteristics? (Select all that apply.) a. CD4+ cell count less than 200/mm3 (0.2 109/L) or less than 14% b. Infection with P. jiroveci c. Positive enzyme-linked immunosorbent assay (ELISA) test for human immune deficiency virus (HIV) d. Presence of HIV wasting syndrome e. Taking antiretroviral medications f. Confusion, dementia, or memory loss

ANS: A,B,D,F A diagnosis of AIDS requires that the person be HIV positive and have either a CD4+ T-cell count of less than 200 cells/mm3 (0.2 109/L) or less than 14% (even if the total CD4+ count is above 200 cells/mm3) or an opportunistic infection such as P. jiroveci and HIV wasting syndrome. Confusion, dementia, and memory loss are central nervous system indications. Having a positive ELISA test and taking antiretroviral medications are not AIDS-defining characteristics.

3. A nurse assesses a client with a brain tumor. Which newly identified assessment findings would alert the nurse to urgently communicate with the primary health care provider? (Select all that apply.) a. Glasgow Coma Scale score of 8 b. Decerebrate posturing c. Reactive pupils d. Uninhibited speech e. Decreasing level of consciousness

ANS: A,B,E The nurse would urgently communicate changes in a patient's neurologic status, including a decrease in the Glasgow Coma Scale score; abnormal flexion or extension; changes in cognition or level of consciousness; and pinpointed, dilated, and nonreactive pupils.

3. The nurse is teaching a client about medications for HIV-II treatment. What drugs are paired with the correct information? (Select all that apply.) a. Abacavir: avoid fatty and fried foods. b. Efavirenz: take 1 hour before or 2 hours after antacids. c. Atazanavir: check pulse daily and report pulse greater than 100 beats/min. d. Dolutegravir: do not take this medication if you become pregnant. e. Enfuvirtide: teach client how to operate syringe infusion pump for administration. f. All drugs: you must adhere to the drug schedule at least 90% of the time for effectiveness.

ANS: A,B,F Abacavir is a nucleoside reverse transcriptase inhibitor and clients are taught to avoid fried and fatty foods because they can lead to digestive upsets and even pancreatitis. Efavirenz is a nonnucleoside reverse transcriptase inhibitor and clients are taught to take them (doraverene) all except spaced 1 hour before or 2 hours after antacids to avoid inhibiting drug absorption. Atazanavir is a protease inhibitor and can cause bradycardia which should be reported. Dolutegravir is an integrase inhibitor and can cause birth defects. Enfuvirtide is a fusion inhibitor and is given subcutaneously. All drugs must be taken as scheduled 90% of the time in order to remain effective.

5. A client is being discharged on warfarin therapy. What discharge instruction is the nurse required to provide? (Select all that apply.) a. Dietary restrictions b. Driving restrictions c. Follow-up laboratory monitoring d. Possible drug-drug interactions e. Reason to take medication f. Wearing a Medic Alert bracelet

ANS: A,C,D,E Best practices state that clients being discharged on warfarin need instruction on follow-up monitoring, dietary restrictions, drug-drug interactions, using a Medic Alert bracelet or necklace, and reason for compliance. Driving is typically not restricted.

3. A client receiving radiation therapy reports severe skin itching and irritation. What actions does the nurse delegate to assistive personnel (AP)? (Select all that apply.) a. Apply approved moisturizers to dry skin. b. Apply steroid creams to the skin. c. Bathe the client using mild soap. d. Help the client pat skin dry after a bath. e. Teach the client to avoid sunlight. f. Make sure no clothing is rubbing the site.

ANS: A,C,D,F The nurse can delegate applying moisturizer approved by the radiation oncologist using mild soap for bathing, and helping the client pat wet skin dry after bathing. Any clothing worn over the site should be soft and not create friction. Steroid creams are not used for this condition. Hot water will worsen the irritation. Client teaching is a nursing function.

4. A client with HIV-III is hospitalized with P. jiroveci pneumonia and is started on the drug of choice for this infection. What laboratory values would be most important for the nurse report to the primary health care provider? (Select all that apply.) a. Aspartate transaminase, alanine transaminase: elevated b. CD4+ cell count: 180/mm3 c. Creatinine: 1.0 mg/dL (88 mcmol/L) d. Platelet count: 80,000/mm3 (80 109/L) e. Serum sodium: 120 mEq/L (120 mmol/L) f. Serum potassium: 3.4 mEq/L (3.4 mmol/L)

ANS: A,D,E The drug of choice to treat P. jiroveci pneumonia is trimethoprim with sulfamethoxazole. Side effects of this drug include hepatitis, hyponatremia, and thrombocytopenia. The elevated liver enzymes, low platelet count, and low sodium would all be reported. The CD4+ cell count is within the expected range for a client with an AIDS-defining infection. The creatinine level is normal and the potassium is just below normal.

8. The nurse working in the emergency department knows that which factors are commonly related to aneurysm formation? (Select all that apply.) a. Atherosclerosis b. Down syndrome c. Frequent heartburn d. History of hypertension e. History of smoking f. Hyperlipidemia

ANS: A,D,E,F Atherosclerosis, hypertension, hyperlipidemia, hyperlipidemia, and smoking are the most commonly related factors. Down syndrome and heartburn have no relation to aneurysm formation.

19. A client with long-standing heart failure being treated for cancer has received a dose of ondansetron for nausea. What action by the nurse is most important? a. Assess the client for a headache or dizziness. b. Request a prescription for cardiac monitoring c. Instruct the client to change positions slowly. d. Weigh the client daily before eating.

ANS: B 5-HT3 antagonists, such as ondansetron, can prolong the QT interval within the cardiac conduction cycle. ECG monitoring is recommended in patients with electrolyte abnormalities (e.g., hypokalemia or hypomagnesemia), heart failure, bradyarrhythmias or patients taking other medications that can cause QT prolongation. The nurse would contact the primary health care provider and request cardiac monitoring. The nurse would assess the client for any other reported changes, but this is not a critical safety factor. Weight is not related directly to this drug.

22. A nurse teaches a client with heart failure about energy conservation. Which statement should the nurse include in this client's teaching? a. "Walk until you become short of breath, and then walk back home." b. "Gather everything you need for a chore before you begin." c. "Pull rather than push or carry items heavier than 5 pounds." d. "Take a walk after dinner every day to build up your strength."

ANS: B A client who has heart failure should be taught to conserve energy. Gathering all supplies needed for a chore at one time decreases the amount of energy needed. The client should not walk until becoming short of breath because he or she may not make it back home. Pushing a cart takes less energy than pulling or lifting. Although walking after dinner may help the client, the nurse should teach the client to complete activities when he or she has the most energy. This is usually in the morning. DIF: Applying/Application REF: 696 KEY: Heart failure| functional ability| patient education MSC: IntegratedProcess:Teaching/Learning NOT: Client Needs Category: Health Promotion and Maintenance

10. A client has a platelet count of 9800/mm3. What action by the nurse is most appropriate? a. Assess the client for calf pain, warmth, and redness. b. Instruct the client to call for help to get out of bed. c. Obtain cultures as per the facilitys standing policy. d. Place the client on protective isolation precautions.

ANS: B A client with a platelet count this low is at high risk for serious bleeding episodes. To prevent injury, the client should be instructed to call for help prior to getting out of bed. Calf pain, warmth, and redness might indicate a deep vein thrombosis, not associated with low platelets. Cultures and isolation relate to low white cell counts.

19. A nurse is caring for a client with a deep vein thrombosis (DVT). What nursing assessment indicates that an important outcome has been met? a. Ambulates with assistance b. Oxygen saturation of 98% c. Pain of 2/10 after medication d. Verbalizing risk factors

ANS: B A critical complication of DVT is pulmonary embolism. A normal oxygen saturation indicates that this has not occurred. The other assessments are also positive, but not as important.

A nurse receives report from the laboratory on a client who was admitted for fever. The laboratory technician states that the client has "a shift to the left" on the white blood cell count. What action by the nurse is most important? a. Document findings and continue monitoring. b. Notify the primary health care provider and request antibiotics. c. Place the client in protective isolation. d. Tell the client this signifies inflammation.

ANS: B A shift to the left indicates an increase in immature neutrophils and is often seen in infections, especially those caused by bacteria. The nurse will notify the primary health care provider and request antibiotics (and cultures). Documentation and teaching need to be done, but the nurse needs to do more. The client does not need protective isolation.

6. A client with HIV-II has had a sudden decline in status with a large increase in viral load. What action would the nurse take first? a. Ask the client about travel to any foreign countries. b. Assess the client for adherence to the drug regimen. c. Determine if the client has any new sexual partners. d. Request information about new living quarters or pets.

ANS: B Adherence to the complex drug regimen needed for HIV treatment can be daunting. Clients must take their medications on time and correctly at a minimum of 90% of the time to be effective. Since this client's viral load has increased dramatically, the nurse would first assess this factor. After this, the other assessments may or may not be needed.

22. A client has an intraventricular catheter. What action by the nurse takes priority? a. Document intracranial pressure readings. b. Perform hand hygiene before client care. c. Measure intracranial pressure per hospital policy. d. Teach the client and family about the device.

ANS: B All of the actions are appropriate for this client. However, performing hand hygiene takes priority because it prevents infection, which is a possibly devastating complication. DIF: Applying/Application REF: 961 KEY: Neurologic disorders| Standard Precautions| infection control MSC: IntegratedProcess:NursingProcess:Implementation NOT: Client Needs Category: Safe and Effective Care Environment: Safety and Infection Control

A nurse assesses a group of clients who have rheumatoid arthritis (RA). Which client would the nurse see first? a. Client who reports jaw pain when eating b. Client with a red, hot, swollen right wrist c. Client who has a puffy-looking area behind the knee d. Client with a worse joint deformity since the last visit

ANS: B All of the options are possible manifestations of RA. However, the presence of one joint that is much redder, hotter, or more swollen that the other joints may indicate infection or an exacerbation of the RA disease process. The nurse needs to see this client first.

The nurse assesses a client with diabetes and osteoarthritis (OA) during a checkup. The nurse notes the client's blood glucose readings have been elevated. What question by the nurse is most appropriate? a. "Are you following the prescribed diabetic diet?" b. "Have you been taking glucosamine supplements?" c. "How much exercise do you really get each week?" d. "You're still taking your diabetic medication, right?"

ANS: B All of the topics are appropriate for a client whose blood glucose readings have been higher than usual. However, since this client also has OA, and glucosamine can increase blood glucose levels, the nurse would ask about its use. The other questions all have an element of nontherapeutic communication in them. Asking how much exercise the client "really" gets is or if the diet is being followed is accusatory. Asking if the client takes his or her medications "right?" is patronizing.

7. A client has hypertension and high risk factors for cardiovascular disease. The client is overwhelmed with the recommended lifestyle changes. What action by the nurse is best? a. Assess the client's support system. b. Assist in finding one change the client can control. c. Determine what stressors the client faces in daily life. d. Inquire about delegating some of the client's obligations.

ANS: B All options are appropriate when assessing stress and responses to stress. However, this client feels overwhelmed by the suggested lifestyle changes. Instead of looking at all the needed changes, the nurse would assist the client in choosing one the client feels optimistic about controlling. Once the client has mastered that change, he or she can move forward with another change. Determining support systems, daily stressors, and delegation opportunities does not directly impact the client's feelings of control.

A nurse assesses a client who is dying. Which manifestation of a dying client should the nurse assess to determine whether the client is near death? a. Level of consciousness b. Respiratory rate c. Bowel sounds d. Pain level on a 0-to-10 scale

ANS: B Although all of these assessments should be performed during the dying process, periods of apnea and Cheyne- Strokes respirations indicate death is near. As peripheral circulation decreases, the clients level of consciousness and bowel sounds decrease, and the client would be unable to provide a numeric number on a pain scale. Even with these other symptoms, the nurse should continue to assess respiratory rate throughout the dying process. As the rate drops significantly and breathing becomes agonal, death is near.

22. A nurse wants to provide community service that helps meet the goals of Healthy People 2020 (HP2020) related to cardiovascular disease and stroke. What activity would best meet this goal? a. Teach high school students heart-healthy living. b. Participate in blood pressure screenings at the mall. c. Provide pamphlets on heart disease at the grocery store. d. Set up an "Ask the nurse" booth at the pet store.

ANS: B An important goal of HP2020 is to increase the proportion of adults who have had their blood pressure measured within the preceding 2 years and can state whether their blood pressure was normal or high. Participating in blood pressure screening in a public spot will best help meet that goal. The other options are all appropriate but do not specifically help meet a goal.

13. A nurse is caring for four clients. Which one would the nurse see first? a. Client who needs a beta blocker, and has a blood pressure of 98/58 mm Hg. b. Client who had a first dose of captopril and needs to use the bathroom. c. Hypertensive client with a blood pressure of 188/92 mm Hg. d. Client who needs pain medication prior to a dressing change of a surgical wound.

ANS: B Angiotensin-converting enzyme inhibitors such as captopril can cause hypotension, especially after the first dose. The nurse would see this client first to prevent falling if the client decides to get up without assistance. The two blood pressure readings are abnormal but not critical. The nurse would check on the client with higher blood pressure next to assess for problems related to the reading. The nurse can administer the beta blocker as standards state to hold it if the systolic blood pressure is below 90 to 100 mm Hg. The client who needs pain medication prior to the dressing change is not a priority over client safety and assisting the other client to the bathroom.

24. A nurse assesses a client who has mitral valve regurgitation. For which cardiac dysrhythmia should the nurse assess? a. Preventricular contractions b. Atrial fibrillation c. Symptomatic bradycardia d. Sinus tachycardia

ANS: B Atrial fibrillation is a clinical manifestation of mitral valve regurgitation and stenosis. Preventricular contractions and bradycardia are not associated with valvular problems. These are usually identified in clients with electrolyte imbalances, myocardial infarction, and sinus node problems. Sinus tachycardia is a manifestation of aortic regurgitation due to a decrease in cardiac output. DIF: Understanding/Comprehension REF: 692 KEY: Valve disorder| cardiac dysrhythmia MSC: IntegratedProcess:NursingProcess:Assessment NOT: Client Needs Category: Physiological Integrity: Reduction of Risk Potential

17. A client is receiving an infusion of alteplase for an intra-arterial clot. The client begins to mumble and is disoriented. What action by the nurse is most important? a. Assess the client's neurologic status. b. Notify the Rapid Response Team. c. Prepare to administer vitamin K. d. Turn down the infusion rate.

ANS: B Clients on fibrinolytic therapy are at high risk of bleeding. The sudden onset of neurologic signs may indicate that the client is having a hemorrhagic stroke. The nurse does need to complete a thorough neurologic examination, but would first call the Rapid Response Team based on the client's manifestations. Vitamin K is not the antidote for this drug. Turning down the infusion rate will not be helpful if the client is still receiving any of the drug.

14. After teaching a client who is being discharged home after mitral valve replacement surgery, the nurse assesses the client's understanding. Which client statement indicates a need for additional teaching? a. "I'll be able to carry heavy loads after 6 months of rest." b. "I will have my teeth cleaned by my dentist in 2 weeks." c. "I must avoid eating foods high in vitamin K, like spinach." d. "I must use an electric razor instead of a straight razor to shave."

ANS: B Clients who have defective or repaired valves are at high risk for endocarditis. The client who has had valve surgery should avoid dental procedures for 6 months because of the risk for endocarditis. When undergoing a mitral valve replacement surgery, the client needs to be placed on anticoagulant therapy to prevent vegetation forming on the new valve. Clients on anticoagulant therapy should be instructed on bleeding precautions, including using an electric razor. If the client is prescribed warfarin, the client should avoid foods high in vitamin K. Clients recovering from open heart valve replacements should not carry anything heavy for 6 months while the chest incision and muscle heal. DIF: Applying/Application REF: 694 KEY: Valve disorder| patient education| hygiene MSC: IntegratedProcess:Teaching/Learning NOT: Client Needs Category: Physiological Integrity: Reduction of Risk Potential

20. A nurse assesses a client who has a history of heart failure. Which question should the nurse ask to assess the extent of the client's heart failure? a. "Do you have trouble breathing or chest pain?" b. "Are you able to walk upstairs without fatigue?" c. "Do you awake with breathlessness during the night?" d. "Do you have new-onset heaviness in your legs?"

ANS: B Clients with a history of heart failure generally have negative findings, such as shortness of breath. The nurse needs to determine whether the client's activity is the same or worse, or whether the client identifies a decrease in activity level. Trouble breathing, chest pain, breathlessness at night, and peripheral edema are symptoms of heart failure, but do not provide data that can determine the extent of the client's heart failure. DIF: Applying/Application REF: 682 KEY: Heart failure| functional ability| respiratory distress/failure MSC: IntegratedProcess:NursingProcess:Assessment NOT: Client Needs Category: Physiological Integrity: Physiological Adaptation

16. A client with a traumatic brain injury is agitated and fighting the ventilator. What drug should the nurse prepare to administer? a. Carbamazepine (Tegretol) b. Dexmedetomidine (Precedex) c. Diazepam (Valium) d. Mannitol (Osmitrol)

ANS: B Dexmedetomidine is often used to manage agitation in the client with traumatic brain injury. Carbamazepine is an antiseizure drug. Diazepam is a benzodiazepine. Mannitol is an osmotic diuretic. DIF: Remembering/Knowledge REF: 955 KEY: Neurologic disorders| sedatives| mechanical ventilation MSC: Integrated Process: Nursing Process: Implementation NOT: Client Needs Category: Physiological Integrity: Pharmacological and Parenteral Therapies

12. A nurse assesses a client with mitral valve stenosis. What clinical manifestation should alert the nurse to the possibility that the client's stenosis has progressed? a. Oxygen saturation of 92% b. Dyspnea on exertion c. Muted systolic murmur d. Upper extremity weakness

ANS: B Dyspnea on exertion develops as the mitral valvular orifice narrows and pressure in the lungs increases. The other manifestations do not relate to the progression of mitral valve stenosis. DIF: Applying/Application REF: 688 KEY: Valve disorder| respiratory distress/failure MSC: IntegratedProcess:NursingProcess:Assessment NOT: Client Needs Category: Physiological Integrity: Reduction of Risk Potential

24. A client with cancer has anorexia and mucositis, and is losing weight. The clients family members continually bring favorite foods to the client and are distressed when the client wont eat them. What action by the nurse is best? a. Explain the pathophysiologic reasons behind the client not eating. b. Help the family show other ways to demonstrate love and caring. c. Suggest foods and liquids the client might be willing to try to eat. d. Tell the family the client isnt able to eat now no matter what they bring.

ANS: B Families often become distressed when their loved ones wont eat. Providing food is a universal sign of caring, and to some people the refusal to eat signifies worsening of the condition. The best option for the nurse is to help the family find other ways to demonstrate caring and love, because with treatment-related anorexia and mucositis, the client is not likely to eat anything right now. Explaining the rationale for the problem is a good idea but does not suggest to the family anything that they can do for the client. Simply telling the family the client is not able to eat does not give them useful information and is dismissive of their concerns.

5. A client is being prepared for a mechanical embolectomy. What action by the nurse takes priority? a. Assess for contraindications to fibrinolytics. b. Ensure that informed consent is on the chart. c. Perform a full neurologic assessment. d. Review the client's medication lists.

ANS: B For this invasive procedure, the client needs to give informed consent. The nurse ensures that this is on the chart prior to the procedure beginning. Fibrinolytics are not used. A neurologic assessment and medication review are important, but the consent is the priority. DIF: Applying/Application REF: 938 KEY: Neurologic disorders| stroke| informed consent MSC: Integrated Process: Communication and Documentation NOT: Client Needs Category: Safe and Effective Care Environment: Management of Care

12. A nurse is caring for a client with HIV-III who was admitted with HAND. What sign or symptom would be most important for the nurse to report to the primary health care provider? a. Nausea b. Change in pupil size c. Weeping open lesions d. Cough

ANS: B HIV-associated neurocognitive disorder (HAND) is a sign of neurologic involvement. The nurse would report any sign of increasing intracranial pressure immediately, including change in pupil size, level of consciousness, vital signs, or limb strength. The other signs and symptoms are not life threatening and would be documented and reported appropriately.

15. A nurse is caring for four clients who might be brain dead. Which client would best meet the criteria to allow assessment of brain death? a. Client with a core temperature of 95° F (35° C) for 2 days b. Client in a coma for 2 weeks from a motor vehicle crash c. Client who is found unresponsive in a remote area of a field by a hunter d. Client with a systolic blood pressure of 92 mm Hg since admission

ANS: B In order to determine brain death, clients must meet four criteria: 1) coma from a known cause, 2) normal or near-normal core temperature, 3) normal systolic blood pressure, and 4) at least one neurologic examination. The client who was in the car crash meets two of these criteria. The clients with the lower temperature and lower blood pressure have only one of these criteria. There is no data to support assessment of brain death in the client found by the hunter. DIF: Remembering/Knowledge REF: 954 KEY: Neurologic disorders| brain death| neurologic assessment MSC: IntegratedProcess:NursingProcess:Assessment NOT: Client Needs Category: Physiological Integrity: Physiological Adaptation

4. A client has been diagnosed with hypertension but does not take the antihypertensive medications because of a lack of symptoms. What response by the nurse is best? a. "Do you have trouble affording your medications?" b. "Most people with hypertension do not have symptoms." c. "You are lucky; most people get severe morning headaches." d. "You need to take your medicine or you will get kidney failure."

ANS: B Most people with hypertension are asymptomatic, although a small percentage do have symptoms such as headache. The nurse would explain this to the client. Asking about paying for medications utilizes closed-ended questioning and is not therapeutic. Threatening the client with possible complications will not increase compliance.

A client asks the nurse about having a total knee arthroplasty to relieve joint pain. Which factor would place the client at the highest risk for impaired postoperative healing? a. Controlled hypertension b. Obesity c. Osteoarthritis d. Mild osteopenia

ANS: B Obesity places a client at high risk for many postoperative complications including slower wound and bone healing. The other factors usually do not affect healing after surgery.

9. After receiving the hand-off report, which client should the oncology nurse see first? a. Client who is afebrile with a heart rate of 108 beats/min b. Older client on chemotherapy with mental status changes c. Client who is neutropenic and in protective isolation d. Client scheduled for radiation therapy today

ANS: B Older clients often do not exhibit classic signs of infection, and often mental status changes are the first observation. Clients on chemotherapy who become neutropenic also often do not exhibit classic signs of infection. The nurse should assess the older client first. The other clients can be seen afterward.

15. A client is 4 hours postoperative after a femoral-popliteal bypass. The client reports throbbing leg pain on the affected side, rated as 7/10. What action by the nurse is most important? a. Administer pain medication as ordered. b. Assess distal pulses and skin color. c. Document the findings in the client's chart. d. Notify the surgeon immediately.

ANS: B Once perfusion has been restored or improved to an extremity, clients can often feel a throbbing pain due to the increased blood flow. However, it is important to differentiate this pain from ischemia. The nurse would assess for other signs of perfusion, such as distal pulses and skin color/temperature. Administering pain medication is done once the nurse determines that the client's perfusion status is normal. Documentation needs to be thorough. Notifying the surgeon is not necessary.

19. A client has received a dose of ondansetron (Zofran) for nausea. What action by the nurse is most important? a. Assess the client for a headache. b. Assist the client in getting out of bed. c. Instruct the client to reduce salt intake. d. Weigh the client daily before the client eats.

ANS: B Ondansetron side effects include postural hypotension, vertigo, and bradycardia, all of which increase the clients risk for injury. The nurse should assist the client when getting out of bed. Headache and fluid retention are not side effects of this drug.

4. A client in the oncology clinic reports her family is frustrated at her ongoing fatigue 4 months after radiation therapy for breast cancer. What response by the nurse is most appropriate? a. Are you getting adequate rest and sleep each day? b. It is normal to be fatigued even for years afterward. c. This is not normal and Ill let the provider know. d. Try adding more vitamins B and C to your diet.

ANS: B Regardless of the cause, radiation-induced fatigue can be debilitating and may last for months or years after treatment has ended. Rest and adequate nutrition can affect fatigue, but it is most important that the client understands this is normal.

3. A nurse assesses a client admitted to the cardiac unit. Which statement by the client alerts the nurse to the possibility of right-sided heart failure? a. "I sleep with four pillows at night." b. "My shoes fit really tight lately." c. "I wake up coughing every night." d. "I have trouble catching my breath."

ANS: B Signs of systemic congestion occur with right-sided heart failure. Fluid is retained, pressure builds in the venous system, and peripheral edema develops. Left-sided heart failure symptoms include respiratory symptoms. Orthopnea, coughing, and difficulty breathing all could be results of left-sided heart failure. DIF: Understanding/Comprehension REF: 683 KEY: Heart failure| assessment/diagnostic examination MSC: IntegratedProcess:NursingProcess:Assessment NOT: Client Needs Category: Health Promotion and Maintenance

7. A student nurse is preparing morning medications for a client who had a stroke. The student plans to hold the docusate sodium (Colace) because the client had a large stool earlier. What action by the supervising nurse is best? a. Have the student ask the client if it is desired or not. b. Inform the student that the docusate should be given. c. Tell the student to document the rationale. d. Tell the student to give it unless the client refuses.

ANS: B Stool softeners should be given to clients with neurologic disorders in order to prevent an elevation in intracranial pressure that accompanies the Valsalva maneuver when constipated. The supervising nurse should instruct the student to administer the docusate. The other options are not appropriate. The medication could be held for diarrhea. DIF: Applying/Application REF: 942 KEY: Neurologic disorders| stool softeners| constipation| intracranial pressure MSC: IntegratedProcess:Teaching/Learning NOT: Client Needs Category: Physiological Integrity: Reduction of Risk Potential

13. A nurse cares for a client recovering from prosthetic valve replacement surgery. The client asks, "Why will I need to take anticoagulants for the rest of my life?" How should the nurse respond? a. "The prosthetic valve places you at greater risk for a heart attack." b. "Blood clots form more easily in artificial replacement valves." c. "The vein taken from your leg reduces circulation in the leg." d. "The surgery left a lot of small clots in your heart and lungs."

ANS: B Synthetic valve prostheses and scar tissue provide surfaces on which platelets can aggregate easily and initiate the formation of blood clots. The other responses are inaccurate. DIF: Applying/Application REF: 696 KEY: Valve disorder| patient education| anticoagulants MSC: IntegratedProcess:Teaching/Learning NOT: Client Needs Category: Physiological Integrity: Reduction of Risk Potential

10. A client's mean arterial pressure is 60 mm Hg and intracranial pressure is 20 mm Hg. Based on the client's cerebral perfusion pressure, what should the nurse anticipate for this client? a. Impending brain herniation b. Poor prognosis and cognitive function c. Probable complete recovery d. Unable to tell from this information

ANS: B The cerebral perfusion pressure (CPP) is the intracranial pressure subtracted from the mean arterial pressure: in this case, 60 - 20 = 40. For optimal outcomes, CPP should be at least 70 mm Hg. This client has very low CPP, which will probably lead to a poorer prognosis with significant cognitive dysfunction should the client survive. This data does not indicate impending brain herniation or complete recovery. DIF: Analyzing/Analysis REF: 949 KEY: Neurologic disorders| nursing assessment| neurologic assessment MSC: Integrated Process: Nursing Process: Analysis NOT: Client Needs Category: Physiological Integrity: Physiological Adaptation

6. When assessing a client who had a traumatic brain injury, the nurse notes that the client is drowsy but easily aroused. What level of consciousness will the nurse document to describe this client's current level of consciousness? a. Alert b. Lethargic c. Stuporous d. Comatose

ANS: B The client is categorized as being lethargic because he or she can be easily aroused even though drowsy. The nurse would carefully monitor the client to determine any decrease in the level of consciousness (LOC).

16. A nurse assesses a client with pericarditis. Which assessment finding should the nurse expect to find? a. Heart rate that speeds up and slows down b. Friction rub at the left lower sternal border c. Presence of a regular gallop rhythm d. Coarse crackles in bilateral lung bases

ANS: B The client with pericarditis may present with a pericardial friction rub at the left lower sternal border. This sound is the result of friction from inflamed pericardial layers when they rub together. The other assessments are not related. DIF: Remembering/Knowledge REF: 699 KEY: Inflammatory response| assessment/diagnostic examination MSC: IntegratedProcess:NursingProcess:Assessment NOT: Client Needs Category: Physiological Integrity: Physiological Adaptation

2. The nurse is evaluating a 3-day diet history with a client who has an elevated lipid panel. What meal selection indicates that the client is managing this condition well with diet? a. A 4-ounce steak, French fries, iceberg lettuce b. Baked chicken breast, broccoli, tomatoes c. Fried catfish, cornbread, peas d. Spaghetti with meat sauce, garlic bread

ANS: B The diet recommended for this client would be low in saturated fats and red meat, high in vegetables and whole grains (fiber), low in salt, and low in trans fat. The best choice is the chicken with broccoli and tomatoes. The French fries have too much fat and the iceberg lettuce has little fiber. The catfish is fried. The spaghetti dinner has too much red meat and no vegetables.

16. A nurse assesses a patient who is recovering from a lumbar puncture (LP). Which complication of this procedure would alert the nurse to urgently contact the primary health care provider? a. Weak pedal pulses b. Nausea and vomiting c. Increased thirst d. Hives on the chest

ANS: B The nurse would immediately contact the provider if the client experiences a severe headache, nausea, vomiting, photophobia, or a change in level of consciousness after an LP, which are all signs of increased intracranial pressure. Weak pedal pulses, increased thirst, and hives are not complications of an LP.

5. A client tells the oncology nurse about an upcoming vacation to the beach to celebrate completing radiation treatments for cancer. What response by the nurse is most appropriate? a. Avoid getting salt water on the radiation site. b. Do not expose the radiation area to direct sunlight. c. Have a wonderful time and enjoy your vacation! d. Remember you should not drink alcohol for a year.

ANS: B The skin overlying the radiation site is extremely sensitive to sunlight after radiation therapy has been completed. The nurse should inform the client to avoid sun exposure to this area. This advice continues for 1 year after treatment has been completed. The other statements are not appropriate.

18. A nurse teaches a client recovering from a heart transplant who is prescribed cyclosporine (Sandimmune). Which statement should the nurse include in this client's discharge teaching? a. "Use a soft-bristled toothbrush and avoid flossing." b. "Avoid large crowds and people who are sick." c. "Change positions slowly to avoid hypotension." d. "Check your heart rate before taking the medication."

ANS: B These agents cause immune suppression, leaving the client more vulnerable to infection. The medication does not place the client at risk for bleeding, orthostatic hypotension, or a change in heart rate. DIF: Applying/Application REF: 703 KEY: Transplant| immune suppressant MSC: Integrated Process: Nursing Process: Implementation NOT: Client Needs Category: Physiological Integrity: Pharmacological and Parenteral Therapies

19. After a craniotomy, the nurse assesses the client and finds dry, sticky mucous membranes and restlessness. The client has IV fluids running at 75 mL/hr. What action by the nurse is best? a. Assess the client's magnesium level. b. Assess the client's sodium level. c. Increase the rate of the IV infusion. d. Provide oral care every hour.

ANS: B This client has manifestations of hypernatremia, which is a possible complication after craniotomy. The nurse should assess the client's serum sodium level. Magnesium level is not related. The nurse does not independently increase the rate of the IV infusion. Providing oral care is also a good option but does not take priority over assessing laboratory results. DIF: Applying/Application REF: 961 KEY: Neurologic disorders| fluid and electrolyte imbalances| nursing assessment MSC: IntegratedProcess:NursingProcess:Assessment NOT: Client Needs Category: Physiological Integrity: Reduction of Risk Potential

1. A client is in the emergency department reporting a brief episode during which he was dizzy, unable to speak, and felt like his legs were very heavy. Currently the client's neurologic examination is normal. About what drug should the nurse plan to teach the client? a. Alteplase (Activase) b. Clopidogrel (Plavix) c. Heparin sodium d. Mannitol (Osmitrol)

ANS: B This client's manifestations are consistent with a transient ischemic attack, and the client would be prescribed aspirin or clopidogrel on discharge. Alteplase is used for ischemic stroke. Heparin and mannitol are not used for this condition. DIF: Remembering/Knowledge REF: 930 KEY: Neurologic disorders| antiplatelet medications| patient education MSC: IntegratedProcess:Teaching/Learning NOT: Client Needs Category: Physiological Integrity: Pharmacological and Parenteral Therapies

22. A client is having a catheter placed in the femoral artery to deliver yttrium-90 beads into a liver tumor. What action by the nurse is most important? a. Assessing the clients abdomen beforehand b. Ensuring that informed consent is on the chart c. Marking the clients bilateral pedal pulses d. Reviewing client teaching done previously

ANS: B This is an invasive procedure requiring informed consent. The nurse should ensure that consent is on the chart. The other actions are also appropriate but not the priority.

A client has been placed on Contact Precautions. The client's family is very afraid to visit for fear of being "contaminated" by the client. What action by the nurse is best? a. Explain to them that these precautions are mandated by law. b. Show the family how to avoid spreading the disease. c. Reassure the family that they will not get the infection. d. Tell the family it is important that they visit the client.

ANS: B Visitors may be apprehensive about visiting a client in Transmission-Based Precautions. The nurse would reassure the visitors that taking appropriate precautions will minimize their risks. The nurse would then demonstrate what precautions were needed. The other options do nothing to ease the family's fears.

20. A client has a deep vein thrombosis (DVT). What comfort measure does the nurse delegate to the assistive personnel (AP)? a. Ambulate the client. b. Apply a warm moist pack. c. Massage the client's leg. d. Provide an ice pack.

ANS: B Warm moist packs will help with the pain of a DVT. Ambulation is not a comfort measure. Massaging the client's legs is contraindicated to prevent complications such as pulmonary embolism. Ice packs are not recommended for DVT.

The nurse is teaching participants in a family-oriented community center ways to prevent their older relatives and friends from getting heat-related illnesses. What information does the nurse include? (Select all that apply.) a. Use sunscreen with an SPF of at least 15 when outdoors. b. Take cool baths or showers after outdoor activities. c. Check on the older adult daily in hot weather. d. Drink plenty of liquids throughout the day. e. Wear light-colored, snugly-fitting clothing to wick sweat away.

ANS: B, C, D To best prevent heat-related illnesses, the nurse would teach individuals to use sunscreen with at least an SPF of 30 for both UVA and UVB rays, to shower or bathe in cool water after being outdoors to reduce body heat, to remain hydrated, and to wear light-colored, loose-fitting clothes. Families and friends should check older adults at least twice a day during a heat wave; however, this may not prevent heat-related illness but could catch it quickly and limit its severity.

The nurse caring for clients understands that which factors must be present to transmit infection? (Select all that apply.) a. Colonization b. Host c. Mode of transmission d. Portal of entry e. Reservoir f. Poor hygiene

ANS: B, C, D, E Factors that must be present in order to transmit an infection include a host with a portal of entry, a mode of transmission, and a reservoir. Colonization is not one of these factors. Poor hygiene may or may not contribute to infection.

1. The student nurse caring for clients who have cancer understands that the general consequences of cancer include which client problems? (Select all that apply.) a. Clotting abnormalities from thrombocythemia b. Increased risk of infection from white blood cell deficits c. Nutritional deficits such as early satiety and cachexia d. Potential for reduced gas exchange e. Various motor and sensory deficits

ANS: B, C, D, E The general consequences of cancer include reduced immunity and blood-producing functions, altered GI structure and function, decreased respiratory function, and motor and sensory deficits. Clotting problems often occur due to thrombocytopenia (not enough platelets), not thrombocythemia (too many platelets).

The nurse is teaching a client with mild rheumatoid arthritis (RA) about how to protect synovial joints. Which health teaching will the nurse include? (Select all that apply.) a. "Use small joints rather than larger ones during tasks." b. "Use both hands instead of one with holding objects." c. "When getting out of bed or a chair, use the palms of your hands." d. "Bend your knees instead of your waist and keep your back straight." e. "Do not use multiple pillows under your head to prevent neck flexion." f. "Use a device or rubber grip to open jars or bottle tops." g. "Use long-handled devices such as a hairbrush with an extended handle."

ANS: B, C, D, E, F, G All of these options are part of health teaching for joint protection except that large joints should be used instead of smaller ones.

9. A nurse assesses a client who is diagnosed with infective endocarditis. Which assessment findings should the nurse expect? (Select all that apply.) a. Weight gain b. Night sweats c. Cardiac murmur d. Abdominal bloating e. Osler's nodes

ANS: B, C, E Clinical manifestations of infective endocarditis include fever with chills, night sweats, malaise and fatigue, anorexia and weight loss, cardiac murmur, and Osler's nodes on palms of the hands and soles of the feet. Abdominal bloating is a manifestation of heart transplantation rejection. DIF: Remembering/Knowledge REF: 697 KEY: Endocarditis MSC: IntegratedProcess:NursingProcess:Assessment NOT: Client Needs Category: Physiological Integrity: Physiological Adaptation

The nurse assesses a client with long-term rheumatoid arthritis (RA) for late signs and symptoms. Which assessment findings will the nurse document as late signs and symptoms of RA? (Select all that apply.) a. Anorexia b. Felty syndrome c. Joint deformity d. Low-grade fever e. Weight loss

ANS: B, C, E Late signs and symptoms of RA include Felty syndrome, joint deformity, weight loss, organ involvement, osteoporosis, extreme fatigue, and anemia, among others. Anorexia and low-grade fever are both seen early in the course of the disease.

A nurse plans care for a client who is at risk for infection. Which interventions will the nurse implement to prevent infection? (Select all that apply.) a. Administer prophylactic antibiotics. b. Monitor white blood cell count and differential. c. Screen all visitors for infections. d. Implement Transmission-Based Precautions. e. Promote sufficient nutritional intake.

ANS: B, C, E Nursing interventions for clients at risk for infection include monitoring white blood cell count and differential, screening visitors for infections and infectious disease, and promoting sufficient nutritional intake. Standard Precautions are required but not Transmission-Based Precautions. Prophylactic antibiotics are not generally used to prevent infections.

A client with an infection has a fever. What actions by the nurse help increase the client's comfort? (Select all that apply.) a. Administer antipyretics around the clock. b. Change the client's gown and linens when damp. c. Offer cool fluids to the client frequently. d. Place ice bags in the armpits and groin. e. Provide a fan to help cool the client. f. Sponging the client with tepid water.

ANS: B, C, F Comfort measures appropriate for this client include offering frequent cool drinks, and changing linens or the gown when damp. Fever is a defense mechanism, and antipyretics will be administered only when the client is uncomfortable. Ice bags can help cool the client quickly but are not comfort measures. Fans are discouraged because they can disperse microbes. Sponging the client's body with tepid water is also helpful.

11. A nurse is dismissing a client from the emergency department who has a mild traumatic brain injury. What information obtained from the client represents a possible barrier to self-management? (Select all that apply.) a. Does not want to purchase a thermometer b. Is allergic to acetaminophen (Tylenol) c. Laughing, says "Strenuous? What's that?" d. Lives alone and is new in town with no friends e. Plans to have a beer and go to bed once home

ANS: B, D, E Clients should take acetaminophen for headache. An allergy to this drug may mean the client takes aspirin or ibuprofen (Motrin), which should be avoided. The client needs neurologic checks every 1 to 2 hours, and this client does not seem to have anyone available who can do that. Alcohol needs to be avoided for at least 24 hours. A thermometer is not needed. The client laughing at strenuous activity probably does not engage in any kind of strenuous activity, but the nurse should confirm this. DIF: Evaluating/Synthesis REF: 957 KEY: Neurologic disorders| patient education MSC: Integrated Process: Nursing Process: Evaluation NOT: Client Needs Category: Health Promotion and Maintenance

3. A nurse is caring for a client after a stroke. What actions may the nurse delegate to the unlicensed assistive personnel (UAP)? (Select all that apply.) a. Assess neurologic status with the Glasgow Coma Scale. b. Check and document oxygen saturation every 1 to 2 hours. c. Cluster client care to allow periods of uninterrupted rest. d. Elevate the head of the bed to 45 degrees to prevent aspiration. e. Position the client supine with the head in a neutral midline position.

ANS: B, E The UAP can take and document vital signs, including oxygen saturation, and keep the client's head in a neutral, midline position with correct direction from the nurse. The nurse assesses the Glasgow Coma Scale score. The nursing staff should not cluster care because this can cause an increase in the intracranial pressure. The head of the bed should be minimally elevated, up to 30 degrees. DIF: Applying/Application REF: 938 KEY: Neurologic disorders| stroke| delegation| unlicensed assistive personnel (UAP) MSC: Integrated Process: Communication and Documentation NOT: Client Needs Category: Safe and Effective Care Environment: Management of Care

1. What nonpharmacologic comfort measures would the nurse include in the plan of care for a client with severe varicose veins? (Select all that apply.) a. Administering mild analgesics for pain b. Applying elastic compression stockings c. Elevating the legs when sitting or lying d. Reminding the client to do leg exercises e. Teaching the client about surgical options f. Encouraging participation in high impact aerobic activity

ANS: B,C,D The three Es of care for varicose veins include elastic compression hose, exercise, and elevation. Mild analgesics are not a nonpharmacologic measure. Teaching about surgical options is not a comfort measure. High impact aerobics is not encouraged and is not a comfort measure.

6. A client with HIV-III is in the hospital with severe diarrhea. What actions does the nurse delegate to assistive personnel (AP)? (Select all that apply.) a. Assessing the client's fluid and electrolyte status b. Assisting the client to get out of bed to prevent falls c. Obtaining a bedside commode if the client is weak d. Providing gentle perianal cleansing after stools e. Reporting any perianal abnormalities

ANS: B,C,D,E The AP can assist the client with getting out of bed, obtain a bedside commode for the client's use, cleanse the client's perianal area after bowel movements, and report any abnormal observations such as redness or open are as the nurse assesses fluid and electrolyte status.

8. A nurse is providing education about HIV risks at a health fair. What groups would the nurse include as needing to be tested for HIV on an annual basis? (Select all that apply.) a. Anyone who received a blood product in 1989 b. Couples planning on getting married c. Those who are sexually active with multiple partners d. Injection drugs users e. Sex workers and their customers f. Adults over the age of 65 years

ANS: B,C,D,E The CDC recommends that HIV testing would be performed on those who received a transfusion between 1978 and 1985 only. People planning on getting married should be tested and all sexually active people should know their HIV status. Those engaged in sex work and their customers should also be tested, as well as injection drug users. Those over the age of 65 years need a one-time screen.

2. An 84-year-old client who is usually alert and oriented experiences an acute cognitive decline. Which of the following factors would the nurse anticipate as contributing to this neurologic change? (Select all that apply.) a. Chronic hearing loss b. Infection c. Drug toxicity d. Dementia e. Hypoxia f. Aging

ANS: B,C,E Acute client conditions that occur in older adults often cause acute confusion and associated emotional behaviors. Infection, drug toxicity, and hypoxia are all acute health problems that can contribute to the client's cognitive decline. Aging does not cause changes in cognition. If the client had dementia, he or she would not be alert and oriented. Having a chronic hearing loss is not a change in the client's condition.

5. A client with HIV-III has oral thrush and difficulty eating. What actions does the nurse delegate to the assistive personnel (AP)? (Select all that apply.) a. Apply oral anesthetic gels before meals. b. Assist the client with oral care every 2 hours. c. Offer the client frequent sips of cool drinks. d. Provide the client with alcohol-based mouthwash. e. Remind the client to use only a soft toothbrush. f. Offer the client soft foods like gelatin or pudding.

ANS: B,C,E,F The AP can help the client with oral care, offer fluids, and remind the client of things the nurse (or other professional) has already taught. Soft foods and liquids are tolerated better than harder foods. Applying medications is performed by the nurse. Alcohol-based mouthwashes are harsh and drying and would not be used.

9. A client with a known abdominal aortic aneurysm reports dizziness and severe abdominal pain. The nurse assesses the client's blood pressure at 82/40 mm Hg. What actions by the nurse are most important? (Select all that apply.) a. Administer pain medication. b. Assess distal pulses every 10 minutes. c. Have the client sign a surgical consent. d. Notify the Rapid Response Team. e. Take vital signs every 10 minutes.

ANS: B,D,E This client may have a ruptured/rupturing aneurysm. The nurse would notify the Rapid Response team and perform frequent client assessments. Giving pain medication will lower the client's blood pressure even further. The nurse cannot have the client sign a consent until the surgeon has explained the procedure.

4. A nurse assesses an older client. Which assessment findings would the nurse identify as normal changes in the nervous system related to aging? (Select all that apply.) a. Long-term memory loss b. Slower processing time c. Increased sensory perception d. Decreased risk for infection e. Change in sleep patterns

ANS: B,E Normal changes in the nervous system related to aging include recent memory loss, slower processing time, decreased sensory perception, an increased risk for infection, changes in sleep patterns, changes in perception of pain, and altered balance and/or decreased coordination.

10. A client presents to the emergency department with a thoracic aortic aneurysm. Which findings are most consistent with this condition? (Select all that apply.) a. Abdominal tenderness b. Difficulty swallowing c. Changes in bowel habits d. Shortness of breath e. Hoarseness

ANS: B,E Signs of a thoracic aortic aneurysm include shortness of breath, hoarseness, and difficulty swallowing. Pain is often rated as a 10 on a 10-point scale. Bowel habits are not related.

A hospice nurse is caring for a variety of clients who are dying. Which end-of-life and death ritual is paired with the correct religion? a. Roman Catholic Autopsies are not allowed except under special circumstances. b. Christian Upon death, a religious leader should perform rituals of bathing and wrapping the body in cloth. c. Judaism A person who is extremely ill and dying should not be left alone. d. Islam An ill or dying person should receive the Sacrament of the Sick.

ANS: C According to Jewish law, a person who is extremely ill or dying should not be left alone. Orthodox Jews do not allow autopsies except under special circumstances. The Islamic faith requires a religious leader to perform rituals of bathing and wrapping the body in cloth upon death. A Catholic priest performs the Sacrament of the Sick for ill or dying people.

The nurse is teaching a client who is planning to have a total hip arthroplasty. What statement by the client indicates a need for further teaching? a. "I will get an IV antibiotic right before surgery to prevent infection." b. "I may request a regional nerve block as part of the surgical anesthesia." c. "I will receive IV heparin before surgery to decrease the risk of clots." d. "I will receive tranexamic acid to help reduce blood loss during surgery."

ANS: C All of the choices are correct except that IV heparin is not given before or after surgery. A different anticoagulant is given after surgery to prevent postoperative venous thromboembolism, such as deep vein thrombosis and pulmonary embolus.

A client is prescribed celecoxib for joint pain. What statement by the client indicates a need for further teaching? a. "I'll report any signs of bleeding or bruising to my primary health care provider." b. "I'll take this drug only as prescribed by my primary health care provider." c. "I'll be sure to take this drug three times a day only on an empty stomach." d. "I'll monitor the amount of urine that I excrete every day and report any changes."

ANS: C All of the choices are correct for this NSAID except that celecoxib can cause GI distress unless taken with meals or food. The drug should not be taken on an empty stomach and is rarely taken more than twice a day.

A client has a left knee arthrocentesis to remove excess joint fluid. What postprocedure health teaching will the nurse include? a. "Take your opioid medication as prescribed by the primary health care provider." b. "Do not bear weight on your left leg for at least a week after you get home." c. "Monitor the site for bleeding or clear fluid leakage when you are home." d. "Tell your employer that you can't come back to work for 2 to 3 weeks."

ANS: C An arthrocentesis is performed as an ambulatory procedure and may require a mild analgesic such as acetaminophen for discomfort. Opioids are not used. The client may bear weight and return to work, but needs to monitor for bleeding or leakage of synovial fluid at the injection site.

17. After teaching a client who is recovering from a heart transplant to change positions slowly, the client asks, "Why is this important?" How should the nurse respond? a. "Rapid position changes can create shear and friction forces, which can tear out your internal vascular sutures." b. "Your new vascular connections are more sensitive to position changes, leading to increased intravascular pressure and dizziness." c. "Your new heart is not connected to the nervous system and is unable to respond to decreases in blood pressure caused by position changes." d. "While your heart is recovering, blood flow is diverted away from the brain, increasing the risk for stroke when you stand up."

ANS: C Because the new heart is denervated, the baroreceptor and other mechanisms that compensate for blood pressure drops caused by position changes do not function. This allows orthostatic hypotension to persist in the postoperative period. The other options are false statements and do not correctly address the client's question. DIF: Understanding/Comprehension REF: 703 KEY: Transplant| patient education MSC: IntegratedProcess:NursingProcess:Implementation NOT: Client Needs Category: Physiological Integrity: Physiological Adaptation

13. A nurse is teaching a client with cerebellar function impairment. Which statement would the nurse include in this client's discharge teaching? a. "Connect a light to flash when your door bell rings." b. "Label your faucet knobs with hot and cold signs." c. "Ask a friend to drive you to your follow-up appointments." d. "Use a natural gas detector with an audible alarm."

ANS: C Cerebellar function enables the client to predict distance or gauge the speed with which one is approaching an object, control voluntary movement, maintain equilibrium, and shift from one skilled movement to another in an orderly sequence. A client who has cerebellar function impairment should not be driving. The client would not have difficulty hearing, distinguishing between hot and cold, or smelling.

2. A nurse assesses a client in an outpatient clinic. Which statement alerts the nurse to the possibility of left-sided heart failure? a. "I have been drinking more water than usual." b. "I am awakened by the need to urinate at night." c. "I must stop halfway up the stairs to catch my breath." d. "I have experienced blurred vision on several occasions."

ANS: C Clients with left-sided heart failure report weakness or fatigue while performing normal activities of daily living, as well as difficulty breathing, or "catching their breath." This occurs as fluid moves into the alveoli. Nocturia is often seen with right-sided heart failure. Thirst and blurred vision are not related to heart failure. DIF: Understanding/Comprehension REF: 682 KEY: Heart failure| assessment/diagnostic examination MSC: IntegratedProcess:NursingProcess:Assessment NOT: Client Needs Category: Health Promotion and Maintenance

21. A nurse is assessing an obese client in the clinic for follow-up after an episode of deep vein thrombosis. The client has lost 20 lb (9.09 Kg) since the last visit. What action by the nurse is best? a. Ask if the weight loss was intended. b. Encourage a high-protein, high-fiber diet. c. Measure for new compression stockings. d. Review a 3-day food recall diary.

ANS: C Compression stockings must fit correctly in order to work. After losing a significant amount of weight, the client would be remeasured and new stockings ordered if needed. The other options are appropriate, but not the most important.

10. A nurse plans care for a 77-year-old client who is experiencing age-related peripheral sensory perception changes. Which intervention would the nurse include in this client's plan of care? a. Provide a call button that requires only minimal pressure to activate. b. Write the date on the client's white board to promote orientation. c. Ensure that the path to the bathroom is free from clutter. d. Encourage the client to season food to stimulate nutritional intake.

ANS: C Dementia and confusion are not common phenomena in older adults. However, physical impairment related to illness can be expected. Providing opportunities for hazard-free ambulation will maintain strength and mobility (and ensure safety). Providing a call button, providing the date, and seasoning food do not address the client's impaired sensory perception.

5. A client asks what "essential hypertension" is. What response by the registered nurse is best? a. "It means it is caused by another disease." b. "It means it is 'essential' that it be treated." c. "It is hypertension with no specific cause." d. "It refers to severe and life-threatening hypertension."

ANS: C Essential hypertension is the most common type of hypertension and has no specific cause such as an underlying disease process. Hypertension that is due to another disease process is called secondary hypertension. A severe, life-threatening form of hypertension is malignant hypertension.

A nurse is observing as an assistive personnel (AP) performs hygiene and provides comfort measures to a client with an infection. What action by the AP requires intervention by the nurse? a. Not using gloves while combing the client's hair b. Rinsing the client's commode pan after use c. Ordering an oscillating fan for the client d. Wearing gloves when providing perianal care

ANS: C Fans in client care areas are discouraged because they can disperse airborne or droplet-borne pathogens. The other actions are appropriate. If the client has a scalp infection or infestation, the AP will wear gloves; otherwise, it is not required for grooming the hair.

8. A client experiences impaired swallowing after a stroke and has worked with speech-language pathology on eating. What nursing assessment best indicates that a priority goal for this problem has been met? a. Chooses preferred items from the menu b. Eats 75% to 100% of all meals and snacks c. Has clear lung sounds on auscultation d. Gains 2 pounds after 1 week

ANS: C Impaired swallowing can lead to aspiration, so the priority goal for this problem is no aspiration. Clear lung sounds is the best indicator that aspiration has not occurred. Choosing menu items is not related to this problem. Eating meals does not indicate the client is not still aspirating. A weight gain indicates improved nutrition but still does not show a lack of aspiration. DIF: Evaluating/Synthesis REF: 942 KEY: Neurologic disorders| stroke| aspiration MSC: IntegratedProcess:NursingProcess:Evaluation NOT: Client Needs Category: Physiological Integrity: Reduction of Risk Potential

The primary health care provider prescribes methotrexate (MTX) for a client with a new diagnosis of rheumatoid arthritis. The nurse provides health teaching about the drug. What statement by the nurse is appropriate to include about methotrexate? a. "It will take at least 1 to 2 weeks for the drug to help relieve your symptoms." b. "The drug is very expensive but there are pharmacy plans to help pay for it." c. "The drug can increase your risk for infection, so you should avoid crowds." d. "It's OK for you to drink about 2 to 3 glasses of wine each week while taking the drug."

ANS: C MTX takes up to 4 to 6 weeks to begin to help relieve RA symptoms and is very inexpensive. Clients should avoid alcohol due to the potential for liver toxicity. MTX suppresses the immune system which makes clients susceptible to infection. The nurse teaches clients to avoid crowds and anyone with a known infection.

16. A client with HIV-III asks the nurse why gabapentin is part of the drug regimen when the client does not have a history of seizures. What response by the nurse is best? a. "Gabapentin can be used as an antidepressant too." b. "I have no idea why you would be taking this drug." c. "This drug helps treat the pain from nerve irritation." d. "You are at risk for seizures due to fungal infections."

ANS: C Many classes of medications are used for neuropathic pain, including tricyclic antidepressants and anticonvulsants such as gabapentin. It is not being used to prevent seizures from fungal infections. If the nurse does not know the answer, he or she would find out for the client.

1. The nurse teaches an 80-year-old client with diminished peripheral sensation. Which statement would the nurse include in this client's teaching? a. "Place soft rugs in your bathroom to decrease pain in your feet." b. "Bathe in warm water to increase your circulation." c. "Look at the placement of your feet when walking." d. "Walk barefoot to decrease pressure injuries from your shoes."

ANS: C Older clients with decreased sensation are at risk of injury from the inability to sense changes in terrain when walking. To compensate for this loss, the client is instructed to look at the placement of his or her feet when walking. Throw rugs can slip and increase fall risk. Bath water that is too warm places the client at risk for thermal injury.

A client has long-term rheumatoid arthritis that especially affects the hands. The client wants to finish quilting a baby blanket before the birth of her grandchild. What response by the nurse is appropriate? a. "Let's ask your provider about increasing your pain pills." b. "Hold ice bags against your hands before quilting." c. "Try a paraffin wax dip 20 minutes before you quilt." d. "You need to stop quilting before it destroys your fingers."

ANS: C Paraffin wax dips are beneficial for decreasing pain in arthritic hands and lead to increased mobility. Increasing pain pills may not help with movement. Ice has limited use unless the client has a "hot" or exacerbated joint. The client wants to finish the project, so the nurse would not negate its importance by telling the client it is destroying her joints.

14. A client is receiving rituximab (Rituxan) and asks how it works. What response by the nurse is best? a. It causes rapid lysis of the cancer cell membranes. b. It destroys the enzymes needed to create cancer cells. c. It prevents the start of cell division in the cancer cells. d. It sensitizes certain cancer cells to chemotherapy.

ANS: C Rituxan prevents the initiation of cancer cell division. The other statements are not accurate.

14. A client is receiving rituximab and asks how it works. What response by the nurse is best? a. "It causes rapid lysis of the cancer cell membranes." b. "It destroys the enzymes needed to create cancer cells." c. "It prevents the start of cell division in the cancer cells." d. "It sensitizes certain cancer cells to chemotherapy."

ANS: C Rituximab prevents the initiation of cancer cell division. The other statements are not accurate.

4. A client with known HIV-II is admitted to the hospital with fever, night sweats, and severe cough. Laboratory results include a CD4+ cell count of 180/mm3 and a negative tuberculosis (TB) skin test 4 days ago. What action would the nurse take first? a. Initiate Droplet Precautions for the client. b. Notify the primary health care provider about the CD4+ results. c. Place the client under Airborne Precautions. d. Use Standard Precautions to provide care.

ANS: C Since this client's CD4+ cell count is so low, he or she may have energy, or the inability to mount an immune response to the TB test. The client also appears to have progressed to HIV-III. The nurse would first place the client on Airborne Precautions to prevent the spread of TB if it is present. Next the nurse notifies the primary health care provider about the low CD4+ count and requests alterative testing for TB. Droplet Precautions are not used for TB. Standard Precautions are not adequate in this case.

15. Four clients are receiving tyrosine kinase inhibitors (TKIs). Which of these four clients should the nurse assess first? a. Client with dry, itchy, peeling skin b. Client with a serum calcium of 9.2 mg/dL c. Client with a serum potassium of 2.8 mEq/L d. Client with a weight gain of 0.5 pound (1.1 kg) in 1 day

ANS: C TKIs can cause electrolyte imbalances. This potassium level is very low, so the nurse should assess this client first. Dry, itchy, peeling skin can be a problem in clients receiving biologic response modifiers, and the nurse should assess that client next because of the potential for discomfort and infection. This calcium level is normal. TKIs can also cause weight gain, but the client with the low potassium level is more critical.

18. A nurse assesses a client with a brain tumor. The client opens his eyes when the nurse calls his name, mumbles in response to questions, and follows simple commands. How would the nurse document this client's assessment using the Glasgow Coma Scale shown below? a. 8 b. 10 c. 12 d. 14

ANS: C The client opens his eyes to speech (Eye Opening: To sound = 3), mumbles in response to questions (Verbal Response: Inappropriate words = 3), and follows simple commands (Motor Response: Obeys commands = 6). Therefore, the client's Glasgow Coma Scale score is 3 + 3 + 6 = 12.

26. A nurse is caring for four clients in the neurologic/neurosurgical intensive care unit. Which client should the nurse assess first? a. Client who has been diagnosed with meningitis with a fever of 101° F (38.3° C) b. Client who had a transient ischemic attack and is waiting for teaching on clopidogrel (Plavix) c. Client receiving tissue plasminogen activator (t-PA) who has a change in respiratory pattern and rate d. Client who is waiting for subarachnoid bolt insertion with the consent form already signed

ANS: C The client receiving t-PA has a change in neurologic status while receiving this fibrinolytic therapy. The nurse assesses this client first as he or she may have an intracerebral bleed. The client with meningitis has expected manifestations. The client waiting for discharge teaching is a lower priority. The client waiting for surgery can be assessed quickly after the nurse sees the client who is receiving t-PA, or the nurse could delegate checking on this client to another nurse. DIF: Analyzing/Analysis REF: 938 KEY: Neurologic disorders| critical rescue| nursing assessment MSC: IntegratedProcess:NursingProcess:Assessment NOT: Client Needs Category: Safe and Effective Care Environment: Management of Care

An intensive care nurse discusses withdrawal of care with a clients family. The family expresses concerns related to discontinuation of therapy. How should the nurse respond? a. I understand your concerns, but in this state, discontinuation of care is not a form of active euthanasia. b. You will need to talk to the provider because I am not legally allowed to participate in the withdrawal of life support. c. I realize this is a difficult decision. Discontinuation of therapy will allow the client to die a natural death. d. There is no need to worry. Most religious organizations support the clients decision to stop medical treatment.

ANS: C The nurse should validate the familys concerns and provide accurate information about the discontinuation of therapy. The other statements address specific issues related to the withdrawal of care but do not provide appropriate information about their purpose. If the clients family asks for specific information about euthanasia, legal, or religious issues, the nurse should provide unbiased information about these topics.

30. A client has a traumatic brain injury and a positive halo sign. The client is in the intensive care unit, sedated and on a ventilator, and is in critical but stable condition. What collaborative problem takes priority at this time? a. Inability to communicate b. Nutritional deficit c. Risk for acquiring an infection d. Risk for skin breakdown

ANS: C The positive halo sign indicates a leak of cerebrospinal fluid. This places the client at high risk of acquiring an infection. Communication and nutrition are not priorities compared with preventing a brain infection. The client has a definite risk for a skin breakdown, but it is not the immediate danger a brain infection would be. DIF: Applying/Application REF: 952 KEY: Neurologic disorders| infection control| asepsis MSC: Integrated Process: Nursing Process: Analysis NOT: Client Needs Category: Physiological Integrity: Reduction of Risk Potential

The nurse is caring for a client who has severe osteoarthritis. What primary joint problems will the nurse expect the client to report? a. Crepitus b. Effusions c. Pain d. Deformities

ANS: C The primary assessment finding typically reported by clients who have osteoarthritis is joint pain, although crepitus, effusions (fluid), and mild deformities may occur.

6. A client had an embolectomy for an arteriovenous malformation (AVM). The client is now reporting a severe headache and has vomited. What action by the nurse takes priority? a. Administer pain medication. b. Assess the client's vital signs. c. Notify the Rapid Response Team. d. Raise the head of the bed.

ANS: C This client may be experiencing a rebleed from the AVM. The most important action is to call the Rapid Response Team as this is an emergency. The nurse can assess vital signs while someone else notifies the Team, but getting immediate medical attention is the priority. Administering pain medication may not be warranted if the client must return to surgery. The optimal position for the client with an AVM has not been determined, but calling the Rapid Response Team takes priority over positioning. DIF: Applying/Application REF: 941 KEY: Neurologic disorders| critical rescue| Rapid Response Team| communication MSC: Integrated Process: Communication and Documentation NOT: Client Needs Category: Safe and Effective Care Environment: Management of Care

After a total knee arthroplasty, a client is on the postoperative nursing unit with a continuous femoral nerve blockade. On assessment, the nurse notes the skin of both legs is pale pink, warm, and dry, but the client is unable to dorsiflex or plantarflex the surgical foot. What action would the nurse take next? a. Document the findings and monitor as prescribed. b. Increase the frequency of monitoring the client. c. Notify the surgeon or anesthesia provider immediately. d. Palpate the client's bladder or perform a bladder scan.

ANS: C With the femoral nerve block, the client would still be able to dorsiflex and plantarflex the affected surgical foot. Since this client has an abnormal finding, the nurse would notify either the surgeon or the anesthesia provider immediately. Documentation is the last priority. Increasing the frequency of assessment may be appropriate, but first the nurse must notify the appropriate provider. Palpating the bladder is not related.

6. Which statements by the client indicate good understanding of foot care in peripheral vascular disease? (Select all that apply.) a. "A good abrasive pumice stone will keep my feet soft." b. "I'll always wear shoes if I can buy cheap flip-flops." c. "I will keep my feet dry, especially between the toes." d. "Lotion is important to keep my feet smooth and soft." e. "Washing my feet in room-temperature water is best." f. "I will inspect my feet daily."

ANS: C,D,E Good foot care includes appropriate hygiene and injury prevention. Keeping the feet dry; wearing good, comfortable shoes; using lotion; washing the feet in room-temperature water; cutting the nails straight across; and inspecting the feet daily are all important measures. Abrasive material such as pumice stones would not be used. Cheap flip-flops may not fit well and won't offer much protection against injury.

8. A nurse assesses a client after administering isosorbide mononitrate (Imdur). The client reports a headache. Which action should the nurse take? a. Initiate oxygen therapy. b. Hold the next dose of Imdur. c. Instruct the client to drink water. d. Administer PRN acetaminophen.

ANS: D The vasodilating effects of isosorbide mononitrate frequently cause clients to have headaches during the initial period of therapy. Clients should be told about this side effect and encouraged to take the medication with food. Some clients obtain relief with mild analgesics, such as acetaminophen. The client's headache is not related to hypoxia or dehydration; therefore, these interventions would not help. The client needs to take the medication as prescribed to prevent angina; the medication should not be held. DIF: Applying/Application REF: 686 KEY: Heart failure| nitroglycerin/nitrates| medication| pharmacologic pain management MSC: Integrated Process: Nursing Process: Implementation NOT: Client Needs Category: Physiological Integrity: Pharmacological and Parenteral Therapies

3. A nurse receives a report on a client who had a left-sided stroke and has homonymous hemianopsia. What action by the nurse is most appropriate for this client? a. Assess for bladder retention and/or incontinence. b. Listen to the client's lungs after eating or drinking. c. Prop the client's right side up when sitting in a chair. d. Rotate the client's meal tray when the client stops eating.

ANS: D This condition is blindness on the same side of both eyes. The client must turn his or her head to see the entire visual field. The client may not see all the food on the tray, so the nurse rotates it so uneaten food is now within the visual field. This condition is not related to bladder function, difficulty swallowing, or lack of trunk control. DIF: Applying/Application REF: 936 KEY: Neurologic disorders| stroke| visual disorders MSC: Integrated Process: Nursing Process: Implementation NOT: Client Needs Category: Physiological Integrity: Basic Care and Comfort

14. The nurse is caring for four clients with traumatic brain injuries. Which client should the nurse assess first? a. Client with cerebral perfusion pressure of 72 mm Hg b. Client who has a Glasgow Coma Scale score of 12 c. Client with a PaCO2 of 36 mm Hg who is on a ventilator d. Client who has a temperature of 102° F (38.9° C)

ANS: D A fever is a poor prognostic indicator in clients with brain injuries. The nurse should see this client first. A Glasgow Coma Scale score of 12, a PaCO2 of 36, and cerebral perfusion pressure of 72 mm Hg are all desired outcomes. DIF: Applying/Application REF: 953 KEY: Neurologic disorders| neurologic assessment MSC: IntegratedProcess:NursingProcess:Assessment NOT: Client Needs Category: Safe and Effective Care Environment: Management of Care

18. A new nurse is caring for a client with an abdominal aneurysm. What action by the new nurse requires the nurse's mentor to intervene? a. Assesses the client for back pain. b. Auscultates over abdominal bruit. c. Measures the abdominal girth. d. Palpates the abdomen in four quadrants.

ANS: D Abdominal aneurysms should never be palpated as this increases the risk of rupture. The nurse mentoring the new nurse would intervene when the new nurse attempts to do this. The other actions are appropriate.

16. A nurse is assessing a female client who is taking progestins. What assessment finding requires the nurse to notify the provider immediately? a. Irregular menses b. Edema in the lower extremities c. Ongoing breast tenderness d. Red, warm, swollen calf

ANS: D All clients receiving progestin therapy are at risk for thromboembolism. A red, warm, swollen calf is a manifestation of deep vein thrombosis and should be reported to the provider. Irregular menses, edema in the lower extremities, and breast tenderness are common side effects of the therapy.

The nurse interviews an older client with moderate osteoarthritis and her husband. What psychosocial assessment question would the nurse include? a. "Do you feel like hurting yourself or others?" b. "Are you planning to retire due to your disease?" c. "Do you ask your husband for assistance?" d. "Do you experience discomfort during sex?"

ANS: D Although some clients can become depressed and anxious as a result of having OA, suicidal ideation is not common. The nurse should not assume that an older adult will want to retire or that the client will need help from her husband. Many clients avoid sexual intercourse because of joint pain and stiffness.

2. The nurse assesses a client's recent memory. Which statement by the client confirms that recent memory is intact? a. "A young girl wrapped in a shroud fell asleep on a bed of clouds." b. "I was born on April 3, 1967, in Johnstown Community Hospital." c. "Apple, chair, and pencil are the words you just stated." d. "I ate oatmeal with wheat toast and orange juice for breakfast."

ANS: D Asking clients about recent events that can be verified, such as what the client ate for breakfast, assesses recent memory. Asking clients about certain facts from the past that can be verified assesses remote or long-term memory. Asking the client to repeat words assesses immediate memory.

10. An older client with peripheral vascular disease (PVD) is explaining the daily foot care regimen to the family practice clinic nurse. What statement by the client may indicate a barrier to proper foot care? a. "I nearly always wear comfy sweatpants and house shoes." b. "I'm glad I get energy assistance so my house isn't so cold." c. "My daughter makes sure I have plenty of lotion for my feet." d. "My hands shake when I try to do things requiring coordination."

ANS: D Clients with PVD need to pay special attention to their feet. Toenails need to be kept short and cut straight across. The client whose hands shake may cause injury when trimming toenails. The nurse would refer this client to a podiatrist. Comfy sweatpants and house shoes are generally loose and not restrictive, which is important for clients with PVD. Keeping the house at a comfortable temperature makes it less likely the client will use alternative heat sources, such as heating pads, to stay warm. The client should keep the feet moist and soft with lotion. 11. A client is taking warfarin and asks the nurse if taking St. John's wort is acceptable. What response by the nurse is best? a. "No, it may interfere with the warfarin." b. "There isn't any information about that." c. "Why would you want to take that?" d. "Yes, it is a good supplement for you."* ANS: A Many foods and drugs interfere with warfarin, St. John's wort being one of them. The nurse would advise the client against taking it. The other answers are not accurate.

10. A nurse teaches a client who has a history of heart failure. Which statement should the nurse include in this client's discharge teaching? a. "Avoid drinking more than 3 quarts of liquids each day." b. "Eat six small meals daily instead of three larger meals." c. "When you feel short of breath, take an additional diuretic." d. "Weigh yourself daily while wearing the same amount of clothing."

ANS: D Clients with heart failure are instructed to weigh themselves daily to detect worsening heart failure early, and thus avoid complications. Other signs of worsening heart failure include increasing dyspnea, exercise intolerance, cold symptoms, and nocturia. Fluid overload increases symptoms of heart failure. The client should be taught to eat a heart-healthy diet, balance intake and output to prevent dehydration and overload, and take medications as prescribed. The most important discharge teaching is daily weights as this provides the best data related to fluid retention. DIF: Applying/Application REF: 687 KEY: Heart failure| patient education MSC: Integrated Process: Teaching/Learning NOT: Client Needs Category: Health Promotion and Maintenance

7. The nurse is assessing a client diagnosed with trigeminal neuralgia affecting cranial nerve V. What assessment findings will the nurse expect for this client? a. Expressive aphasia b. Ptosis (eyelid drooping) c. Slurred speech d. Severe facial pain

ANS: D Cranial nerve (CN) V is the Trigeminal Nerve which has both a motor and sensory function in the face. When affected by a health problem, the client experiences severely facial pain. Expressive aphasia results from damage to the Broca speech area in the frontal lobe of the brain. Ptosis can result from damage to CN III and slurred speech often occurs from either damage to several cranial nerves or from damage to the motor strip in the frontal lobe of the brain.

7. A client with cancer is admitted to a short-term rehabilitation facility. The nurse prepares to administer the clients oral chemotherapy medications. What action by the nurse is most appropriate? a. Crush the medications if the client cannot swallow them. b. Give one medication at a time with a full glass of water. c. No special precautions are needed for these medications. d. Wear personal protective equipment when handling the medications.

ANS: D During the administration of oral chemotherapy agents, nurses must take the same precautions that are used when administering IV chemotherapy. This includes using personal protective equipment. These medications cannot be crushed, split, or chewed. Giving one at a time is not needed.

9. A nurse teaches a client who is prescribed digoxin (Lanoxin) therapy. Which statement should the nurse include in this client's teaching? a. "Avoid taking aspirin or aspirin-containing products." b. "Increase your intake of foods that are high in potassium." c. "Hold this medication if your pulse rate is below 80 beats/min." d. "Do not take this medication within 1 hour of taking an antacid."

ANS: D Gastrointestinal absorption of digoxin is erratic. Many medications, especially antacids, interfere with its absorption. Clients are taught to hold their digoxin for bradycardia; a heart rate of 80 beats/min is too high for this cutoff. Potassium and aspirin have no impact on digoxin absorption, nor do these statements decrease complications of digoxin therapy. DIF: Applying/Application REF: 686 KEY: Heart failure| digoxin| medication| patient education MSC: Integrated Process: Nursing Process: Implementation NOT: Client Needs Category: Physiological Integrity: Pharmacological and Parenteral Therapies

9. A nurse cares for a client who is experiencing deteriorating neurologic functions. The client states, "I am worried I will not be able to care for my young children." How would the nurse respond? a. "Caring for your children is a priority. You may not want to ask for help, but you really have to." b. "Our community has resources that may help you with some household tasks so you have energy to care for your children." c. "You seem distressed. Would you like to talk to a psychologist about adjusting to your changing status?" d. "Can you tell me more about what worries you, so we can see if we can do something to make adjustments?"

ANS: D Investigate specific concerns about situational or role changes before providing additional information. The nurse would not tell the client what is or is not a priority for him or her. Although community resources may be available, they may not be appropriate for the patient. Consulting a psychologist would not be appropriate without obtaining further information from the client related to current concerns.

11. After teaching a patient who is scheduled for magnetic resonance imaging (MRI), the nurse assesses the client's understanding. Which statement indicates client understanding of the teaching? a. "I must increase my fluids because of the dye used for the MRI." b. "My urine will be radioactive so I should not share a bathroom." c. "My gag reflex will be tested before I can eat or drink anything." d. "I can return to my usual activities immediately after the MRI."

ANS: D No postprocedure restrictions are imposed after MRI. The client can return to normal activities after the test is complete. There are no dyes or radioactive materials used for the MRI; therefore, increased fluids are not needed and the client's urine would not be radioactive. The procedure does not impact the client's gag reflex.

23. A nurse is caring for a client with acute pericarditis who reports substernal precordial pain that radiates to the left side of the neck. Which nonpharmacologic comfort measure should the nurse implement? a. Apply an ice pack to the client's chest. b. Provide a neck rub, especially on the left side. c. Allow the client to lie in bed with the lights down. d. Sit the client up with a pillow to lean forward on.

ANS: D Pain from acute pericarditis may worsen when the client lays supine. The nurse should position the client in a comfortable position, which usually is upright and leaning slightly forward. Pain is decreased by using gravity to take pressure off the heart muscle. An ice pack and neck rub will not relieve this pain. DIF: Applying/Application REF: 699 KEY: Nonpharmacologic pain management MSC: Integrated Process: Nursing Process: Implementation NOT: Client Needs Category: Physiological Integrity: Basic Care and Comfort

A client is admitted with possible sepsis. Which action will the nurse perform first? a. Administer antibiotics. b. Give an antipyretic. c. Place the client in isolation. d. Obtain specified cultures.

ANS: D Prior to administering antibiotics, the nurse obtains the prescribed cultures. Broad-spectrum antibiotics will be administered until the culture and sensitivity results are known. Antipyretics are given if the client is uncomfortable; fever is a defense mechanism. Giving antipyretics does not occur before obtaining cultures. The client may or may not need isolation.

19. A nurse cares for a client with end-stage heart failure who is awaiting a transplant. The client appears depressed and states, "I know a transplant is my last chance, but I don't want to become a vegetable." How should the nurse respond? a. "Would you like to speak with a priest or chaplain?" b. "I will arrange for a psychiatrist to speak with you." c. "Do you want to come off the transplant list?" d. "Would you like information about advance directives?"

ANS: D The client is verbalizing a real concern or fear about negative outcomes of the surgery. This anxiety itself can have a negative effect on the outcome of the surgery because of sympathetic stimulation. The best action is to allow the client to verbalize the concern and work toward a positive outcome without making the client feel as though he or she is crazy. The client needs to feel that he or she has some control over the future. The nurse personally provides care to address the client's concerns instead of pushing the client's issues off on a chaplain or psychiatrist. The nurse should not jump to conclusions and suggest taking the client off the transplant list, which is the best treatment option. DIF: Applying/Application REF: 691 KEY: Transplant| psychosocial response| anxiety MSC: Integrated Process: Nursing Process: Implementation NOT: Client Needs Category: Psychosocial Integrity

A client is scheduled to have a total hip arthroplasty. What preoperative teaching by the nurse is most important? a. Teach the need to discontinue all medications for 5 days before surgery. b. Teach the patient about foods high in protein, Vitamin C, and iron. c. Explain to the client the possible need for blood transfusions postoperatively. d. Remind the client to have all dental procedures completed at least 2 weeks prior to surgery.

ANS: D The nurse would include teaching about dental procedures to avoid infection after new joint has been inserted. Planned procedures would be completed at least 2 weeks before surgery and the client will need to tell any future primary health care providers about having a total joint arthroplasty. Only home medications prescribed that increase the risk for bleeding or clotting need to be discontinued 5 to 10 days before surgery. Clients need to be aware that any postoperative anemia may need to be treated with a blood transfusion, but it is not the most important. Diets high in protein, Vitamin C, and iron help with tissue repair, but are not the most important.

1. A nurse is assessing the peripheral vascular system of an older adult. What action by the nurse would cause the supervising nurse to intervene? a. Assessing blood pressure in both upper extremities b. Auscultating the carotid arteries for any bruits c. Classifying capillary filling of 4 seconds as normal d. Palpating both carotid arteries at the same time

ANS: D The nurse would not compress both carotid arteries at the same time to avoid brain ischemia. Blood pressure would be taken and compared in both arms. Prolonged capillary filling is considered to be greater than 5 seconds in an older adult, so classifying refill of 4 seconds as normal would not require intervention. Bruits would be auscultated.

The nurse learning about infection discovers that which factor is the best and most important barrier to infection? a. Colonization by host bacteria b. Gastrointestinal secretions c. Inflammatory processes d. Skin and mucous membranes

ANS: D The skin and mucous membranes are two of the most important barriers against infection. The other options are also barriers, but are considered secondary to skin and mucous membranes.

8. A client with HIV-III and wasting syndrome has inadequate nutrition. What assessment finding by the nurse best indicates that goals have been met for this client problem? a. Chooses high-protein food. b. Has decreased oral discomfort. c. Eats 90% of meals and snacks. d. Has a weight gain of 2 lb (1 kg)/1 mo.

ANS: D The weight gain is the best indicator that goals for this client problem have been met because it demonstrates that the client not only is eating well but also is able to absorb the nutrients. Choosing high-protein food is important, but only if the client eats and absorbs the nutrients.

7. A client is hospitalized with Pneumocystis jiroveci pneumonia. The client reports shortness of breath with activity and extreme fatigue. What intervention is best to promote comfort? a. Administer sleeping medication. b. Perform most activities for the client. c. Increase the client's oxygen during activity. d. Pace activities, allowing for adequate rest.

ANS: D This client has two major reasons for fatigue: decreased oxygenation and systemic illness. The nurse would not do everything for the client but rather let the client do as much as possible within limits and allow for adequate rest in between. Sleeping medications may be needed but not as the first step, and only with caution. Increasing oxygen during activities may or may not be warranted, but first the nurse must try pacing the client's activity.

18. A client in the intensive care unit is scheduled for a lumbar puncture (LP) today. On assessment, the nurse finds the client breathing irregularly with one pupil fixed and dilated. What action by the nurse is best? a. Ensure that informed consent is on the chart. b. Document these findings in the client's record. c. Give the prescribed preprocedure sedation. d. Notify the provider of the findings immediately.

ANS: D This client is exhibiting signs of increased intracranial pressure. The nurse should notify the provider immediately because performing the LP now could lead to herniation. Informed consent is needed for an LP, but this is not the priority. Documentation should be thorough, but again this is not the priority. The preprocedure sedation (or other preprocedure medications) should not be given as the LP will most likely be canceled. DIF: Applying/Application REF: 952 KEY: Neurologic disorders| neurologic assessment| communication MSC: Integrated Process: Communication and Documentation NOT: Client Needs Category: Physiological Integrity: Reduction of Risk Potential

A nurse manager is preparing an educational session for floor nurses on drug-resistant organisms. Which statement below indicates the need to review this information? a. "Methicillin-resistant Staphylococcus aureus can be hospital- or community-acquired." b. "Vancomycin-resistant Enterococcus can live on surfaces and be infectious for weeks." c. Carbapenem-resistant Enterobacteriaceae is hard to treat due to enzymes that break down antibiotics." d. "If you leave work wearing your scrubs, go directly home and wash them right away."

ANS: D To help prevent the transmission of an MDRO, wear scrubs and change clothes before leaving work. Keep work clothes separate from personal clothes. The nursing manager would need to correct his or her knowledge if he or she is letting staff know that wearing scrubs home is alright. The other statements are correct about multi-drug resistant organisms.

10. A client with HIV-III is admitted to the hospital with Toxoplasma gondii infection. Which action by the nurse is most appropriate? a. Initiate Contact Precautions. b. Conduct frequent neurologic assessments. c. Conduct frequent respiratory assessments. d. Initiate Protective Precautions.

ANS: D Toxoplasma gondii infection is an opportunistic infection that causes an encephalitis but poses only a rare threat to immunocompetent individuals The nurse would perform ongoing neurologic assessments. Contact and Protective Precautions are not needed. Good respiratory assessments are important to the client, but toxoplasmosis will demonstrate neurologic signs and symptoms.

A nurse teaches a clients family members about signs and symptoms of approaching death. Which manifestations should the nurse include in this teaching? (Select all that apply.) a. Warm and flushed extremities b. Long periods of insomnia c. Increased respiratory rate d. Decreased appetite e. Congestion and gurgling

ANS: D, E Common physical signs and symptoms of approaching death including coolness of extremities, increased sleeping, irregular and slowed breathing rate, a decrease in fluid and food intake, congestion and gurgling, incontinence, disorientation, and restlessness.

Which statements are true regarding Standard Precautions? (Select all that apply.) a. Always wear a gown when performing hygiene on clients. b. Sneeze into your sleeve or into a tissue that you throw away. c. Remain 3 feet (1 m) away from any client who has an infection. d. Use personal protective equipment as needed for client care. e. Wear gloves when touching clients' excretions or secretions. f. Cohorting clients who have infections caused by the same organism.

ANS: D, E Standard Precautions implies that contact with bodily secretions, excretions, and moist mucous membranes and tissues (excluding perspiration) is potentially infectious. Always wear gloves when coming into contact with such material. Other personal protective equipment is used based on the care being given. For example, if face splashing is expected, you will also wear a mask. Wearing a gown for hygiene is not required. Sneezing into your sleeve or tissue is part of respiratory etiquette. Remaining 3 feet (1 m) away from client is also not part of Standard Precautions. Cohorting infectious clients can be used for deciding room/bed placement, but is not part of Standard Precautions.

2. A nurse is preparing a client for a femoropopliteal bypass operation. What actions does the nurse delegate to the assistive personnel (AP)? (Select all that apply.) a. Administering preoperative medication b. Ensuring that the consent is signed c. Marking pulses with a pen d. Raising the side rails on the bed e. Recording baseline vital signs

ANS: D,E The AP can raise the side rails of the bed for client safety and take and record the vital signs. Administering medications, ensuring that a consent is on the chart, and marking the pulses for later comparison would be done by the registered nurse. This is also often done by the postanesthesia care nurse and is part of the hand-off report.

3. A client is admitted to the emergency department with a probable traumatic brain injury. Which assessment finding would be the priority for the nurse to report to the primary health care provider? a. Mild temporal headache b. Pupils equal and react to light c.Alert and oriented 3 d. Decreasing level of consciousness

Ans: D A decreasing level of consciousness is the first sign of increasing intracranial pressure, a potentially severe and possibly fatal complication of a traumatic brain injury (TBI). A mild headache would be expected for a client having a TBI. Equal reactive pupils and being alert and oriented are normal assessment findings.

A client has received an opioid analgesic for pain. The nurse assesses that the client has a Pasero Scale score of 3 and a respiratory rate of 7 shallow breaths/min. The client's oxygen saturation is 87%. Which action would the nurse perform first? a. Apply oxygen at 4 L/min. b. Attempt to arouse the client. c. Give naloxone (Narcan). d. Notify the Rapid Response Team.

Attempt to arouse the client.

. A nurse assesses the health history of a client who is prescribed ziconotide for chronic low back pain. Which assessment question would the nurse ask? a. "Are you taking a nonsteroidal anti-inflammatory drug?" b. "Have you been diagnosed with a mental health problem?" c. "Are you able to swallow oral medications?" d. "Do you smoke cigarettes or any illegal drugs?"

B

A client is scheduled for a percutaneous endoscopic lumbar discectomy. Which statement by the client indicates a need for further teaching? a. "I should have a lot less pain after surgery." b. "I'll be in the hospital for 2 to 3 days." c. "I should not have any major surgical complications." d. "I could possibly get an infection after surgery.

B

A client who had a complete spinal cord injury at level L5-S1 is admitted with a sacral pressure injury. What other assessment finding will the nurse anticipate for this client? a. Quadriplegia b. Flaccid bowel c. Spastic bladder d. Tetraparesis

B

A nurse teaches a client who is recovering from an open traditional cervical spinal fusion. Which statement would the nurse include in this client's postoperative instructions? a. "Only lift items that are 10 lb (4.5 kg) or less." b. "Wear your neck brace whenever you are out of bed." c. "You must remain in bed for 3 weeks after surgery." d. "You will be prescribed medications to prevent graft rejection."

B

A nurse evaluates prescriptions for a client with chronic atrial fibrillation. Which medication would the nurse expect to find on this client's medication administration record to prevent a common complication of this condition? a. Sotalol b. Warfarin c. Atropine d. Lidocaine

B Atrial fibrillation puts clients at risk for developing emboli. Clients at risk for emboli are treated with anticoagulants, such as heparin, enoxaparin, or warfarin. Sotalol, atropine, and lidocaine are not appropriate for preventing this complication.

A nurse cares for a client who has a heart rate averaging 56 beats/min with no adverse symptoms. Which activity modification would the nurse suggest to avoid further slowing of the heart rate? a. "Make certain that your bath water is warm." b. "Avoid straining while having a bowel movement." c. "Limit your intake of caffeinated drinks to one a day." d. "Avoid strenuous exercise such as running."

B Bearing down strenuously during a bowel movement is one type of Valsalva maneuver, which stimulates the vagus nerve and results in slowing of the heart rate. Such a response is not desirable in a person who has bradycardia. The other instructions are not appropriate for this condition.

A nurse assesses a client with atrial fibrillation. Which manifestation would alert the nurse to the possibility of a serious complication from this condition? a. Sinus tachycardia b. Speech alterations c. Fatigue d. Dyspnea with activity

B Clients with atrial fibrillation are at risk for embolic stroke. Evidence of embolic events includes changes in mentation, speech, sensory function, and motor function. Clients with atrial fibrillation often have a rapid ventricular response as a result. Fatigue is a nonspecific complaint. Clients with atrial fibrillation often have dyspnea as a result of the decreased cardiac output caused by the rhythm disturbance.

A nurse assists with the cardioversion of a client experiencing acute atrial fibrillation. What action would the nurse take prior to the cardioversion? a. Administer intravenous adenosine. b. Turn off oxygen therapy. c. Ensure that a tongue blade is available. d. Position the client on the

B For safety during cardioversion, the nurse would turn off any oxygen therapy to prevent fire. The other interventions are not appropriate for a cardioversion. The client would be placed in a supine position.

After teaching a client who has an implantable cardioverter-defibrillator (ICD), a nurse assesses the client's understanding. Which statement by the client indicates correct understanding of the teaching? a. "I would wear a snug-fitting shirt over the ICD." b. "I will avoid sources of strong electromagnetic fields." c. "I would participate in a strenuous exercise program." d. "Now I can discontinue my antidysrhythmic medication."

B The client being discharged with an ICD is instructed to avoid strong sources of electromagnetic fields, such as devices emitting microwaves (not microwave ovens); transformers; radio, television, and radar transmitters; large electrical generators; metal detectors, including handheld security devices at airports; antitheft devices; arc welding equipment; and sources of 60-cycle (Hz) interference. Also avoid leaning directly over the alternator of a running motor of a car or boat. Clients would avoid tight clothing, which could cause irritation over the ICD generator. The client would be encouraged to exercise but would not engage in strenuous activities that cause the heart rate to meet or exceed the ICD cutoff point because the ICD can discharge inappropriately. The client would continue all prescribed medications.

The nurse is doing home care teaching for a client who has undergone cryotherapy. Which statements by the client indicate a correct understanding of the instructions? (Select all that apply.) A. "I can resume my weight-lifting exercise class tomorrow." B. "I should not use tampons, douche, or have sexual activity." C. "I should shower rather than take a tub bath." D. "There may be a lot of bleeding for a few days." E. "There should be little or no discomfort."

B. "I should not use tampons, douche, or have sexual activity." C. "I should shower rather than take a tub bath." E. "There should be little or no discomfort." Cryotherapy involves freezing of cervical cancer cells and is often painless. Clients are restricted from heavy lifting. They may have a heavy watery discharge for several weeks, but should report any heavy bleeding, foul-smelling drainage, or a fever. The other options are correct.

11. A nurse teaches a client to use a room humidifier after a laryngectomy. Which statement would the nurse include in this patient's teaching? a. "Add peppermint oil to the humidifier to relax the airway." b. "Make sure you clean the humidifier to prevent infection." c. "Keep the humidifier filled with water at all times." d. "Use the humidifier when you sleep, even during daytime naps."

B. "Make sure you clean the humidifier to prevent infection." Priority teaching related to the use of a room humidifier focuses on infection control. Clients would be taught to meticulously clean the humidifier to prevent the spread of mold or other sources of infection. Peppermint oil would not be added to a humidifier. The humidifier would be refilled with water as needed and would be used while awake and asleep.

A client with metastatic prostate cancer has been prescribed leuprolide, a bisphosphonate, and flutamide. Which statement by the client warrants further investigation by the nurse? A. "I go for a short walk each day, even when I am very tired." B. "My wife has noticed my eyes looking a little yellow." C. "I ordered some looser shirts to hide my enlarging breasts." D. "Now I understand my wife's hot flashes with menopause."

B. "My wife has noticed my eyes looking a little yellow." Flutamide is an antiandrogen drug that can cause liver toxicity. The nurse would follow up on the statement that the client's eyes may be looking a little yellow which could indicate the onset of this adverse effect. Leuprolide can cause osteoporosis, hot flashes, and gynecomastia. The statements regarding weight-bearing exercise, enlarging breasts, and hot flashes are not cause for concern.

7. A nurse cares for a client who had a partial laryngectomy 10 days ago. The client states that all food tastes bland. How would the nurse respond? a. "I will consult the speech therapist to ensure you are swallowing properly." b. "This is normal after surgery. What types of food do you like to eat?" c. "I will ask the dietitian to change the consistency of the food in your diet." d. "Replacement of protein, calories, and water is very important after surgery."

B. "This is normal after surgery. What types of food do you like to eat?" Many clients experience changes in taste after surgery. The nurse would identify foods that the client wants to eat to ensure that the client maintains necessary nutrition. Although the nurse would collaborate with the speech therapist and dietitian to ensure appropriate replacement of protein, calories, and water, the other responses do not address the patient's concerns.

A client is scheduled to start external beam radiation therapy (EBRT) for her endometrial cancer. Which teaching by the nurse is accurate? (Select all that apply.) A. "You will need to be hospitalized during this therapy." B. "Your skin needs to be inspected daily for any breakdown." C. "It is not wise to stay out in the sun for long periods of time." D. "The perineal area may become damaged with the radiation." E. "The technician applies new site markings before each treatment." F. "You will not be radioactive or pose any danger to anyone else."

B. "Your skin needs to be inspected daily for any breakdown." C. "It is not wise to stay out in the sun for long periods of time." D. "The perineal area may become damaged with the radiation." F. "You will not be radioactive or pose any danger to anyone else." EBRT is usually performed in ambulatory care and does not require hospitalization. The client needs to know to evaluate the skin, especially in the perineal area, for any breakdown, and avoid sunbathing. The technician does not apply new site markings, so the client needs to avoid washing off the markings that indicate the treatment site.

A new nurse care for several client after radical prostatectomies for prostate cancer. What action by the nurse indicates a need to review care measures for this type of client? A. Delegates emptying and recording contents of the drainage devices. B. Administers a suppository to the client who reports constipation. C. Removes the sequential compression stockings on ambulatory clients. D. Discusses long-term complications such as erectile dysfunction.

B. Administers a suppository to the client who reports constipation. After a radical prostatectomy, the nurse would not provide a rectal suppository for constipation. All rectal treatments are contraindicated. The nurse would delegate emptying and recording drainage, remove the sequential pressure devices when clients being ambulating, discuss long-term complications of operation.

A nurse is caring for a woman who had hysteroscopic surgery for uterine leiomyomas. On initial assessment, the nurse notes the following: pulse: 114 beats/min, respiratory rate: 20 breaths/minute, crackles in bilateral lung bases. What action by the nurse takes priority? A. Assess the client for pain. B. Call the Rapid Response Team. C. Obtain an oxygen saturation. D. Delegate a temperature.

B. Call the Rapid Response Team. The fluid that is used during this procedure to distend the uterine cavity can be absorbed, leading to fluid overload. This client has signs of fluid overload which can be critical. The nurse would notify the Rapid Response Team first, then perform the other actions.

12. A client is wearing a Venturi mask to deliver oxygen and the dinner tray has arrived. What action by the nurse is best? a. Assess the client's oxygen saturation and, if normal, turn off the oxygen. b. Determine if the client can switch to a nasal cannula during the meal. c. Have the client lift the mask off the face when taking bites of food. d. Turn the oxygen off while the client eats the meal and then restart it.

B. Determine if the client can switch to a nasal cannula during the meal. Oxygen is a drug that needs to be delivered constantly. The nurse would determine if the primary health care provider has approved switching to a nasal cannula during meals. If not, the nurse would consult with the primary health care provider about this issue. The primary health care provider would need to prescribe discontinuing oxygen if the client's oxygen saturation is normal. The oxygen would not be turned off. Lifting the mask to eat will alter the FiO2 delivered.

2. A client is scheduled to have a tracheostomy placed in an hour. What action by the nurse is the priority? a. Administer prescribed anxiolytic medication. b. Ensure that informed consent is on the chart. c. Reinforce any teaching done previously. d. Start the preoperative antibiotic infusion.

B. Ensure that informed consent is on the chart. Since this is an operative procedure, the client must sign an informed consent, which must be on the chart. Giving anxiolytics and antibiotics and reinforcing teaching may also be required but do not take priority.

4. A client has a tracheostomy tube in place. When the nurse suctions the client, food particles are noted. What action by the nurse is best? a. Elevate the head of the client's bed. b. Measure and compare cuff pressures. c. Place the client on NPO status. d. Request that the client have a swallow study.

B. Measure and compare cuff pressures. Constant pressure from the tracheostomy tube cuff can cause tracheomalacia, leading to dilation of the tracheal passage. This can be manifested by food particles seen in secretions or by noting that larger and larger amounts of pressure are needed to keep the tracheostomy cuff inflated. The nurse would measure the pressures and compare them to previous ones to detect a trend. Elevating the head of the bed, placing the client on NPO status, and requesting a swallow study will not correct this situation.

A client is admitted to the emergency department with toxic shock syndrome. Which action by the nurse is the most important? A. Administer IV fluids to maintain fluid and electrolyte balance. B. Remove the tampon as the source of infection. C. Collect a blood specimen for culture and sensitivity. D. Transfuse the client to manage low blood count.

B. Remove the tampon as the source of infection The source of infection should be removed first. All of the other answers are possible interventions depending on the client's symptoms and vital signs, but removing the tampon is the priority.

The nurse is teaching a client about side effects and adverse reactions of a PDE5 inhibitor. What information does the nurse include? (Select all that apply.) A. Refrain from eating citrus fruit within 24 hours of taking the medication. B. Stop using this drug if your primary health care provider prescribes a nitrate. C. Do not drink alcohol before having sexual intercourse. D. Muscle cramps, nausea, and vomiting are possible if you take more than 1 pill a day. E. Take this medication within 30 to 60 minutes of having sexual intercourse. F. Change positions slowly especially if you also take an anti-hypertensive drug.

B. Stop using this drug if your primary health care provider prescribes a nitrate. C. Do not drink alcohol before having sexual intercourse. D. Muscle cramps, nausea, and vomiting are possible if you take more than 1 pill a day. F. Change positions slowly especially if you also take an anti-hypertensive drug. A PDE5 inhibitor is used to treat erectile dysfunction. The client should avoid grapefruit or grapefruit juice while taking these drugs. Taking a PDE5 inhibitor along with a nitrate can cause a profound drop in blood pressure. Alcohol may interfere with the ability to have an erection. Muscle cramps, nausea, and vomiting are possible side effects if more than 1 pill a day are taken. Each medication has its own directions for how soon to take it before intercourse, from 15 minutes to 2 hours. Any PDE5 drug can lower blood pressure so the nurse alerts the client of safety precautions.

3. A nurse teaches a client who had a supraglottic laryngectomy. Which technique would the nurse teach the client to prevent aspiration? a. Tilt the head back as far as possible when swallowing. b. Swallow twice while bearing down. c. Breathe slowly and deeply while swallowing. d. Keep the head very still and straight while swallowing.

B. Swallow twice while bearing down. The client post supraglottic laryngectomy has a high risk for aspiration. The nurse or speech language pathologist teaches the client the supraglottic method of swallowing. This includes placing a small amount of food in the mouth, performing the Valsalva maneuver, then swallowing twice. The client sits upright. The client holds the breath while swallowing twice. Keeping the head still and straight will not decrease the risk of aspiration.

A client has returned from a transurethral resection of the prostate with a continuous bladder irrigation. Five hours after the operation, the nurse notes the drainage is bright red with clots. What action should the nurse take first? A. Review the most recent hemoglobin and hematocrit. B. Take vital signs and begin immediate irrigation with sterile water. C. Notify the primary health care provider immediately. D. Remind the client not to pull on the catheter.

B. Take vital signs and begin immediate irrigation with sterile water. Bright red urinary drainage with clots may indicate arterial bleeding. The nurse would notify the primary health care provider immediately and begin irrigating the catheter with sterile normal saline (not sterile water). The nurse can delegate the vital signs. The nurse would review hemoglobin and hematocrit and would remind the client not to pull on the catheter for all clients with bladder irrigation. But for this client who may be bleeding the nurse would take further action to address the problem.

10. A client with a new tracheostomy is being seen in the oncology clinic. What finding by the nurse best indicates that goals for the client's decrease in self-esteem are being met? a. The client demonstrates good understanding of stoma care. b. The client has joined a book club that meets at the library. c. Family members take turns assisting with stoma care. d. Skin around the stoma is intact without signs of infection.

B. The client has joined a book club that meets at the library. The client joining a book club that meets outside the home and requires him or her to go out in public is the best sign that goals for disrupted self-esteem are being met. The other findings are all positive signs but do not relate to this client problem.

The nurse in the outpatient surgery clinic is discussing an upcoming surgical procedure with a client. Which information provided by the nurse is most appropriate for the client's long-term outcome? a. "At least you know that the pain after surgery will diminish quickly." b. "Discuss acceptable pain control after your operation with the surgeon." c. "Opioids often cause nausea but you won't have to take them for long." d. "The nursing staff will give you pain medication when you ask them for it."

B."Discuss acceptable pain control after your operation with the surgeon."

8. A nurse is caring for a client using oxygen while in the hospital. What assessment finding indicates that outcomes for client safety with oxygen therapy are being met? a. 100% of meals being eaten by the client b. Intact skin behind the ears c. The client understanding the need for oxygen d. Unchanged weight for the past 3 days

B.. Intact skin behind the ears Oxygen tubing can cause pressure injuries, so clients using oxygen have a high risk of skin breakdown. Intact skin behind the ears indicates that goals for maintaining client safety with oxygen therapy are being met. Nutrition and weight are not related to using oxygen. Understanding the need for oxygen is important but would not take priority over a physical problem.

A nurse cares for a client who presents with bradycardia secondary to hypothyroidism. Which medication does the nurse prepare to administer? a. Atropine sulfate b. Levothyroxine c. Propranolol d. Epinephrine

B.Levothyroxine

A nurse is caring for four clients on intravenous heparin therapy. Which laboratory value possibly indicates that a serious side effect has occurred? a. Hemoglobin: 14.2 g/dL (142 g/L) b. Platelet count: 82,000/L (82 109 /L) c. Red blood cell count: 4.8/mm3 (4.8 1012/L) d. White blood cell count: 8700/mm3 (8.7 109 /L)

B.Platelet count: 82,000/L

A client who has peptic ulcer disease is prescribed quadruple drug therapy for Helicobacter pylori infection. What health teaching related to bismuth would the nurse include? A: "Report stool changes to your primary health care provider immediately." B: "Do not take aspirin or aspirin products of any kind while on bismuth." C: "Take bismuth about 30 minutes before each meal and at bedtime." D: "Be aware that bismuth can cause frequent vomiting and diarrhea."

B: "Do not take aspirin or aspirin products of any kind while on bismuth." Bismuth is a salicylate drug and causes stool discoloration but not vomiting and diarrhea. It does not have to be taken at a specific time relative to meals. Clients taking bismuth should not take other salicylates, such as aspirin or aspirin-containing products.

8. A nurse is calling the on-call physician about a client who had a hysterectomy 2 days ago and has pain that is unrelieved by the prescribed narcotic pain medication. Which statement is part of the SBAR format for communication? a. A: "I would like you to order a different pain medication." b. B: "This client has allergies to morphine and codeine." c. R: "Dr. Smith doesn't like nonsteroidal anti-inflammatory meds." d. S: "This client had a vaginal hysterectomy 2 days ago."

B: "This client has allergies to morphine and codeine."

A client who had a partial gastrectomy 3 days ago begins to experience vertigo, sweating, and tachycardia about 30 minutes after eating breakfast. What postoperative complication would the nurse suspect? A: Pyloric obstruction B: Dumping syndrome C: Delayed gastric emptying D: Pernicious anemia

B: Dumping syndrome Dumping syndrome causes autonomic symptoms as food quickly leaves the stomach due to its decreased size after surgery.

A client has a nasogastric (NG) tube as a result of an upper gastrointestinal (GI) hemorrhage. What comfort measure would the nurse remind assistive personnel (AP) to provide? A: Lavaging the tube with ice water B: Performing frequent oral care C: Re-positioning the tube every 4 hours D: Taking and recording vital signs

B: Performing frequent oral care Clients with NG tubes need frequent oral care both for comfort and to prevent infection. Lavaging the tube is done by the nurse. Repositioning the tube, if needed, is also done by the nurse. The can take vital signs, but this is not a comfort measure.

A nurse assesses a client with a spinal cord injury at level T5. The client's blood pressure is 184/95 mm Hg, and the client presents with a flushed face and blurred vision. After raising the head of the bed, what action would the nurse take next? a. Initiate oxygen via a nasal cannula. b. Recheck the client's blood pressure. c. Palpate the bladder for distention. d. Administer a prescribed beta blocke

C

A nurse cares for a client with a spinal cord injury. With which interprofessional health team member would the nurse collaborate to assist the client with activities of daily living? a. Social worker b. Physical therapist c. Occupational therapist d. Case manager

C

A nurse is caring for a client with paraplegia who is scheduled to participate in a rehabilitation program. The client states, "I don't understand the need for rehabilitation; the paralysis will not go away and it will not get better." How would the nurse respond? a. "If you don't want to participate in the rehabilitation program, I'll let your primary health care provider know." b. "Rehabilitation programs have helped many patients with your injury. You should give it a chance." c. "The rehabilitation program will teach you how to maintain the functional ability you have and prevent further disability." d. "When new discoveries are made regarding paraplegia, people in rehabilitation programs will benefit first."

C

A nurse has presented an educational program to a community group on Lyme disease. What statement by a participant indicates the need to review the material? A. I should take precautions against ticks, especially in the summer B. A red rash that looks like a bull's eye may be one of the symptoms C. If Lyme disease is not treated successfully it is usually fatal D. For stage I disease antibiotics are usually needed for 14-21 days

C Untreated Lyme disease can lead to chronic complications or stage III Lyme disease, such as arthritis, chronic fatigue, memory/thinking problems. It is not usually a fatal disease so this information would need to be corrected by the nurse. All other statements are correct

A telemetry nurse assesses a client who has a heart rate of 35 beats/min on the cardiac monitor. Which assessment would the nurse complete next? a. Pulmonary auscultation b. Pulse strength and amplitude c. Level of consciousness d. Mobility and gait stability

C A heart rate of 40 beats/min or less could have hemodynamic consequences. The client is at risk for inadequate cerebral perfusion. The nurse would assess for level of consciousness, dizziness, confusion, syncope, chest pain, shortness of breath. Although the other assessments would be completed, the nurse would assess the client's neurologic status next.

A nurse cares for a client with atrial fibrillation who reports fatigue when completing activities of daily living. What intervention would the nurse implement to address this client's concerns? a. Administer oxygen therapy at 2 L per nasal cannula. b. Provide the client with a sleeping pill to stimulate rest. c. Schedule periods of exercise and rest during the day. d. Ask assistive personnel (AP) to help bathe the client.

C Clients who have atrial fibrillation are at risk for decreased cardiac output and fatigue when completing activities of daily living. The nurse would schedule periods of exercise and rest during the day to decrease fatigue. The other interventions will not assist the client with performing self-care activities and there is no indication for oxygen.

A nurse assesses a client who is recovering from an open traditional lumbar laminectomy with fusion. Which complications would the nurse report to the primary health care provider? (Select all that apply.) a. Surgical discomfort b. Redness and itching at the incision site c. Incisional bulging d. Clear drainage on the dressing e. Sudden and severe headache

C, D, E

After teaching a male client with a spinal cord injury at the T4 level, the nurse assesses the his understanding. Which client statements indicate a correct understanding of the teaching related to sexual effects of his injury? (Select all that apply.) a. "I will explore other ways besides intercourse to please my partner." b. "I will not be able to have an erection because of my injury." c. "Ejaculation may not be as predictable as before." d. "I may urinate with ejaculation but this will not cause infection." e. "I should be able to have an erection with stimulation.

C, D, E

A nurse assesses a client with paraplegia from a spinal cord injury and notes reddened areas over the client's hips and sacrum. What actions would the nurse take? (Select all that apply.) a. Apply a barrier cream to protect the skin from excoriation. b. Perform range-of-motion (ROM) exercises for the hip joint. c. Reposition the client off of the reddened areas. d. Get the client out of bed and into a chair several times a day. e. Apply a pressure-reducing mattress.

C, E

A client has been brought to the emergency department with a life-threatening chest injury. What action by the nurse takes priority? a. Apply oxygen at 100%. b. Assess the respiratory rate. c. Ensure a patent airway. d. Start two large-bore IV lines

C, Ensure a patent airway.

The nurse is providing preoperative education to a client prior to having an orchiectomy for testicular cancer. What statement by the client indicates the need to review the information? A. "I can still function sexually without one of my testes." B. "I will investigate sperm banking before the operation." C. "There should be no effect on my ability to reproduce." D. "Testicular self-exam will be important on the remaining testis."

C. "There should be no effect on my ability to reproduce." Oligospermia and azoospermia are common in clients with testicular function and can affect reproduction. The statement that there will be no effect on reproduction requires the nurse to review the information with the client. Sperm banking is an option prior to treatment to store sperm for future use. Normal sexual function is possible with one testis. Self-examination of the remaining testis is important for early detection of another tumor.

A 25-year-old woman is concerned about contracting HPV. What information by the nurse is most appropriate? A. "HPV is a benign infection that usually clears up on its own." B. "You are too old to receive the HPV vaccination." C. "We can provide HPV testing along with your Pap smear." D. "HPV is not a common sexually transmitted disease."

C. "We can provide HPV testing along with your Pap smear." HPV DNA testing can be done at the same time as the pap smear. Most women have HPV infection during their lives; however, it is not always benign. Two types, 16 and 18 are responsible for about 70% of cervical cancers. The vaccination with Gardasil 9 can be given up to age 45.

4. A nurse assesses clients on the medical-surgical unit. Which client is at greatest risk for development of obstructive sleep apnea? a. A 26-year-old woman who is 8 months pregnant. b. A 42-year-old man with gastroesophageal reflux disease. c. A 55-year-old woman who is 50 lb (23 kg) overweight. d. A 73-year-old man with type 2 diabetes mellitus.

C. A 55-year-old woman who is 50 lb (23 kg) overweight. The client at highest risk would be the one who is extremely overweight. None of the other clients have risk factors for sleep apnea. Clients with sleep apnea may develop gastroesophageal reflux.

The nurse is teaching a client who is undergoing brachytherapy about what to immediately report to her primary health care provider . Which signs and symptoms would be included in this teaching? (Select all that apply.) A. Constipation for 3 days B. Temperature of 99° F (37.2° C) C. Abdominal pain D. Visible blood in the urine E. Heavy vaginal bleeding F. Urinary retention

C. Abdominal pain D. Visible blood in the urine E. Heavy vaginal bleeding Health teaching for a client having brachytherapy would emphasize reporting abdominal pain, visible blood in the urine, and heavy vaginal bleeding. Severe diarrhea (not constipation), urethral burning, extreme fatigue, and a fever over 100° F (37.7° C) would also be reported.

8. A nurse cares for a client who is scheduled for a total laryngectomy. What action would the nurse take prior to surgery? a. Assess airway patency, breathing, and circulation. b. Administer prescribed intravenous pain medication. c. Assist the client to choose a communication method. d. Ambulate the client in the hallway to assess gait.

C. Assist the client to choose a communication method. The client will not be able to speak after surgery. The nurse would assist the client to choose a communication method that he or she would like to use after surgery. Assessing the patient's airway and administering IV pain medication are done after the procedure. Although ambulation promotes health and decreases the complications of any surgery, this patient's gait would not be impacted by a total laryngectomy and therefore is not a priority.

12. A nurse is caring for a client who had a modified uvulopalatopharyngoplasty (modUPPP) earlier in the day for obstructive sleep apnea. Which assessment finding indicates that a priority goal has been met? a. Client reports pain is controlled satisfactorily with analgesic regime. b. Client does not have foul odor to the breath or beefy red mucus membranes. c. Client is able to swallow own secretions without drooling. d. Client's vital signs are within normal parameters.

C. Client is able to swallow own secretions without drooling. The priority after a modUPPP is maintaining a patent airway. The client who has a patent airway can swallow his or her own secretions without drooling. Controlled pain is important, but not the priority. Foul breath odor and beefy red mucus membranes indicate possible infection, which probably would not occur this soon after surgery, but preventing infection does not take priority over airway. Vital signs "within normal parameters" are vague.

A client presents to the emergency department reporting vomiting, severe lower abdominal pain, and a tender mass above one testis. What action by the nurse is most important? A. Have the client rate pain using the 0-10 scale. B. Prepare to administer an IV opioid analgesic. C. Determine when he last ate or drank anything. D. Assess risk factors for testicular cancer.

C. Determine when he last ate or drank anything. This client has signs and symptoms of testicular torsion, which is a surgical emergency. For client safety, the nurse assesses last oral intake. Rating the pain is an important intervention too but is not related to safety. The client cannot have opioids prior to signing a surgical consent. The client does not have signs and symptoms of testicular cancer.

The nurse is caring for a newly admitted client who is diagnosed with hyperglycemic-hyperosmolar state (HHS). What is the nurse's priority action at this time? a. Assess the client's blood glucose level. b. Monitor the client's urinary output every hour. C. Establish intravenous access to provide fluids. d. Give regular insulin per agency policy.

C. Establish intravenous access to provide fluids

6. A nurse cares for a client after radiation therapy for neck cancer. The client reports extreme dry mouth. What action by the nurse is most appropriate? a. Ask the client to gargle with mouthwash containing lidocaine. b. Administer IV fluid boluses every 2 hours. c. Explain that xerostomia may be a permanent side effect. d. Assess the client's neck for redness and swelling.

C. Explain that xerostomia may be a permanent side effect. Xerostomia, or dry mouth, is a potential side effect of radiation, particularly if the salivary glands were in the radiation zone. Unfortunately, this may be long term or even permanent. Gargling with lidocaine would not help. Increasing fluids is somewhat helpful, but the client would be encouraged to drink. The client's neck may have redness and swelling, but this finding is not related to the reported dry mouth.

13. The nurse assesses the client using the device pictured below to deliver 50% O2: The nurse finds that the mask fits snugly, the skin under the mask and straps is intact, and the flow rate of the oxygen is 3 L/min. What action by the nurse is best? a. Assess the client's oxygen saturation. b. Document these findings in the chart. c. Immediately increase the flow rate. d. Turn the flow rate down to 2 L/min.

C. Immediately increase the flow rate. For the venturi mask to deliver high flow of oxygen, the flow rate must be set correctly, usually between 4 and 10 L/min. The client's flow rate is too low and the nurse would increase it. After increasing the flow rate, the nurse assesses the oxygen saturation and documents the findings.

A nurse is assisting the health care provider who is intubating a client. The provider has been attempting to intubate for 40 seconds. What action by the nurse takes priority? a. Ensure that the client has adequate sedation. b. Find another qualified provider to intubate. c. Interrupt the procedure to give oxygen. d. Monitor the client's oxygen saturation.

C. Interrupt the procedure to give oxygen

An intubated clients oxygen saturation has dropped to 88%. What action by the nurse takes priority? a. Determine if the tube is kinked. b. Ensure that all connections are patent. c. Listen to the client's lung sounds. d. Suction the endotracheal tube

C. Listen to the clients lung sounds.

A postoperative client has an epidural infusion of morphine and bupivacaine. Which actions does the nurse delegate to the assistive personnel (AP)? (Select all that apply.) a. Ask the client to point out any areas of numbness or tingling. b. Determine how many people are needed to ambulate the client. c. Perform a bladder scan if the client is unable to void after 4 hours. d. Remind the client to use the incentive spirometer every hour. e. Take and record the client's vital signs per agency protocol.

C. Perform a bladder scan if the client is unable to void after 4 hours. D.Remind the client to use the incentive spirometer every hour. E.Take and record the client's vital signs per agency protocol.

A client is on a ventilator and is sedated. What care may the nurse delegate to the unlicensed assistive personnel (UAP)? a. Assess the client for sedation needs. b. Get family permission for restraints. c. Provide frequent oral care per protocol. d. Use nonverbal pain assessment tools.

C. Provide frequent oral care per protocol.

A client returned from a transurethral resection of the prostate 8 hours ago with a continuous bladder irrigation. The client reports headache and dizziness. What action by the nurse is most appropriate? A. Consider starting a blood transfusion. B. Slow the bladder irrigation down. C. Report the findings to the surgeon immediately. D. Take the vital signs every 15 minutes.

C. Report the findings to the surgeon immediately. Headache, dizziness, and shortness of breath are symptoms of possible TURP syndrome in which the irrigation fluid is absorbed, putting strain on the client's heart. The nurse notifies the primary health care provider immediately as the client may need intensive care monitoring. There is no data indicating the client needs a blood transfusion, plus that would add even more fluid in the system. The irrigant may need to be slowed but that is not the first action the nurse would take. Vital signs do need to be taken frequently in this situation, but the nurse notifies the primary health care provider first.

A nurse is providing education to a new 55-year-old African-American client about screening for prostate cancer. What action by the nurse is most appropriate? A. Inform the client that recommendations vary, so screening is a personal choice. B. Let the client know that as an African American, he should be screened annually. C. Teach the client that he is in a high risk group and should discuss screening. D. Give the client written information that discourages screening until age 70.

C. Teach the client that he is in a high risk group and should discuss screening. Clients in certain high risk groups should discuss screening for prostate cancer with their primary health care providers at age 45. High risk groups include African Americans and men with a first-degree relative who was diagnosed with prostate cancer before the age of 65. This new client will be encouraged to discuss screening even though he is past the age of initial discussion. Recommendations do vary somewhat, but he is in a recognized high risk group. The nurse would not say that he "should" be screened annually. Screening is not recommended for men over the age of 70.

A client has a pulmonary embolism and is started on oxygen. The student nurse asks why the clients oxygen saturation has not significantly improved. What response by the nurse is best? a. "Breathing so rapidly interferes with oxygenation." b. "Maybe the client has respiratory distress syndrome." c. "The blood clot interferes with perfusion in the lungs." d. "The client needs immediate intubation and mechanical ventilation."

C. The blood clot interferes with perfusion in the lungs.

6. A nurse is providing tracheostomy care. What action by the nurse requires intervention by the charge nurse? a. Holding the device securely when changing ties b. Suctioning the client first if secretions are present c. Tying a square knot at the back of the neck d. Using half-strength peroxide for cleansing

C. Tying a square knot at the back of the neck To prevent pressure injuries and for client safety, when ties are used that must be knotted, the knot would be placed at the side of the client's neck, not in back. The other actions are appropriate.

A nurse cares for a client who has hypothyroidism as a result of Hashimotothyroiditis. The client asks, "How long will I need to take this thyroid medication?" How would the nurse respond? a. "You will need to take the thyroid medication until the goiter is completely gone." b. "Thyroiditis is cured with antibiotics. Then you won't need thyroid medication." c. "You'll need thyroid pills for life because your thyroid won't start working again." d. "When blood tests indicate normal thyroid function, you can stop the medication."

C."You'll need thyroid pills for life because your thyroid won't start working again."

A nurse assesses a client who is recovering from a subtotal thyroidectomy. On the first postoperative day before discharge, the client states, "I feel numbness and tingling around my mouth." What action does the nurse take? a. Offer mouth care. b. Loosen the dressing. c. Assess for muscle twitching. d. Ask the client orientation questions.

C.Assess for muscle twitching.

A nurse plans care for a client with hypothyroidism. Which priority problem does the nurse address first for this client? a. Heat intolerance b. Body image problems c. Depression and withdrawal d. Obesity and water retention

C.Depression and withdrawal

A nurse is assessing pain on a confused older client who has difficulty with verbal expression. Which pain assessment tool would the nurse choose for this assessment? a. Numeric rating scale b. Verbal Descriptor Scale c. FACES Pain Scale-Revised d. Wong-Baker FACES Pain Scale

C.FACES Pain Scale-Revised

The nurse assesses a client who has possible gastritis. Which assessment finding(s) indicate(s) that the client has chronic gastritis? (Select all that apply.) A: Anorexia B: Dyspepsia C: Intolerance of fatty foods D: Pernicious anemia E: Nausea and vomiting

C: Intolerance of fatty foods D: Pernicious anemia Intolerance of fatty or spicy foods and pernicious anemia are signs of chronic gastritis. Anorexia and nausea/vomiting can be seen in both conditions. Dyspepsia is seen in acute gastritis.

During an interview, the client tells the nurse that the client has a duodenal ulcer. Which assessment finding would the nurse expect? A: Hematemesis B: Pain when eating C: Melena D: Weight loss

C: Melena All of the other assessment findings are more commonly seen in clients who have gastric ulcers rather than duodenal ulcers.

The nurse is caring for a client who has been diagnosed with peptic ulcer disease. For which complication would the nurse monitor? A: Large bowel obstruction B: Dyspepsia C: Upper gastrointestinal (GI) bleeding D: Gastric cancer

C: Upper gastrointestinal (GI) bleeding Peptic ulcer disease (PUD) can cause gastric mucosal damage or perforation, which causes upper GI bleeding. Dyspepsia is a symptom of PUD, gastritis, and gastric cancer. PUD affects the stomach and/or duodenum, not the colon.

A client continues to have persistent low back pain even after using a number of nonpharmacologic pain management strategies. Which prescribed drug would the nurse anticipate that the client might need to manage the pain? a. Oxycontin b. Gabapentin c. Lorazepam d. Tramadol

D

A nurse assesses clients at a community center. Which client is at greatest risk for low back pain? a. A 24-year-old female who is 25 weeks pregnant. b. A 36-year-old male who uses ergonomic techniques. c. A 53-year-old female who uses a walker. d. A 65-year-old female with osteoarthritis

D

The nurse is collaborating with the occupational therapist to assist a client with a complete cervical spinal cord injury to transfer from the bed to the wheelchair. What ambulatory aid would be most appropriate for the client to meet this outcome? a. Rolling walker b. Quad cane c. Adjustable crutches d. Sliding board

D

A nurse is teaching the client with systemic lupus erythematosus about prednisone. What information is the priority? A. Might make the client feel jittery or nervous B. Can cause sodium and fluid retention C. Long-term effects include fat redistribution D. never stop prednisone abruptly

D The nurse teaches the client to avoid stoping the drub abruptly as the priority bc this can lead to a life threatening adrenal crisis. Short term effects do include jitteriness or nervousness, sodium and water retention. One long term side effect is fat redistribution resutlin in "moon face" and "buffalo hump"

A nurse assesses a client's electrocardiograph tracing and observes that not all QRS complexes are preceded by a P wave. How would the nurse interpret this observation? a. The client has hyperkalemia causing irregular QRS complexes. b. Ventricular tachycardia is overriding the normal atrial rhythm. c. The client's chest leads are not making sufficient contact with the skin. d. Ventricular and atrial depolarizations are initiated from different sites.

D Normal rhythm shows one P wave preceding each QRS complex, indicating that all depolarization is initiated at the sinoatrial node. QRS complexes without a P wave indicate a different source of initiation of depolarization. This finding on an electrocardiograph tracing is not an indication of hyperkalemia, ventricular tachycardia, or disconnection of leads.

A client is diagnosed with benign prostatic hyperplasia and seems sad and irritable. After assessing the client's behavior, which statement by the nurse would be the most appropriate? A. "The urine incontinence should not prevent you from socializing." B. "You seem depressed and should seek more pleasant things to do." C. "It is common for men at your age to have changes in mood."s. D. "Nocturia could cause interruption of your sleep and cause changes in mood."

D. "Nocturia could cause interruption of your sleep and cause changes in mood." Frequent visits to the bathroom during the night could cause sleep interruptions and affect the client's mood and mental status. Telling the client his symptoms should not lead to less socialization is patronizing. Instructing the client to seek more pleasant things to do also is patronizing. Neither statement has any information the client could find useful. The statement about age has no validity and again does not offer useful information.

The nurse is caring for a postoperative client following an anterior colporrhaphy. What action can be delegated to the assistive personnel (AP)? A. Reviewing the hematocrit and hemoglobin results B. Teaching the client to avoid lifting her 4-year-old grandson C. Assessing the level of pain and any drainage D. Drawing a shallow hot bath for comfort measures

D. Drawing a shallow hot bath for comfort measures The AP is able to provide comfort through a bath. The registered nurse would review any laboratory results, complete any teaching, and assess pain and discharge.

A client is on mechanical ventilation and the clients spouse wonders why ranitidine (Zantac) is needed since the client only has lung problems. What response by the nurse is best? a. "It will increase the motility of the gastrointestinal tract." b. "It will keep the gastrointestinal tract functioning normally." c. "It will prepare the gastrointestinal tract for enteral feedings." d. "It will prevent ulcers from the stress of mechanical ventilation."

D. It will prevent ulcers from the stress of mechanical ventilation.

A client who had surgery has extreme postoperative pain that is worsened when trying to participate in physical therapy. Which intervention for pain management does the nurse include in the client's care plan? a. As-needed pain medication after therapy b. Pain medications prior to therapy only c. Patient-controlled analgesia with a basal rate d. Round-the-clock analgesia with PRN analgesics

D. Round-the-clock analgesia with PRN analgesics

A nurse is assessing pain in an older adult. Which action by the nurse is best? a. Ask only "yes-or-no" questions so the client doesn't get too tired. b. Give the client a picture of the pain scale and come back later. c. Question the client about new pain only, not normal pain from aging. d. Sit down, ask one question at a time, and allow the client to answer.

D. Sit down, ask one question at a time, and allow the client to answer.

9. A nurse is assessing a client who has a tracheostomy. The nurse notes that the tracheostomy tube is pulsing with the heartbeat as the client's pulse is being taken. No other abnormal findings are noted. What action by the nurse is most appropriate? a. Call the operating room to inform them of a pending emergency case. b. No action is needed at this time; this is a normal finding in some clients. c. Remove the tracheostomy tube; ventilate the client with a bag-valve-mask. d. Stay with the client and have someone else call the primary health care provider immediately.

D. Stay with the client and have someone else call the primary health care provider immediately. This client may have a tracheoinnominate artery fistula, which can be a life-threatening emergency if the artery is breached and the client begins to hemorrhage. Since no bleeding is yet present, the nurse stays with the client and asks someone else to notify the primary health care provider. If the client begins hemorrhaging, the nurse removes the tracheostomy and applies pressure at the bleeding site. The client will need to be prepared for surgery.

A client has undergone a vaginal hysterectomy with a bilateral salpingo-oophorectomy. She is concerned about a loss of libido. What intervention by the nurse would be best? A. Suggest increasing vitamins and supplements daily. B. Discuss the value of a balanced diet and exercise. C. Reinforce that weight gain may be inevitable. D. Teach that estrogen cream inserted vaginally may help.

D. Teach that estrogen cream inserted vaginally may help. Use of vaginal estrogen cream and gentle dilation can help with vaginal changes and loss of libido. Weight gain and masculinization are misperceptions after a vaginal hysterectomy. Vitamins, supplements, a balanced diet, and exercise are helpful for healthy living, but are not necessarily going to increase libido.

The outpatient clinic nurse has assessed a woman who reports a month-long history of feeling full, urinary frequency, and bloating. What action by the nurse is best? A. Obtain a clean catch urine specimen. B. Instruct the client on a 3-day diet history. C. Facilitate having a pelvic ultrasound. D. Teach the woman about CA-125 test.

D. Teach the woman about CA-125 test. Evidence shows that women with ovarian cancer often have recognizable, early signs such as abdominal bloating, urinary frequency or urgency, feeling full or difficulty eating, and pelvic pain. The nurse should "think ovarian" and facilitate the client having a CA-125 blood test, which is a cancer antigen test. The other actions may or may not be needed, but with these symptoms, the client needs to be evaluated for ovarian cancer.

A nurse is caring for a client on mechanical ventilation. When double-checking the ventilator settings with the respiratory therapist, what should the nurse ensure as a priority? a. The client is able to initiate spontaneous breaths. b. The inspired oxygen has adequate humidification. c. The upper peak airway pressure limit alarm is off. d. The upper peak airway pressure limit alarm is on.

D. The upper peak airway pressure limit alarm is on

The nurse is assessing a client's pain and has elicited information on the location, quality, intensity, effect on functioning, aggravating and relieving factors, and onset and duration. Which question by the nurse would be best to ask the client for completing a comprehensive pain assessment? a. "Are you worried about addiction to pain pills?" b. "Do you attach any spiritual meaning to pain?" c. "How high would you say your pain tolerance is?" d. "What pain rating would be acceptable to you?"

D."What pain rating would be acceptable to you?"

A nurse on the postoperative inpatient unit receives hand-off report on four clients using patient-controlled analgesia (PCA) pumps. Which client would the nurse see first? a. Client who appears to be sleeping soundly. b. Client with no bolus request in 6 hours. c. Client who is pressing the button every 10 minutes. d. Client with a respiratory rate of 8 breaths/min

D.Client with a respiratory rate of 8 breaths/min

A nurse assesses a client on the medical-surgical unit. Which statement made by the client alerts the nurse to assess the patient for hypothyroidism? a. "My sister has thyroid problems." b. "I seem to feel the heat more than other people." c. "Food just doesn't taste good without a lot of salt." d. "I am always tired, even with 12 hours of sleep."

D.I am always tired, even with 12 hours of sleep."

The nurse is teaching a client who has been treated for acute gastritis. What statement by the client indicates a need for further teaching? A: "I need to cut down on drinking martinis every might." B: "I should decrease my intake of caffeinated drinks, especially coffee." C: "I will only take ibuprofen once in a while when I really need it." D: "I can continue smoking cigarettes which is better than chewing tobacco."

D: "I can continue smoking cigarettes which is better than chewing tobacco." To prevent another episode of acute gastritis, alcohol, caffeinated drinks, and NSAIDs should be avoided or kept at a minimum. Smoking and all forms of tobacco should also be avoided.

The nurse caring for clients with gastrointestinal disorders would recall that omeprazole is a drug in which classification? A: Gastric acid inhibitor B: Histamine receptor blocker C: Mucosal barrier fortifier D: Proton pump inhibitor

D: Proton pump inhibitor Omeprazole is a proton pump inhibitor.

The nurse learning about cellular regulation understands that which process occurs during the S phase of the cell cycle? a. Actual division (mitosis) b. Doubling of DNA c. Growing extra membrane d. No reproductive activity

Doubling of DNA

A client is put on twice-daily acetaminophen for osteoarthritis. Which finding in the client's health history would lead the nurse to consult with the primary health care provider over the choice of medication? a. 25-pack-year smoking history b. Drinking 3 to 5 beers a day c. Previous peptic ulcer d. Taking warfarin

Drinking 3 to 5 beers a day

What is the first line treatment is a type I hypersensitivity?

Epinephrine

The nurse working with oncology clients understands that interacting factors affect cancer development. Which factors does this include? (Select all that apply.) a. Exposure to carcinogens b. Genetic predisposition c. Immune function d. Normal doubling time e. State of euploidy

Exposure to carcinogens Genetic predisposition Immune function

A nurse is learning the difference between normal cells and benign tumor cells. What information does this include? a. Benign tumors grow through invasion of other tissue. b. Benign tumors have lost their cellular regulation from contact inhibition. c. Growing in the wrong place or time is typical of benign tumors. d. The loss of characteristics of the parent cells is called anaplasia

Growing in the wrong place or time is typical of benign tumors.

A nurse teaches a client who is considering being admitted to hospice. Which statement does the nurse include in the clients teaching?

Hospice care focuses on a holistic approach to health care. It is not designed to hasten death, but rather to relieve symptoms."

A hospitalized client has a history of depression for which sertraline is prescribed. The client also has a morphine allergy and a history of alcoholism. After surgery, several opioid analgesics are prescribed. Which one would the nurse choose? a. Hydrocodone and acetaminophen b. Hydromorphone c. Meperidine d. Tramadol

Hydromorphone

A nurse is participating in primary prevention efforts directed against cancer. In which activities is this nurse most likely to engage? (Select all that apply.) a. Demonstrating breast self-examination methods to women b. Instructing people on the use of chemoprevention c. Providing vaccinations against certain cancers d. Screening teenage girls for cervical cancer e. Teaching teens the dangers of tanning booths f. Educating adults about healthy eating habits

Instructing people on the use of chemoprevention Providing vaccinations against certain cancers Teaching teens the dangers of tanning booths Educating adults about healthy eating habits

A registered nurse is caring for a client who is receiving pain medication via patient-controlled analgesia (PCA). Which action by the nurse indicates the need for further education on pain control with PCA? a. Assesses the client's pain level per agency policy. b. Monitors the client's respiratory rate and sedation. c. Presses the button when the client cannot reach it. d. Reinforces client teaching about using the PCA pump.

Presses the button when the client cannot reach it.

A nurse asks the staff development nurse what "apoptosis" means. What response best? a. Growth by cells enlarging b. Having the normal number of chromosomes c. Inhibition of cell growth d. Programmed cell death

Programmed cell death

What is the MOST COMMON hypersensitivity reaction?

Type I rapid or immediate THIS IS ALSO A MEDICAL EMERGENCY AND TREATMENT IS IMMEDIATE!

Autoimmune hemolytic anemia, good pasture syndrome, myasthenia Travis are all examples of what type of hypersensitivity? What is reacting?

Type II- Cytotoxic Reaction if IgG with host cell membrane or antigen absorbed by host cell membrane Body works against itself

Serum Sickness, vasculitis, SLE (systemic lupus erythematosus) and RA (rheumatoid arthritis) are examples of what type of hypersensitivity?

Type III

A nurse is preparing to give a client ketorolac intravenously for pain. Which assessment findings would lead the nurse to consult with the primary health care provider? a. Bilateral lung crackles b. Hypoactive bowel sounds c. Self-reported pain of 3/10 d. Urine output of 20 mL/2 hr

Urine output of 20 mL/2 hr

The nurse is teaching assistive personnel (AP) about care of a client who has advanced cirrhosis. Which statements would the nurse include in the staff teaching? (Select all that apply.) a. "Apply lotion to the client's dry skin areas." b. "Use a basin with warm water to bathe the patient." c. "For the patient's oral care, use a soft toothbrush." d. "Provide clippers so the patient can trim the fingernails." e. "Bathe with antibacterial and water-based soaps."

a. "Apply lotion to the client's dry skin areas." c. "For the patient's oral care, use a soft toothbrush." d. "Provide clippers so the patient can trim the fingernails." Clients with advanced cirrhosis often have pruritus. Lotion will help decrease itchiness from dry skin. A soft toothbrush would be used to prevent gum bleeding, and the client's nails would need to be trimmed short to prevent the patient from scratching himself or herself. These clients should use cool, not warm, water on their skin, and should not use excessive amounts of soap.

A nurse teaches a client who has chronic obstructive pulmonary disease. Which statements related to nutrition would the nurse include in this client's teaching? (Select all that apply.) a. "Avoid drinking fluids just before and during meals." b. "Rest before meals if you have dyspnea." c. "Have about six small meals a day." d. "Eat high-fiber foods to promote gastric emptying." e. "Use pursed-lip breathing during meals." f. "Choose soft, high-calorie, high-protein foods."

a. "Avoid drinking fluids just before and during meals." b. "Rest before meals if you have dyspnea." c. "Have about six small meals a day." e. "Use pursed-lip breathing during meals." f. "Choose soft, high-calorie, high-protein foods."

The nurse is planning teaching for a client who is starting acarbose for diabetes mellitus type 2. Which statement will the nurse include in the teaching? a. "Be sure to take the drug with each meal." b. "Take the drug every evening before bedtime." c. "Take the drug on an empty stomach in the morning." d. "Decide on the best day of the week to take the drug."

a. "Be sure to take the drug with each meal."

6. A client admitted for pneumonia has been tachypneic for several days. When the nurse starts an IV to give fluids, the client questions this action, saying "I have been drinking tons of water. How am I dehydrated?" What response by the nurse is best? a. "Breathing so quickly can be dehydrating." b. "Everyone with pneumonia is dehydrated." c. "This is really just to administer your antibiotics." d. "Why do you think you are so dehydrated?"

a. "Breathing so quickly can be dehydrating." Tachypnea and mouth breathing (from increased work of breathing), both seen in pneumonia, increase insensible water loss and can lead to a degree of dehydration. The other options do not give the client useful information that addresses this specific concern.

A nurse teaches a client who has severe allergies ways to prevent insect bites. Which statements does the nurse include in this client's teaching? (Select all that apply.) a. "Consult an exterminator to control bugs in and around your home." b. "Do not swat at insects or wasps." c. "Wear sandals whenever you go outside." d. "Keep your prescribed epinephrine autoinjector in a bedside drawer." e. "Use screens in your windows and doors to prevent flying insects from entering." f. "Identify and remove potential nesting sites in your yard."

a. "Consult an exterminator to control bugs in and around your home." b. "Do not swat at insects or wasps." e. "Use screens in your windows and doors to prevent flying insects from entering." f. "Identify and remove potential nesting sites in your yard." To prevent arthropod bites and stings, patients should wear protective clothing, cover garbage cans, use screens in windows and doors, inspect clothing and shoes before putting them on, consult an exterminator, remove nests, avoid swatting at insects, and carry a prescription epinephrine autoinjector at all times if they are known to be allergic to bee or wasp stings. Shoes are needed when working in areas known or suspected to harbor arthropods, but sandals will not protect the feet. Removing nesting sites may help eliminate the population.

A nurse wants to become part of a Disaster Medical Assistance Team (DMAT) but is concerned about maintaining licensure in several different states. Which statement best addresses these concerns? a. "Deployed DMAT providers are federal employees, so their licenses are good in all 50 states." b. "The government has a program for quick licensure activation wherever you are deployed." c. "During a time of crisis, licensure issues would not be the government's priority concern." d. "If you are deployed, you will be issued a temporary license in the state in which you are working."

a. "Deployed DMAT providers are federal employees, so their licenses are good in all 50 states." When deployed, DMAT health care providers act as agents of the federal government, and so are considered federal employees. Thus their licenses are valid in all 50 states. Licensure is an issue that the government would be concerned with, but no programs for temporary licensure or rapid activation are available.

The nurse is caring for a client who is prescribed lactulose. The client states, "I do not want to take this medication because it causes diarrhea." How would the nurse respond? a. "Diarrhea is expected; that's how your body gets rid of ammonia." b. "You may take antidiarrheal medication to prevent loose stools." c"Do not take any more of the medication until your stools firm up." d. "We will need to send a stool specimen to the laboratory as soon as possible."

a. "Diarrhea is expected; that's how your body gets rid of ammonia." The purpose of administering lactulose to this patient is to help ammonia leave the circulatory system through the colon. Lactulose draws water into the bowel with its high osmotic gradient, thereby producing a laxative effect and subsequently evacuating ammonia from the bowel. The patient must understand that this is an expected and therapeutic effect for him or her to remain compliant. The nurse would not suggest administering anything that would decrease the excretion of ammonia or holding the medication. There is no need to send a stool specimen to the laboratory because diarrhea is the therapeutic response to this medication

A nurse assesses a client who presents with renal calculi. Which question would the nurse ask? a. "Do any of your family members have this problem?" b. "Do you drink any cranberry juice?" c. "Do you urinate after sexual intercourse?" d. "Do you experience burning with urination?"

a. "Do any of your family members have this problem?"

A nurse obtains the health history of a client with a suspected diagnosis of bladder cancer. Which question would the nurse ask when determining this client's risk factors? a. "Do you smoke cigarettes?" b. "Do you use any alcohol?" c. "Do you use recreational drugs?" d. "Do you take any prescription drugs?"

a. "Do you smoke cigarettes?"

A nurse teaches a client who is interested in smoking cessation. Which statements would the nurse include in this client's teaching? (Select all that apply.) a. "Find an activity that you enjoy and will keep your hands busy." b. "Keep snacks like potato chips on hand to nibble on." c. "Identify a consequence for yourself in case you backslide." d. "Drink at least eight glasses of water each day." e. "Make a list of reasons you want to stop smoking." f. "Set a quit date and stick to it."

a. "Find an activity that you enjoy and will keep your hands busy. d. "Drink at least eight glasses of water each day." e. "Make a list of reasons you want to stop smoking." f. "Set a quit date and stick to it."

A nurse assesses a client who is prescribed alosetron. Which assessment question would the nurse ask this client before starting the drug? a. "Have you been experiencing any constipation?" b. "Are you eating a diet high in fiber and fluids?" c. "Do you have a history of high blood pressure?" d. "What vitamins and supplements are you taking?"

a. "Have you been experiencing any constipation?

A client comes into the emergency department with a serum creatinine of 2.2 mg/dL (1944 mcmol/L) and a blood urea nitrogen (BUN) of 24 mL/dL (8.57 mmol/L). What question would the nurse ask first when taking this client's history? a. "Have you been taking any aspirin, ibuprofen, or naproxen recently?" b. "Do you have anyone in your family with renal failure?" c. "Have you had a diet that is low in protein recently?" d. "Has a relative had a kidney transplant lately?"

a. "Have you been taking any aspirin, ibuprofen, or naproxen recently?"

A nurse cares for a patient who is prescribed pioglitazone. After 6 months of therapy, the client reports that he has a new onset of ankle edema. What assessment question would the nurse take? a. "Have you gained unexpected weight this week?" b. "Has your urinary output declined recently?" c. "Have you had fever and achiness this week?" d. "Have you had abdominal pain recently?"

a. "Have you gained unexpected weight this week?"

The nurse is caring for a client with hepatitis C. The client's brother states, "I do not want to get this infection, so I'm not going into his hospital room." How would the nurse respond? a. "Hepatitis C is not spread through casual contact." b. "If you wear a gown and gloves, you will not get this virus." c. "This virus is only transmitted through a fecal specimen." d. "I can give you an update on your brother's status from here."

a. "Hepatitis C is not spread through casual contact." Although family members may be afraid that they will contract hepatitis C, the nurse would educate them about how the virus is spread. Hepatitis C is spread via blood-to-blood transmission and is associated with illicit IV drug needle sharing, blood and organ transplantation, accidental needlesticks, unsanitary tattoo equipment, and sharing of intranasal drug paraphernalia. Wearing a gown and gloves will not decrease the transmission of this virus. Hepatitis C is not spread through casual contact or a fecal specimen. The nurse would be violating privacy laws by sharing the client's status with the brother.

The nurse is assessing a client with hepatitis C. The client asks the nurse how it was possible to have this disease. What questions might the nurse ask to help the client determine how the disease was contracted? (Select all that apply.) a. "How old are you?" b. "Do you work in health care? c. "Are you receiving hemodialysis?" d. "Do you use IV drugs?" e. "Did you receive blood before 1992?" f. "Have you even been in prison or jail?

a. "How old are you?" b. "Do you work in health care? c. "Are you receiving hemodialysis?" d. "Do you use IV drugs?" e. "Did you receive blood before 1992?" f. "Have you even been in prison or jail? The nurse would ask all of these questions because "baby boomers," people who use illicit drugs, people on hemodialysis, health workers, and prisoners are at a very high risk for hepatitis C. Additionally, individuals who received blood, blood products, or an organ transplant prior to 1992 before bloodborne disease screening of these products was mandated are at risk for hepatitis C

A client is discharged to home after a modified radical mastectomy with two drainage tubes. Which statement by the client would indicate that further teaching is needed? a. "I am glad that these tubes will fall out at home when I finally shower." b. "I should measure the drainage each day to make sure it is less than an ounce (30 mL)." c. "I should be careful how I lie in bed so that I will not kink the tubing." d. "If there is a foul odor from the drainage, I will contact my primary health care provider.

a. "I am glad that these tubes will fall out at home when I finally shower."

A nurse teaches a community health class about water safety. Which statement by a participant indicates that additional teaching is needed? a. "I can go swimming all by myself because I am a certified lifeguard." b. "I cannot leave my toddler alone in the bathtub for even a minute." c. "I will appoint one adult to supervise the pool at all times during a party." d. "I will make sure that there is a phone near my pool in case of an emergency."

a. "I can go swimming all by myself because I am a certified lifeguard." People would never swim alone, regardless of lifeguard status. The other statements indicate good understanding of the teaching.

A client has recurrent vulvovaginitis. Which statements by the client indicate a need for further teaching? (Select all that apply.) a. "I can take a long, hot bath to relieve itching." b. "I need to take all of my antibiotics as prescribed." c. "I should avoid having sex until my infection is gone." d. "I should not douche or use feminine hygiene sprays." e. "I should use antibacterial soap to clean the area." f. "I should switch to wearing only cotton underwear."

a. "I can take a long, hot bath to relieve itching." b. "I need to take all of my antibiotics as prescribed." e. "I should use antibacterial soap to clean the area."

A nurse cares for a client with end-stage pancreatic cancer. The client asks, "Why is this happening to me?" How would the nurse respond? a. "I don't know. I wish I had an answer for you, but I don't." b. "It's important to keep a positive attitude for your family right now." c. "Scientists have not determined why cancer develops in certain people." d. "I think that this is a trial so you can become a better person because of it."

a. "I don't know. I wish I had an answer for you, but I don't." The client is not asking the nurse to actually explain why the cancer has occurred. The client may be expressing his or her feelings of confusion, frustration, distress, and grief related to this diagnosis. Reminding the client to keep a positive attitude for his or her family does not address the client's emotions or current concerns. The nurse would validate that there is no easy or straightforward answer as to why the client has cancer. Telling a client that cancer is a trial is untrue and may negatively impact the client-nurse relationship

A nurse is assessing coping in older women in a support group for recent widows. Which statement by a participant best indicates potential for successful coping? a. "I have had the same best friend for decades." b. "I think I am coping very well on my own." c. "My kids come to see me every weekend." d. "Oh, I have lots of friends at the senior center."

a. "I have had the same best friend for decades." Friendship and support enhance coping. The quality of the relationship is what is most important, however. People who have close, intimate, stable relationships with others in whom they confide are more likely to cope with crisis. The person who is "coping well on my own" may actually need resources to help with this transition. Having children visit is important but not as important as intimate, long-term friendships. "Friends at the senior center" may refer to good acquaintances and not real friends.

A nurse collaborates with a respiratory therapist to complete pulmonary function tests (PFTs) for a client. Which statements would the nurse include in communications with the respiratory therapist prior to the tests? (Select all that apply.) a. "I held the client's morning bronchodilator medication." b. "The client is ready to go down to radiology for this examination." c. "Physical therapy states the client can run on a treadmill." d. "I advised the client not to smoke for 6 hours prior to the test." e. "The client is alert and can follow your commands."

a. "I held the client's morning bronchodilator medication." . "I advised the client not to smoke for 6 hours prior to the test." e. "The client is alert and can follow your commands."

After teaching a client with a history of renal calculi, the nurse assesses the client's understanding. Which statement made by the client indicates a correct understanding of the teaching? a. "I should drink at least 3 L of fluid every day." b. "I will eliminate all dairy or sources of calcium from my diet." c. "Aspirin and aspirin-containing products can lead to stones." d. "The doctor can give me antibiotics at the first sign of a stone."

a. "I should drink at least 3 L of fluid every day."

After teaching a client who has stress incontinence, the nurse assesses the client's understanding. Which statement made by the client indicates a need for further teaching? a. "I will limit my total intake of fluids." b. "I must avoid drinking alcoholic beverages." c. "I must avoid drinking caffeinated beverages." d. "I shall try to lose about 10% of my body weight."

a. "I will limit my total intake of fluids."

After teaching a client who has chronic pancreatitis and will be discharged with enzyme replacement therapy, a nurse assesses the client's understanding. Which statement by the client indicates a need for further teaching? (Select all that apply.) a. "I will take the enzymes between meals." b. "The enteric-coated preparations cannot be crushed." c. "Swallowing the tables without chewing is best." d. "I will wipe my lips after taking the enzymes." e. "Enzymes should be taken with high-protein foods."

a. "I will take the enzymes between meals." e. "Enzymes should be taken with high-protein foods." Client teaching related to self-management of enzyme replacement therapy would include taking the enzymes with meals and snacks but not mixing enzyme preparations with protein-containing foods. Clients would not crush enteric-coated preparations and should swallow tablets without chewing to minimize oral irritation and allow the drug to be released slowly. Wiping lips after taking enzymes also minimizes skin irritation

After teaching a client with congestive heart failure (CHF), the nurse assesses the client's understanding. Which client statements indicate a correct understanding of the teaching related to nutritional intake? (Select all that apply.) a. "I'll read the nutritional labels on food items for salt content." b. "I will drink at least 3 liters of water each day." c. "Using salt in moderation will reduce the workload of my heart." d. "I will eat oatmeal for breakfast instead of ham and eggs." e. "Substituting fresh vegetables for canned ones will lower my salt intake."

a. "I'll read the nutritional labels on food items for salt content." d. "I will eat oatmeal for breakfast instead of ham and eggs." e. "Substituting fresh vegetables for canned ones will lower my salt intake."

A nurse cares for a client who states, "My husband is repulsed by my colostomy and refuses to be intimate with me." How would the nurse respond? a. "Let's talk to the ostomy nurse to help you and your husband work through this." b. "You could try to wear longer lingerie that will better hide the ostomy appliance." c. "You should empty the pouch first so it will be less noticeable for your husband." d. "If you are not careful, you can hurt the stoma if you engage in sexual activity.

a. "Let's talk to the ostomy nurse to help you and your husband work through this."

The nurse is planning health teaching for a client starting mirabegron for urinary incontinence. What health teaching would the nurse include? (Select all that apply.) a. "Monitor blood tests carefully if you are prescribed warfarin." b. "Avoid crowds and individuals with infection." c. "Report any fever to your primary health care provider." d. "Take your blood pressure frequently at home." e. "Report palpitations or chest soreness that may occur."

a. "Monitor blood tests carefully if you are prescribed warfarin." d. "Take your blood pressure frequently at home."

A nurse collaborates with an unlicensed assistive personnel (UAP) to provide care for a client with congestive heart failure. Which instructions should the nurse provide to the UAP when delegating care for this client? (Select all that apply.) a. "Reposition the client every 2 hours." b. "Teach the client to perform deep-breathing exercises." c. "Accurately record intake and output." d. "Use the same scale to weigh the client each morning." e. "Place the client on oxygen if the client becomes short of breath."

a. "Reposition the client every 2 hours." c. "Accurately record intake and output." d. "Use the same scale to weigh the client each morning."

A nurse teaches a client with polycystic kidney disease (PKD). Which statements would the nurse include in this client's discharge teaching? (Select all that apply.) a. "Take your blood pressure every morning." b. "Weigh yourself at the same time each day." c. "Adjust your diet to prevent diarrhea." d. "Contact your provider if you have visual disturbances." e. "Assess your urine for renal stones."

a. "Take your blood pressure every morning." b. "Weigh yourself at the same time each day." d. "Contact your provider if you have visual disturbances."

After teaching a client with diabetes mellitus to inject insulin, the nurse assesses the client's understanding. Which statement made by the client indicates a need for further teaching? a. "The lower abdomen is the best location because it is closest to the pancreas." b. "I can reach my thigh the best, so I will use the different areas of my thighs." c. "By rotating the sites in one area, my chance of having a reaction is decreased." d. "Changing injection sites from the thigh to the arm will change absorption rates."

a. "The lower abdomen is the best location because it is closest to the pancreas." .

The nurse is teaching a 45-year-old woman about her fibrocystic breast changes. Which statement by the client indicates a lack of understanding? a. "This condition will become malignant over time." b. "I understand that hormone-based drugs have serious adverse effects." c. "One cup of coffee in the morning should be enough for me." d. "This condition makes it more difficult to examine my breasts."

a. "This condition will become malignant over time."

A nurse teaches a young female client who is prescribed cephalexin for a urinary tract infection. Which statement would the nurse include in this client's teaching? a. "Use a second form of birth control while on this medication." b. "You will experience increased menstrual bleeding while on this drug." c. "You may experience an irregular heartbeat while on this drug." d. "Watch for blood in your urine while taking this medication."

a. "Use a second form of birth control while on this medication."

The nurse teaches a community group ways to prevent Escherichia coli infection. Which statements would the nurse include in this group's teaching? (Select all that apply.) a. "Wash your hands after any contact with animals." b. "It is not necessary to buy a meat thermometer." c. "Stay away from people who are ill with diarrhea." d. "Use separate cutting boards for meat and vegetables." e. "Avoid swimming in backyard pools and using hot tubs."

a. "Wash your hands after any contact with animals." d. "Use separate cutting boards for meat and vegetables Washing hands after contact with animals and using separate cutting boards for meat and other foods will help prevent E. coli infection. The other statements are not related to preventing E. coli infection

A nurse is teaching a wilderness survival class. Which statements would the nurse include about the prevention of hypothermia and frostbite? (Select all that apply.) a. "Wear synthetic clothing instead of cotton to keep your skin dry." b. "Drink plenty of fluids. Brandy can be used to keep your body warm." c. "Remove your hat when exercising to prevent overheating." d. "Wear sunglasses to protect skin and eyes from harmful rays." e. "Know your physical limits. Come in out of the cold when limits are reached." f. "Change your gloves and socks if they become wet."

a. "Wear synthetic clothing instead of cotton to keep your skin dry." d. "Wear sunglasses to protect skin and eyes from harmful rays." e. "Know your physical limits. Come in out of the cold when limits are reached." f. "Change your gloves and socks if they become wet." To prevent hypothermia and frostbite, the nurse would teach patients to wear synthetic clothing (which moves moisture away from the body and dries quickly), layer clothing, and a hat, facemask, sunscreen, and sunglasses. The client would also be taught to drink plenty of fluids, but to avoid alcohol when participating in winter activities. Clients need to know their physical limits and come in out of the cold when these limits have been reached. Wet clothing contributes to heat loss so clients would be taught to change any clothing that becomes wet.

A nurse cares for a client with right-sided heart failure. The client asks, "Why do I need to weigh myself every day?" How should the nurse respond? a. "Weight is the best indication that you are gaining or losing fluid." b. "Daily weights will help us make sure that you're eating properly." c. "The hospital requires that all inpatients be weighed daily." d. "You need to lose weight to decrease the incidence of heart failure."

a. "Weight is the best indication that you are gaining or losing fluid."

While obtaining a client's health history, the client states, "I am allergic to avocados, molds, and grass." Which responses by the nurse are best? (Select all that apply.) a. "What happens when you are exposed to those things? b. "How do you treat these allergies?" c. "When was the last time you ate foods containing avocados?" d. "I will document this in your record so all so everyone knows." e. "Have you ever been in the hospital after an allergic response?" f. "How do manage to avoid grass and mold?"

a. "What happens when you are exposed to those things? b. "How do you treat these allergies?" d. "I will document this in your record so all so everyone knows." e. "Have you ever been in the hospital after an allergic response?"

A nurse assesses a client with irritable bowel syndrome (IBS). Which questions would the nurse include in this client's assessment? (Select all that apply.) a. "Which food types cause an exacerbation of symptoms?" b. "Where is your pain or discomfort and what does it feel like?" c. "Have you lost a significant amount of weight lately?" d. "Are your stools soft, watery, and black?" e. "Do you often experience nausea and vomiting"

a. "Which food types cause an exacerbation of symptoms?" b. "Where is your pain or discomfort and what does it feel like?

A nurse cares for an older adult client with heart failure. The client states, "I don't know what to do. I don't want to be a burden to my daughter, but I can't do it alone. Maybe I should die." How should the nurse respond? a. "Would you like to talk more about this?" b. "You are lucky to have such a devoted daughter." c. "It is normal to feel as though you are a burden." d. "Would you like to meet with the chaplain?"

a. "Would you like to talk more about this?"

4. A nurse teaches a client who is being discharged after a jaw wiring for a mandibular fracture. Which statements would the nurse include in this patient's teaching? (Select all that apply.) a. "You will need to cut the wires if you start vomiting." b. "Eat six soft or liquid meals each day while recovering." c. "Use a Waterpik for dental hygiene until you can brush again. d. "Sleep in a semi-Fowler position after the surgery." e. "Gargle with mouthwash that contains hydrogen peroxide once a day."

a. "You will need to cut the wires if you start vomiting." b. "Eat six soft or liquid meals each day while recovering." c. "Use a Waterpik for dental hygiene until you can brush again. d. "Sleep in a semi-Fowler position after the surgery." The client needs to know how to cut the wires in case of emergency. If the client vomits, he or she may aspirate. The client would also be taught to eat soft or liquid meals multiple times a day, irrigate the mouth with a Waterpik to prevent infection, and sleep in a semi-Fowler position to assist in avoiding aspiration. Mouthwash with hydrogen peroxide is not a recommendation.

A nurse cares for a client who has a family history of diabetes mellitus. The client states, "My father has type 1 diabetes mellitus. Will I develop this disease as well?" How would the nurse respond? a. "Your risk of diabetes is higher than the general population, but it may not occur." b. "No genetic risk is associated with the development of type 1 diabetes mellitus." c. "The risk for becoming a diabetic is 50% because of how it is inherited." d. "Female children do not inherit diabetes mellitus, but male children will."

a. "Your risk of diabetes is higher than the general population, but it may not occur."

A home health care nurse assesses an older adult for the intake of nutrients needed in larger amounts than in younger adults. Which foods found in an older adult's kitchen might indicate an adequate intake of these nutrients? (Select all that apply.) a. 1% milk b. Carrots c. Lean ground beef d. Oranges e. Vitamin D supplements f. Cheese sticks

a. 1% milk b. Carrots d. Oranges e. Vitamin D supplements Older adults need increased amounts of calcium; vitamins A, C, and D; and fiber. Milk and cheese have calcium; carrots have vitamin A; vitamin D supplement has vitamin D; and oranges have vitamin C. Lean ground beef is healthier than more fatty cuts, but does not contain these needed nutrients.

The nurse in the emergency department would arrange to transfer which burned clients to a burn center? (Select all that apply.) a. 15% partial-thickness burn b. Lightening injury c. 7% partial-thickness burn d. History of pulmonary edema e. Healthy 67 year old f. 4% partial-thickness burn to perineum

a. 15% partial-thickness burn b. Lightening injury d. History of pulmonary edema e. Healthy 67 year old f. 4% partial-thickness burn to perineum Clients with major burns are transferred to a burn center for specialized care. These include any partial-thickness burn over 10% TBSA; any lightening injury; a burn injury in a client with a history of pre-existing conditions that could complicate care or prolong recovery; adults over the age of 60; and burns to the face, hands, feet, genitalia, perineum, or major joints. The client with a 7% partial-thickness burn could be cared for in a hospital or a burn center.

1. A nurse is providing pneumonia vaccinations in a community setting. Due to limited finances, the event organizers must limit giving the vaccination to priority groups. What clients would be considered a priority when administering the pneumonia vaccination? (Select all that apply.) a. A 22-year-old client with asthma b. Client who had a cholecystectomy last year c. Client with well-controlled diabetes d. Healthy 72-year-old client e. Client who is taking medication for hypertension

a. A 22-year-old client with asthma c. Client with well-controlled diabetes d. Healthy 72-year-old client e. Client who is taking medication for hypertension Clients over 65 years of age and any client (no matter what age) with a chronic health condition would be considered a priority for a pneumonia vaccination. Having a cholecystectomy a year ago does not qualify as a chronic health condition.

5. A nurse is assessing clients on a rehabilitation unit. Which clients are at greatest risk for airway loss related to aspirated oral and nasopharyngeal secretions? (Select all that apply.) a. A 24 year old with a traumatic brain injury b. A 36 year old who fractured his left femur c. A 58 year old getting radiation therapy d. A 66 year old who is a quadriplegic e. An 80-year-old who is aphasic

a. A 24 year old with a traumatic brain injury c. A 58 year old getting radiation therapy d. A 66 year old who is a quadriplegic e. An 80-year-old who is aphasic Thickly crusted, dry secretions that potentially can cause asphyxiation and airway obstruction (inspissated secretions or mucoid impaction) are seen most often in clients who have an altered mental status and level of consciousness (brain injury), are dehydrated, are unable to communicate (aphasic), are unable to cough effectively (quadriplegic), or are at risk for aspiration. The clients with the femur fracture and receiving radiation therapy are not as high of a risk. The location of the radiation is not known.

A nurse triages clients arriving at the hospital after a mass casualty. Which clients are correctly classified? (Select all that apply.) a. A 35-year-old female with severe chest pain: red tag b. A 42-year-old male with full-thickness body burns: green tag c. A 55-year-old female with a scalp laceration: black tag d. A 60-year-old male with an open fracture with distal pulses: yellow tag e. An 88-year-old male with shortness of breath and chest bruises: green tag f. A 33-year-old male unconscious with bilateral leg amputations: yellow tag

a. A 35-year-old female with severe chest pain: red tag d. A 60-year-old male with an open fracture with distal pulses: yellow tag Red-tagged clients need immediate care due to life-threatening injuries. A client with severe chest pain would receive a red tag. Yellow-tagged clients have major injuries that need to be treated within 30 minutes to 2 hours. A client with an open fracture with distal pulses would receive a yellow tag. The client with full-thickness body burns would receive a black tag. The client with a scalp laceration would receive a green tag, and the client with shortness of breath would receive a red tag. The client with the amputated legs will probably be black tagged if the unconsciousness is from massive blood loss.

A nurse assesses clients on a cardiac unit. Which clients should the nurse identify as at greatest risk for the development of acute pericarditis? (Select all that apply.) a. A 36-year-old woman with systemic lupus erythematosus (SLE) b. A 42-year-old man recovering from coronary artery bypass graft surgery c. A 59-year-old woman recovering from a hysterectomy d. An 80-year-old man with a bacterial infection of the respiratory tract e. An 88-year-old woman with a stage III sacral ulcer

a. A 36-year-old woman with systemic lupus erythematosus (SLE) b. A 42-year-old man recovering from coronary artery bypass graft surgery d. An 80-year-old man with a bacterial infection of the respiratory tract

A nurse assesses adults at a health fair. Which adults would the nurse counsel to be tested for diabetes? (Select all that apply.) a. A 56-year-old African-American male b. A 22-year-old female with a 30-lb (13.6 kg) weight gain during pregnancy c. A 60-year-old male with a history of liver trauma d. A 48-year-old female with a sedentary lifestyle e. A 50-year-old male with a body mass index greater than 25 kg/m2 f. A 28-year-old female who gave birth to a baby weighing 9.2 lb (4.2 kg)

a. A 56-year-old African-American male d. A 48-year-old female with a sedentary lifestyle e. A 50-year-old male with a body mass index greater than 25 kg/m2 f. A 28-year-old female who gave birth to a baby weighing 9.2 lb (4.2 kg)

5. A nurse is teaching a client about possible complications and hazards of home oxygen therapy. About which complications does the nurse plan to teach the client? (Select all that apply.) a. Absorptive atelectasis b. Combustion c. Dried mucous membranes d. Alveolar recruitment e. Toxicity

a. Absorptive atelectasis b. Combustion c. Dried mucous membranes e. Toxicity Complications of oxygen therapy include absorptive atelectasis, combustion, dried mucous membranes, and oxygen toxicity. Alveolar recruitment may be a benefit of high-flow nasal cannulas such as Vapotherm, which both humidifies and warms the oxygen.

A client with a new pulmonary embolism (PE) is anxious. What nursing actions are most appropriate? (Select all that apply.) a. Acknowledge the frightening nature of the illness. b. Delegate a back rub to the assistive personnel (AP). c. Give simple explanations of what is happening. d. Request a prescription for antianxiety medication. e. Stay with the client and speak in a quiet, calm voice.

a. Acknowledge the frightening nature of the illness. b. Delegate a back rub to the assistive personnel (AP). c. Give simple explanations of what is happening

A woman is interested in alternative and complementary treatments for the nausea and vomiting caused by the side effects of chemotherapy for breast cancer. Which therapies wound the nurse suggest? (Select all that apply.) a. Acupuncture b. Chiropractic c. Journaling d. Aromatherapy e. Shiatsu f. Black cohosh

a. Acupuncture d. Aromatherapy e. Shiatsu

The nurse caring for mechanically ventilated clients uses best practices to prevent ventilator-associated pneumonia. What actions are included in this practice? (Select all that apply.) a. Adherence to proper hand hygiene b. Administering antiulcer medication c. Elevating the head of the bed d. Providing oral care per protocol e. Suctioning the client on a regular schedule f. Turning and positioning the client at least every 2 hours

a. Adherence to proper hand hygiene b. Administering antiulcer medication c. Elevating the head of the bed d. Providing oral care per protocol f. Turning and positioning the client at least every 2 hours

A client is undergoing hemodialysis. The client's blood pressure at the beginning of the procedure was 136/88 mm Hg, and now it is 110/54 mm Hg. What actions would the nurse perform to maintain blood pressure? (Select all that apply.) a. Adjust the rate of extracorporeal blood flow. b. Place the patient in the Trendelenburg position. c. Stop the hemodialysis treatment. d. Administer a 250-mL bolus of normal saline. e. Contact the primary health care provider.

a. Adjust the rate of extracorporeal blood flow. b. Place the patient in the Trendelenburg position. d. Administer a 250-mL bolus of normal saline.

A nurse assesses a client who has diabetes mellitus and notes that the client is awake and alert, but shaky, diaphoretic, and weak. Five minutes after administering a half-cup (120 mL) of orange juice, the client's signs and symptoms have not changed. What action would the nurse take next? a. Administer another half-cup (120 mL) of orange juice. b. Administer a half-ampule of dextrose 50% intravenously. c. Administer 10 units of regular insulin subcutaneously. d. Administer 1 mg of glucagon intramuscularly.

a. Administer another half-cup (120 mL) of orange juice.

An emergency department nurse cares for a middle-age mountain climber who is confused, ataxic, and exhibits impaired judgement. After administering oxygen, which intervention would the nurse implement next? a. Administer dexamethasone. b. Complete a mini mental state examination. c. Prepare the client for computed tomography of the brain. d. Request a psychiatric consult.

a. Administer dexamethasone. The client is exhibiting signs of mountain sickness and high-altitude cerebral edema (HACE). Dexamethasone reduces cerebral edema by acting as an anti-inflammatory in the central nervous system. The other interventions will not specifically treat HACE, although a thorough mental status exam would be performed.

The nurse is caring for a client who has severe hypoglycemia and is experiencing a seizure. What actions will the nurse take at this time? (Select all that apply.) a. Administer glucagon 1 mg subcutaneously. b. Be sure the bed side rails are in the up position. c. Notify the primary health care provider immediately. d. Monitor the client's blood glucose level. e. Increase the intravenous infusion rate immediately.

a. Administer glucagon 1 mg subcutaneously. b. Be sure the bed side rails are in the up position. c. Notify the primary health care provider immediately. d. Monitor the client's blood glucose level.

A primary health care provider prescribes a rewarming bath for a client who presents with Grade 3 frostbite. What action would the nurse take prior to starting this treatment? a. Administer intravenous morphine. b. Wrap the limb with a compression dressing. c. Massage the frostbitten areas. d. Assess the limb for compartment syndrome.

a. Administer intravenous morphine. Rapid rewarming in a water bath is recommended for all instances of partial-thickness and full-thickness frostbite. Patients experience severe pain during the rewarming process and nurses would administer intravenous analgesics.

18. A nurse admits a client from the emergency department. Client data are listed below: History Physical Assessment Laboratory Values What action by the nurse is the priority? ° 70 years of age ° History of diabetes ° On insulin twice a day ° Reports new onset dyspnea and productive cough ° Crackles and rhonchi heard throughout the lungs ° Dullness to percussion LLL ° Afebrile ° Oriented to person only ° WBC 5,200/mm3 (5.2 109/L) ° PaO2 on room air 85 mm Hg a. Administer oxygen at 4 L per nasal cannula. b. Begin broad-spectrum antibiotics. c. Collect a sputum sample for culture. d. Start an IV of normal saline at 50 mL/hr.

a. Administer oxygen at 4 L per nasal cannula. All actions are appropriate for this client who has signs and symptoms of pneumonia. However, airway and breathing come first, so begin oxygen administration and titrate it to maintain saturations greater than 95%. Start the IV and collect a sputum culture, and then begin antibiotics.

An emergency department nurse plans care for a client who is admitted with heat stroke. Which interventions does the nurse include in this patient's plan of care? (Select all that apply.) a. Administer oxygen via mask or nasal cannula. b. Administer ibuprofen, an antipyretic medication. c. Apply cooling techniques until core body temperature is less than 101° F (38.3° C). d. Infuse 0.9% sodium chloride via a large-bore intravenous cannula. e. Obtain baseline serum electrolytes and cardiac enzymes. f. Insert an indwelling urinary catheter for urine output measurements.

a. Administer oxygen via mask or nasal cannula. d. Infuse 0.9% sodium chloride via a large-bore intravenous cannula. e. Obtain baseline serum electrolytes and cardiac enzymes. Heat stroke is a medical emergency. Oxygen therapy and intravenous fluids would be provided, and baseline laboratory tests would be performed as quickly as possible. Urinary output is measured via an indwelling urinary catheter. The client would be cooled until core body temperature is reduced to 102° F (38.9° C). Antipyretics would not be administered.

A nurse cares for an older adult who is admitted to the hospital with complications of diverticulitis. Which actions would the nurse include in the client's plan of care? (Select all that apply.) a. Administer pain medications as prescribed. b. Palpate the abdomen for distention. c. Assess for sudden changes in mental status. d. Provide the client with a high-fiber diet. e. Evaluate stools for occult blood

a. Administer pain medications as prescribed. b. Palpate the abdomen for distention. c. Assess for sudden changes in mental status. e. Evaluate stools for occult blood When caring for an older adult who has diverticulitis, the nurse would administer analgesics as prescribed, palpate the abdomen for distention and tenderness, assess for confusion and sudden changes in mental status, and check stools for occult or frank bleeding. A low-fiber/residue diet would be provided when symptoms are present and a high-fiber diet when inflammation resolves

The nurse is taking a history of a 68-year-old woman. What assessment findings would indicate a high risk for the development of breast cancer? (Select all that apply.) a. Age greater than 65 years b. Increased breast density c. Osteoporosis d. Multiparity e. Genetic factors f. Early menarche

a. Age greater than 65 years b. Increased breast density e. Genetic factors f. Early menarche

A 70-kg adult client with chronic kidney disease (CKD) is on a 40-g protein diet. The patient has a reduced glomerular filtration rate and is not undergoing dialysis. Which result would be of most concern to the nurse? a. Albumin level of 2.5 g/dL (3.63 mcmol/L) b. Phosphorus level of 5 mg/dL (1.62 mmol/L) c. Sodium level of 135 mEq/L (135 mmol/L) d. Potassium level of 5.5 mEq/L(5.5mmol/)

a. Albumin level of 2.5 g/dL (3.63 mcmol/L)

A client is receiving norepinephrine for shock. What assessment finding best indicates a therapeutic effect from this drug? a. Alert and oriented, answering questions b. Client denies chest pain or chest pressure c. IV site without redness or swelling d. Urine output of 30 mL/hr for 2 hours

a. Alert and oriented, answering questions Normal cognitive function is a good indicator that the client is receiving the benefits of norepinephrine. The brain is very sensitive to changes in oxygenation and perfusion. Norepinephrine can cause chest pain as an adverse reaction, so the absence of chest pain is good but does not indicate therapeutic effect. The IV site is normal. The urine output is normal, but only minimally so.

A nurse is caring for a client in acute respiratory failure who is on mechanical ventilation. What actions will promote comfort in this client? (Select all that apply.) a. Allow visitors at the client's bedside. b. Ensure that the client can communicate if awake. c. Keep the television tuned to a favorite channel. d. Provide back and hand massages when turning. e. Turn the client every 2 hours or more.

a. Allow visitors at the client's bedside. b. Ensure that the client can communicate if awake. d. Provide back and hand massages when turning. e. Turn the client every 2 hours or more.

The nurse caring frequently for older adults in the hospital is aware of risk factors that place them at a higher risk for shock. For what factors would the nurse assess? (Select all that apply.) a. Altered mobility/immobility b. Decreased thirst response c. Diminished immune response d. Malnutrition e. Overhydration f. Use of diuretics

a. Altered mobility/immobility b. Decreased thirst response c. Diminished immune response d. Malnutrition f. Use of diuretics

11. The emergency department (ED) manager is reviewing client charts to determine how well the staff perform when treating clients with community-acquired pneumonia. What outcome demonstrates that goals for this client type have been met? a. Antibiotics started before admission. b. Blood cultures obtained within 20 minutes. c. Chest x-ray obtained within 30 minutes. d. Pulse oximetry obtained on all clients.

a. Antibiotics started before admission Goals for treatment of community-acquired pneumonia include initiating antibiotics prior to inclient admission or within 6 hours of presentation to the ED. Timely collection of blood cultures, chest x-ray, and pulse oximetry are important as well but do not coincide with established goals.

The nurse manager is conducting an annual evaluation of a staff nurse and is appraising the nurse's clinical reasoning. What nurse actions does the manager observe to help form this judgment? (Select all that apply.) a. Anticipating consequences of actions b. Delegating appropriately c. Interpreting data d. Noticing cues e. Setting priorities

a. Anticipating consequences of actions c. Interpreting data d. Noticing cues e. Setting priorities The phases of clinical reasoning include assessing (noticing cues), analyzing (interpreting data), planning (anticipating consequences and setting priorities), implementing, and evaluating. Delegating appropriately is not included in this model.

The nurse is caring for a client who just had a minimally invasive inguinal hernia repair. Which nursing actions would the nurse implement? (Select all that apply.) a. Apply ice to the surgical area for the first 24 hours after surgery. b. Encourage ambulation with assistance within the first few hours after surgery. c. Encourage deep breathing after surgery but teach the client to avoid coughing. d. Assess vital signs frequently for the first few hours after surgery. e. Teach the client to rest for several days after surgery when at home. f. Teach the client not to lift more than 10 lb (4.5 kg) until allowed by the surgeon.

a. Apply ice to the surgical area for the first 24 hours after surgery. b. Encourage ambulation with assistance within the first few hours after surgery. c. Encourage deep breathing after surgery but teach the client to avoid coughing. d. Assess vital signs frequently for the first few hours after surgery. e. Teach the client to rest for several days after surgery when at home. f. Teach the client not to lift more than 10 lb (4.5 kg) until allowed by the surgeon.

8. A nurse is caring for a client who has just experienced a 90-second tonic-clonic seizure. The clients arterial blood gas values are pH 6.88, PaO2 50 mm Hg, PaCO2 60 mm Hg, and HCO3 22 mEq/L. Which action should the nurse take first? a. Apply oxygen by mask or nasal cannula. b. Apply a paper bag over the clients nose and mouth. c. Administer 50 mL of sodium bicarbonate intravenously. d. Administer 50 mL of 20% glucose and 20 units of regular insulin.

a. Apply oxygen by mask or nasal cannula The client has experienced a combination of metabolic and acute respiratory acidosis through heavy skeletal muscle contractions and no gas exchange. When the seizures have stopped and the client can breathe again, the fastest way to return acid-base balance is to administer oxygen. Applying a paper bag over the clients nose and mouth would worsen the acidosis. Sodium bicarbonate should not be administered because the clients arterial bicarbonate level is normal. Glucose and insulin are administered together to decrease serum potassium levels. This action is not appropriate based on the information provided.

A nurse is caring for a client who has a tracheostomy tube. What actions may the nurse delegate to assistive personnel (AP)? (Select all that apply.) a. Applying water-soluble lip balm to the client's lips b. Ensuring that the humidification provided is adequate c. Performing oral care with alcohol-based mouthwash d. Reminding the client to cough and deep breathe often e. Suctioning excess secretions through the tracheostomy f. Holding the new tracheostomy tube while the RN changes the ties

a. Applying water-soluble lip balm to the client's lips d. Reminding the client to cough and deep breathe often The AP can perform hygiene measures such as applying lip balm and reinforce teaching such as reminding the client to perform coughing and deep-breathing exercises. Oral care can be accomplished with normal saline, not products that dry the mouth. Ensuring that the humidity is adequate and suctioning through the tracheostomy are nursing functions. When needed, a second licensed person assists with holding the tracheostomy tube during tie changes; some hospitals require a second licensed person during the first 72 hours after placement.

The nurse assesses a client who has chronic pancreatitis. What assessment findings would the nurse expect for this client? (Select all that apply.) a. Ascites b. Weight gain c. Steatorrhea d. Jaundice e. Polydipsia f. Polyuria

a. Ascites c. Steatorrhea d. Jaundice e. Polydipsia f. Polyuria The client who has chronic pancreatitis has all of these signs and symptoms except he or she loses weight. Ascites and jaundice result from biliary obstruction; ascites is associated with portal hypertension. Steatorrhea is fatty stool that occurs because lipase is not available in the duodenum; because it is released by the disease pancreas into the bloodstream. Polydipsia, polyuria, and polyphagia result from diabetes mellitus, a common problem seen in clients whose pancreas is unable to release adequate amounts of insulin

6. A nurse plans care for a client who has chronic obstructive pulmonary disease and thick, tenacious secretions. Which interventions would the nurse include in this client's plan of care? (Select all that apply.) a. Ask the client to drink 2 L of fluids daily. b. Add humidity to the prescribed oxygen. c. Suction the client every 2 to 3 hours. d. Use a vibrating chest physiotherapy device. e. Encourage diaphragmatic breathing. f. Administer the ordered mucolytic agent.

a. Ask the client to drink 2 L of fluids daily. b. Add humidity to the prescribed oxygen. d. Use a vibrating chest physiotherapy device. f. Administer the ordered mucolytic agent. Interventions to decrease thick tenacious secretions include maintaining adequate hydration and providing humidified oxygen. These actions will help to thin secretions, making them easier to remove by coughing. The use of a vibrating chest physiotherapy device can also help clients remove thick secretions but is usually used in clients with cystic fibrosis. Mucolytic agents help thin secretions, making them easier to bring up. Although suctioning may assist with the removal of secretions, frequent suctioning can cause airway trauma and does not support the client's ability to successfully remove secretions through normal coughing. Diaphragmatic breathing is not used to improve the removal of thick secretions.

15. A client has been hospitalized with tuberculosis (TB). The client's spouse is fearful of entering the room where the client is in isolation and refuses to visit. What action by the nurse is best? a. Ask the spouse to explain the fear of visiting in further detail. b. Inform the spouse that the precautions are meant to keep other clients safe. c. Show the spouse how to follow the Isolation Precautions to avoid illness. d. Tell the spouse that he or she has already been exposed, so it's safe to visit.

a. Ask the spouse to explain the fear of visiting in further detail. The nurse needs to obtain further information about the spouse's specific fears so they can be addressed. This will decrease stress and permit visitation, which will be beneficial for both client and spouse. Precautions for TB prevent transmission to all who come into contact with the client. Explaining Isolation Precautions and what to do when entering the room will be helpful, but this is too narrow in scope to be the best answer. Telling the spouse that it's safe to visit is demeaning of the spouse's feelings.

7. A student nurse asks the faculty to explain best practices when communicating with a person from the lesbian, gay, bisexual, transgender, and queer/questioning (LGBTQ) community. What answer by the faculty is most accurate? a. Avoid embarrassing the client by asking questions. a. Asks if the client has questions before signing a consent c. Most LGBTQ people do not want to share information. d. No differences exist in communicating with this population.

a. Asks if the client has questions before signing a consent

The nurse is caring for a client who has a nasogastric tube (NGT). Which actions would the nurse take for client care? (Select all that apply.) a. Assess for proper placement of the tube every 4 hours or per agency policy. b. Flush the tube with water every hour to ensure patency. c. Secure the NG tube to the client's chin. d. Disconnect suction when auscultating bowel peristalsis. e. Monitor the client's skin around the tube site for irritation.

a. Assess for proper placement of the tube every 4 hours or per agency policy. d. Disconnect suction when auscultating bowel peristalsis. e. Monitor the client's skin around the tube site for irritation.

While assessing a client on a cardiac unit, a nurse identifies the presence of an S3 gallop. Which action should the nurse take next? a. Assess for symptoms of left-sided heart failure. b. Document this as a normal finding. c. Call the health care provider immediately. d. Transfer the client to the intensive care unit.

a. Assess for symptoms of left-sided heart failure. The presence of an S3 gallop is an early diastolic filling sound indicative of increasing left ventricular pressure and left ventricular failure.

7. Emergency medical technicians arrive at the emergency department with an unresponsive client who has an oxygen mask in place. Which action would the nurse take first? a. Assess that the client is breathing adequately. b. Insert a large-bore intravenous line. c. Place the client on a cardiac monitor. d. Assess for the best neurologic response.

a. Assess that the client is breathing adequately. After establishing an airway, the highest priority intervention in the primary survey is to establish that the client is breathing adequately. Even though this client has an oxygen mask on, he or she may not be breathing, or may be breathing inadequately with the device in place. Inserting an IV line and placing the client on a monitor would come after ensuring a patent airway and effective breathing.

13. A nurse evaluates a clients arterial blood gas values (ABGs): pH 7.30, PaO2 86 mm Hg, PaCO2 55 mm Hg, and HCO3 22 mEq/L. Which intervention should the nurse implement first? a. Assess the airway. b. Administer prescribed bronchodilators. c. Provide oxygen. d. Administer prescribed mucolytics.

a. Assess the airway. All interventions are important for clients with respiratory acidosis; this is indicated by the ABGs. However, the priority is assessing and maintaining an airway. Without a patent airway, other interventions will not be helpful.

A visiting nurse is in the home of an older adult and notes a 7-lb weight loss since last month's visit. What actions would the nurse perform first? (Select all that apply.) a. Assess the client's ability to drive or transportation alternatives. b. Determine if the client has dentures that fit appropriately. c. Encourage the client to continue the current exercise plan. d. Have the client complete a 3-day diet recall diary. e. Teach the client about proper nutrition in the older population.

a. Assess the client's ability to drive or transportation alternatives. b. Determine if the client has dentures that fit appropriately. d. Have the client complete a 3-day diet recall diary. Assessment is the first step of the nursing process and would be completed prior to intervening. Asking about transportation to get food, dentures, and normal food patterns would be part of an appropriate assessment for the client. There is no information in the question about the older adult needing to lose weight, so encouraging him or her to continue the current exercise regimen is premature and may not be appropriate. Teaching about proper nutrition is a good idea, but teaching needs to be tailored to the client's needs, which the nurse does not yet know.

A client has been brought to the emergency department after being covered in fertilizer after an explosion and fire at a warehouse. What action by the nurse is best? a. Assess the client's airway. b. Irrigate the client's skin. c. Brush any visible dust off the skin. d. Call poison control for guidance

a. Assess the client's airway With any burn client, assessing and maintaining the airway is paramount. Airway tissues can swell quickly, cutting off the airway. The fertilizer would then be brushed off before irrigation. Poison control may or may not need to be called.

A nurse admits a client who is experiencing an exacerbation of heart failure. Which action should the nurse take first? a. Assess the client's respiratory status. b. Draw blood to assess the client's serum electrolytes. c. Administer intravenous furosemide (Lasix). d. Ask the client about current medications.

a. Assess the client's respiratory status.

4. A nurse wishes to provide client-centered care in all interactions. Which action by the nurse best demonstrates this concept? a. Assesses for cultural influences affecting health care b. Ensures that all the clients' basic needs are met c. Tells the client and family about all upcoming tests d. Thoroughly orients the client and family to the room

a. Assesses for cultural influences affecting health care

The nurse caring for hospitalized clients includes which actions on their care plans to reduce the possibility of the clients developing shock? (Select all that apply.) a. Assessing and identifying clients at risk b. Monitoring the daily white blood cell count c. Performing proper hand hygiene d. Removing invasive lines as soon as possible e. Using aseptic technique during procedures f. limiting the clients visitors until more stable

a. Assessing and identifying clients at risk c. Performing proper hand hygiene d. Removing invasive lines as soon as possible e. Using aseptic technique during procedures

The nurse is caring for a client who has perineal surgical wound. Which actions would the nurse take to promote comfort and wound healing? (Select all that apply.) a. Assist the client into a side-lying position. b. Use a rubber donut device when sitting up. c. Apply warm compresses three to four times a day. d. Instruct the client to wear boxer shorts. e. Place an absorbent dressing over the wound.

a. Assist the client into a side-lying position. c. Apply warm compresses three to four times a day. e. Place an absorbent dressing over the wound.

4. A client has been diagnosed with an empyema. What interventions would the nurse anticipate providing to this client? (Select all that apply.) a. Assisting with chest tube insertion b. Facilitating pleural fluid sampling c. Performing frequent respiratory assessment d. Providing antipyretics as needed e. Suctioning deeply every 4 hours

a. Assisting with chest tube insertion b. Facilitating pleural fluid sampling c. Performing frequent respiratory assessment d. Providing antipyretics as needed The client with an empyema is often treated with chest tube insertion, which facilitates obtaining samples of the pleural fluid for analysis and re-expands the lungs. The nurse would perform frequent respiratory system assessments. Antipyretic medications are also used. Suction is only used when needed and is not done deeply to prevent tissue injury.

4. A nurse manager wants to improve hand-off communication among the staff. What actions by the manager would best help achieve this goal? (Select all that apply.) a. Attend hand-off rounds to coach and mentor. b. Conduct audits of staff using a new template. c. Create a template of topics to include in report. d. Encourage staff to ask questions during hand-off. e. Give raises based on compliance with reporting.

a. Attend hand-off rounds to coach and mentor. b. Conduct audits of staff using a new template. c. Create a template of topics to include in report. d. Encourage staff to ask questions during hand-off.

The expert nurse understands that critical thinking requires which elements to be present? (Select all that apply.) a. Based on logic, creativity, and intuition b. Driven by needs c. Focused on safety and quality d. Grounded in a specific theory e. Guided by standards f. Requires forming options about evidence

a. Based on logic, creativity, and intuition b. Driven by needs c. Focused on safety and quality e. Guided by standards Critical thinking must be based on logic, creativity, and intuition; driven by patient, family, or community needs; focused on safety and quality; guided by standards, policies, ethics, and laws; based on principles of nursing process, problem-solving, and the scientific method (requires forming opinions and making decisions based on evidence); centered on identification of the key problems, issues, and risks; and grounded in strategies that make the most of human potential. It is not dependent on using a specific theory.

A client resuscitated after drowning is admitted to the emergency department. What assessment findings does the nurse recognize as symptoms of a drowning? (Select all that apply.) a. Bilateral crackles b. Bradycardia c. Cyanosis of the lips d. Hypotension e. Flushed, diaphoretic skin

a. Bilateral crackles b. Bradycardia c. Cyanosis of the lips d. Hypotension Drowning victims will exhibit signs of pulmonary edema which includes crackles in one or both lungs, persistent dry cough, and cyanosis of the lips and/or nail beds. The diving reflex as a response to asphyxia produces bradycardia, signs of decreased cardiac output with hypotension, and vasoconstriction of vessels in the intestine, skeletal muscles, and kidneys.

3. A nurse is caring for a postoperative client on the surgical unit. The client's blood pressure was 142/76 mm Hg 30 minutes ago, and now is 88/50 mm Hg. What action by the nurse is best? a. Call the Rapid Response Team. b. Document and continue to monitor. c. Notify the primary care provider. d. Repeat blood pressure measurement in 15 minutes.

a. Call the Rapid Response Team.

2. A nurse assesses a client who is experiencing an acid-base imbalance. The clients arterial blood gas values are pH 7.34, PaO2 88 mm Hg, PaCO2 38 mm Hg, and HCO3 19 mEq/L. Which assessment should the nurse perform first? a. Cardiac rate and rhythm b. Skin and mucous membranes c. Musculoskeletal strength d. Level of orientation

a. Cardiac rate and rhythm Early cardiovascular changes for a client experiencing moderate acidosis include increased heart rate and cardiac output. As the acidosis worsens, the heart rate decreases and electrocardiographic changes will be present. Central nervous system and neuromuscular system changes do not occur with mild acidosis and should be monitored if the acidosis worsens. Skin and mucous membrane assessment is not a priority now, but will change as acidosis worsens.

A client admitted to the emergency department following a lightning strike. What is the priority assessment the nurse focuses on? a. Cardiopulmonary b. Integumentary c. Peripheral vascular d. Renal

a. Cardiopulmonary Lightning strikes can profoundly affect the cardiopulmonary and the central nervous system as a serious cardiac and/or respiratory arrest. The nurse would be alert for reports of chest pain and would watch for dysrhythmias on the cardiac monitor. As impairment of the respiratory center can also be affected, the nurse would assess the respiratory system second.

The nurse caring for mechanically ventilated clients knows that older adults are at higher risk for weaning failure. What age-related changes contribute to this? (Select all that apply.) a. Chest wall stiffness b. Decreased muscle strength c. Inability to cooperate d. Less lung elasticity e. Poor vision and hearing f. Chronic anemia

a. Chest wall stiffness b. Decreased muscle strength d. Less lung elasticity

A nurse is caring for four clients with leukemia. After hand-off report, which client would the nurse assess first? a. Client who had two bloody diarrhea stools this morning. b. Client who has been premedicated for nausea prior to chemotherapy. c. Client with a respiratory rate change from 18 to 22 breaths/min. d. Client with an unchanged lesion to the lower right lateral malleolus.

a. Client who had two bloody diarrhea stools this morning.

The nurse gets the hand-off report on four clients. Which client would the nurse assess first? a. Client with a blood pressure change of 128/74 to 110/88 mm Hg b. Client with oxygen saturation unchanged at 94% c. Client with a pulse change of 100 to 88 beats/min d. Client with urine output of 40 mL/hr for the last 2 hours

a. Client with a blood pressure change of 128/74 to 110/88 mm Hg This client has a falling systolic blood pressure, rising diastolic blood pressure, and narrowing pulse pressure, all of which may be indications of worsening perfusion status and possible shock. The nurse would assess this client first. The client with the unchanged oxygen saturation is stable at this point. Although the client with a change in pulse has a slower rate, it is not an indicator of shock since the pulse is still within the normal range; it may indicate that the client's pain or anxiety has been relieved, or he or she is sleeping or relaxing. A urine output of 40 mL/hr is above the normal range, which is 30 mL/hr.

4. A nurse is planning discharge teaching on tracheostomy care for an older client. What factors does the nurse need to assess before teaching this particular client? (Select all that apply.) a. Cognition b. Dexterity c. Hydration d. Range of motion e. Vision f. Upper arm range of motion

a. Cognition b. Dexterity d. Range of motion e. Vision f. Upper arm range of motion The older adult is at risk for having impairments in cognition, dexterity, range of motion, and vision that could limit the ability to perform tracheostomy care and would be assessed. Upper arm mobility is required to perform tracheostomy self-care. Hydration is not directly related to the ability to perform self-care.

1. A nurse manager wishes to ensure that the nurses on the unit are practicing at their highest levels of competency. Which areas should the manager assess to determine if the nursing staff demonstrate competency according to the Institute of Medicine (IOM) report Health Professions Education: A Bridge to Quality? (Select all that apply.) a. Collaborating with an interdisciplinary team b. Implementing evidence-based care c. Providing family-focused care d. Routinely using informatics in practice e. Using quality improvement in client care

a. Collaborating with an interdisciplinary team b. Implementing evidence-based care d. Routinely using informatics in practice e. Using quality improvement in client care

A nurse cares for many clients with pressure injuries. What actions by the nurse are considered best practice? (Select all that apply.) a. Conduct ongoing assessments that include pain. b. Use normal saline to cleanse around the pressure injury. c. Soak eschar daily until it softens and can be removed. d. Consult with a registered dietitian nutritionist. e. Use antimicrobial agents to clean wounds that are infected. f. Consider the use of adjuvant therapies for nonhealing wounds.

a. Conduct ongoing assessments that include pain. b. Use normal saline to cleanse around the pressure injury. d. Consult with a registered dietitian nutritionist. e. Use antimicrobial agents to clean wounds that are infected. f. Consider the use of adjuvant therapies for nonhealing wounds. Best practice for pressure injury wound management includes ongoing assessments that include pain, using normal saline to clean gently around the wound, ensuring optimal nutrition by involving a registered dietitian nutritionist, using an antimicrobial agent to clean wounds that are anticipated to become infected, and considering the use of adjuvant therapies such as stimulation, negative-pressure wound therapy, ultrasound, hyperbaric oxygen, and topical growth factors. The nurse would not disturb stable eschar.

Which assessment findings would indicate to the nurse that a client has suffered from a heat stroke? (Select all that apply.) a. Confusion and bizarre behavior b. Headache and fatigue c. Hypotension d. Presence of perspiration e. Tachycardia and tachypnea f. Body temperature more than 104° F (40° C)

a. Confusion and bizarre behavior c. Hypotension e. Tachycardia and tachypnea f. Body temperature more than 104° F (40° C) Signs and symptoms of heat stroke include as elevated body temperature (above 104° F [40° C]), mental status changes such as confusion and decreasing level of consciousness, hypotension, tachycardia, and tachypnea. Perspiration is an inconsistent finding.

A nurse working with older adults assesses them for common potential adverse medication effects. For what does the nurse assess? (Select all that apply.) a. Constipation b. Dehydration c. Mania d. Urinary incontinence e. Weakness f. Anorexia

a. Constipation b. Dehydration e. Weakness f. Anorexia Common adverse medication effects include constipation/impaction, dehydration, anorexia, and weakness. Mania and incontinence are not among the common adverse effects, although urinary retention is.

2. A nurse is interested in making interdisciplinary work a high priority. Which actions by the nurse best demonstrate this skill? (Select all that apply.) a. Consults with other disciplines on client care b. Coordinates discharge planning for home safety c. Participates in comprehensive client rounding d. Routinely asks other disciplines about client progress e. Shows the nursing care plans to other disciplines

a. Consults with other disciplines on client care b. Coordinates discharge planning for home safety c. Participates in comprehensive client rounding d. Routinely asks other disciplines about client progress

6. A client is taking ethambutol for tuberculosis. What instructions does the nurse provide the client regarding this drug? (Select all that apply.) a. Contact the primary health care provider if preexisting gout becomes worse. b. Report any changes in vision immediately to the health care provider. c. Avoid drinking alcoholic beverages due to the chance of liver damage. d. Do not take antacids or eat within 2 hours after taking this medication. e. You will take this medication along with some others for 8 weeks. f. Take this medicine with a full glass of water.

a. Contact the primary health care provider if preexisting gout becomes worse. b. Report any changes in vision immediately to the health care provider. e. You will take this medication along with some others for 8 weeks. f. Take this medicine with a full glass of water. The nurse would teach the client that preexisting gout may get worse and the client should report this as medications for gout may need to be adjusted. The nurse would also inform the client about the multi-drug routine used for TB. Optic neuritis can occur with this drug so the client needs to report visual changes right away. The medication should be taken with a full glass of water. Drinking while taking ethambutol causes severe nausea and vomiting. Avoiding antacids and food (within 2 hours) is a precaution with isoniazid.

A nurse cares for a client who reports discomfort related to eczematous dermatitis. Which nonpharmacologic comfort measures would the nurse implement? (Select all that apply.) a. Cool, moist compresses b. Topical corticosteroids c. Heating pad d. Tepid bath with colloidal oatmeal e. Back rub with baby oil

a. Cool, moist compresses d. Tepid bath with colloidal oatmeal For a client with eczematous dermatitis, the goal of comfort measures is to decrease inflammation and help débride crusts and scales. The nurse would implement cool, moist compresses and tepid baths with additives such as colloidal oatmeal. Topical corticosteroids are a pharmacologic intervention. A heating pad and a back rub with baby oil are not appropriate for this client and could increase inflammation and discomfort.

After teaching a patient who has a permanent ileostomy, a nurse assesses the client's understanding. Which dietary items chosen for dinner indicate that the client needs further teaching? (Select all that apply.) a. Corn b. String beans c. Carrots d. Wheat rice e. Squash

a. Corn b. String beans d. Wheat rice Clients with an ileostomy should be cautious of high-fiber and high-cellulose foods including corn, string beans, and rice. Carrots and squash are low-fiber items.

3. A client is being discharged home after having a tracheostomy placed. What suggestions does the nurse offer to help the client maintain self-esteem? (Select all that apply.) a. Create a communication system. b. Don't go out in public alone. c. Find hobbies to enjoy at home. d. Try loose-fitting shirts with collars. e. Wear fashionable scarves.

a. Create a communication system. d. Try loose-fitting shirts with collars. e. Wear fashionable scarves. The client with a tracheostomy may be shy and hesitant to go out in public. The client needs to have a sound communication method to ease frustration. The nurse can also suggest ways of enhancing appearance so the client is willing to leave the house. These can include wearing scarves and loose-fitting shirts to hide the stoma. Keeping the client homebound is not good advice.

A nurse assesses a patient who is experiencing diabetic ketoacidosis (DKA). For which assessment findings would the nurse monitor the client? (Select all that apply.) a. Deep and fast respirations b. Decreased urine output c. Tachycardia d. Dependent pulmonary crackles e. Orthostatic hypotension

a. Deep and fast respirations c. Tachycardia e. Orthostatic hypotension

While on a camping trip, a nurse cares for an adult client who had a drowning incident in a lake and is experiencing agonal breathing with a palpable pulse. What action would the nurse take first? a. Deliver rescue breaths. b. Wrap the client in dry blankets. c. Assess for signs of bleeding. d. Check for a carotid pulse.

a. Deliver rescue breaths. In this emergency situation, the nurse immediately initiates airway clearance and ventilator support measures, including delivering rescue breaths.

14. What is the primary goal of a triage system used by the nurse with clients presenting to the emergency department? a. Determine the acuity of the client's condition to determine priority of care. b. Assess the status of the airway, breathing, circulation, or presence of deficits. c. Determine whether the client is responsive enough to provide needed information. d. Evaluate the emergency department's resources to adequately treat the patient.

a. Determine the acuity of the client's condition to determine priority of care. ED triage is an organized system for sorting and classifying clients into priority levels depending on illness or injury severity. The primary goal of the triage system is to facilitate the ED nurse's ability to prioritize care according to the acuity of the patient, having the clients with the more severe illness or injury seen first. Airway, breathing, and circulation are part of the primary survey. Determining responsiveness is done during the disability phase of the primary survey and is not the primary goal. Evaluating the ED's resources is also not a goal of triage.

The nurse is caring for a client who was recently diagnosed with pancreatic cancer. What factors present risks for developing this type of cancer? (Select all that apply.) a. Diabetes mellitus b. Cirrhosis c. Smoking d. Female gender e. Family history f. Older age

a. Diabetes mellitus b. Cirrhosis c. Smoking e. Family history f. Older age All of these choices are risk factors except that pancreatic cancer occurs most frequently in men

A client with chronic kidney disease (CKD) is refusing to take his medication and has missed two hemodialysis appointments. What is the best initial action for the nurse? a. Discuss what the treatment regimen means to the client. b. Refer the client to a mental health nurse practitioner. c. Reschedule the appointments to another date and time. d. Discuss the option of peritoneal dialysis.

a. Discuss what the treatment regimen means to the client.

A client asks the nurse why she has urinary incontinence. What risk factors would the nurse recall in preparing to respond to the client's question? (Select all that apply.) a. Diuretic therapy b. Anorexia nervosa c. Stroke d. Dementia e. Arthritis f. Parkinson disease

a. Diuretic therapy c. Stroke d. Dementia e. Arthritis f. Parkinson disease

A woman diagnosed with breast cancer had these laboratory tests performed at an office visit: Alkaline phosphatase 125 U/L (2.2 mckat/L) Total calcium 12 mg/dL (3 mmol/L) Hematocrit 39% (0.39) Hemoglobin 14 g/dL (140 mmol/L) Which test results indicate to the nurse that some further diagnostics are needed? a. Elevated alkaline phosphatase and calcium suggests bone involvement. b. Only alkaline phosphatase is decreased, suggesting liver metastasis. c. Hematocrit and hemoglobin are decreased, indicating anemia. d. The elevated hematocrit and hemoglobin indicate dehydration.

a. Elevated alkaline phosphatase and calcium suggests bone involvement.

The nurse is caring for a client who has possible acute pancreatitis. What serum laboratory findings would the nurse expect for this client? (Select all that apply.) a. Elevated amylase b. Elevated lipase c. Elevated glucose d. Decreased calcium e. Elevated bilirubin f. Elevated leukocyte count

a. Elevated amylase b. Elevated lipase c. Elevated glucose d. Decreased calcium e. Elevated bilirubin f. Elevated leukocyte count All of these choices are correct. Amylase and lipase are pancreatic enzymes that are released during pancreatic inflammation and injury. Leukocytes also increased due to his inflammatory response. Pancreatic injury affects the ability of insulin to be released causing increased glucose levels. Bilirubin is also typically increased due to hepatobiliary obstruction. Calcium and magnesium levels decrease because fatty acids bind free calcium and magnesium causing a lowered serum level; these changes occur in the presence of fat necrosis.

According to the WHO, what does primary care involve? (Select all that apply.) a. Empowered people and communities b. Essential public functions c. Multisectoral policy and action d. Primary care e. Priority consideration of chronic diseases f. Elimination of chronic diseases

a. Empowered people and communities b. Essential public functions c. Multisectoral policy and action d. Primary care According to the WHO, primary care involves three main areas: empowered people and communities, primary care and essential public functions, and multisectoral policy and action. Primary care focuses on both prevention and management of chronic disease.

A nurse cares for a client who has a new colostomy. Which action would the nurse take? a. Empty the pouch frequently to remove excess gas collection. b. Change the ostomy pouch and barrier every morning. c. Allow the pouch to completely fill with stool prior to emptying it. d. Use surgical tape to secure the pouch and prevent leakage.

a. Empty the pouch frequently to remove excess gas collection.

10. A nurse assesses a client who is prescribed fluticasone and notes oral lesions. What action would the nurse take? a. Encourage oral rinsing after fluticasone administration. b. Obtain an oral specimen for culture and sensitivity. c. Start the client on a broad-spectrum antibiotic. d. Document the finding as a known side effect.

a. Encourage oral rinsing after fluticasone administration. The drug reduces local immunity and increases the risk for local infection, especially Candida albicans. Rinsing the mouth after using the inhaler will decrease the risk for developing this infection. Use of mouthwash and broad-spectrum antibiotics is not warranted in this situation. The nurse would document the finding, but the best action to take is to have the client start rinsing his or her mouth after using fluticasone. An oral specimen for culture and sensitivity is not necessary to care for this client.

2. A nurse is orienting a new client and family to the inpatient unit. What information does the nurse provide to help the client promote his or her own safety? a. Encourage the client and family to be active partners. b. Have the client monitor hand hygiene in caregivers. c. Offer the family the opportunity to stay with the client. d. Tell the client to always wear his or her armband.

a. Encourage the client and family to be active partners.

8. A home health nurse evaluates a client who has chronic obstructive pulmonary disease. Which assessments would the nurse include in this client's evaluation? (Select all that apply.) a. Examination of mucous membranes and nail beds b. Measurement of rate, depth, and rhythm of respirations c. Auscultation of bowel sounds for abnormal sounds d. Check peripheral veins for distention while at rest e. Determine the client's need and use of oxygen f. Ability to perform activities of daily living

a. Examination of mucous membranes and nail beds b. Measurement of rate, depth, and rhythm of respirations e. Determine the client's need and use of oxygen f. Ability to perform activities of daily living A home health nurse would assess the client's respiratory status and adequacy of ventilation including an examination of mucous membranes and nail beds for evidence of hypoxia, measurement of rate, depth and rhythm of respirations, auscultation of lung fields for abnormal breath sounds, checking neck veins for distention with the client in a sitting position, and determining the client's needs and use of supplemental oxygen. The home health nurse would also determine the client's ability to perform his or her own ADLs. Auscultation of bowel sounds and assessment of peripheral veins are not part of a focused assessment for a client with COPD.

A marathon runner comes into the clinic and states "I have not urinated very much in the last few days." The nurse notes a heart rate of 110 beats/min and a blood pressure of 86/58 mm Hg. Which action by the nurse is most appropriate? a. Give the client a bottle of water immediately. b. Start an intravenous line for fluids. c. Teach the patient to drink 2 to 3 L of water daily. d. Perform an electrocardiogram.

a. Give the client a bottle of water immediately.

10. A nurse is assessing a client with lung cancer. What nonpulmonary signs and symptoms would the nurse be aware of? (Select all that apply.) a. Gynecomastia in male patients b. Frequent shaking and sweating relieved by eating c. Positive Chvostek and Trousseau signs d. "Moon" face and "buffalo" hump e. Expectorating purulent sputum f. General edema

a. Gynecomastia in male patients b. Frequent shaking and sweating relieved by eating d. "Moon" face and "buffalo" hump f. General edema Lung cancer often is associated with paraneoplastic syndromes. Symptoms of these include gynecomastia from ectopic follicle-stimulating hormone release, hypoglycemia from ectopic insulin production (shaking and sweating relieved by eating), and Cushing syndrome (moon facies and buffalo hump) from ectopic adrenocorticotropic hormone. General edema can be caused by antidiuretic hormone.

7. The nurse is learning about endemic pulmonary diseases. Which diseases are matched with correct information? (Select all that apply.) a. Hanta virus: found in urine, droppings, and saliva of infected rodents. b. Aspergillosis: requires a prolonged course of antibiotics. c. Histoplasmosis: sources include soil containing bird and bat droppings. d. Blastomycosis: requires strict adherence to multi-antibiotic regimen. e. Cryptococcosis: has been eradicated due to strategic deforestation. f. Coccidioidomycosis: found in the southwest and far west of the United States.

a. Hanta virus: found in urine, droppings, and saliva of infected rodents. c. Histoplasmosis: sources include soil containing bird and bat droppings. f. Coccidioidomycosis: found in the southwest and far west of the United States. Hanta virus is often seen in the southwest United States and is found in the urine, droppings, and saliva of infected rodents. Histoplasmosis is found in soil containing bird and bat droppings and on surfaces covered with bird droppings. Apergillosis is a common mold found both indoors and outdoors and is treated with a long course of antifungal drugs. Blastomycosis is a fungal disease requiring a prolonged course of antifungal medications. Cryptococcosis is a fungus found on trees and in the soil beneath trees, but has not been eradicated with strategic deforestation. Coccidioidomycosis is found in the southwest and far west of the United States, plus Mexico, and Central and South America.

A nurse is in charge of a first-aid tent at an all-day outdoor sports event on a hot and humid day. A participant comes to the tent reporting a headache, weakness, and nausea. What actions would the nurse take? (Select all that apply.) a. Have the client lie down in a cool place. b. Force fluids with large quantities of plain water. c. Administer acetaminophen and send home. d. Apply cold packs to neck, arm pits, and groin. e. Encourage drinking a sports drink. f. Remove all clothing and cover with a towel.

a. Have the client lie down in a cool place. d. Apply cold packs to neck, arm pits, and groin. e. Encourage drinking a sports drink. Heat exhaustion manifests as flulike symptoms with headache, weakness, nausea, and/or vomiting. Treatment includes stopping the activity, moving to a cool place, and using cooling measures such as cold packs, cool water soaks, or fanning while spraying cool water on skin. Sodium deficits may occur from drinking plain water, so sports drinks or an oral rehydration therapy solution would be provided. The nurse would remove constrictive clothing only.

A nurse assesses a client who experienced a spinal cord injury at the T5 level 12 hours ago. Which assessment findings would the nurse correlate with neurogenic shock? (Select all that apply.) a. Heart rate of 34 beats/min b. Blood pressure of 185/65 mm Hg c. Urine output less than 30 mL/hr d. Decreased level of consciousness e. Increased oxygen saturation

a. Heart rate of 34 beats/min c. Urine output less than 30 mL/hr d. Decreased level of consciousness

A nurse evaluates laboratory results for a client with heart failure. Which results should the nurse expect? (Select all that apply.) a. Hematocrit: 32.8% b. Serum sodium: 130 mEq/L c. Serum potassium: 4.0 mEq/L d. Serum creatinine: 1.0 mg/dL e. Proteinuria f. Microalbuminuria

a. Hematocrit: 32.8% b. Serum sodium: 130 mEq/L e. Proteinuria f. Microalbuminuria

The nurse is teaching a client a client about taking elbasvir for hepatitis C. What information in the client's history would the nurse need prior to drug administration? a. History of hepatitis B b. History of kidney disease c. History of cardiac disease d. History of rectal bleeding

a. History of hepatitis B Elbasvir can cause liver toxicity and therefore the nurse would assess for a history of or current hepatitis B

An emergency department nurse moves to a new city where heat-related illnesses are common. Which clients does the nurse anticipate being at highest risk for heat-related illnesses? (Select all that apply.) a. Homeless individuals b. People with substance abuse disorders c. Caucasians d. Hockey players e. Older adults f. Obese individuals

a. Homeless individuals b. People with substance abuse disorders e. Older adults f. Obese individuals Some of the most vulnerable, at-risk populations for heat-related illness include older adults; people who work outside, such as construction and agricultural workers; homeless people; people who abuse substances; outdoor athletes (recreational and professional); and members of the military who are stationed in countries with hot climates (e.g., Iraq, Afghanistan). Hockey is generally a cold-air game whether played indoors or outdoors and wouldn't have as much risk for heat-related illness as other sports.

A nurse wishes to work in a community-based practice setting. Which areas would this nurse explore for employment? (Select all that apply.) a. Hospice facility b. "Minute clinic" c. Mobile mammography unit d. Small community hospital e. Telehealth f. Home health care

a. Hospice facility b. "Minute clinic" c. Mobile mammography unit e. Telehealth f. Home health care The multiple avenues providing community-based care include hospice, "minute" or retail clinics, mobile screening and diagnostic services, telehealth, private medical practices, outpatient services, freestanding points of care, home health care, long-term ambulatory care, public health, and free clinics. Inpatient services in a hospital are not considered primary care sites.

8. A nurse is teaching a community group about the long-term effects of untreated sleep apnea. What information does the nurse include? (Select all that apply.) a. Hypertension b. Stroke c. Weight gain d. Diabetes e. Cognitive deficits f. Pulmonary disease

a. Hypertension b. Stroke c. Weight gain d. Diabetes e. Cognitive deficits f. Pulmonary disease The long-term effects of untreated sleep apnea include increased risk for hypertension, stroke, cognitive deficits, weight gain, diabetes, and pulmonary and cardiovascular disease.

14. A nurse cares for a client with a 40-year smoking history who is experiencing distended neck veins and dependent edema. Which physiologic process would the nurse correlate with this client's history and clinical signs and symptoms? a. Increased pulmonary pressure creating a higher workload on the right side of the heart b. Exposure to irritants resulting in increased inflammation of the bronchi and bronchioles c. Increased number and size of mucous glands producing large amounts of thick mucus d. Left ventricular hypertrophy creating a decrease in cardiac output

a. Increased pulmonary pressure creating a higher workload on the right side of the heart Smoking increases pulmonary hypertension, resulting in cor pulmonale, or right-sided heart failure. Increased pressures in the lungs make it more difficult for blood to flow through the lungs. Blood backs up into the right side of the heart and then into the peripheral venous system, creating distended neck veins and dependent edema. Inflammation in bronchi and bronchioles creates an airway obstruction which manifests as wheezes. Thick mucus in the lungs has no impact on distended neck veins and edema. Left ventricular hypertrophy is associated with left-heart failure and is not directly caused by a 40-year smoking history.

The nurse assesses a client with diabetic ketoacidosis. Which assessment finding would the nurse correlate with this condition? a. Increased rate and depth of respiration b. Extremity tremors followed by seizure activity c. Oral temperature of 102° F (38.9° C) d. Severe orthostatic hypotension

a. Increased rate and depth of respiration

1. A nurse assesses a client with diabetes mellitus who is admitted with an acid-base imbalance. The clients arterial blood gas values are pH 7.36, PaO2 98 mm Hg, PaCO2 33 mm Hg, and HCO3 18 mEq/L. Which manifestation should the nurse identify as an example of the clients compensation mechanism? a. Increased rate and depth of respirations b. Increased urinary output c. Increased thirst and hunger d. Increased release of acids from the kidneys

a. Increased rate and depth of respirations This client has metabolic acidosis. The respiratory system compensates by increasing its activity and blowing off excess carbon dioxide. Increased urinary output, thirst, and hunger are manifestations of hyperglycemia but are not compensatory mechanisms for acid-base imbalances. The kidneys do not release acids.

A nurse is assessing a client's history of particular matter exposure. What questions are consistent with the I PREPARE tool? (Select all that apply.) a. Investigate all history of known exposures. b. Determine if breathing problems are worse at work. c. Ask the client what type of heating is in the home. d. Gather details about the geographic location of the client's home. e. Have client list all previous jobs and work experiences. f. Assess what hobbies the client and family enjoy.

a. Investigate all history of known exposures. b. Determine if breathing problems are worse at work. c. Ask the client what type of heating is in the home. d. Gather details about the geographic location of the client's home. e. Have client list all previous jobs and work experiences. f. Assess what hobbies the client and family enjoy.

The nurse is caring for a client who has late-stage (advanced) cirrhosis. What assessment findings would the nurse expect? (Select all that apply.) a. Jaundice b. Clay-colored stools c. Icterus d. Ascites e. Petechiae f. Dark urine

a. Jaundice b. Clay-colored stools c. Icterus d. Ascites e. Petechiae f. Dark urine All of these assessment findings are very common for a client who has late-stage cirrhosis due to biliary obstruction and poor liver function. The client has vascular lesions and excess fluid from portal hypertension

An older adult client is in the hospital. The client is ambulatory and independent. What intervention by the nurse would be most helpful in preventing falls in this client? a. Keep the light on in the bathroom at night. b. Order a bedside commode for the client. c. Put the client on a toileting schedule. d. Use side rails to keep the client in bed.

a. Keep the light on in the bathroom at night. Although this older adult is independent and ambulatory, being hospitalized can create confusion. Getting up in a dark, unfamiliar environment can contribute to falls. Keeping the light on in the bathroom will help reduce the likelihood of falling. The client does not need a commode or a toileting schedule. Side rails used to keep the client in bed are considered restraints and would not be used in that fashion.

The nurse is caring for a client who has possible hypothyroidism. What possible risk factors can cause this health problem? (Select all that apply.) a. Lithium drug therapy b. Thyroid cancer c. Autoimmune thyroid disease d. Iodine deficiency e. Laryngitis f. Pituitary tumors

a. Lithium drug therapy b. Thyroid cancer c. Autoimmune thyroid disease d. Iodine deficiency f. Pituitary tumors

A client is hospitalized in the oliguric phase of acute kidney injury (AKI) and is receiving tube feedings. The nurse is teaching the client's spouse about the renal-specific formulation for the enteral solution compared to standard formulas. What components would be discussed in the teaching plan? (Select all that apply.) a. Lower sodium b. Higher calcium c. Lower potassium d. Higher phosphorus e. Higher calories

a. Lower sodium c. Lower potassium e. Higher calories

2. An emergency department (ED) nurse is preparing to transfer a client to the trauma intensive care unit. Which information would the nurse include in the nurse-to-nurse hand-off report? (Select all that apply.) a. Mechanism of injury b. Diagnostic test results c. Immunizations d. List of home medications e. Isolation Precautions f. Safety concerns

a. Mechanism of injury b. Diagnostic test results e. Isolation Precautions f. Safety concerns Hand-off communication would be comprehensive so that the receiving nurse can continue care for the client fluidly. Communication would be concise and would include only the most essential information for a safe transition in care. Hand-off communication would include the client's situation (reason for being in the ED), brief medical history, assessment and diagnostic findings, Transmission-Based Precautions needed, safety concerns interventions provided, and response to those interventions. Immunization history is not usually considered critical unless it relates to the reason for admission. Medication reconciliation will occur when the client reaches the inpatient unit.

A nurse is caring for a client who has lung cancer and is dying. Which prescription should the nurse question? a. Morphine 10 mg sublingual every 6 hours PRN for pain level greater than 5 b. Albuterol (Proventil) metered dose inhaler every 4 hours PRN for wheezes c. Atropine solution 1% sublingual every 4 hours PRN for excessive oral secretions d. Sodium biphosphate (Fleet) enema once a day PRN for impacted stool

a. Morphine 10 mg sublingual every 6 hours PRN for pain level greater than 5 Pain medications should be scheduled around the clock to maintain comfort and prevent reoccurrence of pain. The other medications are appropriate for this client.

The nurse is caring for a client with peritonitis. What assessment findings would the nurse expect? (Select all that apply.) a. Nausea and vomiting b. Distended rigid abdomen c. Abdominal pain d. Bradycardia e. Decreased urinary output f. Fever

a. Nausea and vomiting c. Abdominal pain d. Bradycardia e. Decreased urinary output f. Fever Peritonitis is an acute inflammatory disorder. Therefore, the client would likely have all of these signs and symptoms but would have tachycardia rather than bradycardia due to dehydration from fever

The nurse is caring for a client who is diagnosed with celiac disease and preparing to start natalizumab. Which health teaching would the nurse include in the teaching? (Select all that apply.) a. Need to have drug administered by a primary health care provider. b. Need to avoid crowds and individuals who have infection. c. Need to report injection reactions such as redness and swelling. d. Awareness of a rare but potentially fatal drug complication. e. Need to report any signs and symptoms of infection immediately

a. Need to have drug administered by a primary health care provider. b. Need to avoid crowds and individuals who have infection. d. Awareness of a rare but potentially fatal drug complication. e. Need to report any signs and symptoms of infection immediately All of these choices are correct except that the drug is given intravenously. Therefore, there is no need to teach the client to report injection reactions because the client does not self-administer the medication subcutaneously. Natalizumab can cause progressive multifocal leukoencephalopathy (PML), but it is a very rare disorder causing cognitive, sensory, and/or motor changes

1. A nurse assesses a client who is 6 hours postsurgery for a nasal fracture and has nasal packing in place. What actions would the nurse take? (Select all that apply.) a. Observe for clear drainage. b. Assess for signs of bleeding. c. Watch the client for frequent swallowing. d. Ask the client to open his or her mouth. e. Administer a nasal steroid to decrease edema. f. Change the nasal packing.

a. Observe for clear drainage. b. Assess for signs of bleeding. c. Watch the client for frequent swallowing. d. Ask the client to open his or her mouth. The nurse would observe for clear drainage because of the risk for cerebrospinal fluid leakage. The nurse would assess for signs of bleeding by asking the client to open his or her mouth and observing the back of the throat for bleeding. The nurse would also note whether the client is swallowing frequently because this could indicate postnasal bleeding. A nasal steroid would increase the risk for infection. It is too soon to change the packing, which would be changed by the surgeon the first time.

6. A nurse assesses a client who is at risk for neck cancer. Which symptoms would the nurse assess for? (Select all that apply.) a. Oral mucosa is gray or dark brown b. Pain when drinking grapefruit juice c. Persistent weight gain over the past 2 months d. Oral lesions that are over 2 weeks old e. Changes in the patient's voice quality

a. Oral mucosa is gray or dark brown b. Pain when drinking grapefruit juice Symptoms of head and neck cancer include color changes in the mouth or tongue to gray or dark brown; pain in the mouth, neck, and throat; burning sensation when drinking citrus juices; weight loss; oral lesions or soars that do not heal in 2 weeks; and hoarseness or changes in voice quality.

The nurse plans care for a patient who has hepatopulmonary syndrome. Which interventions would the nurse include in this client's plan of care? (Select all that apply.) a. Oxygen therapy b. Prone position c. Feet elevated on pillows d. Daily weights e. Physical therapy f. Respiratory therapy

a. Oxygen therapy c. Feet elevated on pillows d. Daily weights f. Respiratory therapy Care for a client who has hepatopulmonary syndrome would include oxygen therapy, the head of bed elevated at least 30 degrees or as high as the client wants to improve breathing, elevated feet to decrease dependent edema, and daily weights. There is no need to place the patient in a prone position, on the patient's stomach. Although physical therapy may be helpful to a patient who has been hospitalized for several days, physical therapy is not an intervention specifically for hepatopulmonary syndrome. However, respiratory support from a specialized therapist may be needed.

Emergency medical services (EMS) brings a large number of clients to the emergency department following a mass casualty incident. The nurse identifies the clients with which injuries with yellow tags? (Select all that apply.) a. Partial-thickness burns covering both legs b. Open fractures of both legs with absent pedal pulses c. Neck injury and numbness of both legs d. Small pieces of shrapnel embedded in both eyes e. Head injury and difficult to arouse f. Bruising and pain in the right lower abdomen

a. Partial-thickness burns covering both legs c. Neck injury and numbness of both legs d. Small pieces of shrapnel embedded in both eyes f. Bruising and pain in the right lower abdomen Clients with burns, spine injuries, eye injuries, and stable abdominal injuries would be treated within 30 minutes to 2 hours, and therefore would be identified with yellow tags. The client with the open fractures and the client with the head injury would be classified as urgent with red tags.

After a breast examination, the nurse is documenting assessment findings that indicate possible breast cancer. Which abnormal findings need to be included as part of the client's electronic medical record? (Select all that apply.) a. Peau d'orange b. Dense breast tissue c. Nipple retraction d. Mobile mass at 2 o'clock e. Nontender axillary nodes f. Skin ulceration

a. Peau d'orange c. Nipple retraction d. Mobile mass at 2 o'clock f. Skin ulceration

A nurse plans care for a client who is immobile. Which interventions would the nurse include in this client's plan of care to prevent pressure sores? (Select all that apply.) a. Place a small pillow between bony surfaces. b. Elevate the head of the bed to 45 degrees. c. Limit fluids and proteins in the diet. d. Use a lift sheet to assist with re-positioning. e. Re-position the client who is in a chair every 2 hours. f. Keep the client's heels off the bed surfaces. g. Use a rubber ring to decrease sacral pressure when up in the chair.

a. Place a small pillow between bony surfaces. d. Use a lift sheet to assist with re-positioning. f. Keep the client's heels off the bed surfaces. A small pillow decreases the risk for pressure between bony prominences, a lift sheet decreases friction and shear, and heels have poor circulation and are at high risk for pressure sores, so they would be kept off hard surfaces. Head-of-the-bed elevation greater than 30 degrees increases pressure on pelvic soft tissues. Fluids and proteins are important for maintaining tissue integrity. Clients would be repositioned every hour while sitting in a chair. A rubber ring impairs capillary blood flow, increasing the risk for a pressure sore.

A nurse receives hand-off report from the emergency department on a new admission suspected of having septic shock. The client's qSOFA score is 3. What action by the nurse is best? a. Plan to calculate a full SOFA score on arrival. b. Contact respiratory therapy about ventilator setup. c. Arrange protective precautions to be implemented. d. Call the hospital chaplain to support the family.

a. Plan to calculate a full SOFA score on arrival. The qSOFA score is an abbreviated Sequential Organ Failure Assessment (or "quick"). A score of 3 is high and requires the nurse to assess the client further for organ impairment. The client may or may not need a ventilator, but that in not specified in the score. The client does not need protective precautions. The client's family may well need support, but the nurse would assess their needs and wishes prior to calling the chaplain.

4. A nurse assesses a client who is receiving total parenteral nutrition. For which adverse effects related to an acid-base imbalance should the nurse assess? (Select all that apply.) a. Positive Chvosteks sign b. Elevated blood pressure c. Bradycardia d. Increased muscle strength e. Anxiety and irritability

a. Positive Chvosteks sign e. Anxiety and irritability A client receiving total parenteral nutrition is at risk for metabolic alkalosis. Manifestations of metabolic alkalosis include positive Chvosteks sign, normal or low blood pressure, increased heart rate, skeletal muscle weakness, and anxiety and irritability.

A client in the emergency department has several broken ribs and reports severe pain. What care measure will best promote comfort? a. Prepare to assist with intercostal nerve block. b. Humidify the supplemental oxygen. c. Splint the chest with a large ACE wrap. d. Provide warmed blankets and warmed IV fluids

a. Prepare to assist with intercostal nerve block.

The nurse is reviewing the results of a client's urinalysis. The client has a diagnosis of acute glomerulonephritis. Which urine findings would the nurse expect? (Select all that apply.) a. Presence of protein b. Presence of red blood cells c. Presence of white blood cells d. Acidic urine e. Dilute urine

a. Presence of protein c. Presence of white blood cells d. Acidic urine

A nurse assesses a client with nephrotic syndrome. Which assessment findings would the nurse expect? (Select all that apply.) a. Proteinuria b. Hypoalbuminemia c. Dehydration d. Lipiduria e. Dysuria f. Costovertebral angle (CVA) tenderness

a. Proteinuria b. Hypoalbuminemia d. Lipiduria

A hospital responds to a local mass casualty event. What action would the nurse supervisor take to prevent staff posttraumatic stress disorder during and after the event? a. Provide water and healthy snacks for energy throughout the event. b. Schedule 16-hour shifts to allow for greater rest between shifts. c. Encourage counseling upon deactivation of the emergency response plan. d. Assign staff to different roles and units within the medical facility.

a. Provide water and healthy snacks for energy throughout the event. To prevent staff posttraumatic stress disorder during a mass casualty event, the nurses would use available counseling, encourage and support co-workers, monitor each other's stress level and performance, take breaks when needed, talk about feelings with staff and managers, and drink plenty of water and eat healthy snacks for energy. Nurses would also keep in touch with family, friends, and significant others, and not work for more than 12 hours/day. Encouraging counseling upon deactivation of the plan, or after the emergency response is over, does not prevent stress during the casualty event. Assigning staff to unfamiliar roles or units may increase situational stress and is not an approach to prevent posttraumatic stress disorder. These actions also help mitigate PTSD after the event.

4. The complex care provided during an emergency requires interprofessional collaboration. Which team members are paired with the correct responsibilities? (Select all that apply.) a. Psychiatric crisis nurse—interacts with clients and families when sudden illness, serious injury, or death of a loved one may cause a crisis b. Forensic nurse examiner—performs rapid assessments to ensure that clients with the highest acuity receive the quickest evaluation, treatment, and prioritization of resources c. Triage nurse—provides basic life support interventions such as oxygen, basic wound care, splinting, spinal motion restriction, and monitoring of vital signs d. Emergency medical technician—obtains client histories, collects evidence, and offers counseling and follow up care for victims of rape, child abuse, and domestic violence e. Paramedic—provides prehospital advanced life support, including cardiac monitoring, advanced airway management, and medication administration

a. Psychiatric crisis nurse—interacts with clients and families when sudden illness, serious injury, or death of a loved one may cause a crisis e. Paramedic—provides prehospital advanced life support, including cardiac monitoring, advanced airway management, and medication administration The psychiatric crisis nurse evaluates clients with emotional behaviors or mental illness and facilitates follow-up treatment plans. The psychiatric crisis nurse also works with clients and families when experiencing a crisis. Paramedics are advanced life support providers who can perform advanced techniques that may include cardiac monitoring, advanced airway management and intubation, establishing IV access, and administering drugs en route to the emergency department. The forensic nurse examiner is trained to recognize evidence of abuse and to intervene on the client's behalf. The forensic nurse examiner will obtain client histories, collect evidence, and offer counseling and follow up care for victims of rape, child abuse, and domestic violence. The triage nurse performs rapid assessments to ensure that clients with the highest acuity receive the quickest evaluation, treatment, and prioritization of resources. The emergency medical technician is usually the first caregiver and provides basic life support and transportation to the emergency department.

A nurse is assessing a client with left-sided heart failure. For which clinical manifestations should the nurse assess? (Select all that apply.) a. Pulmonary crackles b. Confusion, restlessness c. Pulmonary hypertension d. Dependent edema e. Cough that worsens at night

a. Pulmonary crackles b. Confusion, restlessness e. Cough that worsens at night

2. A nurse assesses a client who is experiencing an acid-base imbalance. The clients arterial blood gas values are pH 7.32, PaO2 94 mm Hg, PaCO2 34 mm Hg, and HCO3 18 mEq/L. For which clinical manifestations should the nurse assess? (Select all that apply.) a. Reduced deep tendon reflexes b. Drowsiness c. Increased respiratory rate d. Decreased urinary output e. Positive Trousseaus sign

a. Reduced deep tendon reflexes b. Drowsiness c. Increased respiratory rate Metabolic acidosis causes neuromuscular changes, including reduced muscle tone and deep tendon reflexes. Clients usually present with lethargy and drowsiness. The respiratory system will attempt to compensate for the metabolic acidosis; therefore, respirations will increase rate and depth. A positive Trousseaus sign is associated with alkalosis. Decreased urine output is not a manifestation of metabolic acidosis.

1. A nurse is planning interventions that regulate acid-base balance to ensure the pH of a clients blood remains within the normal range. Which abnormal physiologic functions may occur if the client experiences an acid-base imbalance? (Select all that apply.) a. Reduction in the function of hormones b. Fluid and electrolyte imbalances c. Increase in the function of selected enzymes d. Excitable cardiac muscle membranes e. Increase in the effectiveness of many drugs

a. Reduction in the function of hormones b. Fluid and electrolyte imbalances e. Increase in the effectiveness of many drugs Acid-base imbalances interfere with normal physiology, including reducing the function of hormones and enzymes, causing fluid and electrolyte imbalances, making heart membranes more excitable, and decreasing the effectiveness of many drugs.

A nurse collaborates with the interprofessional team to develop a plan of care for a client who is newly diagnosed with diabetes mellitus. Which team members would the nurse include in this interprofessional team meeting? (Select all that apply.) a. Registered dietitian nutritionist b. Clinical pharmacist c. Occupational therapist d. Primary health care provider e. Speech-language pathologist

a. Registered dietitian nutritionist b. Clinical pharmacist d. Primary health care provider

A nurse is caring for a client who has been diagnosed with a small bowel obstruction. Which assessment findings would the nurse correlate with this diagnosis? (Select all that apply.) a. Serum potassium of 2.8 mEq/L (2.8 mmol/L) b. Loss of 15 lb (6.8 kg) without dieting c. Abdominal pain in upper quadrants d. Low-pitched bowel sounds e. Serum sodium of 121 mEq/L (121 mmol/L)

a. Serum potassium of 2.8 mEq/L (2.8 mmol/L) c. Abdominal pain in upper quadrants e. Serum sodium of 121 mEq/L (121 mmol/L)

The nurse assesses a client who has appendicitis. Which assessment finding would the nurse expect? a. Severe, steady right lower quadrant pain b. Abdominal pain associated with nausea and vomiting c. Marked peristalsis and hyperactive bowel sounds d. Abdominal pain that increases with knee flexion

a. Severe, steady right lower quadrant pain Right lower quadrant pain, specifically at McBurney's point, is characteristic of appendicitis. Usually if nausea and vomiting begin first, the client has gastroenteritis. Marked peristalsis and hyperactive bowel sounds are not indicative of appendicitis. Abdominal pain due to appendicitis decreases with knee flexion

A client is placed on a medical regimen of doxorubicin (Adriamycin), cyclophosphamide, and fluorouracil for breast cancer. Which side effect seen in the client would the nurse report to the primary health care provider immediately? a. Shortness of breath b. Nausea and vomiting c. Hair loss d. Mucositis

a. Shortness of breath

A nurse assesses a client who is recovering from a heart transplant. Which assessment findings should alert the nurse to the possibility of heart transplant rejection? (Select all that apply.) a. Shortness of breath b. Abdominal bloating c. New-onset bradycardia d. Increased ejection fraction e. Hypertension

a. Shortness of breath b. Abdominal bloating c. New-onset bradycardia

A nurse cares for a client with infective endocarditis. Which infection control precautions should the nurse use? a. Standard Precautions b. Bleeding precautions c. Reverse isolation d. Contact isolation

a. Standard Precautions

A nurse is constructing a personal preparedness plan in case of a disaster. What does the nurse consider in making this plan? a. Store basic supplies to last for at least 3 days. b. Have short-term arrangements for child care. c. Store enough frozen foods in freezer for 5 days. d. Keep cooking utensils needed in a separate bag.

a. Store basic supplies to last for at least 3 days. Concerns for their home and family can impact the willingness to report in an emergency and can be diminished by being prepared with a personal preparedness plan with enough supplies for 3 days. Any food needs to be nonperishable with no cooking required. Arrangements for children, pets, or older adults would be made for extended period of time.

A nurse cares for clients with urinary incontinence. Which types of incontinence are correctly paired with their description? (Select all that apply.) a. Stress incontinence—urine loss with physical exertion b. Urge incontinence—loss of urine upon feeling the need to void c. Functional incontinence—urine loss results from abnormal detrusor contractions d. Overflow incontinence—constant dribbling of urine e. Reflex incontinence—leakage of urine without lower urinary tract disorder

a. Stress incontinence—urine loss with physical exertion b. Urge incontinence—loss of urine upon feeling the need to void d. Overflow incontinence—constant dribbling of urine

3. A nurse assesses a client who has facial trauma. Which assessment findings require immediate intervention? (Select all that apply.) a. Stridor b. Nasal stuffiness c. Edema of the cheek d. Ecchymosis behind the ear e. Eye pain f. Swollen chin

a. Stridor d. Ecchymosis behind the ear Stridor is a sign of airway obstruction and requires immediate intervention. Ecchymosis, or bruising, behind the ear is called "battle sign" and indicates basilar skull fracture. Nasal stuffiness, edema of the cheek or chin, and eye pain do not interfere with respirations or neurologic function, and therefore are not priorities for immediate intervention.

A nurse is assessing clients with pressure injuries. Which wound description is correctly matched to its description? a. Suspected deep tissue injury: nonblanchable deep purple or maroon. b. Stage 2: may have visible adipose tissue and slough. c. Stage 3: may have a pink or red wound bed. d. Stage 4: wound bed is obscured with eschar or slough.

a. Suspected deep tissue injury: nonblanchable deep purple or maroon. A suspected deep tissue injury is characterized by persistent, nonblanchable purple or maroon discoloration. A stage 2 wound may have a pink of red would bed with granulation tissue. The stage 3 wound may have visible adipose tissue and slough. A stage 4 wound is full-thickness skin loss with exposed or palpable fascia, muscle, tendon, ligament, cartilage, or bone. An unstageable wound is obscured by eschar or slough making assessment impossible.

A nurse prepares to discharge a client who has heart failure. Based on the Heart Failure Core Measure Set, which actions should the nurse complete prior to discharging this client? (Select all that apply.) a. Teach the client about dietary restrictions. b. Ensure the client is prescribed an angiotensin-converting enzyme (ACE) inhibitor. c. Encourage the client to take a baby aspirin each day. d. Confirm that an echocardiogram has been completed. e. Consult a social worker for additional resources.

a. Teach the client about dietary restrictions. b. Ensure the client is prescribed an angiotensin-converting enzyme (ACE) inhibitor. d. Confirm that an echocardiogram has been completed.

A nurse wants to become involved in community disaster preparedness and is interested in helping setup and staff first-aid stations or community acute care centers in the event of a disaster. Which organization is the best fit for this nurse's interests? a. The Medical Reserve Corps b. The National Guard c. The health department d. A Disaster Medical Assistance Team

a. The Medical Reserve Corps The Medical Reserve Corps (MRC) consists of volunteer medical and public health care professionals who support the community during times of need. They may help staff hospitals, establish first-aid stations or special needs shelters, or set up acute care centers in the community. The National Guard often performs search-and-rescue operations and law enforcement. The health department focuses on communicable disease tracking, treatment, and prevention. A Disaster Medical Assistance Team is deployed to a disaster area for up to 72 hours, providing many types of relief services.

1. A home health nurse is visiting a new client who uses oxygen in the home. For which factors does the nurse assess when determining if the client is using the oxygen safely? (Select all that apply.) a. The client does not allow smoking in the house. b. Electrical cords are in good working order. c. Flammable liquids are stored in the garage. d. Household light bulbs are the fluorescent type. e. The client does not have pets inside the home. f. No alcohol-based hand sanitizers are present.

a. The client does not allow smoking in the house. b. Electrical cords are in good working order. c. Flammable liquids are stored in the garage. Oxygen it enhances combustion, so precautions are needed whenever using it. The nurse would assess if the client allows smoking in the house, whether electrical cords are in good shape or are frayed, and if flammable liquids are stored (and used) in the garage away from the oxygen. Light bulbs and pets are not related to oxygen safety. Alcohol-based hand sanitizers are permitted.

A nurse evaluates a client with acute glomerulonephritis (GN). Which assessment finding would the nurse recognize as a positive response to the prescribed treatment? a. The client lost 11 lb (5 kg) in the past 10 days. b. The client's urine specific gravity is 1.048. c. No blood is observed in the client's urine. d. The client's blood pressure is 152/88 mm Hg.

a. The client lost 11 lb (5 kg) in the past 10 days.

5. A client, who has become increasingly dyspneic over a year, has been diagnosed with pulmonary fibrosis. What information would the nurse plan to include in teaching this client? (Select all that apply.) a. The need to avoid large crowds and people who are ill b. Safety measures to take if home oxygen is needed c. Information about appropriate use of the drug nintedanib d. Genetic therapy to stop the progression of the disease e. Measures to avoid fatigue during the day f. The possibility of receiving a lung transplant if infection-free for a year

a. The need to avoid large crowds and people who are ill b. Safety measures to take if home oxygen is needed c. Information about appropriate use of the drug nintedanib e. Measures to avoid fatigue during the day Pulmonary fibrosis is a progressive disorder with no cure. Therapy focuses on slowing progression and managing dyspnea. Clients need to avoid contracting infections so should be taught to stay away from large crowds and sick people. Home oxygen is needed and the nurse would teach safety measures related to oxygen. The drug nintedanib has shown to improve cellular regulation and slow progression of the disease. Gene therapy is not available. Energy conservation measures are also an important topic. Lung transplantation is an unlikely option due to selection criteria.

The nurse learns that which age-related changes increase the potential for complications of burns? (Select all that apply.) a. Thinner skin b. Slower healing time c. Decreased mobility d. Hyperresponsive immune response e. Increased risk of unnoticed sepsis f. Pre-existing conditions

a. Thinner skin b. Slower healing time c. Decreased mobility e. Increased risk of unnoticed sepsis f. Pre-existing conditions Age-related differences that can increase the risk of burns and complications of burns include thinner skin, slower healing, decreased mobility, increased risk of infection that goes unnoticed, and pre-existing conditions that can complicate recovery. The older adult has decreased inflammatory and immune responses.

A 100-kg client has developed ARDS and needs mechanical ventilation. Which of the following are potentially correct ventilator management choices? (Select all that apply.) a. Tidal volume: 600 mL b. Volume-controlled ventilation c. PEEP based on oxygen saturation d. Suctioning every hour e. High-frequency oscillatory ventilation f. Limited turning for ventilator pressures

a. Tidal volume: 600 mL c. PEEP based on oxygen saturation e. High-frequency oscillatory ventilation

A nurse is caring for a postoperative 70-kg client who had major blood loss during surgery. Which findings by the nurse would prompt immediate action to prevent acute kidney injury? (Select all that apply.) a. Urine output of 100 mL in 4 hours b. Urine output of 500 mL in 12 hours c. Large amount of sediment in the urine d. Amber, odorless urine e. Blood pressure of 90/60 mm Hg

a. Urine output of 100 mL in 4 hours c. Large amount of sediment in the urine e. Blood pressure of 90/60 mm Hg ANS: A, C, E T

The nurse assesses a client who is recovering from a paracentesis 1 hour ago. Which assessment finding would require immediate action by the nurse? a. Urine output via indwelling urinary catheter is 20 mL/hr b. Blood pressure increases from 110/58 to 120/62 mm Hg c. Respiratory rate decreases from 22 to 16 breaths/min d. A decrease in the client's weight by 3 lb (1.4 kg

a. Urine output via indwelling urinary catheter is 20 mL/hr

The nurse assesses a client who is recovering from a paracentesis 1 hour ago. Which assessment finding would require immediate action by the nurse? a. Urine output via indwelling urinary catheter is 20 mL/hr b. Blood pressure increases from 110/58 to 120/62 mm Hg c. Respiratory rate decreases from 22 to 16 breaths/min d. A decrease in the client's weight by 3 lb (1.4 kg)

a. Urine output via indwelling urinary catheter is 20 mL/hr Rapid removal of ascitic fluid causes decreased abdominal pressure, which can contribute to hypovolemia. This can be manifested by a decrease in urine output to below 30 mL/hr. A slight increase in systolic blood pressure is insignificant. A decrease in respiratory rate indicates that breathing has been made easier by the procedure. The nurse would expect the client's weight to drop as fluid is removed. To prevent hypovolemic shock, no more than 2000 mL are usually removed from the abdomen at one time. The patient's weight typically only decreases by less than 2 kg or 4.4 lb

A nurse cares for a client with diabetes mellitus who asks, "Why do I need to administer more than one injection of insulin each day?" How would the nurse respond? a. "You need to start with multiple injections until you become more proficient at self-injection." b. "A single dose of insulin each day would not match your blood insulin levels and your food intake patterns." c. "A regimen of a single dose of insulin injected each day would require that you eat fewer carbohydrates." d. "A single dose of insulin would be too large to be absorbed, predictably putting you at risk for insulin shock."

b. "A single dose of insulin each day would not match your blood insulin levels and your food intake patterns."

A nurse assesses a client who has a history of heart failure. Which question should the nurse ask to assess the extent of the client's heart failure? a. "Do you have trouble breathing or chest pain?" b. "Are you able to walk upstairs without fatigue?" c. "Do you awake with breathlessness during the night?" d. "Do you have new-onset heaviness in your legs?"

b. "Are you able to walk upstairs without fatigue?"

A nurse teaches a client recovering from a heart transplant who is prescribed cyclosporine (Sandimmune). Which statement should the nurse include in this client's discharge teaching? a. "Use a soft-bristled toothbrush and avoid flossing." b. "Avoid large crowds and people who are sick." c. "Change positions slowly to avoid hypotension." d. "Check your heart rate before taking the medication."

b. "Avoid large crowds and people who are sick."

A nurse teaches a client with type 2 diabetes mellitus who has been prescribed glipizide (Glucotrol). Which statement would the nurse include in this client's teaching? a. "Change positions slowly when you get out of bed." b. "Avoid taking nonsteroidal anti-inflammatory drugs (NSAIDs)." c. "If you miss a dose of this drug, you can double the next dose." d. "Discontinue the medication if you develop a urinary infection."

b. "Avoid taking nonsteroidal anti-inflammatory drugs (NSAIDs)."

A client is concerned about the risk of lymphedema after a mastectomy. Which response by the nurse is best? a. "You do not need to worry about lymphedema since you did not have radiation therapy." b. "Be careful not to injure that arm or get any infection in that arm." c. "Numbness, tingling, and swelling are common sensations after a mastectomy." d. "The risk for lymphedema is a real threat and can be very self-limiting."

b. "Be careful not to injure that arm or get any infection in that arm.

The nurse is caring for a client who has been prescribed lubiprostone for irritable bowel syndrome (IBS-C). What health teaching will the nurse include about taking this drug? a. "This drug will make you very dry because it will decrease your diarrhea." b. "Be sure to take this drug with food and water to help manage constipation." c. "Avoid people who have infection as this drug will suppress your immune system." d. "Include high-fiber foods in your diet to help produce more solid stools."

b. "Be sure to take this drug with food and water to help manage constipation."

A nurse asks the charge nurse to explain the difference between critical thinking and clinical judgment. What statement by the charge nurse is best? a. "Clinical judgment is often clouded by erroneous hypotheses." b. "Clinical judgment is the observable outcome of critical thinking." c. "Critical thinking requires synthesizing interactions within a situation." d. "Critical thinking is the highest level of nursing judgment."

b. "Clinical judgment is the observable outcome of critical thinking." Clinical judgment is the observable outcome of critical thinking and decision making. It can be, but most often is not, clouded by erroneous hypotheses. Recognizing, understanding, and synthesizing interactions and interdependencies in a set of components designed for a specific purpose is systems thinking. Critical thinking is not the highest level of nursing judgment.

After a hospital's emergency department (ED) has efficiently triaged, treated, and transferred clients from a community disaster to appropriate units, the hospital incident command officer wants to "stand down" from the emergency plan. Which question would the nursing supervisor ask at this time? a. "Are you sure no more victims are coming into the ED?" b. "Do all areas of the hospital have the supplies and personnel they need?" c. "Have all ED staff had the chance to eat and rest recently?" d. "Does the Chief Medical Officer agree this disaster is under control?"

b. "Do all areas of the hospital have the supplies and personnel they need?" Before "standing down," the incident command officer ensures that the needs of the other hospital departments have been taken care of because they may still be stressed and may need continued support to keep functioning. Many more "walking wounded" victims may present to the ED; that number may not be predictable. Giving staff the chance to eat and rest is important, but all areas of the facility need that too. Although the Chief Medical Officer (CMO) may be involved in the incident, the CMO does not determine when the hospital can "stand down."

The nurse is caring for a client who is prescribed sulfasalazine. Which question would the nurse ask the client before starting this drug? a. "Are you taking Vitamin C or B? b. "Do you have any allergy to sulfa drugs?" c. "Can you swallow pills pretty easily?" d. "Do you have insurance to cover this drug?"

b. "Do you have any allergy to sulfa drugs?" Sulfasalazine is a sulfa drug given for clients who have ulcerative colitis. However, it should not be given to those who have an allergy to sulfa and sulfa drugs to prevent a hypersensitivity reaction

A nurse observes that a client's anteroposterior (AP) chest diameter is the same as the lateral chest diameter. Which question would the nurse ask the client in response to this finding? a. "Are you taking any medications or herbal supplements?" b. "Do you have any chronic breathing problems?" c. "How often do you perform aerobic exercise?" d. "What is your occupation and what are your hobbies?"

b. "Do you have any chronic breathing problems?"

4. A nurse assesses a client with chronic obstructive pulmonary disease. Which questions would the nurse ask to determine the client's activity tolerance? (Select all that apply.) a. "What color is your sputum?" b. "Do you have any difficulty sleeping?" c. "How long does it take to perform your morning routine?" d. "Do you walk upstairs every day?" e. "Have you lost any weight lately?" f. "How does your activity compare to this time last year?"

b. "Do you have any difficulty sleeping?" c. "How long does it take to perform your morning routine?" e. "Have you lost any weight lately?" f. "How does your activity compare to this time last year?" Difficulty sleeping could indicate worsening breathlessness, as could taking longer to perform activities of daily living. Weight loss could mean increased dyspnea as the client becomes too fatigued to eat. The color of the client's sputum would not assist in determining activity tolerance. Asking whether the client walks upstairs every day is not as pertinent as determining if the client becomes short of breath on walking upstairs, or if the client goes upstairs less often than previously. The nurse would ask the client to compare his or her current level of activity with that of a month or even a year ago.

10. A newly graduated nurse in the hospital states that, since she is so new, she cannot participate in quality improvement (QI) projects. What response by the precepting nurse is best? a. "All staff nurses are required to participate in quality improvement here." b. "Even being new, you can implement activities designed to improve care." c. "It's easy to identify what indicators should be used to measure quality." d. "You should ask to be assigned to the research and quality committee."

b. "Even being new, you can implement activities designed to improve care."

A nurse teaches a client with heart failure about energy conservation. Which statement should the nurse include in this client's teaching? a. "Walk until you become short of breath, and then walk back home." b. "Gather everything you need for a chore before you begin." c. "Pull rather than push or carry items heavier than 5 pounds." d. "Take a walk after dinner every day to build up your strength."

b. "Gather everything you need for a chore before you begin."

A nurse assesses a client who presents with an increase in psoriatic lesions. Which questions would the nurse ask to identify a possible trigger for worsening of this client's psoriatic lesions? (Select all that apply.) a. "Have you eaten a large amount of chocolate lately?" b. "Have you been under a lot of stress lately?" c. "Have you recently used a public shower?" d. "Have you been out of the country recently?" e. "Have you recently had any other health problems?" f. "Have you changed any medications recently?"

b. "Have you been under a lot of stress lately?" e. "Have you recently had any other health problems?" f. "Have you changed any medications recently?" Outbreaks of psoriasis can be induced by stress, environmental triggers, certain medications, skin injuries, infections, smoking, alcohol use, and obesity. Psoriatic lesions are not triggered by chocolate, public showers, or international travel.

A nurse is teaching a client who has itchy, raised red patches covered with a silvery white scale how to care for this disorder. What statement by the client shows a need for further information? a. "At the next family reunion, I'm going to ask my relatives if they have anything similar." b. "I have to make sure I keep my lesions covered, so I do not spread this to others." c. "I must avoid large crowds and sick people while I am taking adalimumab." d. "I will buy a good quality emollient to put on my skin each day."

b. "I have to make sure I keep my lesions covered, so I do not spread this to others." This client has plaque psoriasis which is not a contagious disorder. The client does not have to worry about spreading the condition to others. It is a condition that has hereditary links so it would be correct for the client to inquire about other family members who are affects. Adalimumab is a drug used to treat psoriasis and it has a black box warning about serious infection risk and cancer risk, so the client needs to take precautions to avoid infectious individuals. Emollients help keep the plaques soft and reduce itching.

A client is being discharged home after a large myocardial infarction and subsequent coronary artery bypass grafting surgery. The client's sternal wound has not yet healed. What statement by the client most indicates a higher risk of developing sepsis after discharge? a. "All my friends and neighbors are planning a party for me." b. "I hope I can get my water turned back on when I get home." c. "I am going to have my daughter scoop the cat litter box." d. "My grandkids are so excited to have me coming home!"

b. "I hope I can get my water turned back on when I get home." All these statements indicate a potential for leading to infection once the client gets back home. A large party might include individuals who are themselves ill and contagious. Having litter boxes in the home can expose the client to microbes that can lead to infection. Small children often have upper respiratory infections and poor hand hygiene that spread germs. However, the most worrisome statement is the lack of running water for handwashing and general hygiene and cleaning purposes.

After teaching a client who has been diagnosed with hepatitis A, the nurse assesses the client's understanding. Which statement by the client indicates correct understanding of the teaching? a. "Some medications have been known to cause hepatitis A." b. "I may have been exposed when we ate shrimp last weekend." c. "I was infected with hepatitis A through a recent blood transfusion." d. "My infection with Epstein-Barr virus can co-infect me with hepatitis A."

b. "I may have been exposed when we ate shrimp last weekend." The route of transmission for hepatitis A infection is through close personal contact or ingestion of contaminated water or shellfish. Hepatitis A is not transmitted through medications, blood transfusions, or Epstein-Barr virus. Toxic and drug-induced hepatitis is caused from exposure to hepatotoxins, but this is not a form of hepatitis A. Hepatitis B can be spread through blood transfusions. Epstein-Barr virus causes a secondary infection that is not associated with hepatitis A

After teaching a client with early polycystic kidney disease (PKD) about nutritional therapy, the nurse assesses the client's understanding. Which statement made by the client indicates a correct understanding of the teaching? a. "I will take a laxative every night before going to bed." b. "I must increase my intake of dietary fiber and fluids." c. "I shall only use salt when I am cooking my own food." d. "I'll eat white bread to minimize gastrointestinal gas.

b. "I must increase my intake of dietary fiber and fluids."

A nurse is giving discharge instructions to a client recently diagnosed with chronic kidney disease (CKD). Which statements made by the client indicate a correct understanding of the teaching? (Select all that apply.) a. "I can continue to take antacids to relieve heartburn." b. "I need to ask for an antibiotic when scheduling a dental appointment." c. "I'll need to check my blood sugar often to prevent hypoglycemia." d. "The dose of my pain medication may have to be adjusted." e. "I should watch for bleeding when taking my anticoagulants."

b. "I need to ask for an antibiotic when scheduling a dental appointment." c. "I'll need to check my blood sugar often to prevent hypoglycemia." d. "The dose of my pain medication may have to be adjusted." e. "I should watch for bleeding when taking my anticoagulants."

16. A clinic nurse is reviewing care measures with a client who has asthma, Step 3. What statement by the client indicates the need to review the information? a. "I still will use my rapid-acting inhaler for an asthma attack." b. "I will always use the spacer with my dry powder inhaler." c. "If I am stable for 3 months, I might be able to reduce my drugs." d. "My inhaled corticosteroid must be taken regularly to work well."

b. "I will always use the spacer with my dry powder inhaler." Dry powder inhalers are not used with a spacer. The other statements are accurate.

After teaching a client who is being discharged home after mitral valve replacement surgery, the nurse assesses the client's understanding. Which client statement indicates a need for additional teaching? a. "I'll be able to carry heavy loads after 6 months of rest." b. "I will have my teeth cleaned by my dentist in 2 weeks." c. "I must avoid eating foods high in vitamin K, like spinach." d. "I must use an electric razor instead of a straight razor to shave."

b. "I will have my teeth cleaned by my dentist in 2 weeks."

After teaching a client with nephrotic syndrome and a normal glomerular filtration, the nurse assesses the client's understanding. Which statement made by the client indicates a correct understanding of the diet therapy for this condition? a. "I must decrease my intake of fat." b. "I will increase my intake of protein." c. "A decreased intake of carbohydrates will be required." d. "An increased intake of vitamin C is necessary."

b. "I will increase my intake of protein."

The nurse is caring for a client who is planning to have a laparoscopic colon resection for colorectal cancer tomorrow. Which statement by the client indicates a need for further teaching? a. "I should have less pain after this surgery compared to having a large incision." b. "I will probably be in the hospital for 3 to 4 days after surgery." c. "I will be able to walk around a little on the same day as the surgery." d. "I will be able to return to work in a week or two depending on how I do."

b. "I will probably be in the hospital for 3 to 4 days after surgery."

After teaching a client who is prescribed adalimumab for severe ulcerative colitis (UC), the nurse assesses the client's understanding. Which statement made by the client indicates a need for further teaching? a. "I will avoid large crowds and people who are sick." b. "I will take this medication with my breakfast each morning." c. "Nausea and vomiting are common side effects of this drug." d. "I should wash my hands after I play with my dog."

b. "I will take this medication with my breakfast each morning." Adalimumab is an immune modulator that is given via subcutaneous injection. It does not need to be given with food or milk. Nausea and vomiting are two common side effects. Adalimumab can cause immune suppression, so clients receiving the medication should avoid large crowds and people who are sick, and should practice good handwashing.

After teaching a client with hypertension secondary to renal disease, the nurse assesses the client's understanding. Which statement made by the client indicates a need for additional teaching? a. "I can prevent more damage to my kidneys by managing my blood pressure." b. "If I have increased urination at night, I need to drink less fluid during the day." c. "I need to see the registered dietitian to discuss limiting my protein intake." d. "It is important that I take my antihypertensive medications as directed."

b. "If I have increased urination at night, I need to drink less fluid during the day."

A nurse is confused on why systems thinking is important since working on the unit involves caring for a few specific clients. What explanation by the nurse manager is best? a. "It's a good way to conduct root-cause analysis." b. "It is important for quality improvement and safety." c. "Systems thinking helps you see the bigger picture." d. "You may enter management 1 day and need to know this."

b. "It is important for quality improvement and safety." A systems thinking approach to care reinforces the nurse's role in safety and quality improvement while expanding clinical judgment to include the patient's place within the greater health care system in the context of care decisions. Root-cause analyses would be a small portion of systems thinking. It does give the nurse a big-picture view, but this answer is vague. The nurse may or may not ever join management.

A primary health care provider prescribes diazepam to a client who was bitten by a black widow spider. The client asks, "What is this medication for?" How does the nurse respond? a. "This medication is an antivenom for this type of bite." b. "It will relieve your muscle rigidity and spasms." c. "It prevents respiratory difficulty from excessive secretions." d. "This medication will prevent respiratory failure."

b. "It will relieve your muscle rigidity and spasms." Black widow spider venom can produce muscle rigidity and spasms, which are treated with the muscle relaxant, diazepam. It does not prevent respiratory difficulty or failure nor is it antivenom.

The nurse is caring for a client with hepatic portal-systemic encephalopathy (PSE). The client is thin and cachectic, and the family expresses distress that the patient is receiving little dietary protein. How would the nurse respond? a. "A low-protein diet will help the liver rest and will restore liver function." b. "Less protein in the diet will help prevent confusion associated with liver failure." c. "Increasing dietary protein will help the patient gain weight and muscle mass." d. "Low dietary protein is needed to prevent fluid from leaking into the abdomen."

b. "Less protein in the diet will help prevent confusion associated with liver failure." A low-protein diet is prescribed when serum ammonia levels increase and/or the client shows signs of PSE. A low-protein diet helps reduce excessive breakdown of protein into ammonia by intestinal bacteria. Encephalopathy is caused by excess ammonia. A low-protein diet has no impact on restoring liver function. Increasing the patient's dietary protein will cause complications of liver failure and would not be suggested. Increased intravascular protein will help prevent ascites, but clients with liver failure are not able to effectively synthesize dietary protein.

A nurse teaches a client with diabetes mellitus about sick-day management. Which statement would the nurse include in this client's teaching? a. "When ill, avoid eating or drinking to reduce vomiting and diarrhea." b. "Monitor your blood glucose levels at least every 4 hours while sick." c. "If vomiting, do not use insulin or take your oral antidiabetic agent." d. "Try to continue your prescribed exercise regimen even if you are sick."

b. "Monitor your blood glucose levels at least every 4 hours while sick."

A nurse assesses a client admitted to the cardiac unit. Which statement by the client alerts the nurse to the possibility of right-sided heart failure? a. "I sleep with four pillows at night." b. "My shoes fit really tight lately." c. "I wake up coughing every night." d. "I have trouble catching my breath."

b. "My shoes fit really tight lately."

7. A nurse teaches a client who has epistaxis and recently had his nasal packing removed. Which statements indicate that the client correctly understood the teaching? (Select all that apply.) a. "I will vigorously blow my nose multiple times each day." b. "Nasal saline sprays will help to prevent rebleeding." c. "I will wait at least 1 month before resuming weight lifting." d. "Ibuprofen will decrease nasal swelling and pain." e. "I will apply a small amount of petroleum jelly to my nares."

b. "Nasal saline sprays will help to prevent rebleeding." c. "I will wait at least 1 month before resuming weight lifting." e. "I will apply a small amount of petroleum jelly to my nares." A nurse would teach a client to avoid vigorous nose blowing, the use of aspirin or other NSAIDs, and strenuous activities such as heavy lifting for at least 1 month. The nurse would also teach the client to apply petroleum jelly sparingly to the nares for lubrication and comfort, and to use nasal saline sprays and humidification to prevent rebleeding.

7. An older adult is brought to the emergency department by a family member, who reports a moderate change in mental status and mild cough. The client is afebrile. The primary health care provider orders a chest x-ray. The family member questions why this is needed since the symptoms seem so vague. What response by the nurse is best? a. "Chest x-rays are always ordered when we suspect pneumonia." b. "Older people often have vague symptoms, so an x-ray is essential." c. "The x-ray can be done and read before laboratory work is reported." d. "We are testing for any possible source of infection in the client."

b. "Older people often have vague symptoms, so an x-ray is essential." It is essential to obtain an early chest x-ray in older adults suspected of having pneumonia because symptoms are often vague. Waiting until definitive signs and symptoms are present to obtain the x-ray leads to a costly delay in treatment. Stating that chest x-rays are always ordered does not give the family definitive information. The x-ray can be done while laboratory values are still pending, but this also does not provide specific information about the importance of a chest x-ray in this client. The client has symptoms of pneumonia, so the staff is not testing for any possible source of infection but rather is testing for a suspected disorder.

A client has an external percutaneous transhepatic biliary catheter inserted for a biliary obstruction. What health teaching about catheter care would the nurse provide for the client? a. "Cap the catheter drain at night to prevent leakage and skin damage." b. "Position the drainage bag lower than the catheter insertion site." c. "Irrigate the catheter with an ounce of saline every night." d. "Pierce a hole in the top of the drainage bag to get rid of odors."

b. "Position the drainage bag lower than the catheter insertion site." An external temporary or permanent catheter drains bile by gravity into a bag that collects bile. Therefore, the drainage bag should be lower that the catheter insertion site. The catheter should not be capped or irrigated, and no holes should be made in the bag to prevent bile from having contact with the skin

A nurse cares for a client with a new ileostomy. The client states, "I don't think my friends will accept me with this ostomy." How would the nurse respond? a. "Your friends will be happy that you are alive." b. "Tell me more about your concerns." c. "A therapist can help you resolve your concerns." d. "With time you will accept your new body."

b. "Tell me more about your concerns." Social anxiety and apprehension are common in clients with a new ileostomy. The nurse would encourage the client to discuss concerns by restating them in an open-ended manner. The nurse would not minimize the client's concerns or provide false reassurance

9. A nurse is teaching a client how to perform pursed-lip breathing. Which instructions would the nurse include in this teaching? (Select all that apply.) a. "Open your mouth and breathe deeply." b. "Use your abdominal muscles to squeeze air out of your lungs." c. "Breath out slowly without puffing your cheeks." d. "Focus on inhaling and holding your breath as long as you can." e. "Exhale at least twice the amount of time it took to breathe in." f. "Lie on your back with your knees bent."

b. "Use your abdominal muscles to squeeze air out of your lungs." c. "Breath out slowly without puffing your cheeks." e. "Exhale at least twice the amount of time it took to breathe in." A nurse would teach a client to close his or her mouth and breathe in through his or her nose, purse his or her lips and breathe out slowly without puffing his or her cheeks, and use his or her abdominal muscles to squeeze out every bit of air. The nurse would also remind the client to use pursed-lip breathing during any physical activity, to focus on exhaling, and to never hold his or her breath. Lying on the back with bent knees is the preferred position for diaphragmatic breathing.

The nurse assesses a client with a history of urinary incontinence who presents with extreme dry mouth, constipation, and an inability to void. Which question would the nurse ask first? a. "Are you drinking plenty of water?" b. "What medications are you taking?" c. "Have you tried laxatives or enemas?" d. "Has this type of thing ever happened before?"

b. "What medications are you taking?"

A client is preparing to have a laparoscopic restorative proctocolectomy with ileo pouch-anal anastomosis (RCA-IPAA). Which preoperative health teaching would the nurse include? a. "You will have to wear an appliance for your permanent ileostomy." b. "You should be able to have better bowel continence after healing occurs." c. "You will have a large abdominal incision that will require irrigation." d. "This procedure can be performed under general or regional anesthesia."

b. "You should be able to have better bowel continence after healing occurs." A RCA-IPAA can improve bowel continence although leakage may still occur for some clients. The procedure is a 2-step process performed under general anesthesia using a laparoscope which does not require an abdominal incision or permanent ileostomy.

A telehealth nurse speaks with a client who is recovering from a liver transplant 2 weeks ago. The client states, "I'm having right belly pain and have a temperature of 101° F (38.3° C)." How would the nurse respond? a. "The anti-rejection drugs you are taking make you susceptible to infection." b. "You should go to the hospital immediately to get checked out." c. "You should take an additional dose of cyclosporine today." d. "Take acetaminophen every 4 hours until you feel better soon."

b. "You should go to the hospital immediately to get checked out." Fever, right abdominal quadrant pain, and jaundice are signs of possible liver transplant rejection; the client would be admitted to the hospital as soon as possible for intervention. Antirejection drugs do make a client more susceptible to infection, but this client has signs of rejection, not infection. The nurse would not advise the client to take an additional dose of cyclosporine or acetaminophen as these medications will not treat the acute rejection

A nurse is teaching a client with diabetes mellitus who asks, "Why is it necessary to maintain my blood glucose levels no lower than about 60 mg/dL (3.3 mmol/L)?" How would the nurse respond? a. "Glucose is the only fuel used by the body to produce the energy that it needs." b. "Your brain needs a constant supply of glucose because it cannot store it." c. "Without a minimum level of glucose, your body does not make red blood cells." d. "Glucose in the blood prevents the formation of lactic acid and prevents acidosis."

b. "Your brain needs a constant supply of glucose because it cannot store it."

The nurse is caring for a client who has diabetes mellitus. The nurse administers 6 units of regular insulin and 10 units of NPH insulin at 7:00 a.m. (0700). At which time would the nurse assess the client for potential hypoglycemia related to the NPH insulin? a. 8:00 a.m. (0800) b. 4:00 p.m. (1600) c. 8:00 p.m. (2000) d. 11:00 p.m. (2300)

b. 4:00 p.m. (1600)

5. A nurse is triaging clients in the emergency department (ED). Which client would the nurse prioritize to receive care first? a. A 22 year old with a painful and swollen right wrist b. A 45 year old reporting chest pain and diaphoresis c. A 60 year old reporting difficulty swallowing and nausea d. An 81 year old, respiratory rate 28 breaths/min and temperature of 101° F (38.8°C)

b. A 45 year old reporting chest pain and diaphoresis A client experiencing chest pain and diaphoresis would be classified as emergent and would be triaged immediately to a treatment room in the ED. The other clients are more stable.

17. A pulmonary nurse cares for clients who have chronic obstructive pulmonary disease (COPD). Which client would the nurse assess first? a. A 46 year old with a 30-pack-year history of smoking b. A 52 year old in a tripod position using accessory muscles to breathe c. A 68 year old who has dependent edema and clubbed fingers d. A 74 year old with a chronic cough and thick, tenacious secretions

b. A 52 year old in a tripod position using accessory muscles to breathe The client who is in a tripod position and using accessory muscles is working to breathe. This client must be assessed first to establish how effectively the client is breathing and provide interventions to minimize respiratory distress. The other clients are not in acute distress.

A middle-age mountain hiker is admitted to the emergency department exhibiting a cough with pink, frothy sputum and cyanosis of lips and nail beds. What priority action would the nurse implement? a. Administer acetazolamide. b. Administer oxygen via a nonrebreather mask. c. Complete a thorough pulmonary assessment. d. Obtain arterial blood gas (ABG) specimen for analysis.

b. Administer oxygen via a nonrebreather mask. The client is exhibiting signs of AMS with high-altitude pulmonary edema (HAPE). Cyanosis indicates hypoxia and must be treated immediately. A complete pulmonary assessment and ABG analysis are indicated but the priority is oxygen administration. Acetazolamide is used to prevent AMS.

13. A client has been taking isoniazid for tuberculosis for 3 weeks. What laboratory results need to be reported to the primary health care provider immediately? a. Albumin: 5.1 g/dL (7.4 mcmol/L) b. Alanine aminotransferase (ALT): 180 U/L c. Red blood cell (RBC) count: 5.2/million/μL (5.2 1012/L) d.White blood cell (WBC) count: 12,500/mm3 (12.5 109/L)

b. Alanine aminotransferase (ALT): 180 U/L INH can cause liver damage, especially if the client drinks alcohol. The ALT (one of the liver enzymes) is extremely high and needs to be reported immediately. The albumin and RBCs are normal. The WBCs are slightly high, but that would be an expected finding in a client with an infection.

The nurse is caring for a client who has a postoperative paralytic ileus following abdominal surgery. What drug is appropriate to manage this nonmechanical bowel obstruction? a. Alosetron b. Alvimopan c. Amitiptyline d. Amlodipine

b. Alvimopan

The nurse is caring for a client with a new diagnosis of chronic kidney disease. Which priority complications would the nurse anticipate? (Select all that apply.) a. Dehydration b. Anemia c. Hypertension d. Dysrhythmias e. Heart failure

b. Anemia c. Hypertension d. Dysrhythmias e. Heart failure

A nurse assesses a patient who has celiac disease. Which signs and symptoms would the nurse expect? (Select all that apply.) a. Weight gain b. Anorexia c. Constipation d. Anal fistula e. Abdominal pain

b. Anorexia c. Constipation e. Abdominal pain Signs and symptoms of celiac disease include weight loss, anorexia, constipation, and abdominal pain. Anal fistulas are not associated with celiac disease

10. A nurse evaluates the following arterial blood gas values in a client: pH 7.48, PaO2 98 mm Hg, PaCO2 28 mm Hg, and HCO3 22 mEq/L. Which client condition should the nurse correlate with these results? a. Diarrhea and vomiting for 36 hours b. Anxiety-induced hyperventilation c. Chronic obstructive pulmonary disease (COPD) d. Diabetic ketoacidosis and emphysema

b. Anxiety-induced hyperventilation The elevated pH level indicates alkalosis. The bicarbonate level is normal, and so is the oxygen partial pressure. Loss of carbon dioxide is the cause of the alkalosis, which would occur in response to hyperventilation. Diarrhea and vomiting would cause metabolic alterations, COPD would lead to respiratory acidosis, and the client with emphysema most likely would have combined metabolic acidosis on top of a mild, chronic respiratory acidosis.

2. A nurse cares for a client with arthritis who reports frequent asthma attacks. What action would the nurse take first? a. Review the client's pulmonary function test results. b. Ask about medications the client is currently taking. c. Assess how frequently the client uses a bronchodilator. d. Consult the primary health care provider and request arterial blood gases.

b. Ask about medications the client is currently taking. Aspirin and other nonsteroidal anti-inflammatory drugs (NSAIDs) can trigger asthma in some people. This results from increased production of leukotriene when aspirin or NSAIDs suppress other inflammatory pathways and is a likely culprit given the client's history. Reviewing pulmonary function test results will not address the immediate problem of frequent asthma attacks. This is a good time to review response to bronchodilators, but assessing triggers is more important. Questioning the client about the use of bronchodilators will address interventions for the attacks but not their cause. Reviewing arterial blood gas results would not be of use in a client between attacks because many clients are asymptomatic when not having attacks.

A nurse is caring for a terminally ill client who has just died in a hospital setting with family members at the bedside. Which action should the nurse take first? a. Call for emergency assistance so that resuscitation procedures can begin. b. Ask family members if they would like to spend time alone with the client. c. Ensure that a death certificate has been completed by the physician. d. Request family members to prepare the clients body for the funeral home.

b. Ask family members if they would like to spend time alone with the client. Before moving the clients body to the funeral home, the nurse should ask family members if they would like to be alone with the client. Emergency assistance will not be necessary. Although it is important to ensure that a death certificate has been completed before the client is moved to the mortuary, the nurse first should ask family members if they would like to be alone with the client. The clients family should not be expected to prepare the body for the funeral home.

A nurse is caring for a client after surgery. The client's respiratory rate has increased from 12 to 18 breaths/min and the pulse rate increased from 86 to 98 beats/min since the client was last assessed 4 hours ago. What action by the nurse is best? a. Ask if the client needs pain medication. b. Assess using the MEWS score. c. Document the findings in the client's chart. d. Increase the rate of the client's IV infusion

b. Assess using the MEWS score. Signs of the earliest stage of shock are subtle and may manifest in slight increases in heart rate, respiratory rate, or blood pressure. Even though these readings are not out of the normal range, the nurse would conduct a thorough assessment of the patient, focusing on indicators of perfusion. The MEWS score (Modified Early Warning Score) was developed to identify clients at risk for deterioration. The client may need pain medication, but this is not the priority at this time. Documentation would be done thoroughly but would be done after the assessment. The nurse would not increase the rate of the IV infusion without an order.

A nurse assesses a client who is recovering from a nephrostomy. Which assessment findings would alert the nurse to urgently contact the primary health care provider? (Select all that apply.) a. Clear drainage b. Bloody drainage at site c. Patient reports headache d. Foul-smelling drainage e. Urine draining from site

b. Bloody drainage at site d. Foul-smelling drainage e. Urine draining from site

A nurse assesses clients on a medical-surgical unit, all of whom have stage 2 or 3 pressure injuries. Which client would the nurse evaluate further for a wound infection? a. WBC 9200 mm/L3 (9.2 109) b. Boggy feel to granulation tissue c. Increased size after debridement d. Requesting pain medication

b. Boggy feel to granulation tissue Wound infection may or may not occur in the presence of signs of systemic infection, but a change in the appearance, texture, color, drainage, or size of a wound (except after debridement) is indicative of possible infection. The nurse would assess the client with boggy granulation tissue further. The WBC is normal. After debridement, the wound may look larger. If the client needs a sudden increase in the amount or frequency of pain medication that would be another indicator, but there is no evidence this client has more pain than usual.

After teaching a client with irritable bowel syndrome (IBS), a nurse assesses the client's understanding. Which menu selection indicates that the client correctly understands the dietary teaching? a. Ham sandwich on white bread, cup of applesauce, carbonated beverage b. Broiled chicken with brown rice, steamed broccoli, glass of apple juice c. Grilled cheese sandwich, small banana, cup of hot tea with lemon d. Baked tilapia, fresh green beans, cup of coffee with low-fat milk

b. Broiled chicken with brown rice, steamed broccoli, glass of apple juice

After delegating care to assistive personnel (AP) for a client who is prescribed habit training to manage incontinence, a nurse evaluates the AP's understanding. Which action indicates that the AP needs additional teaching? a. Toileting the client after breakfast b. Changing the client's incontinence brief when wet c. Encouraging the client to drink fluids d. Recording the client's incontinence episodes

b. Changing the client's incontinence brief when wet

After teaching a client who has a stage 2 pressure injury, a nurse assesses the client's understanding. Which dietary choice by the client indicates a good understanding of the teaching? a. Green salad, a banana, whole wheat dinner roll, coffee b. Chicken breast, broccoli, baked potato, ice water c. Vegetable lasagna and green salad, iced tea d. Hamburger, fruit cup, cookie, diet pop

b. Chicken breast, broccoli, baked potato, ice water Successful healing of pressure injuries depends on adequate intake of calories, protein, vitamins, minerals, and water. The dinner with the chicken breast meets all these criteria. The other dinners while having some healthy items each, are not as nutritious.

21. A nurse administers medications to a client who has asthma. Which medication classification is paired correctly with its physiologic action? a. Bronchodilator—stabilizes the membranes of mast cells and prevents the release of inflammatory mediators. b. Cholinergic antagonist—causes bronchodilation by inhibiting the parasympathetic nervous system. c. Corticosteroid—relaxes bronchiolar smooth muscles by binding to and activating pulmonary beta2 receptors. d. Cromone—disrupts the production of pathways of inflammatory mediators.

b. Cholinergic antagonist—causes bronchodilation by inhibiting the parasympathetic nervous system. Cholinergic antagonist drugs cause bronchodilation by inhibiting the parasympathetic nervous system. This allows the sympathetic nervous system to dominate and release norepinephrine that activates beta2 receptors. Bronchodilators relax bronchiolar smooth muscles by binding to and activating pulmonary beta2 receptors. Corticosteroids disrupt the production of pathways of inflammatory mediators. Cromones stabilize the membranes of mast cells and prevent the release of inflammatory mediators.

A hospital prepares to receive large numbers of casualties from a community disaster. Which clients would the nurse identify as appropriate for discharge or transfer to another facility? (Select all that apply.) a. Older adult in the medical decision unit for evaluation of chest pain b. Client who had open reduction and internal fixation of a femur fracture 3 days ago c. Client admitted last night with community-acquired pneumonia d. Infant who has a fever of unknown origin e. Client on the medical unit for wound care f. Client with symptoms of influenza after traveling abroad

b. Client who had open reduction and internal fixation of a femur fracture 3 days ago e. Client on the medical unit for wound care The client with the femur fracture could be transferred to a rehabilitation facility, and the client on the medical unit for wound care could be transferred home with home health or to a long-term care facility for ongoing wound care. The client in the medical decision unit would be identified for dismissal if diagnostic testing reveals a noncardiac source of chest pain. The newly admitted client with pneumonia would not be a good choice because culture results are not yet available and antibiotics have not been administered long enough. The infant does not have a definitive diagnosis. The client who has recently traveled abroad may have either seasonal influenza or may have a novel or potential pandemic respiratory virus and should not be transferred to avoid spreading the illness.

The nurse is caring for four clients with chronic kidney disease (CKD). Which client would the nurse assess first upon initial rounding? a. Client with a blood pressure of 158/90 mm Hg b. Client with Kussmaul respirations c. Client with skin itching from head to toe d. Client with halitosis and stomatitis

b. Client with Kussmaul respirations

A nurse is caring for five clients. For which clients would the nurse assess a high risk for developing a pulmonary embolism (PE)? (Select all that apply.) a. Client who had a reaction to contrast dye yesterday b. Client with a new spinal cord injury on a rotating bed c. Middle-age client with an exacerbation of asthma d. Older client who is 1 day post-hip replacement surgery e. Young obese client with a fractured femur f. Middle-age adult with a history of deep vein thrombosis

b. Client with a new spinal cord injury on a rotating bed d. Older client who is 1 day post-hip replacement surgery e. Young obese client with a fractured femur

A nurse evaluates the following data in a client's chart: Admission Note : A 78-year-old male with a past medical history of atrial fibrillation is admitted with a chronic leg wound Prescriptions: Warfarin sodium (Coumadin) Sotalol (Betapace) Wound Care: Negative-pressure wound therapy (NPWT) to leg wound Based on this information, which action would the nurse take first? a. Assess the client's vital signs and initiate continuous telemetry monitoring. b. Contact the primary health care provider to discuss the treatment c. Consult the wound care nurse to apply the VAC device. d. Obtain a prescription for a low-fat, high-protein diet with vitamin supplements.

b. Contact the primary health care provider to discuss the treatment A client on anticoagulants is not a candidate for NPWT because of the incidence of bleeding complications. The health care primary health care provider needs this information quickly to plan other therapy for the client's wound. The nurse would contact the wound care nurse after alternative orders for wound care are prescribed. Vital signs and telemetry monitoring are appropriate for a client who has a history of atrial fibrillation and would be implemented as routine care for this client. A low-fat, high-protein diet with vitamin supplements will provide the client with necessary nutrients for wound healing but can be implemented after wound care, vital signs, and telemetry monitoring.

A nurse working in a medical home would do which of the following as part of the job? a. Advocate with insurance companies. b. Coordinate interprofessional care. c. Hold monthly team meetings. d. Provide out-of-network specialty referrals.

b. Coordinate interprofessional care. The medical home concept came into being to decrease the fragmentation of care. On a daily basis, this nurse would expect to coordinate with the interprofessional care team. Advocating with insurance companies would not be a daily function. Monthly team meetings may or may not be needed. Out of network referrals would not be needed as the interprofessional team strives to provide comprehensive care.

9. While assessing a client who is 12 hours postoperative after a thoracotomy for lung cancer, a nurse notices that the chest tube is dislodged. Which action by the nurse is best? a. Assess for drainage from the site. b. Cover the insertion site with sterile gauze. c. Contact the primary health care provider. d. Reinsert the tube using sterile technique.

b. Cover the insertion site with sterile gauze. Immediately covering the insertion site helps prevent air from entering the pleural space and causing a pneumothorax. The area will not reseal quickly enough to prevent air from entering the chest. The nurse would not leave the client to obtain a suture kit. An occlusive dressing may cause a tension pneumothorax. The nurse does not need to assess the site at this moment. The primary health care provider would be called to reinsert the chest tube or prescribe other treatment options.

An older adult is brought to the emergency department because of sudden onset of confusion. After the client is stabilized and comfortable, what assessment by the nurse is most important? a. Assess for orthostatic hypotension. b. Determine if there are new medications. c. Evaluate the client for gait abnormalities. d. Perform a delirium screening test.

b. Determine if there are new medications. Medication side effects and adverse effects are common in the older population. Something as simple as a new antibiotic can cause confusion and memory loss. The nurse would determine if the client is taking any new medications. Assessments for orthostatic hypotension, gait abnormalities, and delirium may be important once more is known about the client's condition.

A nurse caring for an older client in the hospital is concerned the client is not competent to give consent for upcoming surgery. What action by the nurse is best? a. Call Adult Protective Services. b. Discuss concerns with the health care team. c. Do not allow the client to sign the consent. d. Have the client's family sign the consent.

b. Discuss concerns with the health care team. In this situation, each facility will have a policy designed for assessing competence. The nurse would bring these concerns to an interprofessional care team meeting. There may be physiologic reasons for the client to be temporarily too confused or incompetent to give consent. If an acute condition is ruled out, the staff would follow the legal procedure and policies in their facility and state for determining competence. The key is to bring the concerns forward. Calling Adult Protective Services is not appropriate at this time. Signing the consent would wait until competence is determined unless it is an emergency, in which case the next of kin can sign if there are grave doubts as to the client's ability to provide consent. Simply not allowing the client to sign does not address the problem.

8. A trauma client with multiple open wounds is brought to the emergency department in cardiac arrest. Which action would the nurse take prior to providing advanced cardiac life support? a. Contact the on-call orthopedic surgeon. b. Don personal protective equipment. c. Notify the Rapid Response Team. d. Obtain a complete history from the paramedic.

b. Don personal protective equipment. Nurses must recognize and plan for a high risk of contamination with blood and body fluids when engaging in trauma resuscitation. Standard Precautions would be taken in all resuscitation situations and at other times when exposure to blood and body fluids is likely. Proper attire consists of an impervious cover gown, gloves, eye protection, a facemask, a surgical cap, and shoe covers. It is not known if this client has orthopedic injuries. The Rapid Response Team is not needed in the ED. A complete history is needed but the staff's protection comes first.

A nurse works at a community center for older adults. What self-management measure can the nurse teach the clients to prevent shock? a. Do not get dehydrated in warm weather. b. Drink fluids on a regular schedule. c. Seek attention for any lacerations. d. Take medications as prescribed.

b. Drink fluids on a regular schedule. Preventing dehydration in older adults is important because the age-related decrease in the thirst mechanism makes them prone to dehydration. Having older adults drink fluids on a regular schedule will help keep them hydrated without the influence of thirst (or lack of thirst). Telling clients not to get dehydrated is important, but not the best answer because it doesn't give them the tools to prevent it from occurring. Older adults should seek attention for lacerations, but this is not as important an issue as staying hydrated. Taking medications as prescribed may or may not be related to hydration.

A nurse assesses a client who has cirrhosis of the liver. Which laboratory findings would the nurse expect in clients with this disorder? (Select all that apply.) a. Elevated aspartate transaminase b. Elevated international normalized ratio (INR) c. Decreased serum globulin levels d. Decreased serum alkaline phosphatase e. Elevated serum ammonia f. Elevated prothrombin time (PT)

b. Elevated international normalized ratio (INR) e. Elevated serum ammonia f. Elevated prothrombin time (PT) Elevated INR and PT are indications of clotting disturbances and alert the nurse to the increased possibility of hemorrhage. Elevated ammonia levels increase the client's confusion. The other values are abnormal and associated with liver disease but do not necessarily place the client at increased risk for complications

A nurse is caring for a client with an electrical burn. The client has entrance wounds on the hands and exit wounds on the feet. What information is most important to include when planning care? a. The client may have memory and cognitive issues postburn. b. Everything between the entry and exit wounds can be damaged. c. The respiratory system requires close monitoring for signs of swelling. d. Electrical burns increase the risk of developing future cancers.

b. Everything between the entry and exit wounds can be damaged. As the electricity enters the body, travels through various tissues, and exits, it damages all the tissue it flows through. There may be severe internal injury that is not yet apparent. The client may have cognitive issues postburn but this is not as important as vigilant monitoring for complications. Respiratory system swelling is associated with thermal burns and smoke inhalation. Exposure to radiation increases cancer risk.

A nurse working in an Acute Care of the Elderly unit learns that frailty in the older population includes which components? (Select all that apply.) a. Dementia b. Exhaustion c. Slowed physical activity d. Weakness e. Weight gain f. Frequent illness

b. Exhaustion c. Slowed physical activity d. Weakness Frailty is a syndrome consisting of unintentional weight loss, slowed physical activity and exhaustion, and weakness. Weight gain and dementia are not part of this syndrome. Frequent illness could occur due to frailty, but is also not part of the syndrome.

A nurse is caring for a client on the medical stepdown unit. The following data are related to this client: Subjective Information Laboratory Analysis Physical Assessment Shortness of breath for 20 pH: 7.32 Pulse: 120 beats/min minutes PaCO2: 28 mm Hg Respiratory rate: 34 Reports feeling frightened PaO2: 78 mm Hg breaths/min "Can't catch my breath" SaO2: 88% Blood pressure 158/92 mm Hg Lungs have crackles What action by the nurse is most appropriate? a. Call respiratory therapy for a breathing treatment. b. Facilitate a STAT pulmonary angiography. c. Prepare for immediate endotracheal intubation. d. Prepare to administer intravenous anticoagulants.

b. Facilitate a STAT pulmonary angiography.

A nurse assesses a client who has a family history of polycystic kidney disease (PKD). Which assessment findings would the nurse expect? (Select all that apply.) a. Nocturia b. Flank pain c. Increased abdominal girth d. Dysuria e. Hematuria f. Diarrhea

b. Flank pain c. Increased abdominal girth e. Hematuria

A nurse assesses a client with pericarditis. Which assessment finding should the nurse expect to find? a. Heart rate that speeds up and slows down b. Friction rub at the left lower sternal border c. Presence of a regular gallop rhythm d. Coarse crackles in bilateral lung bases

b. Friction rub at the left lower sternal border

A client with ARDS is receiving minimal amounts of IV fluids. The new nurse notes the client is scheduled to receive a diuretic at this time. The nurse consults the Staff Development Nurse to determine the best course of action. What will the new nurse do? a. Contact the primary health care provider. b. Give the ordered diuretic as scheduled. c. Request an increase in the IV rate. d. Calculate the client's 24-hour fluid balance.

b. Give the ordered diuretic as scheduled.

A nurse reviews laboratory results for a client with diabetes mellitus who is prescribed an intensified insulin regimen: · Fasting blood glucose: 75 mg/dL (4.2 mmol/L) · Postprandial blood glucose: 200 mg/dL (11.1 mmol/L) · Hemoglobin A1C level: 5.5% How would the nurse interpret these laboratory findings? a. Increased risk for developing ketoacidosis b. Good control of blood glucose c. Increased risk for developing hyperglycemia d. Signs of insulin resistance

b. Good control of blood glucose

A nurse teaches a client who is considering being admitted to hospice. Which statement should the nurse include in this clients teaching? a. Hospice admission has specific criteria. You may not be a viable candidate, so we will look at alternative plans for your discharge. b. Hospice care focuses on a holistic approach to health care. It is designed not to hasten death, but rather to relieve symptoms. c. Hospice care will not help with your symptoms of depression. I will refer you to the facilitys counseling services instead. d. You seem to be experiencing some difficulty with this stage of the grieving process. Lets talk about your feelings.

b. Hospice care focuses on a holistic approach to health care. It is designed not to hasten death, but rather to relieve symptoms. As both a philosophy and a system of care, hospice care uses an interdisciplinary approach to assess and address the holistic needs of clients and families to facilitate quality of life and a peaceful death. This holistic approach neither hastens nor postpones death but provides relief of symptoms experienced by the dying client.

A hospital prepares for a mass casualty event. Which functions are correctly paired with the personnel role? (Select all that apply.) a. Paramedic—decides the number, acuity, and resource needs of clients b. Hospital incident commander—assumes overall leadership for implementing the emergency plan c. Public information officer—provides advanced life support during transportation to the hospital d. Triage officer—rapidly evaluates each client to determine priorities for treatment e. Medical command physician—serves as a liaison between the health care facility and the media

b. Hospital incident commander—assumes overall leadership for implementing the emergency plan d. Triage officer—rapidly evaluates each client to determine priorities for treatment The hospital incident commander assumes overall leadership for implementing the emergency plan. The triage officer rapidly evaluates each client to determine priorities for treatment. The paramedic provides advanced life support during transportation to the hospital. The public information officer serves as a liaison between the health care facility and the media. The medical command physician decides the number, acuity, and resource needs of clients.

An older client had hip replacement surgery and the surgeon prescribed morphine sulfate for pain. The client is allergic to morphine and reports pain and muscle spasms. When the nurse calls the surgeon, which medication would he or she suggest in place of the morphine? a. Cyclobenzaprine b. Hydromorphone hydrochloride c. Ketorolac d. Meperidine

b. Hydromorphone hydrochloride Cyclobenzaprine (used for muscle spasms), ketorolac, and meperidine (both used for pain) are all on the Beers list of potentially inappropriate medications for use in older adults and would not be suggested. The nurse would suggest hydromorphone hydrochloride.

The nurse is teaching a family member about various types of complementary therapies that might be effective for relieving the dying clients anxiety and restlessness. Which statement made by the family member indicates understanding of the nurses teaching? a. Maybe we should just hire an around-the-clock sitter to stay with Grandmother. b. I have some of her favorite hymns on a CD that I could bring for music therapy. c. I dont think that shell need pain medication along with her herbal treatments. d. I will burn therapeutic incense in the room so we can stop the anxiety pills.

b. I have some of her favorite hymns on a CD that I could bring for music therapy. Music therapy is a complementary therapy that may produce relaxation by quieting the mind and removing a clients inner restlessness. Hiring an around-the-clock sitter does not demonstrate that the clients family understands complementary therapies. Complementary therapies are used in conjunction with traditional therapy. Complementary therapy would not replace pain or anxiety medication but may help decrease the need for these medications.

9. After teaching a client who was malnourished and is being discharged, a nurse assesses the clients understanding. Which statement indicates the client correctly understood teaching to decrease risk for the development of metabolic acidosis? a. I will drink at least three glasses of milk each day. b. I will eat three well-balanced meals and a snack daily c. I will not take pain medication and antihistamines together. d. I will avoid salting my food when cooking or during meals.

b. I will eat three well-balanced meals and a snack daily Starvation or a diet with too few carbohydrates can lead to metabolic acidosis by forcing cells to switch to using fats for fuel and by creating ketoacids as a by-product of excessive fat metabolism. Eating sufficient calories from all food groups helps reduce this risk.

14. A client seen in the emergency department reports fever, fatigue, and dry cough but no other upper respiratory symptoms. A chest x-ray reveals mediastinal widening. What action by the nurse is best? a. Collect a sputum sample for culture by deep suctioning. b. Inform the client that oral antibiotics will be needed for 60 days. c. Place the client on Airborne Precautions immediately. d. Tell the client that directly observed therapy is needed.

b. Inform the client that oral antibiotics will be needed for 60 days This client has signs and symptoms of early inhalation anthrax. For treatment, after IV antibiotics are finished, oral antibiotics are continued for at least 60 days. Sputum cultures are not needed. Anthrax is not transmissible from person to person, so Standard Precautions are adequate. Directly observed therapy is often used for tuberculosis.

5. The charge nurse on a medical unit is preparing to admit several "clients" who have possible pandemic flu during a preparedness drill. What action by the nurse is best? a. Admit the "clients" on Contact Precautions. b. Inquire as to recent travel outside the United States. c. Do not allow pregnant caregivers to care for these "clients." d. Place the "clients" on enhanced Droplet Precautions.

b. Inquire as to recent travel outside the United States Preventing the spread of pandemic flu is equally important as caring for the clients who have it. Preventing the spread of disease is vital. The nurse would ask the "clients" about recent overseas travel to assess the risk of a pandemic flu. Clients with possible pandemic flu need to be in Contact and Airborne Precautions the infectious organism is identified and routes of transmission known. There is no specific danger to pregnant caregivers. Droplet Precautions are not appropriate.

A home health care nurse has conducted a home safety assessment for an older adult. There are five concrete steps leading out from the front door. Which intervention would be most helpful in keeping the older adult safe on the steps? a. Have the client use a walker or cane on the steps. b. Install contrasting color strips at the edge of each step. c. Instruct the client to use the garage door instead. d. Tell the client to use a two-footed gait on the steps.

b. Install contrasting color strips at the edge of each step. As a person ages, he or she may experience a decreased sense of touch. The older adult may not be aware of where his or her foot is on the step. Combined with diminished visual acuity, this can create a fall hazard. Holding the handrail would help keep the person safer. If the client does not need an assistive device, he or she would not use a cane or walker just on stairs. Using an alternative door may be necessary but does not address making the front steps safer. A two-footed gait may not help if the client is unaware of where the foot is on the step.

After administering newly prescribed captopril (Capoten) to a client with heart failure, the nurse implements interventions to decrease complications. Which priority intervention should the nurse implement for this client? a. Provide food to decrease nausea and aid in absorption. b. Instruct the client to ask for assistance when rising from bed. c. Collaborate with unlicensed assistive personnel to bathe the client. d. Monitor potassium levels and check for symptoms of hypokalemia.

b. Instruct the client to ask for assistance when rising from bed. (ACE) inhibitors is often associated with hypotension, usually termed first-dose effect.

A client in the emergency department is taking rifampin for tuberculosis. The client reports yellowing of the sclera and skin and bleeding after minor trauma. What laboratory results correlate to this condition? (Select all that apply.) a. Blood urea nitrogen (BUN): 19 mg/dL (6.7 mmol/L) b. International normalized ratio (INR): 6.3 c. Prothrombin time: 35 seconds d. Serum sodium: 130 mEq/L (130 mmol/L) e. White blood cell (WBC) count: 72,000/mm3 (72 109/L)

b. International normalized ratio (INR): 6.3 c. Prothrombin time: 35 seconds Rifampin can cause liver damage, evidenced by the client's high INR and prothrombin time. The BUN and WBC count are normal. The sodium level is low, but that is not related to this client's problem.

A nurse performs a skin screening for a client who has numerous skin lesions. Which lesion does the nurse evaluate first? a. Beige freckles on the backs of both hands. b. Irregular mole with multiple colors on the leg. c. Large cluster of pustules in the right axilla. d. Thick, reddened papules covered by white scales.

b. Irregular mole with multiple colors on the leg. This mole fits two of the criteria for being cancerous or precancerous: variation of color within one lesion, and an indistinct or irregular border. Freckles are a benign condition. Pustules could mean an infection, but it is more important to assess the potentially cancerous lesion first. Psoriasis vulgaris manifests as thick reddened papules covered by white scales. This is a chronic disorder and is not the priority.

5. A nurse is assessing a client who has acute pancreatitis and is at risk for an acid-base imbalance. For which manifestation of this acid-base imbalance should the nurse assess? a. Agitation b. Kussmaul respirations c. Seizures d. Positive Chvosteks sign

b. Kussmaul respirations The pancreas is a major site of bicarbonate production. Pancreatitis can cause a relative metabolic acidosis through underproduction of bicarbonate ions. Manifestations of acidosis include lethargy and Kussmaul respirations. Agitation, seizures, and a positive Chvosteks sign are manifestations of the electrolyte imbalances that accompany alkalosis.

A nurse is caring for several clients at risk for shock. Which laboratory value requires the nurse to communicate with the primary health care provider? a. Creatinine: 0.9 mg/dL (68.6 mcmol/L) b. Lactate: 5.4 mg/dL (6 mmol/L) c. Sodium: 150 mEq/L (150 mmol/L) d. White blood cell count: 11,000/mm3 (11 × 109 /L)

b. Lactate: 5.4 mg/dL (6 mmol/L) A lactate level of 5.4 mg/dL (6 mmol/L) is high and is indicative of possible shock. A creatinine level of 0.9 mg/dL (68.6 mcmol/L) is normal. A sodium level of 150 mEq/L (150 mmol/L) is slightly high but does not need to be communicated. A white blood cell count of 11,000/mm3 (11 × 109 /L) is slightly high but is not as critical as the lactate level.

6. A nurse is evaluating levels and functions of trauma centers. Which function is appropriately paired with the level of the trauma center? a. Level I—located within remote areas and provides advanced life support within resource capabilities b. Level II—located within community hospitals and provides care to most injured clients c. Level III—located in rural communities and provides only basic care to clients d. Level IV—located in large teaching hospitals and provides a full continuum of trauma care for all clients

b. Level II—located within community hospitals and provides care to most injured clients Level I trauma centers are usually located in large teaching hospital systems and provide a full continuum of trauma care for all clients. Both Level II and Level III facilities are usually located in community hospitals. These trauma centers provide care for most clients and transport to Level I centers when client needs exceed resource capabilities. Level IV trauma centers are usually located in rural and remote areas. These centers provide basic care, stabilization, and advanced life support while transfer arrangements to higher level trauma centers are made.

A nurse wishes to participate in an activity that will influence health outcomes. What action by the nurse best meets this objective? a. Creating a transportation system for health care appointments b. Lobbying with a national organization for health care policy c. Organizing a food pantry in an impoverished community d. Running for election to the county public health board

b. Lobbying with a national organization for health care policy All options are good choices for an altruistic nurse wishing to influence health outcomes; however, being involved in policy creation and health care reform is an activity specifically recognized to improve health outcomes. This action will also affect a wider population than the more local options.

A nurse is caring for a client who suffered massive blood loss after trauma. How does the nurse correlate the blood loss with the client's mean arterial pressure (MAP)? a. It causes vasoconstriction and increased MAP. b. Lower blood volume lowers MAP. c. There is no direct correlation to MAP. d. it raises cardiac output and MAP

b. Lower blood volume lowers MAP. Lower blood volume will decrease MAP. The other answers are not accurate.

A nurse is caring for a client after surgery who is restless and apprehensive. The assistive personnel (AP) reports the vital signs and the nurse sees that they are only slightly different from previous readings. What action does the nurse delegate next to the AP? a. Assess the client for pain or discomfort. b. Measure urine output from the catheter. c. Reposition the client to the side. d. Stay with the client and reassure him or her.

b. Measure urine output from the catheter. Urine output changes are a sensitive early indicator of shock. The nurse would delegate emptying the urinary catheter and measuring output to the AP as a baseline for hourly urine output measurements. The AP cannot assess for pain. Repositioning may or may not be effective for decreasing restlessness, but does not take priority over physical assessments. Reassurance is a therapeutic nursing action, but the nurse needs to do more in this situation.

3. A nurse is assessing clients who are at risk for acid-base imbalance. Which clients are correctly paired with the acid-base imbalance? (Select all that apply.) a. Metabolic alkalosis Young adult who is prescribed intravenous morphine sulfate for pain b. Metabolic acidosis Older adult who is following a carbohydrate-free diet c. Respiratory alkalosis Client on mechanical ventilation at a rate of 28 breaths/min d. Respiratory acidosis Postoperative client who received 6 units of packed red blood cells e. Metabolic alkalosis Older client prescribed antacids for gastroesophageal reflux disease

b. Metabolic acidosis Older adult who is following a carbohydrate-free diet c. Respiratory alkalosis Client on mechanical ventilation at a rate of 28 breaths/min e. Metabolic alkalosis Older client prescribed antacids for gastroesophageal reflux disease Respiratory acidosis often occurs as the result of underventilation. The client who is taking opioids, especially IV opioids, is at risk for respiratory depression and respiratory acidosis. One cause of metabolic acidosis is a strict low-calorie diet or one that is low in carbohydrate content. Such a diet increases the rate of fat catabolism and results in the formation of excessive ketoacids. A ventilator set at a high respiratory rate or tidal volume will cause the client to lose too much carbon dioxide, leading to an acid deficit and respiratory alkalosis. Citrate is a substance used as a preservative in blood products. It is not only a base, it is also a precursor for bicarbonate. Multiple units of packed red blood cells could cause metabolic alkalosis. Sodium bicarbonate antacids may increase the risk of metabolic alkalosis.

3. An emergency department nurse is caring for a trauma patient. Which interventions does the nurse perform during the primary survey? (Select all that apply.) a. Foley catheterization b. Needle decompression c. Initiating IV fluids d. Splinting open fractures e. Endotracheal intubation f. Removing wet clothing g. Laceration repair

b. Needle decompression c. Initiating IV fluids e. Endotracheal intubation f. Removing wet clothing The primary survey for a trauma client organizes the approach to the client so that life-threatening injuries are rapidly identified and managed. The primary survey is based on the standard mnemonic ABC, with an added D and E: Airway and cervical spinal motion restriction; Breathing; Circulation; Disability; and Exposure. After the completion of primary diagnostic and laboratory studies, and the insertion of gastric and urinary tubes, the secondary survey (a complete head-to-toe assessment) can be carried out.

The nurse is caring for a client who has diabetes mellitus type 1 and is experiencing hypoglycemia. Which assessment findings will the nurse expect? (Select all that apply.) a. Warm, dry skin b. Nervousness c. Rapid deep respirations d. Dehydration e. Ketoacidosis f. Blurred vision

b. Nervousness f. Blurred vision

A nurse assesses a client who is recovering from a thoracentesis. Which assessment findings would alert the nurse to a potential pneumothorax? (Select all that apply.) a. Bradycardia b. New-onset cough c. Purulent sputum d. Tachypnea e. Pain with respirations f. Rapid, shallow respirations

b. New-onset cough d. Tachypnea e. Pain with respirations

A nurse assesses a client who is diagnosed with infective endocarditis. Which assessment findings should the nurse expect? (Select all that apply.) a. Weight gain b. Night sweats c. Cardiac murmur d. Abdominal bloating e. Osler's nodes

b. Night sweats c. Cardiac murmur e. Osler's nodes

A nurse is caring for a client who received benzocaine spray prior to a recent bronchoscopy. The client presents with continuous cyanosis even with oxygen therapy. What action would the nurse take next? a. Administer an albuterol treatment. b. Notify the Rapid Response Team. c. Assess the client's peripheral pulses. d. Obtain blood and sputum cultures.

b. Notify the Rapid Response Team.

A nurse reviews the laboratory findings of a client with a urinary tract infection (bacterial cystitis). The laboratory report notes a "shift to the left" in the client's white blood cell count. What action would the nurse take? a. Request that the laboratory perform a differential analysis on the white blood cells. b. Notify the primary health care provider and start an intravenous line for parenteral antibiotics. c. Ask assistive personnel (AP) to strain the client's urine for renal calculi. d. Assess the client for a potential allergic reaction and anaphylactic shock

b. Notify the primary health care provider and start an intravenous line for parenteral antibiotics.

3. The nurse utilizing evidence-based practice (EBP) considers factors when planning care? (Select all that apply.) a. Cost-saving measures b. Nurse's expertise c. Client preferences d. Research findings e. Values of the client

b. Nurse's expertise c. Client preferences d. Research findings e. Values of the client

A client is having a peritoneal dialysis treatment. The nurse notes an opaque color to the effluent. What is the priority action by the nurse? a. Warm the dialysate solution in a microwave before instillation. b. Obtain a sample of the effluent and send to the laboratory. c. Flush the tubing with normal saline to maintain patency of the catheter. d. Check the peritoneal catheter for kinking and curling.

b. Obtain a sample of the effluent and send to the laboratory.

A nurse is assessing a client who is recovering from a lung biopsy. The client's breath sounds are absent. While another nurse calls the Rapid Response Team, what action by the nurse takes is most important? a. Take a full set of vital signs. b. Obtain pulse oximetry reading. c. Ask the patient about hemoptysis. d. Inspect the biopsy site.

b. Obtain pulse oximetry reading.

2. The nurse is teaching a client with obstructive sleep apnea (OSA) about the prescribed CPAP. What information does the nurse include? (Select all that apply.) a. Insurance will cover the cost if you wear it at least 4 hours a day. b. Once the delivery mask is adjusted, do not loosen the straps. c. The CPAP provides pressure that holds your upper airways open. d. You need to clean the mask at least once a week to prevent infection. e. The humidification increases the risk of fungal infections. f. Be patient when first using the system, it can be frustrating at first.

b. Once the delivery mask is adjusted, do not loosen the straps. c. The CPAP provides pressure that holds your upper airways open. e. The humidification increases the risk of fungal infections. f. Be patient when first using the system, it can be frustrating at first. A CPAP for OSA provides pressure that keeps the upper airway open. A properly fitting mask or nasal pillows is necessary to provide the pressure. Humidification in the system leads to an increased risk for fungal infections. Patients may have anxiety about using the equipment and worry about it being disruptive; most clients have a period of adjustment when first starting to use a CPAP. Medicare will usually cover the cost if the client wears the CPAP at least 6 hours a day. The mask or pillows should be cleaned daily.

A nurse cares for a dying client. Which manifestation of dying should the nurse treat first? a. Anorexia b. Pain c. Nausea d. Hair loss

b. Pain Only symptoms that cause distress for a dying client should be treated. Such symptoms include pain, nausea and vomiting, dyspnea, and agitation. These problems interfere with the clients comfort. Even when symptoms, such as anorexia or hair loss, disturb the family, they should be treated only if the client is distressed by their presence. The nurse should treat the clients pain first.

A nurse assesses a client with polycystic kidney disease (PKD). Which assessment finding would alert the nurse to immediately contact the primary health care provider? a. Flank pain b. Periorbital edema c. Bloody and cloudy urine d. Enlarged abdomen

b. Periorbital edema

11. The nurse is preparing to teach a community group about warning signs of lung cancer. What information does the nurse include? (Select all that apply.) a. Over 10-pack-year history of smoking b. Persistent coughing c. Rusty or blood-tinged sputum d. Dyspnea e. Hoarseness f. Fatigue

b. Persistent coughing c. Rusty or blood-tinged sputum d. Dyspnea e. Hoarseness Some common signs of lung cancer include persistent cough, rusty or blood-tinged sputum, dyspnea, and hoarseness. Fatigue is common to many conditions. Smoking history is a risk factor for lung cancer.

The nurse is caring for a client who is recovering from an open traditional Whipple surgical procedure. What action would the nurse take? a. Clamp the nasogastric tube. b. Place the patient in semi-Fowler position. c. Assess vital signs once every shift. d. Provide oral rehydration

b. Place the patient in semi-Fowler position. Postoperative care for a patient recovering from an open Whipple procedure would include placing the client in a semi-Fowler position to reduce tension on the suture line and anastomosis sites and promote breathing, setting the nasogastric tube to low continuous suction to remove free air buildup and pressure, assessing vital signs frequently to assess fluid and electrolyte complications, and providing intravenous fluids

The nurse is assessing a client who has hepatitis C. What extrahepatic complications would the nurse anticipate? (Select all that apply.) a. Pancreatitis b. Polyarthritis c. Heart disease d. Myalgia e. Peptic ulcer disease f. Ulcerative colitis

b. Polyarthritis c. Heart disease d. Myalgia

The nurse is caring for a client who has cirrhosis of the liver. What nursing action is appropriate to help control ascites? a. Monitor intake and output. b. Provide a low-sodium diet. c. Increase oral fluid intake. d. Weigh the patient daily.

b. Provide a low-sodium diet. A low-sodium diet is one means of controlling abdominal fluid collection. Monitoring intake and output does not control fluid accumulation, nor does weighing the client. These interventions merely assess or monitor the situation. Increasing fluid intake would not be helpful.

A nurse is caring for a client whose Braden Scale score is 9. What intervention demonstrates a lack of evidence-based knowledge? a. Requests a referral to a registered dietitian nutritionist. b. Raises the head of the bed no more than 45 degrees. c. Performs perineal cleansing every 2 hours. d. Assesses the client's entire skin surface daily.

b. Raises the head of the bed no more than 45 degrees. A client with a Braden Scale score of 9 is at high risk for skin breakdown and requires moderate to maximum assistance to prevent further breakdown. The nurse needs to keep the head of the bed elevated to no more than 30 degrees to prevent shearing. An RDN consultation, frequent perineal cleaning, and assessing the client's entire skin surface are all appropriate actions.

A nurse evaluates the following data in a client's chart: Admission Note: Laboratory Results: Wound Care Note: A 66-year-old male with a health history of a cerebral vascular accident and left-side paralysis White blood cell count: 8000/mm3 (8 109/L) Prealbumin: 15.2 mg/dL (152 mg/L) Albumin: 4.2 mg/dL (42 mg/L) Lymphocyte count: 2000/mm3 (2 109/L) Sacral ulcer: 4 2 1.5 cm Admission Note: • A 66-year-old male with a health history of a cerebral vascular accident and left-side paralysis Laboratory Results: • White blood cell count: 8000/mm3 • Prealbumin: 15.2 mg/dL • Albumin: 4.2 mg/dL • Lymphocyte count: 2000/mm3 Wound Care Note: • Sacral ulcer: 1.5 cm Based on this information, which action would the nurse take? a. Perform a neuromuscular assessment. b. Request a dietary consult. c. Initiate Contact Precautions. d. Assess the client's vital signs.

b. Request a dietary consult. The white blood cell count is not directly related to nutritional status. Albumin, prealbumin, and lymphocyte counts all give information related to nutritional status. The prealbumin count is a more specific indicator of nutritional status than is the albumin count. The albumin and lymphocyte counts given are normal, but the prealbumin count is low. This puts the client at risk for inadequate wound healing, so the nurse would request a dietary consult. The other interventions do not address the information provided.

A nurse assesses a client who has psoriasis. Which action would the nurse take first? a. Don gloves and an isolation gown. b. Shake the client's hand and introduce self. c. Assess for signs and symptoms of infections. d. Ask the client if she might be pregnant.

b. Shake the client's hand and introduce self. Clients with psoriatic lesions are often self-conscious of their skin. The nurse would first provide direct contact and touch without gloves to establish a good report with the client. Psoriasis is not an infectious disease, nor is it contagious. The nurse would not need to wear gloves or an isolation gown. Obtaining a health history and assessing for an infection and pregnancy would be completed after establishing a report with the client.

12. A nurse is caring for a client who is experiencing excessive diarrhea. The clients arterial blood gas values are pH 7.28, PaO2 98 mm Hg, PaCO2 45 mm Hg, and HCO3 16 mEq/L. Which provider order should the nurse expect to receive? a. Furosemide (Lasix) 40 mg intravenous push b. Sodium bicarbonate 100 mEq diluted in 1 L of D5W c. Mechanical ventilation d. Indwelling urinary catheter

b. Sodium bicarbonate 100 mEq diluted in 1 L of D5W This clients arterial blood gas values represent metabolic acidosis related to a loss of bicarbonate ions from diarrhea. The bicarbonate should be replaced to help restore this clients acid-base balance. Furosemide would cause an increase in acid fluid and acid elimination via the urinary tract; although this may improve the clients pH, the client has excessive diarrhea and cannot afford to lose more fluid. Mechanical ventilation is used to treat respiratory acidosis for clients who cannot keep their oxygen saturation at 90%, or who have respirator muscle fatigue. Mechanical ventilation and an indwelling urinary catheter would not be prescribed for this client.

On a hot humid day, an emergency department nurse is caring for a client who is confused and has these vital signs: temperature 104.1° F (40.1° C), pulse 132 beats/min, respirations 26 breaths/min, and blood pressure 106/66 mm Hg. What action would the nurse take? a. Encourage the client to drink cool water or sports drinks. b. Start an intravenous line and infuse 0.9% saline solution. c. Administer acetaminophen (Tylenol) 650 mg orally. d. Encourage rest and reassess in 15 minutes.

b. Start an intravenous line and infuse 0.9% saline solution. The client demonstrates signs of heat stroke. This is a medical emergency and priority care includes oxygen therapy, IV infusion with 0.9% saline solution, insertion of a urinary catheter, and aggressive interventions to cool the patient, including external cooling and internal cooling methods. Oral hydration would not be appropriate for a client who has symptoms of heat stroke because oral fluids would not provide necessary rapid rehydration, and the confused client would be at risk for aspiration. Acetaminophen would not decrease this patient's temperature or improve the patient's symptoms. The client needs immediate medical treatment; therefore, rest and reassessing in 15 minutes are inappropriate.

The nurse is teaching a client who had a descending colostomy 2 days ago about the ostomy stoma. Which changes in the stoma would the nurse teach the client to report to the primary health care provider? (Select all that apply.) a. Stool consistency is similar to paste. b. Stoma becomes dark and dull. c. Skin around the stoma becomes excoriated. d. Skin around stoma becomes protruded. e. Stoma becomes retracted into the abdomen.

b. Stoma becomes dark and dull. c. Skin around the stoma becomes excoriated. d. Skin around stoma becomes protruded. e. Stoma becomes retracted into the abdomen.

4. After teaching a client how to perform diaphragmatic breathing, the nurse assesses the client's understanding. Which action demonstrates that the client correctly understands the teaching? a. The client lies on his or her side with knees bent. b. The client places his or her hands on the abdomen. c. The client lies in a prone position with straight. d. The client places his or her hands above the head.

b. The client places his or her hands on the abdomen. To perform diaphragmatic breathing correctly, the client would place his or her hands on the abdomen to create resistance. This type of breathing cannot be performed effectively while lying on the side or with hands over the head. This type of breathing would not be as effective lying prone.

2. A nurse assesses a client who has a mediastinal chest tube. Which symptoms require the nurse's immediate intervention? (Select all that apply.) a. Production of pink sputum b. Tracheal deviation c. Pain at insertion site d. Sudden onset of shortness of breath e. Drainage greater than 70 mL/hr f. Disconnection at Y site

b. Tracheal deviation d. Sudden onset of shortness of breath e. Drainage greater than 70 mL/hr f. Disconnection at Y site Immediate intervention is warranted if the client has tracheal deviation because this could indicate a tension pneumothorax. Sudden shortness of breath could indicate dislodgment of the tube, occlusion of the tube, or pneumothorax. Drainage greater than 70 mL/hr could indicate hemorrhage. Disconnection at the Y site could result in air entering the tubing. Production of pink sputum and pain at the insertion site are not signs/symptoms that would require immediate intervention.

A nurse cares for a patient who has a chronic inflammatory bowel disease. Which actions would the nurse take to prevent skin excoriation? (Select all that apply.) a. Cleanse the perineum with an antibacterial soap. b. Use medicated wipes instead of toilet paper. c. Identify foods that decrease constipation. d. Apply a thin coat of aloe cream to the perineum. e. Gently pat the perineum dry after cleansing.

b. Use medicated wipes instead of toilet paper. d. Apply a thin coat of aloe cream to the perineum. e. Gently pat the perineum dry after cleansing. To prevent skin excoriation from frequent bowel movements associated with inflammatory bowel disease, the nurse would encourage good skin care with a mild soap and water and gently patting the area dry after each bowel movement. Using medicated wipes instead of toilet paper and applying a thin coat of aloe cream are appropriate. The client should identify and avoid foods that increase diarrhea. Antibacterial soaps are harsh and should not be used

1. A nurse is caring for clients in a busy emergency department. What actions would the nurse take to ensure client and staff safety? (Select all that apply.) a. Leave the stretcher in the lowest position with rails down so that the client can access the bathroom. b. Use two identifiers before each intervention and before mediation administration. c. Attempt de-escalation strategies for clients who demonstrate aggressive behaviors. d. Search the belongings of clients with altered mental status to gain essential medical information. e. Use facility policy identification procedures for "Jane/John Doe" clients. f. Check clients for a medical alert bracelets or necklaces. g. Avoid using Security personnel to prevent escalation of client behaviors.

b. Use two identifiers before each intervention and before mediation administration. c. Attempt de-escalation strategies for clients who demonstrate aggressive behaviors. d. Search the belongings of clients with altered mental status to gain essential medical information. e. Use facility policy identification procedures for "Jane/John Doe" clients. f. Check clients for a medical alert bracelets or necklaces. Best practices for client and staff safety in the emergency department include leaving beds in the lowest position with side rails up, using two unique identifiers for medications and procedures, using de-escalation strategies for clients or visitors showing hostile or aggressive behaviors, searching the belongings of confused clients for medical information, using facility identification systems for Jane/John Doe clients, observing for medical alert jewelry, and using security staff as needed.

A nurse prepares a client who is scheduled for a bronchoscopy with transbronchial biopsy procedure at 9:00 AM (0900). What actions would the nurse take? (Select all that apply.) a. Provide a clear liquid breakfast. b. Verify that the informed consent was obtained. c. Document the client's allergies. d. Review laboratory results. e. Hold the client's bronchodilator. f. Monitor the client for at least 24 hours afterwards.

b. Verify that the informed consent was obtained. c. Document the client's allergies. d. Review laboratory results. f. Monitor the client for at least 24 hours afterwards

14. A nurse is planning care for a client who is hyperventilating. The clients arterial blood gas values are pH 7.52 PaO2 94 mm Hg, PaCO2 31 mm Hg, and HCO3 26 mEq/L. Which question should the nurse ask when developing this clients plan of care? a. Do you take any over-the-counter medications? b. You appear anxious. What is causing your distress? c. Do you have a history of anxiety attacks? d. You are breathing fast. Is this causing you to feel light-headed?

b. You appear anxious. What is causing your distress? The nurse should assist the client who is experiencing anxiety-induced respiratory alkalosis to identify causes of the anxiety. The other questions will not identify the cause of the acid-base imbalance.

A diabetic client becomes septic after a bowel resection and is having problems with respiratory distress. The nurse reviews the labs and finds the following ABG results: pH 7.50, PaCO2 30, HCO3 : 24, and PaO2 68. What does the nurse recognize as the primary factor causing this the acid-base imbalance? a. Atelectasis due to respiratory muscle fatigue b. Hyperventilation due to poor oxygenation c. Hypoventilation due to morphine PCA d. Kussmaul respirations due to glucose of 102 mg/dL (5.7 mmol/L

b. hyperventilation due to poor oxygenation The ABG results indicate respiratory alkalosis. The client has low oxygenation as indicated by low partial pressure of arterial oxygen causing a compensatory mechanism of increased respirations and hyperventilation. Respiratory muscle fatigue and hypoventilation would cause respiratory acidosis with a low pH and high PaCO2. Kussmaul respirations are characterized by deep labored breathing and are a compensatory mechanism to metabolic acidosis, not hypoxemia or alkalosis.

A nurse answers a call light and finds a client anxious, short of breath, reporting chest pain, and having a blood pressure of 88/52 mm Hg on the cardiac monitor. What action by the nurse takes priority? a. Assess the client's lung sounds. b. Notify the Rapid Response Team. c. Provide reassurance to the client. d. Take a full set of vital signs.

b. notify rapid response team

A nurse assesses a client who has diabetes mellitus. Which arterial blood gas values would the nurse identify as potential ketoacidosis in this client? a. pH 7.38, HCO3 22 mEq/L (22 mmol/L), PCO2 38 mm Hg, PO2 98 mm Hg b. pH 7.28, HCO3 18 mEq/L (18 mmol/L), PCO2 28 mm Hg, PO2 98 mm Hg c. pH 7.48, HCO3 28 mEq/L (28 mmol/L), PCO2 38 mm Hg, PO2 98 mm Hg d. pH 7.32, HCO3 22 mEq/L (22 mmol/L), PCO2 58 mm Hg, PO2 88 mm Hg

b. pH 7.28, HCO3 18 mEq/L (18 mmol/L), PCO2 28 mm Hg, PO2 98 mm Hg

After teaching a client with bacterial cystitis who is prescribed phenazopyridine, the nurse assesses the client's understanding. Which statement made by the client indicates a correct understanding of the teaching? a. "I will not take this drug with food or milk." b. "I will have my partners tested for STIs." c. "An orange color in my urine should not alarm me." d. "I will drink two glasses of cranberry juice daily."

c. "An orange color in my urine should not alarm me."

The nurse is examining a woman's breast and notes multiple small mobile lumps. Which question would be most appropriate for the nurse to ask? a. "When was your last mammogram at the clinic?" b. "How many cans of caffeinated soda do you drink in a day?" c. "Do the small lumps seem to change with your menstrual period?" d. "Do you have a first-degree relative who has breast cancer?"

c. "Do the small lumps seem to change with your menstrual period?"

15. A nurse cares for a client with chronic obstructive pulmonary disease (COPD) who appears thin and disheveled. Which question would the nurse ask first? a. "Do you have a strong support system?" b. "What do you understand about your disease?" c. "Do you experience shortness of breath with basic activities?" d. "What medications are you prescribed to take each day?"

c. "Do you experience shortness of breath with basic activities?" Clients with severe COPD may not be able to perform daily activities, including bathing and eating, because of excessive shortness of breath. The nurse would ask the client if shortness of breath is interfering with basic activities. Although the nurse would need to know about the client's support systems, current knowledge, and medications, these questions do not address the client's appearance.

A nurse cares for a middle-age female client with diabetes mellitus who is being treated for the third episode of acute pyelonephritis in the past year. The client asks, "What can I do to help prevent these infections?" How would the nurse respond? a. "Test your urine daily for the presence of ketone bodies and proteins." b. "Use tampons rather than sanitary napkins during your menstrual period." c. "Drink more water and empty your bladder more frequently during the day." d. "Keep your hemoglobin A1C under 9% by keeping your blood sugar controlled."

c. "Drink more water and empty your bladder more frequently during the day."

8. A nurse is teaching a client who has cystic fibrosis (CF). Which statement would the nurse include in this client's teaching? a. "Take an antibiotic each day." b. "You should get genetic screening." c. "Eat a well-balanced, nutritious diet." d. "Plan to exercise for 30 minutes every day."

c. "Eat a well-balanced, nutritious diet." Clients with CF often are malnourished due to vitamin deficiency and pancreatic malfunction. Maintaining nutrition is essential. Daily antibiotics and daily exercise are not essential actions. Genetic screening might be an option; however, the nurse would not just tell the client to do something like that.

The nurse is planning to teach for a client who is starting exenatide extended-release (ER) for diabetes mellitus type 2. Which statement will the nurse include in the teaching? a. "Be sure to take the drug once a day before breakfast." b. "Take the drug every evening before bedtime." c. "Give your drug injection the same day every week." d. "Take the drug with dinner at the same time each day."

c. "Give your drug injection the same day every week."

A nurse is working with an older client admitted with mild dehydration. What teaching does the nurse provide to best address this issue? a. "Cut some sodium out of your diet." b. "Dehydration can cause incontinence." c. "Have something to drink every 1 to 2 hours." d. "Take your diuretic in the morning."

c. "Have something to drink every 1 to 2 hours." Older adults often lose their sense of thirst. Plus older adults have less body water than younger people. Since they should drink 1 to 2 L of water a day, the best remedy is to have the older adult drink something each hour or two, whether or not he or she is thirsty. Cutting "some" sodium from the diet will not address this issue and is vague. Although dehydration can cause incontinence from the irritation of concentrated urine, this information will not help prevent the problem of dehydration. Instructing the client to take a diuretic in the morning rather than in the evening also will not directly address this issue.

After teaching a client who is recovering from a colon resection to treat early-stage colorectal cancer (CRC), the nurse assesses the client's understanding. Which statements by the client indicate understanding of the teaching? (Select all that apply.) a. "I must change the ostomy appliance daily and as needed." b. "I will use warm water and a soft washcloth to clean around the stoma." c. "I might start bicycling and swimming again once my incision has healed." d. "I will make sure that I make lifestyle changes to prevent constipation." e. "I will be sure to have the recommended colonoscopies."

c. "I might start bicycling and swimming again once my incision has healed." d. "I will make sure that I make lifestyle changes to prevent constipation." e. "I will be sure to have the recommended colonoscopies."

A nurse assesses a client in an outpatient clinic. Which statement alerts the nurse to the possibility of left-sided heart failure? a. "I have been drinking more water than usual." b. "I am awakened by the need to urinate at night." c. "I must stop halfway up the stairs to catch my breath." d. "I have experienced blurred vision on several occasions."

c. "I must stop halfway up the stairs to catch my breath."

After teaching a client who has alcohol-induced cirrhosis, a nurse assesses the client's understanding. Which statement made by the client indicates a need for further teaching? a. "I cannot drink any alcohol at all anymore." b. "I should not take over-the-counter medications." c. "I need to avoid protein in my diet." d. "I should eat small, frequent, balanced meals."

c. "I need to avoid protein in my diet." Based on the degree of liver involvement and decreased function, protein intake may have to be decreased. However, some protein is necessary for the synthesis of albumin and normal healing. The other statements indicate accurate understanding of self-care measures for this client

After teaching a client who is newly diagnosed with type 2 diabetes mellitus, the nurse assesses the client's understanding. Which statement made by the client indicates a need for additional teaching? a. "I should increase my intake of vegetables with higher amounts of dietary fiber." b. "My intake of saturated fats should be no more than 10% of my total calorie intake." c. "I should decrease my intake of protein and eliminate carbohydrates from my diet." d. "My intake of water is not restricted by my treatment plan or medication regimen.

c. "I should decrease my intake of protein and eliminate carbohydrates from my diet."

The nurse is caring for a client who has cirrhosis from substance abuse. The client states, "All of my family hates me." How would the nurse respond? a. "You should make peace with your family." b. "This is not unusual. My family hates me too." c. "I will help you identify a support system." d. "You must attend Alcoholics Anonymous."

c. "I will help you identify a support system." Clients who have cirrhosis due to addiction may have alienated relatives over the years because of substance abuse. The nurse would assist the client to identify a friend, neighbor, clergy/spiritual leader, or group for support. The nurse would not minimize the patient's concerns. Attending AA may be appropriate, but this response doesn't address the client's concern. "Making peace" with the client's family may not be possible. This statement is not client-centered.

A nurse cares for a client with colorectal cancer who has a new colostomy. The client states, "I think it would be helpful to talk with someone who has had a similar experience." How would the nurse respond? a. "I have a good friend with a colostomy who would be willing to talk with you." b. "The ostomy nurse will be able to answer all of your questions." c. "I will make a referral to the United Ostomy Associations of America." d. "You'll find that most people with colostomies don't want to talk about them."

c. "I will make a referral to the United Ostomy Associations of America."

3. After teaching a client who is prescribed a long-acting beta2 agonist medication, a nurse assesses the client's understanding. Which statement indicates that the client comprehends the teaching? a. "I will carry this medication with me at all times in case I need it." b. "I will take this medication when I start to experience an asthma attack." c. "I will take this medication every morning to help prevent an acute attack." d. "I will be weaned off this medication when I no longer need it."

c. "I will take this medication every morning to help prevent an acute attack." Long-acting beta2 agonist medications will help prevent an acute asthma attack because they are long acting. The client will take this medication every day for best effect. The client does not have to always keep this medication with him or her because it is not used as a rescue medication. This is not the medication the client will use during an acute asthma attack because it does not have an immediate onset of action. The client will not be weaned off this medication because this is likely to be one of his or her daily medications.

6. After teaching a client who is prescribed salmeterol, the nurse assesses the client's understanding. Which statement by the client indicates a need for additional teaching? a. "I will be certain to shake the inhaler well before I use it." b. "It may take a while before I notice a change in my asthma." c. "I will use the drug when I have an asthma attack." d. "I will be careful not to let the drug escape out of my nose and mouth."

c. "I will use the drug when I have an asthma attack." Salmeterol is a long-acting beta2 agonist designed to prevent an asthma attack; it does not relieve or reverse symptoms. Salmeterol has a slow onset of action; therefore, it would not be used as a rescue drug. The drug must be shaken well because it has a tendency to separate easily. Poor technique on the client's part allows the drug to escape through the nose and mouth.

7. A nurse cares for a client with chronic obstructive pulmonary disease (COPD). The client states that going out with friends is no longer enjoyable. How would the nurse respond? a. "There are a variety of support groups for people who have COPD." b. "I will ask your primary health care provider to prescribe an antianxiety agent." c. "I'd like to hear about thoughts and feelings causing you to limit social activities." d. "Friends can be a good support system for clients with chronic disorders."

c. "I'd like to hear about thoughts and feelings causing you to limit social activities." Many clients with moderate to severe COPD become socially isolated because they are embarrassed by frequent coughing and mucus production. They also can experience fatigue, which limits their activities. The nurse needs to encourage the client to verbalize thoughts and feelings so that appropriate inteGrvReAnDtioEnSs LcaAnBb.e CseOleMcted. Joining a support group would not decrease feelings of social isolation if the client does not verbalize feelings. Antianxiety agents will not help the client with social isolation. While friends can be good sources of support, the client specifically is discussing going out of the home.

A nurse cares for victims during a community-wide disaster drill. One of the victims asks, "Why are the individuals with black tags not receiving any care?" How does the nurse respond? a. "To do the greatest good for the greatest number of people, it is necessary to sacrifice some." b. "Not everyone will survive a disaster, so it is best to identify those people early and move on." c. "In a disaster, extensive resources are not used for one person at the expense of many others." d. "With black tags, volunteers can identify those who are dying and can give them comfort care."

c. "In a disaster, extensive resources are not used for one person at the expense of many others." In a disaster, military-style triage is used; this approach identifies the dead or expectant dead with black tags. This practice helps to maintain the goal of triage, which is doing the most good for the most people. Precious resources are not used for those with overwhelming critical injury or illness, so that they can be allocated to others who have a reasonable expectation of survival. Victims are not "sacrificed." Telling victims that is important to move on after identifying the expectant dead does not provide an adequate explanation and is callous. Victims are not black-tagged to allow volunteers to give comfort care.

A nurse cares for a young client with a new ileostomy. The client states, "I cannot go to prom with an ostomy." How would the nurse respond? a. "Sure you can. Purchase a prom dress one size larger to hide the ostomy appliance." b. "The pouch won't be as noticeable if you avoid broccoli and carbonated drinks prior to the prom." c. "Let's talk to the ostomy nurse about options for ostomy supplies and dress styles." d. "You can remove the pouch from your ostomy appliance when you are at the prom so that it is less noticeable."

c. "Let's talk to the ostomy nurse about options for ostomy supplies and dress styles." The ostomy nurse is a valuable resource for patients, providing suggestions for supplies and methods to manage the ostomy. A larger dress size will not necessarily help hide the ostomy appliance. Avoiding broccoli and carbonated drinks does not offer reassurance for the client. Ileostomies have an almost constant liquid effluent, so pouch removal during the prom is not feasible.

A client with pneumonia and dementia is admitted with an indwelling urinary catheter in place. During interprofessional rounds the following day, which question would the nurse ask the primary health care provider? a. "Do you want daily weights on this client?" b. "Will the client be able to return home?" c. "May we discontinue the indwelling catheter?" d. "Should we get another chest x-ray today?"

c. "May we discontinue the indwelling catheter?"

After teaching a client how to prevent altitude-related illnesses, a nurse assesses the client's understanding. Which statement indicates that the client needs additional teaching? a. "If my climbing partner can't think straight, we should descend to a lower altitude." b. "I will ask my primary health care provider about medications to help prevent acute mountain sickness." c. "My partner and I will plan to sleep at a higher elevation to acclimate more quickly." d. "I will drink plenty of fluids to stay hydrated while on the mountain."

c. "My partner and I will plan to sleep at a higher elevation to acclimate more quickly." Teaching to prevent altitude-related illness would include descending when symptoms start, staying hydrated, and taking acetazolamide, which is commonly used to prevent and treat acute mountain sickness. The nurse would teach the client to sleep at a lower elevation.

The new nurse asks the preceptor how context affects clinical judgment. What response by the preceptor is best? a. "Context considers the whole of the patient's story and circumstances." b. "It shouldn't, only nursing knowledge would affect clinical judgment." c. "Outside influences such as environment in which you provide care, influence your decisions." d. "The context of the situation provides an extra layer of complexity to consider."

c. "Outside influences such as environment in which you provide care, influence your decisions." The context of a situation considers and supports clinical judgment. The factors within this layer—such as environment, time pressure, availability or content of electronic health records, resources, and individual nursing knowledge—have a direct impact on clinical judgment. The other two options are too vague to provide appropriate information.

20. A nurse cares for a client who has a family history of cystic fibrosis. The client asks, "Will my children have cystic fibrosis?" How would the nurse respond? a. "Since many of your family members are carriers, your children will also be carriers of the gene." b. "Cystic fibrosis is an autosomal recessive disorder. If you are a carrier, your children will have the disorder." c. "Since you have a family history of cystic fibrosis, I would encourage you and your partner to be tested." d. "Cystic fibrosis is caused by a protein that controls the movement of chloride. Adjusting your diet will decrease the spread of this disorder."

c. "Since you have a family history of cystic fibrosis, I would encourage you and your partner to be tested." Cystic fibrosis is an autosomal recessive disorder in which both gene alleles must be mutated for the disorder to be expressed. The nurse would encourage both the client and partner to be tested for the abnormal gene. The other statements are not true.

A nurse teaches a client who is prescribed nicotine replacement therapy. Which statement would the nurse include in this client's teaching? a. "Make a list of reasons why smoking is a bad habit." b. "Rise slowly when getting out of bed in the morning." c. "Smoking while taking this medication will increase your risk of a stroke." d. "Stopping this medication suddenly increases your risk for a heart attack."

c. "Smoking while taking this medication will increase your risk of a stroke."

After teaching a client who is prescribed pancreatic enzyme replacement therapy, the nurse assesses the client's understanding. Which statement by the client indicates a need for further teaching? a. "The capsules can be opened and the powder sprinkled on applesauce if needed." b. "I will wipe my lips carefully after I drink the enzyme preparation." c. "The best time to take the enzymes is immediately after I have a meal or a snack." d. "I will not mix the enzyme powder with food or liquids that contain protein."

c. "The best time to take the enzymes is immediately after I have a meal or a snack." The enzymes must be taken immediately before eating meals or snacks. If the client cannot swallow the capsules whole, they can be opened up and the powder sprinkled on applesauce, mashed fruit, or rice cereal. The client should wipe his or her lips carefully after drinking the enzyme preparation because the liquid could damage the skin. Protein items will be dissolved by the enzymes if they are mixed together

13. The nurse is caring for a client who has cystic fibrosis (CF). The client asks for information about gene therapy. What response by the nurse is best? a. "Unfortunately, gene therapy is only provided to children upon diagnosis." b. "Do you know that you will have to have genetic testing?" c. "There is a good treatment for the most common genetic defect in CF." d. "Gene therapy will only help improve your pulmonary symptoms."

c. "There is a good treatment for the most common genetic defect in CF." The drug ivacaftor/lumacaftor is effective as therapy for patients whose CF is caused by the F508del (also known as the Phe508del) mutation, the most common mutation involved in CF, even in patients who are homozygous for the mutation with both alleles being affected. The nurse would provide that information as the best response. Asking if the client understands he or she will have to undergo genetic testing is a correct statement, but is a yes/no question which is not therapeutic and might sound paternalistic. It also does not provide any information on the therapy itself. The drug is not limited to children and helps move chloride closer to the membrane surfaces so it would have an effect on any organ compromised by CF.

2. A nurse in a family practice clinic is preparing discharge instructions for a client reporting facial pain that is worse when bending over, tenderness across the cheeks, and postnasal discharge. What instruction will be most helpful? a. "Ice packs may help with the facial pain." b. "Limit fluids to dry out your sinuses." c. "Try warm, moist heat packs on your face." d. "We will schedule a computed tomography scan this week."

c. "Try warm, moist heat packs on your face." This client has rhinosinusitis. Comfort measures for this condition include humidification, hot packs, nasal saline irrigations, sleeping with the head elevated, increased fluids, and avoiding cigarette smoke. The client does not need a CT scan.

A nurse assesses a young female client who is prescribed tazarotene. Which question should the nurse ask prior to starting this therapy? a. "Do you spend a great deal of time in the sun?" b. "Have you or any family members ever had skin cancer?" c. "Which method of contraception are you using?" d. "Do you drink alcoholic beverages?"

c. "Which method of contraception are you using?" Tazarotene has many side effects. It is a known teratogen and can cause severe birth defects. Strict birth control measures must be used during therapy. The other questions are not directly related to this medication.

After teaching a client who is recovering from a heart transplant to change positions slowly, the client asks, "Why is this important?" How should the nurse respond? a. "Rapid position changes can create shear and friction forces, which can tear out your internal vascular sutures." b. "Your new vascular connections are more sensitive to position changes, leading to increased intravascular pressure and dizziness." c. "Your new heart is not connected to the nervous system and is unable to respond to decreases in blood pressure caused by position changes." d. "While your heart is recovering, blood flow is diverted away from the brain, increasing the risk for stroke when you stand up."

c. "Your new heart is not connected to the nervous system and is unable to respond to decreases in blood pressure caused by position changes." DISCONNECTED HEART </3

19. A nurse cares for a client who tests positive for alpha1-antitrypsin (AAT) deficiency. The client asks, "What does this mean?" How would the nurse respond? a. "Your children will be at high risk for chronic obstructive pulmonary disease." b. "I will contact a genetic counselor to discuss your condition." c. "Your risk for chronic obstructive pulmonary disease is higher, especially if you smoke." d. "This is a recessive gene and would have no impact on your health."

c. "Your risk for chronic obstructive pulmonary disease is higher, especially if you smoke." Alpha1-antitrypsin deficiency is an important risk factor for COPD. The gene for AAT is a recessive gene. Clients with only one allele produce enough AAT to prevent COPD unless the client smokes or there is sufficient exposure to other inhalants. A client with two alleles is at high risk for COPD even if not exposed to smoke or other irritants. The client is a carrier, and children may or may not be at high risk depending on the partner's AAT levels. Contacting a genetic counselor may be helpful but does not address the client's current question.

3. An emergency department nurse is triaging victims of a multi-casualty event. Which client would receive care first? a. A 30-year-old distraught mother holding her crying child b. A 65-year-old conscious male with a head laceration c. A 26-year-old male who has pale, cool, clammy skin d. A 48 year old with a simple fracture of the lower leg

c. A 26-year-old male who has pale, cool, clammy skin The client with pale, cool, clammy skin may be in shock and needs immediate medical attention. The mother does not have injuries and so would be the lowest priority. The other two people need medical attention soon, but not at the expense of a person in shock.

13. A nurse is triaging clients in the emergency department. Which client would the nurse classify as "nonurgent?" a. A 44 year old with chest pain and diaphoresis b. A 50 year old with chest trauma and absent breath sounds c. A 62 year old with a simple fracture of the left arm d. A 79 year old with a temperature of 104°F.(40.0° C)

c. A 62 year old with a simple fracture of the left arm A client in a nonurgent category can tolerate waiting several hours for health care services without a significant risk of clinical deterioration. The client with a simple arm fracture and palpable radial pulses is currently stable, is not at significant risk of clinical deterioration, and would be considered nonurgent. The client with chest pain and diaphoresis and the client with chest trauma are emergent owing to the potential for clinical deterioration and would be seen immediately. The client with a high fever may be stable now but also has a risk of deterioration.

A nurse assesses clients on a medical-surgical unit. Which client is at greatest risk for pressure injury development? a. A 44 year old prescribed IV antibiotics for pneumonia b. A 26 year old who is bedridden with a fractured leg c. A 65 year old with hemiparesis and incontinence d. A 78 year old requiring assistance to ambulate with a walker

c. A 65 year old with hemiparesis and incontinence Risk factors for development of a pressure injury include lack of mobility, exposure of skin to excessive moisture (e.g., urinary or fecal incontinence), malnourishment, and aging skin. The client with hemiparesis and incontinence has two risk factors. The client with pneumonia has no identified risk factors. The other two are at lower risk if they are not very mobile, but having two risk factors is a higher risk.

1. A nurse assesses a client with asthma and notes bilateral wheezing, decreased pulse oxygen saturation, and suprasternal retraction on inhalation. What actions by the nurse are best? (Select all that apply.) a. Administer prescribed salmeterol inhaler. b. Assess the client for a tracheal deviation. c. Administer oxygen and place client on an oximeter. d. Perform peak expiratory flow readings. e. Administer prescribed albuterol inhaler. f. Assess the client's lung sounds after administering the inhaler.

c. Administer oxygen and place client on an oximeter. e. Administer prescribed albuterol inhaler. f. Assess the client's lung sounds after administering the inhaler. Suprasternal retraction caused by inhalation usually indicates that the client is using accessory muscles and is having difficulty moving air into the respiratory passages because of airway narrowing. Wheezing indicates a narrowed airway; a decreased pulse oxygen saturation also supports this finding. The asthma is becoming unstable, and intervention is needed. Administration of a rescue inhaler is indicated, probably along with administration of oxygen. The nurse would reassess the lung sounds after the rescue inhaler. The nurse would not do a peak flow reading at this time, nor would a code be called. The nurse could assess for tracheal deviation after administering oxygen and albuterol.

After teaching a client about advance directives, a nurse assesses the clients understanding. Which statement indicates the client correctly understands the teaching? a. An advance directive will keep my children from selling my home when Im old. b. An advance directive will be completed as soon as Im incapacitated and cant think for myself. c. An advance directive will specify what I want done when I can no longer make decisions about health care. d. An advance directive will allow me to keep my money out of the reach of my family.

c. An advance directive will specify what I want done when I can no longer make decisions about health care. An advance directive is a written document prepared by a competent individual that specifies what, if any, extraordinary actions a person would want taken when he or she can no longer make decisions about personal health care. It does not address issues such as the clients residence or financial matters.

An emergency department charge nurse notes an increase in sick calls and bickering among the staff after a week with multiple trauma incidents. What action would the nurse take? a. Organize a pizza party for each shift. b. Remind the staff of the facility's sick-leave policy. c. Arrange for postincident crisis support. d. Talk individually with staff members.

c. Arrange for postincident crisis support. The staff may be suffering from stress related to the multiple traumas and needs to have crisis support. A crisis support team can assist the staff with developing appropriate coping methods. Speaking with staff members individually does not provide the same level of support as trained health care providers who can offer emotional first aid. Organizing a party and revisiting the sick-leave policy may be helpful, but are not as important and beneficial as formalized crisis support.

A nurse cares for a client who had a bronchoscopy 2 hours ago. The client asks for a drink of water. What action would the nurse take next? a. Call the primary health care provider and request food and water for the client. b. Provide the client with ice chips instead of a drink of water. c. Assess the client's gag reflex before giving any food or water. d. Let the client have a small sip to see whether he or she can swallow.

c. Assess the client's gag reflex before giving any food or water.

A nurse assesses a client recently bitten by a coral snake. Which assessment would the nurse complete first? a. Unilateral peripheral swelling b. Clotting times c. Cardiopulmonary status d. Electrocardiogram rhythm

c. Cardiopulmonary status Signs and symptoms of coral snake envenomation are the result of its neurotoxic properties. The physiologic effect is to block neurotransmission, which produces ascending paralysis, reduced perception of pain, and, ultimately, respiratory paralysis. The nurse would monitor for respiratory rate and depth. Severe swelling and clotting problems do not occur with coral snakes but do occur with pit viper snakes. Electrocardiogram rhythm is not affected by neurotoxins.

A nurse assesses a client who has cholecystitis. Which sign or symptom indicates that this condition is chronic rather than acute? a. Temperature of 100.1° F (37.8° C) b. Positive Murphy sign c. Clay-colored stools d. Upper abdominal pain after eating

c. Clay-colored stools Jaundice, clay-colored stools, and dark urine are more commonly seen with chronic cholecystitis. The other symptoms are seen in clients with either chronic or acute cholecystitis

A nurse assesses a client after an open lung biopsy. Which assessment finding is matched with the correct intervention? a. Client reports being dizzy—nurse calls the Rapid Response Team. b. Client's heart rate is 55 beats/min—nurse withholds pain medication. c. Client has reduced breath sounds—nurse calls primary health care provider immediately. d. Client's respiratory rate is 18 breaths/min—nurse decreases oxygen flow rate.

c. Client has reduced breath sounds—nurse calls primary health care provider immediately.

A nurse is volunteering in a temporary shelter for survivors of a hurricane. Which client does the nurse anticipate has the highest need for further assessment and referral? a. Client who is still trying to locate relatives who are missing b. Family awaiting the ability to travel out of state for temporary housing c. Client with a score of 48 on the Impact of Event Scale-Revised (IES-R) d. Client who has trouble sleeping and who startles easily

c. Client with a score of 48 on the Impact of Event Scale-Revised (IES-R) The IES-R is an assessment tool is a 22-item self-administered questionnaire that scores individuals on signs and symptoms of acute stress disorder or posttraumatic stress disorder. A score of 33 or higher out of 88 is a positive finding and this client would be referred a psychiatrist or other licensed mental health care provider. The nurse would administer the assessment to the client with difficulty sleeping after ensuring he or she can read at the 10th grade level, which is the reading level of the tool. The other two clients do not show evidence of particular needs for referral beyond what is usually provided in a natural disaster.

1. A nurse working in a geriatric clinic sees clients with "cold" symptoms and rhinitis. The primary health care provider (PHCP) often leaves a prescription for diphenhydramine. What action by the nurse is best? a. Teach the client about possible drowsiness. b. Instruct the client to drink plenty of water. c. Consult with the PHCP about the medication. d. Encourage the client to take the medication with food.

c. Consult with the PHCP about the medication. First-generation antihistamines are not appropriate for use in the older population. These drugs include chlorpheniramine, diphenhydramine, and hydroxyzine. The nurse would consult with the PHCP and request a different medication. Diphenhydramine does cause drowsiness, but the nurse would request a different medication. Drinking plenty of fluids is appropriate for the condition and is not related to the medication. Antihistamines can be taken without regard to food.

An emergency department nurse is caring for a client who had been hiking in the mountains for the past 2 days. What are the most important indicators that a client is experiencing high-altitude pulmonary edema (HAPE)? (Select all that apply.) a. Ataxia b. Confusion c. Crackles in both lung fields d. Decreased level of consciousness e. Persistent dry cough f. Reports "feeling hung over"

c. Crackles in both lung fields e. Persistent dry cough Signs and symptoms of high-altitude pulmonary edema (HAPE) include poor exercise tolerance, prolonged recovery time after exertion, fatigue, and weakness that progresses to a persistent dry cough and cyanosis of lips and nail beds. Crackles may be auscultated in one or both lung fields. A late sign of HAPE is pink, frothy sputum. Ataxia and confusion or decreased level of consciousness are seen in HACE—high-altitude cerebral edema. Acute mountain sickness produces a syndrome similar to an alcohol-induced hangover.

The nurse is teaching a client how to avoid the formation of hemorrhoids. What lifestyle change would the nurse include? a. Avoiding alcohol b. Quitting smoking c. Decreasing fluid intake d. Increasing dietary fiber

c. Decreasing fluid intake

The nurse is working with a male client who has gynecomastia. What action by the nurse is most appropriate? a. Teach the client to perform self-breast examination. b. Review the plan for chemotherapy after surgery. c. Educate him on the side effects of tamoxifen. d. Assess his usual daily alcohol intake.

c. Educate him on the side effects of tamoxifen.

A client is admitted with acute kidney injury (AKI) and a urine output of 2000 mL/day. What is the major concern of the nurse regarding this patient's care? a. Edema and pain b. Cardiac and respiratory status c. Electrolyte and fluid imbalance d. Mental health status

c. Electrolyte and fluid imbalance

The nurse is caring for a client who is diagnosed with a complete small bowel obstruction. For what priority problem is this client most likely at risk? a. Abdominal distention b. Nausea c. Electrolyte imbalance d. Obstipation

c. Electrolyte imbalance

The nurse reviews the laboratory results for a client who has possible appendicitis. Which laboratory test finding would the nurse expect? a. Decreased potassium level b. Increased sodium level c. Elevated leukocyte count d. Decreased thrombocyte count

c. Elevated leukocyte count Appendicitis is an acute inflammatory disorder that frequently results in elevation of leukocytes (white blood cells). Serum electrolytes are not affected because the client does not usually have diarrhea. Thrombocyte (platelet) count is unrelated to this GI disorder

1. An emergency department nurse assesses a client who has been raped. With which health care team member would the nurse collaborate when planning this client's care? a. Primary health care provider b. Case manager c. Forensic nurse examiner d. Psychiatric crisis nurse

c. Forensic nurse examiner All other members of the health care team listed may be used in the management of this client's care. However, the forensic nurse examiner is educated to obtain client histories and collect evidence dealing with the assault, and can offer the counseling and follow-up needed when dealing with the victim of an assault.

A nurse assesses a client who has ulcerative colitis and severe diarrhea. Which assessment would the nurse complete first? a. Inspection of oral mucosa b. Recent dietary intake c. Heart rate and rhythm d. Percussion of abdomen

c. Heart rate and rhythm Although the client with severe diarrhea may experience skin irritation and hypovolemia, the client is most at risk for cardiac dysrhythmias secondary to potassium and magnesium loss from severe diarrhea. The client would have her or his electrolyte levels monitored, and electrolyte replacement may be necessary. Oral mucosa inspection, recent dietary intake, and abdominal percussion are important parts of physical assessment but are lower priority for this patient than heart rate and rhythm.

The nurse assesses a client who is hospitalized with an exacerbation of Crohn disease. Which assessment finding would the nurse expect? a. Positive Murphy sign with rebound tenderness to palpitation b. Dull, hypoactive bowel sounds in the lower abdominal quadrants c. High-pitched, rushing bowel sounds in the right lower quadrant d. Reports of abdominal cramping that is worse at night

c. High-pitched, rushing bowel sounds in the right lower quadrant The nurse expects high-pitched, rushing bowel sounds due to narrowing of the bowel lumen in Crohn disease. A positive Murphy sign is indicative of gallbladder disease, and rebound tenderness often indicates peritonitis. Dullness in the lower abdominal quadrants and hypoactive bowel sounds is not commonly found with Crohn disease. Nightly worsening of abdominal cramping is not consistent with Crohn disease

11. After providing discharge teaching, a nurse assesses the clients understanding regarding increased risk for metabolic alkalosis. Which statement indicates the client needs additional teaching? a. I dont drink milk because it gives me gas and diarrhea. b. I have been taking digoxin every day for the last 15 years. c. I take sodium bicarbonate after every meal to prevent heartburn. d. In hot weather, I sweat so much that I drink six glasses of water each day.

c. I take sodium bicarbonate after every meal to prevent heartburn. Excessive oral ingestion of sodium bicarbonate and other bicarbonate-based antacids can cause metabolic alkalosis. Avoiding milk, taking digoxin, and sweating would not lead to increased risk of metabolic alkalosis.

17. A client is admitted with suspected pneumonia from the emergency department. The client went to the primary health care provider a "few days ago" and shows the nurse the results of what the client calls "an allergy test," as shown below: The reddened area is firm. What action by the nurse is best? a. Assess the client for possible items to which he or she is allergic. b. Call the primary health care provider's office to request records. c. Immediately place the client on Airborne Precautions. d. Prepare to begin administration of intravenous antibiotics.

c. Immediately place the client on Airborne Precautions. This "allergy test" is actually a positive tuberculosis test. The client would be placed on Airborne Precautions immediately. The other options do not take priority over preventing the spread of the disease.

A nurse reviews the laboratory values of a client who returned from kidney transplantation 12 hours ago: Sodium: 136 mEq/L (135 mmol/L) Potassium: 5 mEq/L (5 mmol/L) Blood urea nitrogen (BUN): 44 mg/dL (15.7 mmol/L) Serum creatinine: 2.5 mg/dL (221 mcmol/L) What initial intervention would the nurse anticipate? a. Start hemodialysis immediately. b. Discuss the need for peritoneal dialysis. c. Increase the dose of immunosuppression. d. Return the client to surgery for exploration.

c. Increase the dose of immunosuppression.

11. A nurse cares for a client who is infected with Burkholderia cepacia. What action would the nurse take first when admitting this client to a pulmonary care unit? a. Instruct the client to wash his or her hands after contact with other people. b. Implement Droplet Precautions and don a surgical mask. c. Keep the client separated from other clients with cystic fibrosis. d. Obtain blood, sputum, and urine culture specimens.

c. Keep the client separated from other clients with cystic fibrosis. B. cepacia infection is spread through casual contact between cystic fibrosis clients, thus the need for infected clients to be separated from noninfected clients. Strict isolation measures will not be necessary. Although the client would wash his or her hands frequently, the most important measure that can be implemented on the unit is isolation of the client from other clients with cystic fibrosis. There is no need to implement Droplet Precautions or don a surgical mask when caring for this client. Obtaining blood, sputum, and urine culture specimens will not provide information necessary to care for a client with B. cepacia infection.

12. An emergency department nurse is caring for a client who is homeless. Which action would the nurse take to gain the client's trust? a. Speak in a quiet and monotone voice. b. Avoid eye contact with the patient. c. Listen to the client's concerns and needs. d. Ask security to store the client's belongings.

c. Listen to the client's concerns and needs. To demonstrate behaviors that promote trust with homeless clients, the emergency department nurse makes eye contact (if culturally appropriate), speaks calmly, avoids any prejudicial or stereotypical remarks, shows genuine care and concern by listening, and follows through on promises. The nurse would also respect the client's belongings and personal space.

A nurse obtains the health history of a client who is recently diagnosed with lung cancer and identifies that the client has a 60-pack-year smoking history. Which action is most important for the nurse to take when interviewing this client? a. Tell the client that he or she needs to quit smoking to stop further cancer development. b. Encourage the client to be completely honest about both tobacco and marijuana use. c. Maintain a nonjudgmental attitude to avoid causing the client to feel guilty. d. Avoid giving the client false hope regarding cancer treatment and prognosis.

c. Maintain a nonjudgmental attitude to avoid causing the client to feel guilty

A nurse is caring for a client with elevated triiodothyronine and thyroxine, and normal thyroid-stimulating hormone levels. What actions does the nurse take? (Select all that apply.) a. Administer levothyroxine. b. Administer propranolol. c. Monitor the apical pulse. d. Assess for Trousseau sign. e. Initiate telemetry monitoring.

c. Monitor the apical pulse. e. Initiate telemetry monitoring.

15. An elderly client who has fallen from a roof is transported to the emergency department by ambulance. The client was unconscious at the scene but is conscious on arrival and is triaged as urgent. What is the priority assessment the nurse includes during the primary survey of the patient? a. A full set of vital signs b. Cardiac rhythm c. Neurologic status d. Client history

c. Neurologic status The primary survey for a trauma client organizes the approach to the client so that life-threatening injuries are rapidly identified and managed. Injuries from this type of fall have a high risk for cervical spine injuries. In addition, with the loss of consciousness at the scene the client would be at risk for head trauma. A full set of vital signs is obtained as part of the secondary survey. The cardiac rhythm is important but not specifically related to this client's presentation. Client history would be obtained as able.

A nurse caring for a client notes the following assessments: white blood cell count 3800/mm3 (3.8 × 109 /L), blood glucose level 198 mg/dL (11 mmol/L), and temperature 96.2° F (35.6° C). What action by the nurse takes priority? a. Document the findings in the client's chart. b. Give the client warmed blankets for comfort. c. Notify the primary health care provider immediately. d. Prepare to administer insulin per sliding scale.

c. Notify the primary health care provider immediately. This client has several indicators of sepsis with systemic inflammatory response. The nurse would notify the primary health care provider immediately. Documentation needs to be thorough but does not take priority. The client may appreciate warm blankets, but comfort measures do not take priority. The client may need insulin if blood glucose is being regulated tightly.

5. A nurse cares for a client who has developed esophagitis after undergoing radiation therapy for lung cancer. Which diet selection would the nurse provide for this client? a. Spaghetti with meat sauce, ice cream b. Chicken soup, grilled cheese sandwich c. Omelet, soft whole-wheat bread d. Pasta salad, custard, orange juice

c. Omelet, soft whole-wheat bread Side effects of radiation therapy may include inflammation of the esophagus. Clients would be taught that bland, soft, high-calorie foods are best, along with liquid nutritional supplements. Tomato sauce may prove too spicy for a client with esophagitis. A grilled cheese sandwich is too difficult to swallow with this condition, and orange juice and other foods with citric acid are too caustic.

The nurse assesses a client who has possible bladder cancer. What common assessment finding associated with this type of cancer would the nurse expect? a. Urinary retention b. Urinary incontinence c. Painless hematuria d. Difficulty urinating

c. Painless hematuria

10. A charge nurse is rounding on several older clients on ventilators in the Intensive Care Unit whom the nurse identifies as being at high risk for ventilator-associated pneumonia. To reduce this risk, what activity would the nurse delegate to the assistive personnel (AP)? a. Encourage between-meal snacks. b. Monitor temperature every 4 hours. c. Provide oral care every 4 hours. d. Report any new onset of cough.

c. Provide oral care every 4 hours. Oral colonization by gram-negative bacteria is a risk factor for health care-associated pneumonia. Good, frequent oral care can help prevent this from developing and is a task that can be delegated to the AP. Encouraging good nutrition is important, but this will not prevent pneumonia. Monitoring temperature and reporting new cough in clients are important to detect the onset of possible pneumonia but do not prevent it.

11. An emergency department (ED) case manager is consulted for a client who is homeless. Which intervention would the case manager provide? a. Communicate client needs and restrictions to support staff. b. Prescribe low-cost antibiotics to treat community-acquired infection. c. Provide referrals to subsidized community-based health clinics. d. Offer counseling for substance abuse and mental health disorders.

c. Provide referrals to subsidized community-based health clinics. Case management interventions include facilitating referrals to primary care providers who are accepting new clients or to subsidized community-based health clinics for clients or families in need of routine services. The ED nurse is accountable for communicating pertinent staff considerations, client needs, and restrictions to support staff (e.g., physical limitations, Isolation Precautions) to ensure that ongoing client and staff safety issues are addressed. The ED provider prescribes medications and treatments. The psychiatric nurse team evaluates clients with emotional behaviors or mental illness and facilitates the follow-up treatment plan, including possible admission to an appropriate psychiatric facility.

An older adult client takes medication three times a day and becomes confused about which medication should be taken at which time. The client refuses to use a pill sorter with slots for different times, saying "Those are for old people." What action by the nurse would be most helpful? a. Arrange medications by time in a drawer. b. Encourage the client to use easy-open tops. c. Put color-coded stickers on the bottle caps. d. Write a list of when to take each medication.

c. Put color-coded stickers on the bottle caps. Color-coded stickers are a fast, easy-to-remember system. One color is for morning meds, one for evening meds, and the third color is for nighttime meds. Arranging medications by time in a drawer might be helpful if the person doesn't accidentally put them back in the wrong spot. Easy-open tops are not related. Writing a list might be helpful, but not if it gets misplaced. With stickers on the medication bottles themselves, the reminder is always with the medication.

A nurse assessing a client with colorectal cancer auscultates high-pitched bowel sounds and notes the presence of visible peristaltic waves. Which action would the nurse take? a. Ask if the client is experiencing pain in the right shoulder. b. Perform a rectal examination and assess for polyps. c. Recommend that the client have computed tomography. d. Administer a laxative to increase bowel movement activity

c. Recommend that the client have computed tomography.

A client with MODS has been started on dobutamine. What assessment finding requires the nurse to communicate with the primary health care provider immediately? a. Blood pressure of 98/68 mm Hg b. Pedal pulses 1+/4+ bilaterally c. Report of chest heaviness d. Urine output of 32 mL/hr

c. Report of chest heaviness Chest heaviness or pain indicates myocardial ischemia, a possible adverse effect of dobutamine. While taking dobutamine, the oxygen requirements of the heart are increased due to increased myocardial workload, and may cause ischemia. Without knowing the client's previous blood pressure or pedal pulses, there is not enough information to determine if these are an improvement or not. A urine output of 32 mL/hr is acceptable.

An older adult recently retired and reports "being depressed and lonely." What information would the nurse assess as a priority? a. History of previous depression b. Previous stressful events c. Role of work in the adult's life d. Usual leisure time activities

c. Role of work in the adult's life Establishing and maintaining relationships with others throughout life are especially important to the older person's happiness. When people retire, they may lose much of their social network, leading them to feeling depressed and lonely. This loss from a sudden change in lifestyle can easily lead to depression. The nurse would first assess the role that work played in the client's life. The other factors can be assessed as well, but this circumstance is commonly seen in the older population.

5. The emergency department nurse is participating in a bioterrorism drill in which several "clients" are suspected to have inhalation anthrax. Which "clients" would the nurse see as the priorities? (Select all that apply.) a. Widened mediastinum on chest x-ray b. Dry cough c. Stridor d. Oxygen saturation of 91% e. Diaphoresis f. Oral temperature of 99.9° F (37.7° C)

c. Stridor d. Oxygen saturation of 91% e. Diaphoresis Clients with fulminant anthrax may exhibit stridor, hypoxia, and diaphoresis. Although an oxygen saturation of 91% is not critical, it is anormally low. These clients would be seen as the priority. A widened mediastinum and dry cough are usually seen in the prodromal phase when the temperature elevation is not as severe.

A client has just returned from a right radical mastectomy. Which action by the assistive personnel (AP) would require the nurse consider to intervene? a. Checking the amount of urine in the catheter collection bag b. Elevating the right arm on a pillow c. Taking the blood pressure on the right arm d. Encouraging the client to squeeze a rolled washcloth

c. Taking the blood pressure on the right arm

An emergency department manager wants to mitigate the possible acute and chronic stress after mass casualty events in the staff. What action would the manager take? a. Encourage all staff to join a Disaster Medical Assistance Team. b. Instruct all staff members to prepare go bags for all family members. c. Use available resources for broad education and training in disaster management. d. Provide incentives and bonuses for responding to mass casualty events.

c. Use available resources for broad education and training in disaster management. Research indicates that education and training in disaster management before an incident occurs is associated with improved confidence and better coping after the incident. Go bags are important to maintain for all family members but would not be effective in mitigating stress. A DMAT is a medical relief team made up of civilian medical, paraprofessional, and support personnel that is deployed to a disaster area with enough medical equipment and supplies to sustain operations for 72 hours. Incentives and bonuses will not help mitigate stress.

A nurse caring for an older adult has provided education on high-fiber foods. Which menu selection by the client demonstrates a need for further review? a. Barley soup b. Black beans c. White rice d. Whole-wheat bread

c. White rice Older adults need 35 to 50 g of fiber a day. White rice is low in fiber. Foods high in fiber include barley, beans, and whole-wheat products.

After teaching a young adult client who is newly diagnosed with type 1 diabetes mellitus, the nurse assesses the client's understanding. Which statement made by the client indicates a correct understanding of the need for eye examinations? a. "At my age, I should continue seeing the ophthalmologist as I usually do." b. "I will see the eye doctor when I have a vision problem and yearly after age 40." c. "My vision will change quickly. I should see the ophthalmologist twice a year." d. "Diabetes can cause blindness, so I should see the ophthalmologist yearly."

d. "Diabetes can cause blindness, so I should see the ophthalmologist yearly."

A nurse teaches a client who is prescribed digoxin (Lanoxin) therapy. Which statement should the nurse include in this client's teaching? a. "Avoid taking aspirin or aspirin-containing products." b. "Increase your intake of foods that are high in potassium." c. "Hold this medication if your pulse rate is below 80 beats/min." d. "Do not take this medication within 1 hour of taking an antacid."

d. "Do not take this medication within 1 hour of taking an antacid." Gastrointestinal absorption of digoxin is erratic. Many medications, especially antacids, interfere with its absorption.

The nurse is teaching a client with chronic kidney disease (CKD) about the sodium restriction needed in the diet to prevent edema and hypertension. Which statement by the client indicates that more teaching is needed? a. "I will probably lose weight by cutting out potato chips." b. "I will cut out bacon with my eggs every morning." c. "My cooking style will change by not adding salt." d. "I am thrilled that I can continue to eat fast food."

d. "I am thrilled that I can continue to eat fast food."

A nurse teaches a client who has pruritus. Which statement by the client shows a need to review the information? a. "I will shower daily using a super-fatted soap." b. "I can try taking a bath with colloidal oatmeal." c. "I will pat my skin dry instead of rubbing it with a towel." d. "I will be careful to keep my nails filed smoothly."

d. "I will be careful to keep my nails filed smoothly." The client with pruritus should shower only every other day, although super-fatted soap is an appropriate choice. Colloidal oatmeal baths are very soothing. Patting the skin dry avoids trauma and injury. Keeping nails filed smoothly also prevents injury.

A client contacts the clinic to report a life-long mole has developed a crust with occasional bleeding. What instruction by the nurse is most appropriate? a. "Take monthly photographs of it so you can document any changes." b. "Wash daily with warm water and gentle soap to prevent infection." c. "Keep the lesion covered with a bandage and triple antibiotic ointment." d. "Please make an appointment to be seen here as soon as possible."

d. "Please make an appointment to be seen here as soon as possible." A lesion demonstrating a change in characteristics, such as oozing, crusting, bleeding, or scaling, is suspicious for skin cancer. The nurse would instruct the client to come in for evaluation. Monthly photographs are a good way to document skin changes, but the client needs an assessment for skin cancer. The lesion can be washed and covered with a bandage and ointment, but again, the client needs an evaluation for skin cancer.

The nurse is preparing to teach a client with chronic hepatitis B about lamivudine therapy. What health teaching would the nurse include? a. "Follow up on all appointments to monitor your lab values." b. "Do not take amiodorone at any time while on this drug." c. "Monitor for jaundice, rash, and itchy skin while on this drug." d. "Report any changes in urinary elimination while on this drug."

d. "Report any changes in urinary elimination while on this drug." Lamivudine can cause renal impairment and the nurse would remind the client of changes that may indicate kidney damage.

When teaching a client recently diagnosed with type 1 diabetes mellitus, the client states, "I will never be able to stick myself with a needle." How would the nurse respond? a. "I can give your injections to you while you are here in the hospital." b. "Everyone gets used to giving themselves injections. It really does not hurt." c. "Your disease will not be managed properly if you refuse to administer the shots." d. "Tell me what it is about the injections that are concerning you."

d. "Tell me what it is about the injections that are concerning you."

A nurse teaches a client with type 1 diabetes mellitus. Which statement would the nurse include in this client's teaching to decrease the client's insulin needs? a. "Limit your fluid intake to 2 L a day." b. "Animal organ meat is high in insulin." c. "Limit your carbohydrate intake to 80 g a day." d. "Walk at a moderate pace for 1 mile daily."

d. "Walk at a moderate pace for 1 mile daily."

A nurse teaches a client who has a history of heart failure. Which statement should the nurse include in this client's discharge teaching? a. "Avoid drinking more than 3 quarts of liquids each day." b. "Eat six small meals daily instead of three larger meals." c. "When you feel short of breath, take an additional diuretic." d. "Weigh yourself daily while wearing the same amount of clothing."

d. "Weigh yourself daily while wearing the same amount of clothing."

9. A client has been admitted for suspected inhalation anthrax infection. What question by the nurse is most important? a. "Are any family members also ill?" b. "Have you traveled recently?" c. "How long have you been ill?" d. "What is your occupation?"

d. "What is your occupation?" Inhalation anthrax is rare and is an occupational hazard among people who work with animal wool, bone meal, hides, and skin, such as taxidermists and veterinarians. Inhalation anthrax seen in someone without an occupational risk is considered a bioterrorism event and must be reported to authorities immediately. The other questions are appropriate for anyone with an infection.

A nurse cares for a client with end-stage heart failure who is awaiting a transplant. The client appears depressed and states, "I know a transplant is my last chance, but I don't want to become a vegetable." How should the nurse respond? a. "Would you like to speak with a priest or chaplain?" b. "I will arrange for a psychiatrist to speak with you." c. "Do you want to come off the transplant list?" d. "Would you like information about advance directives?"

d. "Would you like information about advance directives?"

A client who is hospitalized with burns after losing the family home in a fire becomes angry and screams at a nurse when dinner is served late. How would the nurse respond? a. "Do you need something for pain right now?" b. "Please stop yelling. I brought dinner as soon as I could." c. "I suggest that you get control of yourself." d. "You seem upset. I have time to talk if you'd like."

d. "You seem upset. I have time to talk if you'd like." Clients would be allowed to ventilate their feelings of anger and despair after a catastrophic event. The nurse establishes rapport through active listening and honest communication and by recognizing cues that the client wishes to talk. Asking whether the client is in pain as the first response closes the door to open communication and limits the client's options. Simply telling the client to stop yelling and to gain control does nothing to promote therapeutic communication.

A nurse cares for a client who has a family history of colorectal cancer. The client states, "My father and my brother had colon cancer. What is the chance that I will get cancer?" How would the nurse respond? a. "If you eat a low-fat and low-fiber diet, your chances decrease significantly." b. "You are safe. This is an autosomal dominant disorder that skips generations." c. "Preemptive surgery and chemotherapy will remove cancer cells and prevent cancer." d. "You should have a colonoscopy more frequently to identify abnormal polyps early."

d. "You should have a colonoscopy more frequently to identify abnormal polyps early."

A client is scheduled for a hepatobiliary iminodiacetic acid (HIDA) scan. What would the nurse include in client teaching about this diagnostic test? a. "You'll have to drink a contrast medium right before the test." b. "You'll need to do a bowel prep the nursing before the test." c. "You'll be able to drink liquids up until the test begins." d. "You'll have a large camera close to you during the test."

d. "You'll have a large camera close to you during the test." Clients having a HIDA scan are NPO and receive an injectable nuclear medicine contrast. No bowel preparation is required. A large camera is close to the client for most of the test which can be a problem for clients who are claustrophobic.

A nurse reviews the laboratory test values for a client with a new diagnosis of diabetes mellitus type 2. Which A1C value would the nurse expect? a. 5.0% b. 5.7% c. 6.2% d. 7.4%

d. 7.4%

1. A nurse assesses several clients who have a history of respiratory disorders. Which client would the nurse assess first? a. A 66-year-old client with a barrel chest and clubbed fingernails b. A 48-year-old client with an oxygen saturation level of 92% at rest c. A 35-year-old client who reports orthopnea in bed d. A 27-year-old client with a heart rate of 120 beats/min

d. A 27-year-old client with a heart rate of 120 beats/min Tachycardia can indicate hypoxemia as the body tries to circulate the oxygen that is available. A barrel chest is not an emergency finding. Likewise, a pulse oximetry level of 92% is not considered an acute finding. Orthopnea at night in bed is breathlessness when lying down but is not an acute finding at this moment.

A nurse assesses clients who are at risk for diabetes mellitus. Which client is at greatest risk? a. A 19-year-old Caucasian b. A 22-year-old African American c. A 44-year-old Asian American d. A 58-year-old American Indian

d. A 58-year-old American Indian

A nurse assesses clients at a community health center. Which client is at highest risk for developing colorectal cancer? a. A 37-year-old who drinks eight cups of coffee daily. b. A 44-year-old with irritable bowel syndrome (IBS). c. A 60-year-old lawyer who works 65 hours per week. d. A 72-year-old who eats fast food frequently.

d. A 72-year-old who eats fast food frequently

While at a public park, a nurse encounters a person immediately after a bee sting. The person's lips are swollen, and wheezes are audible. What action would the nurse take first? a. Elevate the site and notify the person's next of kin. b. Remove the stinger with tweezers and encourage rest. c. Administer diphenhydramine and apply ice. d. Administer an epinephrine autoinjector and call 911.

d. Administer an epinephrine autoinjector and call 911.

While at a public park, a nurse encounters a person immediately after a bee sting. The person's lips are swollen, and wheezes are audible. What action would the nurse take first? a. Elevate the site and notify the person's next of kin. b. Remove the stinger with tweezers and encourage rest. c. Administer diphenhydramine and apply ice. d. Administer an epinephrine autoinjector and call 911.

d. Administer an epinephrine autoinjector and call 911. The client's swollen lips indicate that anaphylaxis may be developing, and this is a medical emergency. The nurse would call 911 would immediately, and the client transported to the emergency department as quickly as possible. If an EpiPen is available, it would be administered at the first sign of an anaphylactic reaction. The other answers do not provide adequate interventions to treat airway obstruction due to anaphylaxis, although the nurse would remove the stinger as soon as possible after administering the autoinjector.

12. A nurse cares for a client who had a chest tube placed 6 hours ago and refuses to take deep breaths because of the pain. What action would the nurse take? a. Ambulate the client in the hallway to promote deep breathing. b. Auscultate the client's anterior and posterior lung fields. c. Encourage the client to take shallow breaths to help with the pain. d. Administer pain medication and encourage the client to take deep breaths.

d. Administer pain medication and encourage the client to take deep breaths. A chest tube is placed in the pleural space and may be uncomfortable for a client. The nurse would provide pain medication to minimize discomfort and encourage the client to take deep breaths. The other responses do not address the client's discomfort and need to take deep breaths to prevent complications.

A nurse reviews the laboratory findings of a client with a urinary tract infection (bacterial cystitis). The laboratory report notes a "shift to the left" in the client's white blood cell count. What action would the nurse take? a. Request that the laboratory perform a differential analysis on the white blood cells. b. Notify the primary health care provider and start an intravenous line for parenteral antibiotics. c. Ask assistive personnel (AP) to strain the client's urine for renal calculi. d. Assess the client for a potential allergic reaction and anaphylactic shock.

d. Assess the client for a potential allergic reaction and anaphylactic shock

A nurse assesses a client who is recovering from a radical nephrectomy for renal cell carcinoma. The nurse notes that the client's blood pressure has decreased from 134/90 to 100/56 mm Hg and urine output is 20 mL for this past hour. What action would the nurse take? a. Position the client to lay on the surgical incision. b. Measure the specific gravity of the client's urine. c. Administer intravenous pain medications. d. Assess the rate and quality of the client's pulse.

d. Assess the rate and quality of the client's pulse.

A nurse on the general medical-surgical unit is caring for a client in shock and assesses the following: Respiratory rate: 10 breaths/min Pulse: 136 beats/min Blood pressure: 92/78 mm Hg Level of consciousness: responds to voice Temperature: 101.5° F (38.5° C) Urine output for the last 2 hours: 40 mL/hr. What action by the nurse is best? a. Transfer the client to the Intensive Care Unit. b. Continue monitoring every 30 minutes. c. Notify the unit charge nurse immediately. d. Call the Rapid Response Team.

d. Call the Rapid Response Team. This client has a MEWS score of 7 (RR: 0, P: 3, SBP: 1, LOC: 1, Temperature: 1, UO: 1). Scores above 5 are associated with a high risk of death and ICU admission. The most important action for the nurse is to notify the Rapid Response Team so that timely interventions can be initiated. The client most likely will be transferred to the ICU, but an order is required. Monitoring the client every 30 minutes is appropriate, but the nurse needs to obtain care for the client. The charge nurse is a valuable resource, but the best action is to notify the Rapid Response Team.

The nurse is caring for a client scheduled to have a transjugular intrahepatic portal-systemic shunt (TIPS) procedure. What client assessment would the nurse perform prior to this procedure? a. Musculoskeletal assessment b. Neurologic assessment c. Mental health assessment d. Cardiovascular assessment

d. Cardiovascular assessment A postprocedure complication of a TIPS procedure is right-sided heart failure. Therefore, the nurse would perform a cardiovascular assessment before the procedure to determine if the client has signs and symptoms of heart failure.

10. An emergency department nurse is caring for a client who has died from a suspected homicide. Which action does the nurse take? a. Remove all tubes and wires in preparation for the medical examiner. b. Limit the number of visitors to minimize the family's trauma. c. Consult the bereavement committee to follow up with the grieving family. d. Communicate the client's death to the family in a simple and concrete manner.

d. Communicate the client's death to the family in a simple and concrete manner. When dealing with clients and families in crisis, communicate in a simple and concrete manner to minimize confusion. Tubes must remain in place for the medical examiner. Family would be allowed to view the body. Offering to call for additional family support during the crisis is suggested. The bereavement committee would be consulted, but this is not the priority at this time.

A nurse assesses a client who is recovering from a subtotal thyroidectomy and observes the development of stridor. What is the priority action for the nurse to take? a. Apply oxygen via nasal cannula at 2 L/min. b. Document the finding and assess the client hourly. c. Place the client in high-Fowler position in the bed. d. Contact the Rapid Response Team and prepare for intubation.

d. Contact the Rapid Response Team and prepare for intubation.

An emergency department (ED) charge nurse prepares to receive clients from a mass casualty within the community. What is the role of this nurse during the event? a. Ask nursing staff to discharge clients from the medical-surgical units in order to make room for critically injured victims. b. Call additional medical-surgical and critical care nursing staff to come to the hospital to assist when victims are brought in. c. Inform the incident commander at the mass casualty scene about how many victims may be handled by the ED. d. Direct medical-surgical and critical care nurses to assist with clients currently in the ED while emergency staff prepare to receive the mass casualty victims.

d. Direct medical-surgical and critical care nurses to assist with clients currently in the ED while emergency staff prepare to receive the mass casualty victims. The ED charge nurse would direct additional nursing staff to help care for current ED clients while the ED staff prepares to receive mass casualty victims; however, they would not be assigned to the most critically ill or injured clients. The hospital incident commander's role is to take a global view of the entire situation and facilitate patient movement through the system, while bringing in personnel and supply resources to meet patient needs. The medical command physician would kept the incident commander informed about victims and capacity of the ED.

A client had an open traditional Whipple procedure this morning. For what priority complication would the nurse assess? a. Urinary tract infection b. Chronic kidney disease c. Heart failure d. Fluid and electrolyte imbalances

d. Fluid and electrolyte imbalances Due to the length and complexity of this type of surgery, the client is at risk for fluid and electrolyte imbalances. The nurse would assess for signs and symptoms of these imbalances so they can be managed early to prevent potentially life-threatening complications.

The nurse administers epoetin alfa to a client who has chronic kidney disease (CKD). Which laboratory test value would the nurse monitor to determine this drug's effectiveness? a. Potassium b. Sodium c. Renin d. Hemoglobin

d. Hemoglobin

The nurse is caring for a client who has cirrhosis of the liver. Which risk factor is the leading cause of cirrhosis? a. Metabolic syndrome b. Liver cancer c. Nonalcoholic fatty liver disease d. Hepatitis C

d. Hepatitis C Hepatitis C is the leading cause of cirrhosis and an also cause liver cancer. Clients with nonalcoholic fatty liver disease often have metabolic syndrome and can also develop cirrhosis

The nurse teaches a client who has stress incontinence methods to regain more urinary continence. Which health teaching is the most important for the nurse to include for this client? a. What type of incontinence pads to use? b. What types of liquids to drink and when? c. Need to perform intermittent catheterizations. d. How to do Kegel exercises to strengthen muscles?

d. How to do Kegel exercises to strengthen muscles?

The nurse is caring for a newly admitted older adult who has a blood glucose of 300 mg/dL (16.7 mmol/L), a urine output of 185 mL in the past 8 hours, and a blood urea nitrogen (BUN) of 44 mg/dL (15.7 mmol/L). What diabetic complication does the nurse suspect? a. Diabetic ketoacidosis (DKA) b. Severe hypoglycemia c. Chronic kidney disease (CKD) d. Hyperglycemic-hyperosmolar state (HHS)

d. Hyperglycemic-hyperosmolar state (HHS)

A nurse is caring for a dying client. The clients spouse states, I think he is choking to death. How should the nurse respond? a. Do not worry. The choking sound is normal during the dying process. b. I will administer more morphine to keep your husband comfortable. c. I can ask the respiratory therapist to suction secretions out through his nose. d. I will have another nurse assist me to turn your husband on his side.

d. I will have another nurse assist me to turn your husband on his side. The choking sound or death rattle is common in dying clients. The nurse should acknowledge the spouses concerns and provide interventions that will reduce the choking sounds. Repositioning the client onto one side with a towel under the mouth to collect secretions is the best intervention. The nurse should not minimize the spouses concerns. Morphine will assist with comfort but will not decrease the choking sounds. Nasotracheal suctioning is not appropriate in a dying client.

22. A nurse evaluates the following arterial blood gas and vital sign results for a client with chronic obstructive pulmonary disease (COPD): Arterial Blood Gas Results Vital Signs pH = 7.32 PaCO2 = 62 mm Hg PaO2 = 46 mm Hg HCO3 = 28 mEq/L (28 mmol/L) Heart rate = 110 beats/min Respiratory rate = 12 breaths/min Blood pressure = 145/65 mm Hg Oxygen saturation = 76%What action would the nurse take first? a. Administer a short-acting beta2 agonist inhaler. b. Document the findings as normal for a client with COPD. c. Teach the client diaphragmatic breathing techniques. d. Initiate oxygenation therapy to increase saturation to 88% to 92%.

d. Initiate oxygenation therapy to increase saturation to 88% to 92%. Oxygen would be administered to a client who is hypoxic even if the client has COPD and is a carbon dioxide retainer. The other interventions do not address the client's hypoxia, which is the major issue. There is no indication the client needs an inhaler. Diaphragmatic breathing techniques would not be taught to a client in distress. These findings are not normal for all clients with COPD.

A nurse assesses a client with diabetes mellitus who self-administers subcutaneous insulin. The nurse notes a spongy, swelling area at the site the client uses most frequently for insulin injection. What action would the nurse take? a. Apply ice to the site to reduce inflammation. b. Consult the provider for a new administration route. c. Assess the client for other signs of cellulitis. d. Instruct the client to rotate sites for insulin injection.

d. Instruct the client to rotate sites for insulin injection.

3. A nurse assesses a client who is prescribed furosemide (Lasix) for hypertension. For which acid-base imbalance should the nurse assess to prevent complications of this therapy? a. Respiratory acidosis b. Respiratory alkalosis c. Metabolic acidosis d. Metabolic alkalosis

d. Metabolic alkalosis Many diuretics, especially loop diuretics, increase the excretion of hydrogen ions, leading to excess acid loss through the renal system. This situation is an acid deficit of metabolic origin.

A nurse is caring for a client who has a nonhealing pressure injury on the right ankle. Which action would the nurse take first? a. Draw blood for albumin, prealbumin, and total protein. b. Prepare for and assist with obtaining a wound culture. c. Instruct the client to elevate the foot. d. Assess the right leg for pulses, skin color, and temperature.

d. Multi-fiber superabsorbent dressing A client with an ulcer on the foot would be assessed for interruption in arterial flow to the area. This begins with the assessment of pulses and color and temperature of the skin. The nurse can also assess for pulses noninvasively with a Doppler flowmeter if unable to palpate with his or her fingers. Tests to determine nutritional status and risk assessment would be completed after the initial assessment is done. Wound cultures are done after it has been determined that drainage, odor, and other risks for infection are present. Elevation of the foot would impair the ability of arterial blood to flow to the area.

A nurse cares for a client who has a stage 3 pressure injury with copious exudate. What type of dressing does the nurse use on this wound? a. Wet-to-damp saline moistened gauze b. None, the wound is left open to the air c. A transparent film d. Multi-fiber superabsorbent dressing

d. Multi-fiber superabsorbent dressing This pressure injury requires a superabsorbent dressing that will collect the exudate but not stick to the wound itself. A wet-to-damp gauze dressing provides mechanical removal of necrotic tissue. A draining wound would not be left open. A transparent film is a good choice for a noninfected stage 2 pressure injury.

A nurse is field-triaging clients after an industrial accident. Which client condition would the nurse triage with a red tag? a. Dislocated right hip and an open fracture of the right lower leg b. Large contusion to the forehead and a bloody nose c. Closed fracture of the right clavicle and arm numbness d. Multiple fractured ribs and shortness of breath

d. Multiple fractured ribs and shortness of breath Clients who have an immediate threat to life are given the highest priority, are placed in the emergent or class I category, and are given a red triage tag. The client with multiple rib fractures and shortness of breath has a threat to oxygenation and is the most critical. The client with the hip and leg problem and the client with the clavicle fracture would be classified as class II (urgent, yellow tag); these major but stable injuries can wait for 30 minutes to 2 hours for definitive care. The client with facial wounds would be considered the "walking wounded" and classified as nonurgent (class III, green tag).

A nurse assesses a client's respiratory status. Which information is most important for the nurse to obtain? a. Average daily fluid intake. b. Neck circumference. c. Height and weight. d. Occupation and hobbies.

d. Occupation and hobbies.

A younger woman from an unfamiliar culture is at high risk for breast cancer and is considering a prophylactic mastectomy and oophorectomy. What action by the nurse is most appropriate? a. Discourage this surgery since the woman is still of childbearing age. b. Reassure the client that reconstructive surgery is as easy as breast augmentation. c. Inform the client that this surgery removes all mammary tissue and cancer risk. d. Offer to include support people, such as the male partner, in the decision making

d. Offer to include support people, such as the male partner, in the decision making

Once the nurse has considered all possible collaborative and client problems, what action does the nurse take next? a. Act on the observed cues. b. Determine desired outcomes. c. Generate solutions. d. Prioritize the hypotheses.

d. Prioritize the hypotheses. Analyzing cues lead to a list of potential hypotheses. The nurse prioritizes them, determines the desired outcomes, generates solutions, and acts. This is part of the six-step clinical judgment model.

A nurse admits an older adult from a home environment. The client lives with an adult son and daughter-in-law. The client has urine burns on the skin, no dentures, and several pressure injuries. What action by the nurse is most appropriate? a. Ask the family how these problems occurred. b. Call the police department and file a report. c. Notify Adult Protective Services. d. Report the findings as per agency policy.

d. Report the findings as per agency policy. These findings are suspicious for abuse. Health care providers are mandatory reporters for suspected abuse. The nurse would notify social work, case management, or whomever is designated in facility policies. That person can then assess the situation further. If the police need to be notified, that is the person who will notify them. Adult Protective Services is notified in the community setting.

To demonstrate clinical reasoning skills, what action does the nurse take? a. Collaborating with co-workers to buddy up for lunch breaks b. Delegating frequent vital signs on a new postoperative patient c. Documenting a complete history and physical on an admission d. Requesting the provider order medication for a client with high potassium

d. Requesting the provider order medication for a client with high potassium The components of clinical reasoning include assessing, analyzing, planning, implementing, and evaluating. This nurse shows the ability to analyze by interpreting the meaning of the lab value, to plan by anticipating the consequences of the lab value, and to implement by taking action.

After teaching a client who has a history of cholelithiasis, the nurse assesses the client's understanding. Which menu selection indicates that the client understands the dietary teaching? a. Lasagna, tossed salad with Italian dressing, and low-fat milk b. Grilled cheese sandwich, tomato soup, and coffee with cream c. Cream of potato soup, Caesar salad with chicken, and a diet cola d. Roasted chicken breast, baked potato with chives, and orange juice

d. Roasted chicken breast, baked potato with chives, and orange juice Clients with cholelithiasis should avoid foods high in fat and cholesterol, such as whole milk, butter, and fried foods. Lasagna, low-fat milk, grilled cheese, cream, and cream of potato soup all have high levels of fat. The meal with the least amount of fat is the chicken breast dinner

5. A nurse prepares to discharge an older adult client home from the emergency department (ED). What actions does the nurse take to prevent future ED visits? (Select all that apply.) a. Provide medical supplies to the family. b. Consult a home health agency. c. Encourage participation in community activities. d. Screen for depression and suicide. e. Complete a functional assessment.

d. Screen for depression and suicide. e. Complete a functional assessment. Due to the high rate of suicide among older adults, a nurse would assess all older adults for depression and suicide. The nurse would also screen older adults for functional assessment, cognitive assessment, and risk for falls to prevent future ED visits.

The nurse is caring for a client with a large bowel obstruction due to fecal impaction. What position would be appropriate for the client while in bed? a. Prone b. Supine c. Recumbent d. Semi-Fowler

d. Semi-Fowler

A nurse reviews the laboratory results of a client who is receiving intravenous insulin. Which would alert the nurse to intervene immediately? a. Serum chloride level of 98 mEq/L (98 mmol/L) b. Serum calcium level of 8.8 mg/dL (2.2 mmol/L) c. Serum sodium level of 132 mEq (132 mmol/L) d. Serum potassium level of 2.5 mEq/L (2.5 mmol/L)

d. Serum potassium level of 2.5 mEq/L (2.5 mmol/L)

A nurse is caring for a client with acute pericarditis who reports substernal precordial pain that radiates to the left side of the neck. Which nonpharmacologic comfort measure should the nurse implement? a. Apply an ice pack to the client's chest. b. Provide a neck rub, especially on the left side. c. Allow the client to lie in bed with the lights down. d. Sit the client up with a pillow to lean forward on.

d. Sit the client up with a pillow to lean forward on. Pain from acute pericarditis may worsen when the client lays supine. The nurse should position the client in a comfortable position, which usually is upright and leaning slightly forwar

4. A client is in the family practice clinic reporting a severe "cold" that started 4 days ago. On examination, the nurse notes that the client also has a severe headache and muscle aches. What action by the nurse is best? a. Educate the client on oseltamivir. b. Facilitate admission to the hospital. c. Instruct the client to have a flu vaccine. d. Teach the client to sneeze in the upper sleeve.

d. Teach the client to sneeze in the upper sleeve. Sneezing and coughing into one's sleeve helps prevent the spread of upper respiratory infections. The client does have symptoms of the flu (influenza), but it is too late to start antiviral medications; to be effective, they must be started within 24 to 48 hours of symptom onset. The client does not need hospital admission. The client would be instructed to have a flu vaccination, but now that he or she has the flu, vaccination will have to wait until next year.

A nurse assesses a client after a thoracentesis. Which assessment finding warrants immediate action? a. The client rates pain as a 5/10 at the site of the procedure. b. A small amount of drainage from the site is noted. c. Pulse oximetry is 93% on 2 L of oxygen. d. The trachea is shifted toward the opposite side of the neck.

d. The trachea is shifted toward the opposite side of the neck.

A nurse plans care for a client with overflow incontinence. Which intervention does the nurse include in this client's plan of care to assist with elimination? a. Stroke the medial aspect of the thigh. b. Use intermittent catheterization. c. Provide digital anal stimulation. d. Use the Valsalva maneuver.

d. Use the Valsalva maneuver.

A nurse is caring for a client who is scheduled to undergo a thoracentesis. Which intervention would the nurse complete prior to the procedure? a. Measure oxygen saturation before and after a 12-minute walk. b. Verify that the client understands all possible complications. c. Explain the procedure in detail to the client and the family. d. Validate that informed consent has been given by the client.

d. Validate that informed consent has been given by the client.

16. A client is being discharged on long-term therapy for tuberculosis (TB). What referral by the nurse is most appropriate? a. Community social worker for Meals on Wheels b. Occupational therapy for job retraining c. Physical therapy for homebound therapy services d. Visiting nurses for directly observed therapy

d. Visiting nurses for directly observed therapy Directly observed therapy is often utilized for managing clients with TB in the community. Meals on Wheels, job retraining, and home therapy may or may not be appropriate.

A client tells the nurse that, even though it has been 4 months since her sisters death, she frequently finds herself crying uncontrollably. How should the nurse respond? a. Most people move on within a few months. You should see a grief counselor. b. Whenever you start to cry, distract yourself from thoughts of your sister. c. You should try not to cry. Im sure your sister is in a better place now. d. Your feelings are completely normal and may continue for a long time.

d. Your feelings are completely normal and may continue for a long time. Frequent crying is not an abnormal response. The nurse should let the client know that this is normal and okay. Although the client may benefit from talking with a grief counselor, it is not unusual for her to still be grieving after a few months. The other responses are not as therapeutic because they justify or minimize the clients response.


Related study sets

Examples of questions from the textbook Quiz 3

View Set

Computers in Healthcare Admin. Final Exam

View Set

Lesson 5: Small Business, Entrepreneurship, and Franchises

View Set

303 Practice Exam - need to study

View Set

Sociology Exam 2 -- Important Terms

View Set

Chapter 3: Corporate Social Responsibility and Citizens

View Set